EXAM 5 (CH 26,27,28,29,32,33)

Ace your homework & exams now with Quizwiz!

Best indicators of placenta previa

-clinical presentations -decreasing urinary output These are better indicators of acute blood loss than vital signs alone FHR normal painless bright red bleeding soft relaxed nondistender uterus with normal tone Fundal height in placenta previa -greater than expected for gestational age

Risk factors for chorioamnionitis

-long labor -prolonged membrane rupture -multiple vaginal examinations -use of internal FHR and contraction monitoring -young maternal age -lowe SES -nulliparity -preexisting infection of the lower genital tract

Risk factors for vasa previa

-low-lying placentas -pregnancies resulting from assisted reproductive technology -multiple gestations

Placenta Previa

-minimal to severe bleeding -life threatening -bright red blood -normal uterine tonicity -painless -abnormal location of placenta -presenting part is high (not engaged) -fetus is in breech, transverse, or oblique position -normal fetal heart rate and pattern

Considerations for use of Indomethacin (indocin)

-use only if gestation age <32 weeks -administer for 48 hours or less -do not use in woman with peptic ulcer disease, poorly controlled hypertension, asthma, coagulation disorders, renal or hepatic disease -administer with food to decrease GI distress

Caput Succedaneum

-usually disappears in 3-5 days after birth

Hyperemesis Gravidarum

-when vomiting during pregnancy becomes excessive enough to cause weight loss, electrolyte imbalance, nutritional deficiencies, and ketonuria -occurs in 0.5% of all births -1/10 of women with it have it for the entire duration of the pregnancy

What is the antidote for mag sulfate?

calcium chloride or Calcium Gluconate 10 ml of 10% solution administered IV push over 3 min. If administered too fast: vasodilatation, bradycardia, dysrhythmias, & ventricular fibrillation may occur.

Poor glycemic control before and during pregnancy in women who have diabetes

can lead to maternal complications such as miscarriage, infection, and dystocia (abnormally slow or difficult labor) caused by fetal macrosomia.

risk factors for cervical insufficiency

congenital: - collagen disorders - uterine anomalies - ingestion of DES aquired: - previous cervical trauma from lacerations during birth - mechanical dilation of cervix during OB procedures

S/S of cardiac decompensation:

cough and resp congestion, dyspnea and fatigue, palpitations and tachycardia, peripheral edema, chest pain, signs of resp infection, signs of HF and pulmonary edema

Daily Fetal Movement Count (DFMC)

count once daily for 60 min

Considerations for use of terbutaline (Brethine)

-Assess maternal glucose and potassium levels before treatment is initially Notify physician if patient has the following: -maternal HR >130 bpm -BP <90/60 mmHg -signs of pulmonary edema -fetal HR >180 bpm

Women with preexisting diabetes are at risk for what complications?

-Hypertension -Preeclampsia -Cesarean birth -Preterm birth -Maternal mortality *Risk of complications increases with the duration and severity of diabetes

A client who received an epidural for her operative delivery has vomited twice since the surgery. Which of the following prn medications ordered by the anesthesiologist should the nurse administer at this time? 1. Reglan (metoclopramide). 2. Demerol (meperidine). 3. Seconal (secobarbital). 4. Benadryl (diphenhydramine).

1. Reglan (metoclopramide).

A client is on magnesium sulfate via IV pump for severe preeclampsia. Other than patellar reflex assessments, which of the following noninvasive assessments should the nurse perform to monitor the client for early signs of magnesium sulfate toxicity? 1. Serial grip strengths. 2. Kernig assessments. 3. Pupillary responses. 4. Apical heart rate checks.

1. Serial grip strengths.

Assessment for Pulmonary Embolism

1. Sudden dyspnea and chest pain 2. Tachypnea and tachycardia 3. Cough and lung crackles 4. Hemoptysis 5. Feeling of impending doom

Three types of thrombophlebitis:

1. Superficial thrombophlebitis 2. Femoral thrombophlebitis 3. Pelvic thrombophlebitis

An ultrasound is being done on an Rh-negative woman. Which of the following pregnancy findings would indicate that the baby has developed erythroblastosis fetalis? 1. Caudal agenesis. 2. Cardiomegaly. 3. Oligohydramnios. 4. Hyperemia.

2. Cardiomegaly is one of the common signs of erythroblastosis fetalis. TEST-TAKING TIP: Erythroblastosis fetalis is the fetal condition that results when an Rh(-) mother who is sensitized to Rh(+) blood is pregnant with an Rh(+) baby. Maternal antibodies cross the placenta and destroy the fetal red blood cells. As a result, the baby becomes severely anemic. Cardiomegaly is one of the complications that occurs as a result of the severe anemia. 84. 1. RhoGAM

The nurse should expect to observe which behavior in a 3-week-multigravid postpartum client with postpartum depression? 1. Feelings of infanticide. 2. Difficulty with breastfeeding latch. 3. Feelings of failure as a mother. 4. Concerns about sibling jealousy

3. Feelings of failure as a mother.

The nurse is caring for a postoperative cesarean client. The woman is obese and is an insulin-dependent diabetic. For which of the following complications should the nurse carefully monitor this client? 1. Failed lactogenesis. 2. Dysfunctional parenting. 3. Wound dehiscence. 4. Projectile vomiting.

3. Wound dehiscence.

Which ratio would be used to restore effective circulating volume in a postpartum client who is experiencing hypovolemic shock? 4:1 2:1 1:1 3:1

3:1 Rationale A 3:1 ratio of 3 ml infused for every 1 ml of estimated blood lost is recommended to restore circulating volume.

Labor that has progressed to a cervical dilation of ____ cm or more is likely to lead to inevitable preterm birth

4 cm

The nurse in a birthing room is monitoring a client with dysfunctional labor for signs of fetal or maternal compromise. Which assessment finding should alert the nurse to a compromise? 1. maternal fatigue 2. coordinated uterine contractions 3, progressive changes in the cervix 4. persistent nonreassuring fetal heart rate

4. persistent nonreassuring fetal heart rate

An insulin-dependent diabetic is in active labor. The physician has written the following order: Administer regular insulin 5 units per hour via IV pump. The insulin has been diluted as follows: 50 units/500 mL normal saline. At what rate should the nurse set the pump? __________ mL/hr.

50 mL/hr

Hemorrhage is bleeding of ____ mL or more after delivery.

500 ml

What is used to confirm gestational diabetes?

A 3 hour oral glucose tolerance test

Chorioamnionitis

A bacterial infection of the amniotic cavity that is potentially life threatening for the fetus and the woman; it is the most common maternal complication of PROM.

Achieving and maintaining euglycemia are the primary goals of medical therapy for the pregnant woman with diabetes. These goals are achieved through a combination of diet, insulin, exercise, and blood glucose monitoring. The target blood glucose levels 1 hour after a meal should be _____________.

ANS: 110 to 129 mg/dl Target levels of blood glucose during pregnancy are lower than nonpregnant values. Accepted fasting levels are between 60 and 99 mg/dl, and 1-hour postmeal levels should be between 110 to 129 mg/dl. Two-hour postmeal levels should be 120 mg/dl or less.

What would a steady trickle of bright red blood from the vagina in the presence of a firm fundus suggest to the nurse? a. Uterine atony b. Lacerations of the genital tract c. Perineal hematoma d. Infection of the uterus

ANS: B Undetected lacerations will bleed slowly and continuously. Bleeding from lacerations is uncontrolled by uterine contraction. The fundus is not firm in the presence of uterine atony. A hematoma would develop internally. Swelling and discoloration would be noticeable; however, bright bleeding would not be. With an infection of the uterus, an odor to the lochia and systemic symptoms such as fever and malaise would be present.

Assessment for a hematoma?

Abnormal severe pain, pressure in perineal area (client states that she feels like she has to have a bowel movement), palpable sensitive swelling in the perineal area with discolored skin, inability to void, decreased hemoglobin and hematocrit levels, signs of shock such as pallor tachycardia and hypotension if significant blood loss has occurred

What is Hydralazine (Apresoline)

Antihypertensive that relaxes smooth muscle to reduce BP. Used in preeclampsia when BP is elevated to degree that might be associated with intracranial bleeding. IV bolus 5-10 mg may be administered as often as every 20 min. Continuous infusion of 100 mg in 200 mL NS with infusion pump. Titrate to maintain diastolic blood pressure at 80-90 mm Hg. cannot give w/ mag sulfate

Postpartum Infection

Any infection of the reproductive organs that occurs within 28 days of delivery or abortion

When is long-acting insulin given?

At bedtime

External cephalic version (ECV)

Attempt to turn the fetus from a breech or shoulder presentation to a vertex presentation for birth by exerting gentle, constant pressure on the abdomen.

Susan, a 34-year-old pregnant client, has had a consistently high BP ranging from 148/92 mm Hg to 160/98 mm Hg since she was 28 years old. Her weight gain has followed normal patterns and urinalysis remains normal as well.

B. Chronic hypertension

assessemnt and nursing diagnosis of preeclampsia

BP edema (distribution, degree, pitting) dependent edema if walking (feet and ankles) ir not (sacral) deep tendon reflexes - bicep and patellar reflexes) clonus ( Clonus is present if foot "jerks" or taps against examiner's hand after quickly pulling up on the foot. 3-4 taps = very close to seizure)Clonus indicates more pronounced hyperflexia & is indicative of CNS irritability. presence of proteinurea - evaluation of a 24-hr urine collection ( >300mg)

What is chorioamnionitis?

Bacterial infection of the amniotic cavity, can result from PROM, vaginitis, amniocentesis, or intrauterine procedures, may result in the development of postpartum endometritis and neonatal sepsis

When do most miscarriages occur?

Before 12 weeks of gestation (80%)

terbutaline (Brethine)

Beta2-adrenergic receptor agonist administered subcutaneously; it relaxes uterine smooth muscles and is used to diagnose preterm labor, temporarily suppress preterm labor, or treat uterine tachysystole."

When should pregnant women be screened for gestational diabetes?

Between 24 and 28 weeks of pregnancy

tocolytic

Classification of medications given to arrest labor after uterine con- tractions and cervical change have occurred."

Intrauterine growth restriction (IUGR)

Condition of inadequate fetal growth not necessarily correlated with the initiation of labor.

Ultrasonography

Diagnostic test that involves the use of sound having a frequency higher than that detectable by humans to examine structures inside the body. During pregnancy it can be doen by using either the transvaginal or transabdominal approach.

What is mainly used to treat diabetes?

Diet alone, some clients may need insulin

post partumum insulin care

During the first 24 hours postpartum, insulin requirements __decrease substantially because the placenta has been removed Possible postpartum complications for diabetics -preeclampsia -eclampsia -hemorrhage -infection (e.g. endometritis) Breastfeeding has an antidiabetogenic effect for the children of women with diabetes Breastfeeding women are at increased risk for hypoglycemia, especially in the early postpartum period and after breastfeeding sessions Insulin dose during breastfeeding is decreased possible hemorrhage due to overdistended uterus

When can excessive bleeding occur?

During the period between the separation of the placenta and its expulsion/removal -due to incomplete placental separation, undue manipulation of the fundus, or excessive traction on the cord

When does pregnancy exert a diabetogenic effect on the maternal metabolic status?

During the second and third trimesters

When is the risk of fetal compromise highest in diabetic moms?

During the third trimester

Polyphagia

Eating excessive amounts of food.

A nurse is providing home care for a patient who experienced the loss of an infant. The nurse notices that both the patient and partner appear unkempt and have little interest in day-to-day activities. Which intervention is appropriate for the nurse to take? Avoid discussing the loss of the infant. Reassure the parents that this is a normal part of grieving. Call the parents' family members and ask them to visit with the family. Encourage parents to share their feelings about the loss of their infant.

Encourage parents to share their feelings about the loss of their infant. Encouraging the parents to verbalize their feelings is the most appropriate intervention for this patient. This action is important to assist in the grieving process.

When do maternal insulin requirements increase?

From 18-24 weeks to 36 weeks of gestation

Which statement is most likely to be associated with a breech presentation? Least common malpresentation Descent is rapid Diagnosis by ultrasound only High rate of neuromuscular disorders

High rate of neuromuscular disorders Rationale Fetuses with neuromuscular disorders have a higher rate of breech presentation, perhaps because they are less capable of movement within the uterus. Breech is the most common malpresentation, affecting 3% to 4% of all labors. Descent is often slow because the breech is not as good a dilating wedge as is the fetal head. Diagnosis is made by abdominal palpation and vaginal examination. It is confirmed by ultrasound.

Nsg interventions for an HIV client during the intrapartum period?

Highest risk for exposure to fetus exists during delivery through the birth canal, avoid the use of internal scalp electrobes for fetal monitoring, avoid episiotomy to decrease amount of maternal blood in birth canal, aboid the administration of oxytocin, place heavy absorbent pads under the mother's hips to absorb amniotic fluid and maternal blood, minimize the neonates exposure to maternal blood and body fluid, remove neonate promplty from blood, suction fluids immediately from neonate, prepare to administer zidovudine as prescribed to mother during labor and delivery

What is the most common medical complication reported during pregnancy?

Hypertension

S/S Preeclampsia

Hypertension Proteinuria

Major Contraindications for use of ergonovine or methyelergonovine

Hypertension and CVD

When is ECV most successful

In a multiparous women who has a normal amount of amniotic fluid and whose fetus is not yet engaged in the pelvis

name the two ways an ultra sound can be performed

abdominally and transvaginally a full bladder to displace the uterus upward to provide a better image of the fetus.

Placenta accreta

abnormally adherent placenta

complications of gestational hypertension

abruptio placentae disseminated intravascule coagulation thrombocytopenia placental insufficiency intrauterine growth restriction intrauterine fetal death HEEP syndrome

hematoma

accumulation of blood in the connective tissue as a result of blood vessel damage

care management PROM

activ pursuit of labor (34-36 weeks) before 32 weeks - managed expectantly, hospitalized to prolong pregnancy assessment fetal non stress test fetal movements antenatal corticosteroids if less than 32 weeks antibiotics

Hyperemesis Gravidarum

Intractable N/V during 1st trimester that causes disturbances in nutrition and F&E balance. S/S nausea most pronounced on arising, persistent vomiting, wt loss, S/S dehydration, F&E imbalances.

What is anemia?

Iron deficiency anemia is a condition that develops as a result of an inadequate amount of serum iron, anemia predisposes the client to postpartum infection

Secondary arrest—no change

Labor pattern defined as no progress in dilation for 2 hours or more in both nulliparas and multiparas.

Protracted descent

Labor pattern during which fetal progress through the birth canal occurs at a rate of less than 1 cm/hour in nulliparas and less than 2 cm/hour in multiparas.

Nsg interventions for Hepatitis B?

Limit the number of vaginal exams (risk for intrapartum ascending infection), remove maternal blood from the neonate immediately after birth, suction the fluids from the neonate immediately after birth, bathe the neonate before any invasive procedure, clean and dry the face and eyes of the neonate before instilling eye prophylaxis

Maternal and Fetal contraindications to tocolysis

Maternal -preeclampsia with severe features of eclampsia -bleeding with hemodynamic instability -contraindications to specific tocolytic medications Fetal -Intrauterine fetal demise -lethal fetal anomaly -nonreassuring fetal status -chorioamnionitis -preterm PROM

Postprandial (postmeal) blood glucose

Measurement of blood glucose levels 1 to 2 hours after eating; acceptable results for 1 hour after meals range from 130 to 140 and for 2 hours after meals it should be 120."

Fasting (preprandial) blood glucose

Measurement of blood glucose levels prior to eating. Acceptable results for this test range from 66 to 90 during pregnancy."

The common bleeding disorders of early pregnancy.

Miscarriage (spontaneous abortion); reduced cervical competence (premature dilation of the cervix); ectopic pregnancy; hydatidiform mole (molar pregnancy)

What is used to ripen the cervix?

Misoprostol; prostaglandins -tends to help reduce the amount of oxytocin necessary

Nsg interventions for pregnant client with diabetes during labor?

Monitor fetal status continuously for signs of distress and if noted prepare the client for immediate c-section, carefully regulate insulin and provide glucose intravenously as prescribed because labor depletes glycogen

What are the symptoms of HELLP syndrome?

Nausea (50%), Vomiting (50%), Malaise (90%), Flulike symptoms, or Epigastric pain (65%)

Gestational hypertension

Onset of hypertension without proteinuria after the 20th week of pregnancy; it does not persist longer than 12 weeks postpartum, usually resolving during the first postpartum week.

Pulmonary Embolism

Passage of a thrombus, often originating in a uterine or other pelvic vein, into the lungs, where it disrupts the circulation of the blood.

Postpartum women experience an increased risk for urinary tract infection. A prevention measure the nurse could teach the postpartum woman would be to what? Acidify the urine by drinking three glasses of orange juice each day. Maintain a fluid intake of 1 to 2 L/day. Empty her bladder every 4 hours throughout the day. Perform perineal care on a regular basis.

Perform perineal care on a regular basis. Rationale Urine is acidified with cranberry juice. The woman should drink at least 3 L of fluid each day. The woman should empty her bladder every 2 hours to prevent stasis of urine. Keeping the perineum clean will help prevent a urinary tract infection.

oxytocin (Pitocin)

Pituitary hormone used to stimulate uterine contractions in the augmentation or induction of labor.

Which term is used to describe perforation of the uterus due to placental adherence to the uterine walls? Placenta previa Placenta increta Placenta accreta Placenta percreta

Placenta percreta Rationale Placenta percreta is the condition in which the endometrium is perforated due to the adherence of the placenta. Placenta previa is a complication in which the placenta is implanted near the cervix. Deep penetration of the endometrium by the placenta is known as placenta increta. Placenta accreta is the condition in which the placenta penetrates slightly into the endometrium.

A pregnant client after 20 weeks of gestation reports painless bright red vaginal bleeding. Upon assessment the nurse finds that the client's vital signs are normal. Which condition does the nurse suspect in the patient? Eclampsia Preeclampsia Pyelonephritis Placenta previa

Placenta previa Rationale Placenta previa is indicated by painless bright red vaginal bleeding during the second or third trimester of pregnancy. The client's vital signs may be normal even after blood loss, because a pregnant client can lose up to 40% of the blood volume without any signs of shock. Eclampsia is the onset of seizure activity in a client with preeclampsia. Preeclampsia is indicated by hypertension and proteinuria after 20 weeks of gestation. Pyelonephritis, which is identified by fever, shaking chills, and aching in the lumbar area of the back, is an infection caused by the E. coli organism.

Human chorionic gonadotropin (hCG)

Placental hormone used in the diagnosis of pregnancy and pregnancy loss.

A nurse participates in the stabilization of a neonate born at 26 weeks' gestation. The neonate's skin is reddened, gelatinous, and thin, and the neonate is grunting and has severe substernal retractions. Which priority intervention should the nurse implement during stabilization? Placing a hat on the neonate's head Starting a peripheral IV in the neonate's hand Placing the neonate in a polyethylene wrap before drying Placing cardiopulmonary monitor leads on the neonate's chest

Placing the neonate in a polyethylene wrap before drying Placing the neonate in a polyethylene wrap before drying decreases the mechanisms of heat loss. Thermoregulation is a priority during the stabilization process.

Poor glycemic control at the beginning of pregnancy increases risk of what for the mom?

Preeclampsia

Spontaneous miscarriage (abortion);

Pregnancy that ends as a result of natural causes before 20 weeks of gestation, the point of viability when the fetus may survive outside the uterus. The five types are threatened; inevitable; incomplete; complete; missed

Amniocentesis

Prenatal diagnostic test performed to obtain amniotic fluid to examine the fetal cells it contains

Percutaneous umbilical blood sampling

Prenatal diagonstic test that provides direct access to the fetal circulation during the second and third trimesters

Missed abortion

Products of conception are retained in utero after fetal death

Which clinical reports does the nurse evaluate to identify ectopic pregnancy in a client? Select all that apply. Quantitative human chorionic gonadotropin (β-hCG) levels Transvaginal ultrasound Progesterone level Thyroid test reports Kleihauer-Betke (KB) test

Quantitative human chorionic gonadotropin (β-hCG) levels Transvaginal ultrasound Progesterone level Rationale An ectopic pregnancy is indicated when β-hCG levels are greater than 1500 milli-international units/ml, but no intrauterine pregnancy is seen on the transvaginal ultrasound. A transvaginal ultrasound is repeated to verify if the pregnancy is inside the uterus. A progesterone level less than 5 ng/ml indicates ectopic pregnancy. Thyroid test reports need to be evaluated in case the patient has hyperemesis gravidarum, because hyperthyroidism is associated with this disorder. The KB test is used to determine transplacental hemorrhage.

When do maternal tissues regain their prepregnancy sensitivity to insulin?

Quickly after the expulsion of the fetus and placenta -for nonbreastfeeding mother- usually returns within 7-10 days of birth

Glycosuria

Results when unusable glucose is excreted."

UTI

Risk factors: -frequent pelvic examinations -regional anesthesia -Genital tract injury -urinary catheterization -c-section -history of UTI UTI Risk Factors catheters, epidurals, pelvic exams, genital tract injuries, history of UTI, and C-section. Signs and symptoms of UTI dysuria, frequency, urgency, low-grade fever, urinary retention, hematuria, and pyuria.

Placenta Accreta, Percreta, and Increta

Risk factors: -placenta previa -prior uterine surgery -endometrial defects -submucosal fibroids -parity -maternal age Can be diagnosed before birth using ultrasound and MRI -can lead to excessive bleeding when delivery of placenta is attempted -hysterectomy may be indicated for all 3 types of placental adherence of bleeding is uncontrolled Treatment- blood component replacement

Match the risk factor with the most likely condition.

Superficial venous thrombosis (SVT) Decreased risk of pulmonary embolism Deep venous thrombosis (DVT) Can progress to pulmonary embolism Pulmonary embolism (PE) Can very quickly result in maternal mortality

dysmaturitysyndrome

Syndrome characterized by dry, cracked, peeling skin; long nails; meconium staining of skin, nails, and umbilical cord; and perhaps loss of subcutaneous fat and muscle mass.

Recurrent (habitual) miscarriage

Term used to refer to three or more spontaneous pregnancy losses prior to 20 weeks of gestation

Induction of labor

The chemical or mechanical initiation of uterine contractions before their spontaneous onset

gestational age

before 22 weeks ultrasound performed before 22 weeks of gestation is the best rather than menstrual dates because the ultra sound dating is more accurate than even "sure: menstrual dates

The nurse finds diuresis, weight loss, and muscle atrophy in a pregnant client with mild preeclampsia. What could the nurse conclude from these findings? The client was mostly on a liquid diet. The client was on prolonged bed rest. The client has developed HELLP syndrome. The client is at risk for placental abruption.

The client was on prolonged bed rest. Rationale Prolonged bed rest in clients with preeclampsia may result in diuresis and fluid, electrolyte, and weight loss. Therefore the nurse advises the client to restrict activity instead of taking complete bed rest. A liquid diet may contribute to weight loss, but does not cause diuresis or muscle atrophy. HELLP syndrome is characterized by hemolysis (H), elevated liver enzymes (EL), and low platelet count (LP) in a patient with preeclampsia. Vaginal bleeding will indicate placental abruption in the client.

Amniotic Fluid embolism

The escape of amniotic fluid into maternal circulation. -Debris-containing amniotic fluid deposits in pulmonary arterioles and fatal to mothers S/S Abrupt onset of respiratory distress and chest pain, Cyanosis, Fetal bradycardia and distress

Occipitoposterior

The most common fetal malposition.

Breech

The most common form of malpresentation.

What is the major source of insulin resistance during pregnancy?

The placenta

Placenta previa

The placenta is implanted in the lower uterine segment such that it completely or partially covers the cervix (causes bleeding when cervix dilates or lower uterine segment effaces) -can be complete or marginal

Premature Rupture of Membranes (PROM)

The spontaneous rupture of the amniotic sac and leakage of amniotic fluid beginning before the onset of labor at any gestational age

How would the nurse determine that the mother's body cannot metabolize glucose? Urine positive for blood Urine positive for bacteria Urine positive for ketones Urine positive for white blood cells

Urine positive for ketones Ketones in urine indicate breakdown of muscle tissue for energy when glucose cannot be metabolized by body cells.

What can precipitate uterine rupture?

Uterine tachysystole

The nurse should avoid which assessment in a patient who is pregnant, presents with vaginal bleeding, and has unknown placental location? Urinalysis Vaginal exam Nonstress test Abdominal ultrasound

Vaginal exam Vaginal exams should be avoided when placental location is unknown and there is vaginal bleeding. The patient may have placenta previa, and a digital vaginal exam can cause placental separation.

Which medications can cause neural tube defects in the fetus? Aspirin (Acuprin) Folic acid supplements Pyridoxine supplements Valproic acid (Depakote) Carbamazepine (Tegretol)

Valproic acid (Depakote) Carbamazepine (Tegretol) Rationale Valproic acid (Depakote) and carbamazepine (Tegretol) are anticonvulsants that have teratogenic effects. These medications impair neural tube formation and cause neural tube defects in the fetus. Folic acid supplements help to prevent neural tube defects because they do not impair neural tube formation. Aspirin (Acuprin) is an analgesic and does not have teratogenic effects. Pyridoxine or vitamin B 6 supplements do not affect neural tube formation, nor do they cause neural tube defects.

Low birth weight (LBW)

Weight at the time of birth of 2500 g or less.

How can you distinguish PPH due to lacerations of the cervix, vagina, or perineum?

When the uterine fundus is firm and contracted but bleeding continues

Define abortion

a pregnancy that ends before 20 weeks gestation either spontaneously or electively

complete rupture of the uterus

direct communication between the uterine and eritoneal cavities

coagulopathy

disorder where coagulation is compromised, results in continuous bleeding

uteroplacental insufficiency (UPI)

ecline in placental function-echange of gases, nutrients, and waste-leading to fetal hypoxia and acidosis; evidenced by late fetal heart rate decelerations in response to uterine contractions

Precipitous labo

is defined as labor that lasts than 3 hours from the onset of contractions to the time of birth.

Ectopic pregnancy

is one in which the fertilized ovum is implanted outside the uterine cavity; this is a significant cause of maternal morbidity and mortality.

biophysical profile (BPP)

noninvasive dynamic assessment of a fetus that is based on acute and chronic markers of fetal disease. It includes AFV, FMBs, fetal movements, and fetal tone determined by means of the NST BPP 8-10 normal limitation - sleep cycle

A count of fewer than 3 fetal movements within 1 hour warrants further evaluation by a

nonstress test (NST) or a contraction stress test and a complete or modified biophysical profile

Soft Tissue Dystocia

results from obstruction of the birth passage by an anatomic abnormality other than that involving the bony pelvis -can be due to placenta previa -uterine fibroids -ovarian tumors

recognized causes of this delay/subinvolution include

retained placental fragments and pelvic infection

sociodemographic

risks arise from the mother and her family

psychological

risks consist of maternal behaviors and adverse lifestyles that have a negative effect on the health of the mother or fetus

biophysical

risks that include factors that originate within the mother or fetus and affect the development or functioning of either on or both

In vasa previa

the fetal vessels lie over the cervical os, and the vessels are implanted into the fetal membranes rather than into the placenta

What are surgical management options for PPH?

uterine tamponade, uterine packing, uterine compression sutures or Hysterectomy.

uterine inversion

uterus completely or patially turn inside out risk factors: fundal implantation of the placenta, manual extraction of the placenta, short umbelical cord, uterine atony, leiomyomas, abnormally adherent placental tissue assessment : depression in the fundal area of the uterus; interior of the uterus may be seen through the cervix or protuding through the vagina; clinet is in severe pain, hemorrhage, shock interventions - monitor for hemorrhage, shock, treat shock; laparatomy with replacemrntof uterus to corrcr position

fetal alarm signal

when fetal movements cease entirely for 12 hours

Predisposing Factors for Postpartum Hemorrhag

■ Previous history of postpartum hemorrhage ■ Placenta previa ■ Abruptio placentae ■ Overdistention of the uterus—polyhydramnios, multiple gestation, large neonate ■ Infection ■ Multiparity ■ Dystocia or labor that is prolonged ■ Operative delivery—cesarean or forceps delivery, intrauterine manipulation

Assessment for Pelvic Thrombophlebitis

■ Severe chills ■ Dramatic body temperature changes ■ Pulmonary embolism may be the first sign

Polyuria

Excretion of large volumes of urine."

What often develops during the 3rd trimester of pregnancy in women with diabetes?

(poly)hydramnios- amniotic fluid index >25 cm Complications of polyhydramnios -placental abruption -uterine dysfunction -postpartum hemorrjage

The nurse is assisting in the delivery of a baby via vacuum extraction. Which of the following nursing diagnoses for the gravida is appropriate at this time? 1. Risk for impaired skin integrity. 2. Risk for body image disturbance. 3. Risk for impaired parenting. 4. Risk for ineffective sexuality pattern

. The woman is at risk of impaired skin integrity. TEST-TAKING TIP: Clients who are delivered by vacuum extraction are at high risk for lacerations. Their skin integrity, therefore, is at risk. Although a mother may have emotional responses as a result of the delivery method, they will not appear at this time. They would be evident during the postpartum period, not when the delivery is occurring

On ultrasound, it is noted that the pregnancy of a hospitalized woman who is carrying monochorionic twins is complicated by twin-to-twin transfusion. The nurse should carefully monitor this client for which of the following? 1. Rapid fundal growth. 2. Vaginal bleeding. 3. Projectile vomiting. 4. Congestive heart failure.

1. Fundal growth is often accelerated. TEST-TAKING TIP: Fundal growth is accelerated for two reasons: (a) with two babies in utero, uterine growth is increased and (b) the recipient twin-the twin receiving blood from the other twin-often produces large quantities of urine resulting in polyhydramnios.

A client is receiving IV heparin for deep vein thrombosis. Which of the following medications should the nurse obtain from the pharmacy to have on hand in case of heparin overdose? 1. Vitamin K. 2. Protamine. 3. Vitamin E. 4. Mannitol.

2. Protamine.

A woman, 40 2⁄7 weeks' gestation, has had ruptured membranes for 15 hours with no labor contractions. Her obstetrician has ordered 10 units oxytocin (Pitocin) to be diluted in 1000 mL D51⁄2 NS. The order reads: administer oxytocin IV at 0.5 milliunits per min. Calculate the drip rate for the infusion pump to be programmed. __________ mL/hr.

3 mL/hr.

The nurse is assessing the Bishop score on a postdates client. Which of the following measurements will the nurse assess? Select all that apply. 1. Gestational age. 2. Rupture of membranes. 3. Cervical dilatation. 4. Fetal station. 5. Cervical position.

3, 4, and 5 are correct. 3. Cervical dilatation is part of the Bishop score. 4. Fetal station is part of the Bishop score. 5. Cervical position is part of the Bishop score. TEST-TAKING TIP: The Bishop score is calculated to determine the inducibility of the cervix. Although gestational age may be an indication for calculating the score, it does not have a direct impact on status of the cervix. Similarly, although rupture of the membranes may be an indication for calculating the score, that fact does not have a direct impact on status of the cervix.

On admission to the labor and delivery suite, the nurse assesses the discharge needs of a primipara who will be discharged home 4 days after a cesarean delivery. Which of the following questions should the nurse ask the client? 1. "Have you ever had anesthesia before?" 2. "Do you have any allergies?" 3. "Do you scar easily?" 4. "Are there many stairs in your home?"

4. "Are there many stairs in your home?"

The nurse is caring for a client, G3 P2002, whose infant has been diagnosed with a treatable birth defect. Which of the following is an appropriate statement for the nurse to make? 1. "Thank goodness. It could have been untreatable." 2. "I'm so happy that you have other children who are healthy." 3. "These things happen. They are the will of God." 4. "It is appropriate for you to cry at a time like this."

4. "It is appropriate for you to cry at a time like this."

The nurse caring for a client who is 28 hours postpartum notes a temperature of 101.2 ° F. What should the nurse do?

A temperature of 100.4 ° F is normal during the first 24 hours postpartum because of dehydration; a temperature of 100.4 ° F or greater after 24 hours postpartum indicates infection. Therefore, if the temperature is 101.2 ° F 28 hours postpartum, the nurse should report the finding to the health care provider.

infection/ temperature

A temperature of 100.4 ° F is normal during the first 24 hours postpartum because of dehydration; a temperature of 100.4 ° F or greater after 24 hours postpartum indicates:

betamethasone

An antenatal glucocorticoid administered intramuscularly to acceler- ate fetal lung maturation when there is risk for preterm birth; its use results in an increase in the production and release of surfactant."

Cullen sign

An ecchymotic blueness around the umbilicus indicating hematoperitoneum; it may develop in an undiagnosed intraabdominal ectopic pregnancy.

What can partial hydatidiform moles be mistaken for?

An incomplete or missed miscarriage

Cystitis

An infection of the bladder, can occur in the postpartum period and the postpartum woman should be encouraged to consume adequate fluids and void frequently to avoid bladder distention.

Assessment of endometritis?

An infection of the lining of the uterus occurring in the postpartum period and caused by bacteria that invade the uterus at the placental site, the infection may spread and involve the entire endometrium and cause peritonitis or pelvic thrombophlebitis

Incompetent cervix interventions

Bed rest Hydration Tocolysis to inhibit uterine contractions -Prepare for cervical cerclage- band of fascia or nonabsorable ribbon place around cervix beneath mucosa to constrict internal os & removed at 37wks or c-section performed. -After cervical cerclage- teach client to refrain from intercourse and avoid prolonged standing and heavy lifting. -Monitor for contractions, rupture of membranes and signs of infection -Report vaginal bleeding or increased uterine contractions

Which test does the RN anticipate that the HCP will order after considering... appendicites or ectopic pregnancy

CBC,UA,hCG,McBurney's, Urine Preg Test

Amniotomy

Artificial rupture of membranes -used to induce labor when the condition of the cervix is ripe to accelerate progress -risk of chorioamnioniits if labor is not stimulated -risk of variable decelerations with cord prolapse -actual rupture itself is painless but woman may experinence pain when sharp amnihook is inserted through vagina and cervix Considerations: -presenting part must be engaged before rupture to prevent prolapsed cord -woman should be free of active infection of genital tract (e.g. herpees) and HIV -FHR should be assessed before and immediately after amniotomy to detect any changes -tachycardia is okay

Amniotomy

Artificial rupture of the membranes often used to induce labor when the cervix is ripe or to augment labor if progress begins to slow."

Vacuum-assisted birth; vacuum extraction

Birth method involving the attachment of a vacuum cup to the fetal head using negative pressure.

prostaglandin

Classification of medications that can be used to ripen the cervix, stimulate uterine contractions, or both.

Dawn has been hypertensive since her 24th week of preg- nancy. Urinalysis indicates a protein content of 3+. Further testing reveals a platelet count of 95,000 and elevated aspartate aminotransferase (AST) and alanine aminotransferase (ALT) levels; she has begun to experience nausea with some vomiting and epigastric pain.

D. HELLP syndrome

Oligohydraminos

Decreased or deficient amount of aminotic fluid

Premature separation of the placenta (abruptio placentae)

Detachment of part or all of a normally implanted placenta from the uterus.

Preterm birth

ny birth that occurs between 20 and 37 weeks of gestation.

Polydipsia "

Excessive thirst."

Chronic hypertension

Hypertension that is present before pregnancy

There is an association between hyperemesis gravidarum and what other condition?

Hyperthyroidism

What contraceptive method should not be used in patients who just dealt with a molar pregnancy?

IUD

preeclampsia causes

IUGR fetal demise increase rate of stillbirth

Placenta previa

Implantation of the placenta in the lower uterine segment such that it completely or partially covers the cervix or is close enough to the cervix to cause bleeding when dilation or effacement occurs.

Prostaglandin Synthetase Inhibitors (NSAIDs)

Indomethacin (indocin) -relaxes uterine smooth muscle by inhibiting prostaglands Maternal side effects: -nausea and vomiting -heartburn Fetal side effects: -constriction of ductus arteriosus -oligohydramnios -neonatal pulmonary hypertension Severe maternal adverse effects: -GI bleeding -prolonged bleeding time -asthma in aspirin-sensitive clients

At a 2-month follow-up visit, a postpartum woman reports pain and tenderness in one breast. Which behaviors by the woman could indicate mastitis? Changing positions during feedings Using a supplemental nursing system (SNS) Keeping the straps snug while wearing an infant carrier Pumping the breasts if the infant is asleep during a scheduled feeding time

Keeping the straps snug while wearing an infant carrier Wearing a tight or constricting infant carrier may compress the breasts and increases the risk of mastitis. This behavior would indicate possible mastitis infection.

The nurse observes that a client has a decreased uterine size and has lost 5 lbs. at 42 weeks of gestation. Which signs should the nurse assess in the neonate after birth? Seizures Long nails Dry and peeling skin High body temperature Meconium-stained skin

Long nails Dry and peeling skin Meconium-stained skin Rationale Reduced uterine size and a maternal weight loss of 3 lb/week or more are clinical manifestations of postterm pregnancies. Postterm infants are at an increased risk of meconium aspiration. These infants are most likely to have postmaturity syndrome, which is characterized by long nails, dry and peeling skin, and meconium-stained skin. Seizures (indicative of cerebral palsy) and high body temperature (indicative ofsepsis) are the complications associated with infants who are born to women with chorioamnionitis during labor.

Is breastfeeding contraindicated if the neonate has been vaccinated?

No, support breast feeding after neonatal treatment for hepB

While waiting for a cesarean delivery because of cord prolapse, a patient in the Trendelenburg position reports discomfort and asks for her head to be lifted up. The nurse offers to reposition her onto her side with pillows supporting her hips. Which is the nursing rationale for this action? Pillows maintain elevation of the hips. A side-lying position relieves pressure on the patient's back. The head should not be lifted to prevent orthostatic hypotension. A side-lying position reduces the risk of precipitate birth while awaiting a cesarean section.

Pillows maintain elevation of the hips. Using pillows to elevate the hips in a side-lying position helps maintain fetal oxygenation by relieving pressure on the umbilical cord.

Diagnosis of Abruptio Placentae

Primarily based on symptoms (sometimes it shows up on US) -evidence of DIC -confirmed after birth by visualization of placenta

Cerclage

Procedure in which a suture is placed around the cervix beneath the mucosa to constrict the internal os of the cervix.

2 Categories of Preterm Births

Spontaneous and indicated

Lamicel

Synthetic cervical dilator containing magnesium sulfate

Diabetes in pregnancy puts the fetus at risk in several ways. Nurses should be aware that: With good control of maternal glucose levels, sudden and unexplained stillbirth is no longer a major concern. The most important cause of perinatal loss in diabetic pregnancy is congenital malformations. Infants of mothers with diabetes have the same risks for respiratory distress syndrome because of the careful monitoring. At birth, the neonate of a diabetic mother is no longer in any greater risk

The most important cause of perinatal loss in diabetic pregnancy is congenital malformations. Congenital malformations account for 30% to 50% of perinatal deaths in diabetic pregnancies. Even with good control, sudden and unexplained stillbirth remains a major concern. Infants of diabetic mothers are at increased risk for respiratory distress syndrome, and the transition to extrauterine life is often marked by hypoglycemia and other metabolic abnormalities.

A pregnant client is diagnosed with succenturiate placenta. What should the nurse understand about the client's condition? The placenta separates prematurely from the uterus. The placenta is implanted in the lower uterine segment. The placental trophoblasts have undergone proliferative growth. The placenta is divided into more lobes, rather than a single lobe.

The placenta is divided into more lobes, rather than a single lobe. Rationale When the placenta is divided into two or more lobes, rather than as a single mass, it is called a succenturiate placenta. Premature separation of the placenta from the uterus is called abruptio placentae. A placenta implanted in the lower uterine segment is known as placenta previa. Benign proliferative growth of the placental trophoblast is known as molar pregnancy.

A woman with a body mass index (BMI) of 30 delivered an infant after 12 hours of labor. The woman has the greatest risk for which complication? Hematoma Uterine atony Subinvolution Perineal laceration

Uterine atony Obesity and prolonged labor put the woman at increased risk for uterine atony.

Assessment for chorioamnionitis?

Uterine tenderness and contractions, elevated temperature, maternal or fetal tachycardia, foul odor to amniotic fluid, leukocytosis

essential component of the care management for women with premature rupture of membranes

Vigilance for signs of infection

A 40-year-old woman with a high body mass index is 10 weeks pregnant. Which diagnostic tool is appropriate to suggest to her at this time? Biophysical profile Amniocentesis Maternal serum alpha-fetoprotein (MSAFP) Transvaginal ultrasound

Transvaginal ultrasound Rationale Transvaginal ultrasound is useful for obese women whose thick abdominal layers cannot be penetrated adequately with the abdominal approach. A biophysical profile is a method of biophysical assessment of fetal well-being in the third trimester. An amniocentesis is performed after the fourteenth week of pregnancy. A MSAFP test is performed from week 15 to week 22 of the gestation (weeks 16 to 18 are ideal). An ultrasound is the method of biophysical assessment of the infant that is performed at this gestational age.

What should be done before external cephalic version is attempted?

Use an altrasound to: -Determine the fetal position -Locate the umbilical cord -Rule out placenta previa -Determine adequacy of maternal pelvis -Assess the amount of amniotic fluid, the gestational age, and the presence of any anomalies - A nonstress test is done to confirm fetal well-being Give a tocolytic agent is given to relax the uterus and facilitate the maneuver

Unplanned cesarean birth

emergency

Marginal placenta previa

he edge of the placenta is seen on transvaginal ultrasound to be 2.5 cm closer to the internal cervical os

Macrosomia

large-bodied baby commonly seen in diabetic pregnancies

Which statement made by a woman diagnosed with gestational diabetes indicates to the nurse that the teaching about dietary practices was effective? "If I feel like the baby's not moving a lot, I eat a candy bar." "I have been eating three small meals and two snacks every day." "I use my husband's diabetes pills when I run out of insulin." "I know my baby needs extra sugar, so I have been eating only fruit."

"I have been eating three small meals and two snacks every day." Calories should be distributed among three meals and two or more snacks.

Types of Miscarriages

1. Threatened 2. Inevitable 3. Incomplete 4. Complete 5. Missed

A woman has just had a macrosomic baby after a 12-hour labor. For which of the following complications should the woman be carefully monitored? 1. Uterine atony. 2. Hypoprolactinemia. 3. Infection. 4. Mastitis.

1. Uterine atony.

The nurse should suspect puerperal infection when a client exhibits which of the following? 1. Temperature of 100.2°F. 2. White blood cell count of 14,500 cells/mm3. 3. Diaphoresis during the night. 4. Malodorous lochial discharge.

4. Malodorous lochial discharge

What signs in a pregnant preeclamptic client would indicate the invasion stage of the tonic-clonic seizures? Select all that apply. Fixed eyes Bloodshot eyes Protruding eyes Stertorous inhalation Twitching of facial muscles

Fixed eyes Twitching of facial muscles Rationale If a pregnant client with preeclampsia is in the tonic-clonic convulsion stage, the nurse may observe signs such as fixed eyes and twitching of facial muscles. These signs last for only 2 to 3 seconds. Bloodshot eyes and protruding eyes are signs of the stage of contraction. Stertorous inhalation can be observed in the stage of convulsion but is not seen in the stage of invasion.

Transient HTN

Gestational HTN with NO signs of preeclampsia present at time of birth and HTN resolves by 12 week after birth

Risks of maternal blood loss for mom and fetus

Increased risk for: -hypovolemia - anemia -infection -preterm labor - oxygen delivery to the fetus For fetus: --blood loss -anemia -hypoxemia -hypoxia -anoxia -preterm birth

Glycemic control"

Key to optimal outcome of a diabetic pregnancy before conception and throughout pregnancy."

What is the newborn born to a diabetic mother at risk for?

Macrosomia, hypoglycemia, hyperbilirubinemia, respiratory distress syndrome, hypocalcemia, and congenital anomalies

Should nifedipine (procardia) and magnesium sulfate be administered simultaneously?

No- can cause skeletal muscle blockade

Prolapse of umbilical cord

Obstetric emergency in which the umbilical cord lies below the presenting part of the fetus; it may be occult (hidden) or more commonly frank (visible).

Nsg interventions for ectopic pregnancy?

Obtain assessment data and vital signs, monitor bleeding and initiate measures to prevent rupture and shock, prepare client for laparotomy and removal of the pregnancy and tube if necessary or repair of the tube, administer antibiotics, Rho(D) immune globulin is prescribed for Rh-negative women

Operative Vaginal Births

Performed using either forceps or a vacuum extractor

Insulin

Released by beta cells in the islets of Langerhands in the pancreas -regulate blood glucose levels by enabling glucose to enter adipose and muscle cells where it is used for energy

amniotic fluid amount

amniotic fluid index - sum of the four quadrants of amniotic fluid normal 10 cm-25 cm 5-10 lw less than 5 oligohydrominos more than 25 polyhydromnios

HELLP syndrome

is a laboratory diagnosis for a variant of severe preeclampsia that involves hepatic dysfunction

Postterm pregnancy

may be complicated by abnormal fetal growth, decreased amniotic fluid, and an increased risk for hemorrhage and infection.

common bleeding disorders in early pregnancy

miscarriage cervical insufficiency ectopic pregnancy hydatidiform mole molar pregnancy

assymetric growth suggests

placental insufficiency hypertension renal disease cardiovascular disease

The fetus produces its own insulin and pulls glucose from the mother

predisposes the mother to hypoglycemic reactions

placenta increta

term used to refer to DEEP penetration of the MYOMETRIUM by the placenta

placenta accreta

term used to refer to SLIGHT penetration of the MYOMETRIUM by placenta

Vasa Previa

when fetal vessels lie over the cervical os -vessels are implanted into the fetal membranes rather than into the placenta -vessles are not protected by Wharton's jelly -at risk of rupture or compression

Abruptio Placentae

-premature separation of the placenta from the uterus -occurs after 20 weeks of gestation but before birth

What decreases risk of hyperemesis gravidarum?

-Age of >/= 30 years -women who smoke

The nurse correctly explains to the nursing student that the majority of ectopic pregnancies are located where? Uterine fundus Cervical os Ampulla Fimbriae

Ampulla Rationale The majority of ectopic pregnancies, approximately 80 percent, are located in the ampulla, or the largest portion of the tube. A pregnancy within the uterus would be considered a normal pregnancy. Implantation of the pregnancy at the cervical os would be a significant abnormality.

What is endometritis?

An infection of the lining of the uterus occurring in the postpartum period and caused by bacteria that invade the uterus at the placental site, the infection may spread and involve the entire endometrium and cause peritonitis or pelvic thrombophlebitis

Hydatidiform mole

is a benign proliferative growth and a gestational trophoblastic disease characterized by grapelike clusters. GTD is a group of pregnancy-related trophoblastic proliferative disorders without a viable fetus that are caused by abnormal fertilization.

Amniotic fluid embolism interventions

Institute emergency measures to maintain life Admin oxygen 8-10 l face mask Prepare for intubation and mechanical ventilation Position client on side Admin IV fluids, blood products and meds Monitor fetal status Prepare for emergency delivery when client stabilized.

Postpartum hemorrhage

is a leading cause of maternal morbidity and mortality throughout the world. Diagnosis, expected outcomes of care, and intervention are based on the cause of PPH.

The nurse is reviewing the HCP prescriptions for a client admitted for premature rupture of the membranes.. Gestational age of the fetus is determined to be 37 weeks. Which prescription should the nurse question? 1. monitor fetal HR continuously 2. monitor maternal vital signs frequently 3. perform a vaginal exam every shift 4. administer an antibiotic per HCP prescription and per agency protocol

3. perform a vaginal exam every shift

The charge nurse is planning nursing assignments on a busy antepartum unit. For which patient does the nurse know that cesarean delivery is contraindicated? A patient with preeclampsia A patient with cephalopelvic disproportion A patient with a confirmed absence of fetal heartbeat A patient with premature rupture of membranes (PROM)

A patient with a confirmed absence of fetal heartbeat Fetal death is a contraindication for cesarean delivery. The surgical risk(s) to the mother (infection, hemorrhage, atelectasis, and anesthesia complications) outweigh the benefit of cesarean delivery in this case.

A client is 36 hours post-cesarean section. Which of the following assessments would indicate that the client may have a paralytic ileus? 1. Abdominal striae. 2. Oliguria. 3. Omphalocele. 4. Absent bowel sounds

4. Absent bowel sounds

Which woman is at greatest risk for late postpartum hemorrhage? A 17-year-old nullipara A woman with preeclampsia A 38-year-old woman with placenta accreta A woman whose labor was induced and who delivered twins naturally

A 38-year-old woman with placenta accreta Advanced maternal age and placenta accreta are risk factors for development of late postpartum hemorrhage.

Which patient is at greatest risk for gestational trophoblastic disease? A 17-year-old who douches daily A 25-year-old who smokes cigarettes A 30-year-old who uses an intrauterine device A 47-year-old with a history of molar pregnancies

A 47-year-old with a history of molar pregnancies Women who have had one molar pregnancy are at greater risk for having another. Age (at both ends of the reproductive spectrum) is an additional risk factor.

nifedipine (Procardia)

A calcium channel blocker administered orally; it relaxes smooth muscles, including those of the contracting uterus; maternal hypo- tension is a concern."

Which woman is at greatest risk for postpartum psychosis? A woman with a seizure disorder A woman who chose to surrender her newborn for adoption A woman whose aunt was diagnosed with postpartum psychosis A woman who was diagnosed with bipolar disorder type I as an adolescent

A woman whose aunt was diagnosed with postpartum psychosis Women with a family history of postpartum psychosis are at an increased risk for developing postpartum psychosis.

Which statement concerning the complication of maternal diabetes is the most accurate? a. Diabetic ketoacidosis (DKA) can lead to fetal death at any time during pregnancy. b. Hydramnios occurs approximately twice as often in diabetic pregnancies than in nondiabetic pregnancies. c. Infections occur about as often and are considered about as serious in both diabetic and nondiabetic pregnancies. d. Even mild-to-moderate hypoglycemic episodes can have significant effects on fetal well-being.

ANS: A Prompt treatment of DKA is necessary to save the fetus and the mother. Hydramnios occurs 10 times more often in diabetic pregnancies. Infections are more common and more serious in pregnant women with diabetes. Mild-to-moderate hypoglycemic episodes do not appear to have significant effects on fetal well-being.

What nursing diagnosis is the most appropriate for a woman experiencing severe preeclampsia? a. Risk for injury to mother and fetus, related to central nervous system (CNS) irritability b. Risk for altered gas exchange c. Risk for deficient fluid volume, related to increased sodium retention secondary to the administration of magnesium sulfate d. Risk for increased cardiac output, related to the use of antihypertensive drugs

ANS: A Risk for injury is the most appropriate nursing diagnosis for this client scenario. Gas exchange is more likely to become impaired, attributable to pulmonary edema. A risk for excess, not deficient, fluid volume, related to increased sodium retention, is increased, and a risk for decreased, not increased, cardiac output, related to the use of antihypertensive drugs, also is increased.

Gestational Diabetes Mellitus (GDM)- Definition

Carbohydrate intolerance with the onset or first recognition occurring during pregnancy

Which statement related to cephalopelvic disproportion (CPD) is the least accurate? a. CPD can be related to either fetal size or fetal position. b. The fetus cannot be born vaginally. c. CPD can be accurately predicted. d. Causes of CPD may have maternal or fetal origins.

ANS: C Unfortunately, accurately predicting CPD is not possible. Although CPD is often related to excessive fetal size (macrosomia), malposition of the fetal presenting part is the problem in many cases, not true CPD. When CPD is present, the fetus cannot fit through the maternal pelvis to be born vaginally. CPD may be related to either fetal origins such as macrosomia or malposition or maternal origins such as a too small or malformed pelvis.

What are the predisposing conditions for DIC?

Abruptio placentae, amniotic fluid embolism, gestational hypertension, intrauterine fetal death, liver disease, and sepsis

Assessment of fetal death in utero?

Absence of fetal movement, absence of fetal heart tones, maternal weight loss, lack of fetal growth or decrease in fundal height, no evidence of fetal cardiac activity

A 3-day-old premature neonate is NPO due to abdominal distension, vomiting, and bloody stools. Which intervention would the nurse anticipate for this neonate? Weighing the neonate in the morning and evening Swabbing the neonate's mouth with 1 ml colostrum Administering total parenteral nutrition at the ordered rate Increasing the IV fluid rate from 100 ml/kg/day to 150 ml/kg/day

Administering total parenteral nutrition at the ordered rate Total parenteral nutrition will provide the nutrients and calories needed for weight gain in the neonate who is NPO.

Upon assessment the nurse finds that the client who has undergone a cesarean delivery is at risk of postpartum hemorrhage (PPH). What would be the most likely cause of PPH in this client? Anesthesia Coagulopathy Placenta previa Chorioamnionitis

Anesthesia Rationale Anesthesia blocks the neurologic impulses that stimulate uterine contractions (UCs). This causes uterine atony and can lead to PPH. Chorioamnionitis is a serious condition in which the fetal membranes are infected, and is associated with other signs, such as fever. Coagulopathy is a clotting disorder, which occurs due to protein defects in the plasma that interrupt the coagulation cascade and cause blood coagulation. Placenta previa is an obstetric complication in which the placenta penetrates partially in the lower uterine segment. This is a common complication in pregnant women who smoke and consume cocaine.

Preterm birth

Any birth that occurs between 20 0/7 and 36 6/7 (between 20 and 37 weeks) Very Preterm <32 weeks gestation Moderately preterm 32-34 weeks gestation Later preterm 34-36 weeks of gestation Low birth weigh <2500 g -can be caused by preterm birth or IUGR

Postpartum/puerperal infection

Any clinical infection of the genital tract that occurs within 28 days after miscarriage, induced abortion, or birth -presence of a fever of >/= 38oC (100.4oF) on 2 successive days of the first 10 postpartum days (not counting the first 25 hours after birth) Most common infecting organisms: -streptococcus More common in women who had an operative birth or immunosuppresive conditions Risk factors: -prolonged ROM -prolonged labor -intrauterine monitoring

As part of preparing a 24-year-old woman at 42 weeks of gestation for a nonstress test, the nurse should: A. Tell the woman to fast for 8 hours before the test B. Explain that the test will evaluate how well her baby is moving inside her uterus C. Show her how to indicate when her baby moves D. Attach a spiral electrode to the presenting part to determine FHR patterns"

Choice C is correct; food and fluid are not restricted before the test; the test will evaluate the response of the FHR to fetal movement—acceleration is ex- pected; external, not internal, monitoring is used." C. Show her how to indicate when her baby moves

blood loss during pregnancy

Blood loss during pregnancy should always be regarded as a warning sign until the cause is determined. • Maternal blood loss decreases oxygen-carrying capacity, putting the woman at increased risk for hypovolemia, anemia, infection, and preterm labor, and adversely affects oxygen delivery to the fetus.

dinoprostone (Prepidil)

Cervical ripening agent in the form of a gel that is inserted in the cervical canal just below the internal os.

A neonate in the NICU requires continuous nasal cannula oxygen. Which action should the nurse take prior to discharging this neonate? Provide the parents with information about an in-house nurse. Contact a home oxygen company for home oxygen use set-up. Observe the parents changing diapers and swaddling the neonate. Provide the parents with the record of immunizations given in the NICU.

Contact a home oxygen company for home oxygen use set-up. A neonate requiring continuous nasal cannula oxygen will need oxygen therapy at home. To ensure continuity of care, the nurse must ensure that home oxygen services are set-up prior to the discharge of the neonate.

Which sign would the nurse observe in a client with hydatidiform mole? Clear vaginal discharge A small uterus Decreased fetal heart rate Dark brown vaginal discharge

Dark brown vaginal discharge Rationale Hydatidiform mole is characterized by the degeneration of the chorionic villi, in which the villi become vesicle-like. These vesicle-like substances are expelled from the vagina through a dark-brown or bright-red discharge. These are definite signs that the client has hydatidiform mole. Clear vaginal discharge may be a normal finding during pregnancy and may not indicate any complications. The fundal height is unusually more in molar pregnancy. The fetal heartbeat is absent as there is no viable fetus.

Diagnosis of fetal anomalies

Goal of antepartal assessment during the first and second trimesters

Which statement is most likely to be associated with a breech presentation? Least common malpresentation Descent is rapid Diagnosis by ultrasound only High rate of neuromuscular disorders

High rate of neuromuscular disorders Rationale Fetuses with neuromuscular disorders have a higher rate of breech presentation, perhaps because they are less capable of movement within the uterus. Breech is the most common malpresentation, affecting 3% to 4% of all labors. Descent is often slow because the breech is not as good a dilating wedge as is the fetal head. Diagnosis is made by abdominal palpation and vaginal examination. It is confirmed by ultrasound.

a leading cause of maternal and perinatal morbidity and mortality worldwide

Hypertensive disorders during pregnancy are a leading cause of maternal and perinatal morbidity and mortality worldwide, primarily in relation to uteroplacental insufficiency and premature birth

What factor is definitely associated with preterm labor?

Infection

Which PPH conditions are considered medical emergencies that require immediate treatment? Inversion of the uterus and hypovolemic shock Hypotonic uterus and coagulopathies Subinvolution of the uterus and idiopathic thrombocytopenic purpura Uterine atony

Inversion of the uterus and hypovolemic shock Rationale Inversion of the uterus and hypovolemic shock are considered medical emergencies. A hypotonic uterus can be managed with massage and oxytocin. Coagulopathies should be identified prior to delivery and treated accordingly. Although subinvolution of the uterus and ITP are serious conditions, they do not always require immediate treatment. ITP can be safely managed with corticosteroids or IV immunoglobulin. Uterine atony is a very serious obstetric complication; however, uterine inversion is a medical emergency requiring immediate intervention.

A nurse is caring for a woman diagnosed with gestational diabetes. Which recommendations would the nurse make? Select all that apply. Keep caloric intake to 30 kcal/kg/day Self-monitor blood glucose (SMBG) levels only in the morning Self-monitor blood glucose (SMBG) levels three times daily Self-monitor blood glucose (SMBG) levels after feelings of confusion Text self-monitored blood glucose (SMBG) results to the health care provider

Keep caloric intake to 30 kcal/kg/day The recommended caloric intake for a pregnant woman with diabetes is 30 kcal/kg/day. Self-monitor blood glucose (SMBG) levels after feelings of confusion Women should perform SMBG with any symptoms of hypoglycemia, such as feelings of confusion.

Pregestational diabetes"

Label given to type 1 or type 2 diabetes that existed before pregnancy."

can reduce the rate of cesarean birth and increase the VBAC rate

Labor management that emphasizes one-to-one support of the laboring woman by another woman (e.g., doula, nurse, nurse-midwife)

Battledore placenta

Marginal insertion of the cord into the placenta.

What is an appropriate indicator for performing a contraction stress test? Increased fetal movement and small for gestational age Maternal diabetes mellitus and postmaturity Adolescent pregnancy and poor prenatal care History of preterm labor and intrauterine growth restriction

Maternal diabetes mellitus and postmaturity Decreased fetal movement is an indicator for performing a contraction stress test; the size (small for gestational age) is not an indicator. Although adolescent pregnancy and poor prenatal care are risk factors for poor fetal outcomes, they are not indicators for performing a contraction stress test. Intrauterine growth restriction is an indicator; history of a previous stillbirth, not preterm labor, is another indicator.

Hyperemesis Gravidarum Interventions

Meds for nausea IV meds for wt loss and F&E imbalances Monitor VS, I&Os, wt, and calorie count Labs for dehydration/electrolyte imbalances Monitor urine for ketones Monitor FHR, activity and growth -Encourage intake of small portions of food (low fat, easily digestible carbs (cereals, rice, pasta). Liquids b/w meals to avoid distending stomach & triggering vomiting. -Sit upright after meals

What interventions are implemented if the initial treatment for excessive bleeding postpartum is ineffective?

Misopostol (Cytotec), advantage is that can be given orally, rectally, vaginally; Methergine and Hemabate also given IM.

Assessment of ectopic pregnancy?

Missed menstrual period, abdominal pain, vaginal spotting to bleeding that is dark red or brown, rupture: increased pain, referred shoulder pain, signs of shock

Which are the most important physical assessments for the nurse to perform for a patient with a hemorrhage during late pregnancy? Select all that apply. Monitor urinary output Monitor vaginal bleeding Assess vital signs for the patient Monitor fetal heart rate and contraction patterns Assess the patient's knowledge of medical care for a hemorrhage during late pregnancy

Monitor urinary output Monitoring urinary output is a priority assessment for the nurse. Monitor vaginal bleeding Monitoring vaginal bleeding is a priority assessment for the nurse. Assess vital signs for the patient Assessing vital signs for the patient is a priority assessment for the nurse. Monitor fetal heart rate and contraction patterns Monitoring fetal heart rate and contraction patterns are priority assessments for the nurse.

antepartum evaluation of high risk women

NST mBPP

The common bleeding disorders of late pregnancy.

Placenta previa; premature separation of the placenta (abruptio placentae or placental abruption)

Which condition in a pregnant client with severe preeclampsia is an indication for administering magnesium sulfate? Seizure activity Renal dysfunction Pulmonary edema Low blood pressure (BP)

Seizure activity Rationale Severe preeclampsia may cause seizure activity or eclampsia in the client, which is treated with magnesium sulfate. Magnesium sulfate is not administered for renal dysfunction and can cause magnesium toxicity in the patient. Pulmonary enema can be prevented by restricting the client's fluid intake to 125 ml/hr. Increasing magnesium toxicity can cause low BP in the client.

Women with hyperthyroidism are at increased risk for what?

Severe preeclampsia and heart failure or Thyroid storm which can occur in response to stress such as labor and vaginal birth, infection, preeclampsia, or surgery -symptoms= fever, restlessness, tachycarida, vomiting, hypotension, or stupor

A 37-year-old woman whose placenta was manually removed during delivery is at greatest risk for which complication? Subinvolution Cervical lacerations Uterine overdistention Retroperitoneal hematoma

Subinvolution Subinvolution, delayed return of the uterus to its nonpregnant size, is usually caused by retained placental fragments. Advanced maternal age and manual removal of the placenta put the woman at risk for retained placental fragments and subinvolution.

What medications commonly given during pregnancy may increase the risk for hyperglycemia and diabetic ketoacidosis?

Terbutaline (brethine) for tocolysis and corticosteroids given to enhance fetal lung maturity

Beta-Adrenergic Agonist

Terubtaline (brethine) -subcutaneous injection -relaxes smooth muscle, inhibiting uterine activity and causing bronchodilation -should not be used for more than 24 hours Maternal side effects: -tachycardia, palpitations -tremors, dizziness -headache -nasal congestion -nausea and vomiting -hypokalemia -hyperglycemia -hypotension Fetal Adverse effects: -tachycardia -hyperinsulinemia -hyperglycemia

Nonstress test

Test based on the fact that the heart rate of a healthy akwake fetus with an intact central nervous system usually accelerates in response to movement.

Cell free DNA screening

Test that provides a definitive diagnosis noninvasively for fetal Rh status, fetal gender, and certain paternally transmitted single gene disorders

Maternal serum alpha-fetoprotein

Test used as a screening tool for neural tube defects in pregnancy. The test is ideally performed between 16 and 18 weeks of gestation.

Trial of Labor (TOL)

The observance of a woman and her fetus for a reasonable period (4-6 hours) of spontaneous active labor to assess the safety of vaginal birth for the mother and infant -most common reason- if woman wants to have a vaginal birth after a previous c-section

Which behavioral factors during pregnancy can contribute to a higher risk of thromboembolic disorder? Moving to a new house Working from home at a desk Teaching classes in a lecture hall Living with a partner who smokes Switching from jogging to walking daily

Working from home at a desk Working from home at a desk contributes to a higher risk of thromboembolic disorder due to sitting for prolonged periods of time. Correct Teaching classes in a lecture hall Teaching classes in a lecture hall contributes to a higher risk of thromboembolic disorders due to standing for prolonged periods of time. Correct Living with a partner who smokes Living with a partner who smokes contributes to a higher risk of thromboembolic disorders due to secondhand smoke exposure.

What is recommended for the prevention of maternal-to-fetal HIV infections?

Zidovudine (Retrovir). Administered orally beginning after 14 weeks' gestation, intravenously during labor, and in the form of syrup to the newborn for 6 weeks after birth High risk for infection for mother because she is vulnerable to infections

complications and risks of c-section

anesthesia events - drug reaction, aspiration pneumonia - hemorrhage - bowel or bladder injury - amniotic fluid embolism - air embolism possible postpartum complications: - atelectasis - endometriosis - UTI - abdominal wound hematoma - dehisence infection - necrotizing fascitis - thromboembolic disease - bowel dysfunction

predictors of preterm birth

additionally to risk factors - poverty -lack of education - living conditions - lack of access to prenatal care - genetically related - endocervical length

nonstress test (NST)

antepartum evaluation of the fetus; heart rate patterns in response to fetal movement, terrine contractions, or stimulation easy, quickly, noninvasive, easy to perform and interpret, inexpensive, no contraindications used FHR by doppler transducer and tocodynamometer somitines use vibroacoustic stimulation to stimulate activity method of testing antepartum FHR response; performed in conjunction with the NST uses a combination of sound sand vibration to stimulate the fetus (FAST) or can take orange juice prior tesr

A home care nurse visits a pregnant client who has a diagnosis of mild preeclampsia. Which assessment finding indicates a worsening of the preeclampsia and the need to notify the physician? a. urinary output has increased b. dependent edema has resolved c. bp reading is at the prenatal baseline d. the client complaint of a headache and blusrred vision

d. the client complaint of a headache and blusrred vision

complications of amniocentesis in fetus

death hemorrhage infection direct injury from the needle

subinvolution

delayed return of the enlarge uterus to normal size and function following birth

hemorrhagic (hypovolemic) shock

emergency situation where profuse blood loss can result in severely compromised perfusion of body organs DEATH MAY OCCUR

mild gestational hypertension and preeclampsia without severe features / goal

ensure safety deliver a healthy newborn as close to term as possible home management is BP < 150/100 and no increase in proteinurea, normal platelet count, normal liver enzymes

tests for HIV

enzyme-linked immunosorbent assay (ELIZA) Western blot immunofluorescence assay (IFA)

mastitis

infection of the breast soon after childbirth most often affecting primiparous breastfeeding women

Amniotic fluid embolus

is a rare but devastating complication of pregnancy characterized by the sudden, acute onset of hypoxia, hypotension, cardiovascular collapse, and coagulopathy.

Complete inversion of uterus

lining of fundus crosses through cervical os and forms mass in vagina.

Complete abortion

loss of all products of conception

ultrasound during first trimester

number, size and location of gestational sacs, presence or absence of fetal cardiac and body movements presence of absence of uterine abnormalities pregnancy dating

Prolapsed inversion of uterus

obvious, large red, rounded mass and protrudes 20-30 cm outside introitus.

The major causes of bleeding in late pregnancy

placenta previa and placental abruption, are differentiated by type of bleeding, uterine tonicity, and presence or absence of pain. Management of late-pregnancy bleeding requires immediate evaluation; care is based on gestational age, amount of bleeding, and fetal condition.

Pelvic Dystocia

occurs when contractures of the pelvic diameter reduce the capacity of the bony pelvis -can be caused by congenital abnormalities, maternal malnutrition, neoplams, or lower spinal disorders

contraindication for CST

on women who should not give birth vaginally preterm labor placenta previa vasa previa reduced cervical competenc multiple gestations previous classic incision for C-birth

neonatal risk of choreoamnionitis

pneumonia bacerimia sepsis prevent with broad spectrum antibiotics

uterine atony

poorly contracted uterus that does not adequately compress large open vessels at the placental site, results in hemorrhage assessment - soft/boggy uterus interventions - massage uterus until firm; empty the woman's bladder, notify HCP if interventions didn't help

spontaneous abortion

pregnancy ends because of natural causes

percutaneous umbilical blood sampling (PUBS)

procedure during which a fetal umbilical vessel is accessed for blood sampling or for transfusions; insertion of a needle directly into a fetal umbilical vessel, under ultrasound guidance preferrsable insert into umbelical cord near insertion performed for evaluatiion of mosaic results in amniocentesis or CVS complications - loss of pregnancy, hematomas, bleeding from the puncture site in umbelical cord, transient fetal bradicardia, fetomaternal hemorrhage fetus with isoimmune hemolitic anemia - PUBS identifies fetal blood type and RBC

Preeclampsia s/s

protein lost in urine uric acid clearance decreased serum uric acid increase sodium and watwr retained intravascular volume reduced tissue edema Hct increases impaired liver function -> elevated liver enzymes may epigastric or right upper quadrant pain cerebral edema increased irritability of CNS - hyperreflexia, headache, seizures, positive anckle clonus visual disturbances ( dim, blurred, double vision) hypertension proteinouria FGR, decreased amniotic fluid low birth weight preterm birth

eclampsia symptoms

seizure begins with twitching around mouth body become regid in a state of tonic muscular contraction 15-20 sec facial muscles and then body muscles alternately contract and relax in a rabit succession respiration ceases during seizure because diaphragm tends to remain fixed ( breathing resumes shortly after seizure) postictal sleep occurs

preeclampsia without severe features labs

serum createnine platelet count liver enzymes 24-hrs proteine assessment fetal movemet count and nonstress test Q1-2 week liver enzymes and platelet count, proteinuria - weekly BP - Q2week simptoms - severe headache, blurred doubled vision, mental confusion, right upper quadrant or epigastric pain, nausea, vomiting, shortmness of breath, decreased urinary output

Habitual abortion

spontaneous abortions occur in 3 or more successive pregnancies

Cytomegalovirus

transmitted Through close personal contact 2. Across the placenta to the baby, or the baby could be infected through the birth canal 3. mother may be asymptomatic and most infants are as well at birth 4. causes - LBW, intrauterine growth restriction, enlarged liver and spleen, jaundice, blindness, hearing loss, and seizures 5 antivirals for mother, but they are TOXIC and may only temporarily suppress shedding of the virus

inversion of the uterus

turning of the uterus inside out AFTER birth it can be incomplete, complete or prolapsed

von willebrand disease

type of hemophilia probably the most common of all hereditary bleeding disorders

Thyroid dysfunction

—hyperthyroidism or hypothyroidism—during pregnancy requires close monitoring of thyroid hormone levels to regulate therapy and prevent fetal insult.

Maternal indications for labor induction

-Hypertensive complications of pregnancy -Fetal death -Chorioamnionitis

Late pregnancy bleeding

-Placenta previa -abruptio placentae

Lifestyle Modifications Required by Preterm Labor

1. Activity restriction, including bed rest and limited work 2. Restriction of sexual activity

what do we generally want fetal movements to be?

10 times in one hour mother count fetal movements at the same time every day

When does the fetus begin to produce its own thyroid hormones

18 weeks of gestation -until then, hypothyroidism in the mom can lead to long-term neuropsychologic damage in children

Which of the following comments suggest that a client whose baby was born with a congenital defect is in the bargaining phase of grief? 1. "I hate myself. I caused my baby to be sick." 2. "I'll take him to a specialist. Then he will get better." 3. "I can't seem to stop crying." 4. "This can't be happening."

2. "I'll take him to a specialist. Then he will get better."

A nurse has administered Methergine (methylergonovine) 0.2 mg po to a grand multipara who delivered vaginally 30 minutes earlier. Which of the following outcomes indicates that the medication is effective? 1. Blood pressure 120/80. 2. Pulse rate 80 bpm and regular. 3. Fundus firm at umbilicus. 4. Increase in prothrombin time.

3. Fundus firm at umbilicus

A nurse on the postpartum unit is caring for two postoperative cesarean clients. One client had spinal anesthesia for the delivery and the other client had an epidural. Which of the following complications will the nurse monitor the spinal client for that the epidural client is much less high risk for? 1. Pruritus. 2. Nausea. 3. Postural headache. 4. Respiratory depression.

3. Postural headache.

A woman has been diagnosed with a ruptured ectopic pregnancy. Which of the following signs/symptoms is characteristic of this diagnosis? 1. Dark brown rectal bleeding. 2. Severe nausea and vomiting. 3. Sharp unilateral pain. 4. Marked hyperthermia.

3. Sharp unilateral pain is a common symptom of a ruptured ectopic. The most common location for an ectopic pregnancy to implant is in a fallopian tube. Because the tubes are nonelastic, when the pregnancy becomes too big, the tube ruptures. Unilateral pain can develop because only one tube is being affected by the condition. In addition, some women complain of generalized abdominal pain.

Protracted active phase—dilation

Labor pattern defined as an active phase during which cervical dilation occurs at a rate of less than 1.2 cm/hour in nulliparas and less than 1.5 cm/hour in multiparas.

High Risk Pregnancy

A pregnancy in which the life or health of the mother or her fetus is jeopardized by a disorder coincidental with or unique to pregnancy.

Arrest of descent

Labor pattern defined as no progress in fetal descent for 1 hour or more in a nullipara and 30 minutes or more in a multipara.

When does the fetus begin to secrete its own insulin?

Around the 10th week of gestation

Bright blood

Arterial -can indicate deep lacerations of the cervix

fFN

Fetal fibronectin -a diagnostic test for preterm labor- vaginal swab done on mother -if fFN is present in the late second and early third trimester may be related to placental inflammation which may lead to preterm birth -preterm birth unlikely to occur with a negative fFN result

Secondary Powers

Bearing down efforts -compromised when large amounts of analgesic are giv

factors place the pregnancy, fetus, or neonate at risk.

Biophysical, sociodemographic, psychosocial, and environmental factors Biophysical assessment techniques include daily fetal movement count, ultrasonography, and MRI. Biochemical monitoring techniques include amniocentesis, percutaneous umbilical blood sampling, chorionic villus sampling, maternal serum alpha-fetoprotein measurement, multiple marker screens, Coombs test, and cell-free DNA screening in maternal blood. Most assessment tests have some degree of risk for the mother and fetus and usually cause some anxiety for the woman and her family

After assessing a client, the nurse concludes that the client has threatened miscarriage. What signs in the client would be consistent with the diagnosis? Select all that apply. Blood spotting Heavy bleeding Closed cervical os Mild uterine cramping Decreased urine output

Blood spotting Closed cervical os Mild uterine cramping Rationale A pregnancy that ends from natural causes before 20 weeks of gestation results in miscarriage, or spontaneous abortion. Blood spotting, a closed cervical os, and mild uterine cramping are signs of a threatened miscarriage. Heavy bleeding is observed in inevitable and incomplete miscarriages. A threatened miscarriage does not affect urine output. Decreased urine output is a sign of placenta previa.

The nurse observes that intravenous (IV) administration of magnesium sulfate has resulted in magnesium toxicity in a pregnant client with preeclampsia. The nurse immediately discontinues the infusion and reports to the primary health care provider (PHP). For which drug does the nurse obtain a prescription from the PHP? Calcium gluconate Nifedipine (Adalat) Hydralazine (Apresoline) Labetalol hydrochloride (Normodyne)

Calcium gluconate Rationale The nurse needs to obtain a prescription for calcium gluconate, because it acts as an antidote to magnesium toxicity. Nifedipine (Adalat) and labetalol hydrochloride (Normodyne) are antihypertensive medications, which are prescribed for gestational hypertension or severe preeclampsia. Hydralazine (Apresoline) is also an antihypertensive medication used for treating hypertension intrapartum.

What can the client develop from fetal death in utero?

Can develop DIC if the dead fetus is retained in uterus for 3-4 weeks or longer

Lacerations of the Genital Tract

Causes: -Size of fetus operative birth precipitous or rapid birth congenital anomalies pf maternal soft tissue -Abnormal presentation and position of the fetus -Relative size of the presenting part and the birth canal -Previous scarring from infection, injury, or operation -Vulvar, perineal, and vaginal varicosities usually identified and sutured immediately after birth; increase fiber and fluids to reduce risk of constipation and stool softeners to reestablish bowel habits without straining and putting stress on suture lines.

Common challenge associated with prematurity in newborn

Cerebral palsy -administration of magnesium sulfate may help prevent it because it facilitates vasodilation, reduces inflammation, and reduces calcium uptake into cells -mag sulfate is recommended for protection against cerebral palsy in women with anticipated preterm birth at <32 weeks of gestation

dinoprostone (Cervidil)

Cervical ripening agent in the form of a vaginal insert that is placed in the posterior fornix of the vagina.

misoprostol (Cytotec)

Cervical ripening agent, used in the form of a tablet that is most commonly inserted intravaginally in the posterior fornix."

success of labor induction.

Cervical ripening using chemical or mechanical measures can improve the success of labor induction.

Induction of labor

Chemical or mechanical initiation of uterine contractions before their spontane- ous onset for the purpose of bringing about the birth.

Recurrent (habitual) early miscarriage

Defined as 3+ spontaneous pregnancy losses before 20 weeks of gestation Most common causes: -parental chromosomal abnormalities -antiphospholipid antibody syndrome -certain uterine abnormalities Management: -prohpylactic cerclage if cervical insufficinecy suspected -karyotyping of both partners and miscarriage assessment -testing the woman for antiphospholipid antibody syndrome and thyroid disease -assessment of placenta and woman's uterine cavity

During a prenatal visit at 30 weeks of gestation, Angela's blood pressure (BP) was 156/98 mm Hg; it has ranged be- tween 142/92 and 150/90 mm Hg since the 28th week of her pregnancy; her urinalysis indicated a protein level of 2+ using a dipstick; her biceps and patellar reflexes are 2+.

E. Preeclampsia (mild) without severe features

The nurse is caring for a client in early pregnancy with hyperemesis. What changes in diet will benefit this client? Avoiding dairy products and cold foods Eating a high-protein snack at bedtime Eating foods that are served warm Avoiding dry, bland, and low-fat foods

Eating a high-protein snack at bedtime Rationale Eating a high-protein snack at bedtime may help the client overcome protein deficiency and reduce hyperemesis. The client can have dairy products, which are easily accepted by the body. Dairy products also contain proteins. Clients with hyperemesis are able to tolerate cold foods better than foods that are served warm. The client can have dry, bland, and low-fat foods, because they do not induce hyperemesis.

How often should woman's temperature be checked after rupture of membranes?

Every 2 hours -if her temperature is 100.4oF or higher, nurse should notify HCP

Hyperglycemia "

Excess glucose in the blood.

Gestational diabetes may cause congenital abnormalities in the fetus. Match the congenital malformation with its definition.

Fetal size greater than 4000 grams Macrosomia Malformations in heart structures Cardiac defects Abnormal closure of spinal cord Neural tube defects Malformation of sacrum, lumbar spine, and lower extremities Caudal regression syndrome

The nurse is preparing a diet plan for a pregnant client with preeclampsia. What does the nurse include in the client's diet? Select all that apply. Food with low fiber content Four to five cups of coffee per day Food with low sodium content Food with high zinc content Six to eight glasses of water per day

Food with low sodium content Food with high zinc content Six to eight glasses of water per day Rationale Patients with preeclampsia may have edema, which may worsen with excessive salt intake. Therefore, the diet plan of a pregnant client with preeclampsia should include not more than 1.5 gm of sodium per day. The diet plan should also include food with high zinc content to prevent anemia caused by preeclampsia. The patient should drink six to eight 8-ounce glasses (approximately a liter) of water per day to maintain adequate fluid in her body. A pregnant patient should consume fiber-rich food to prevent constipation. The pregnant client with preeclampsia should limit her caffeine intake and should not consume excessive coffee.

maternal blood glucose

For pregnant women who have diabetes and are insulin dependent, maternal blood glucose levels are normally reduced during the first trimester. Insulin requirements are increased as the pregnancy progresses and may double or quadruple by the end of pregnancy as a result of insulin resistance created by placental hormones, insulinase, and cortisol. After birth, levels decrease dramatically; breastfeeding affects insulin needs.

Which is the most common cause of spontaneous abortion? Infection Genetic factors Vaginal bleeding Anomalies of the maternal reproductive tract

Genetic factors Severe congenital abnormalities that are incompatible with life are the most common causes of spontaneous abortions.

Postterm Pregnancy

Has completed 42 weeks of gestation -more common in primiparous women but a woman with a hx of postterm pregnancy is more likely to experience it again Clinical Manifestations: -maternal weight loss (approximately 3 lbs./wk) -decreased uterine size (related to decreased amniotic fluid) -meconium in amniotic fluid -advanced bone maturation in fetal skeleton

At a 1-month follow-up visit, a new mother indicates that she is not sleeping or eating well and she believes that she is unable to care for her newborn adequately. She indicates that her partner is no help to her and she doesn't have anyone else to turn to. Which is a priority nursing intervention for this woman? Referring the woman to a couples' counselor Helping the woman verbalize feelings of frustration Reminding the woman of her attachment to her newborn Reassuring the woman that parenting is most difficult in the first few months

Helping the woman verbalize feelings of frustration Helping the woman verbalize her feelings is a priority nursing action that helps the woman acknowledge that her feelings are real. Repressing negative feelings is not encourages, as repression can reinforce or fuel feelings of sadness and depression.

What should be done if bleeding is excessive or continuous or excessive and there are concerns about the condition of the fetus in a woman with placenta previa?

Induction regardless of gestational age -preferably wait until 36 weeks if she is stable, but if she is not, induction

The most prevalent clinical manifestation of abruptio placentae (as opposed to placenta previa) is what? Bleeding Intense abdominal pain Uterine activity Cramping

Intense abdominal pain Rationale Pain is absent with placenta previa but may be agonizing with abruptio placentae. Bleeding may be present in varying degrees for both placental conditions. Uterine activity may be present with both placental conditions. Cramping is a form of uterine activity that may be present in both placental conditions.

The priority assessment in evaluating a pregnant woman with severe nausea and vomiting is: Fasting blood glucose level. Ketonuria. Bilirubin. White blood cell count

Ketonuria. Determination of ketonuria would be a critical assessment that would lead towards determination of hyperemesis. A pregnant patient with severe nausea and vomiting may have hyperemesis gravidarum and as such requires critical monitoring to determine the nature of the problem. An FBS measurement, although informative, would not be the priority assessment at this time, nor would a bilirubin measurement. A WBC count would indicate the possibility of an infectious source but it would not be a priority assessment in terms of the patient's presentation.

Which of the following findings is not likely to be seen in a pregnant patient who has hypothyroidism? Miscarriage Macrosomia Gestational hypertension Placental abruption

Macrosomia Infants born to mothers with hypothyroidism are more likely to be of low birth weight or preterm; these outcomes can be improved with early diagnosis and treatment. Hypothyroidism is often associated with both infertility and an increased risk of miscarriage. Pregnant women with hypothyroidism are more likely to experience both preeclampsia and gestational hypertension. Placental abruption and stillbirth are risks associated with hypothyroidism.

What is the main concern for a pregnant client with cardiac disease?

May be unable to physiologically cope with the added plasma volume and increased cardiac output that occur during pregnancy blood volume peaks at 32-34 weeks and declines slightly at 40

Nsg interventions for endometritis?

Monitor vitals, position client in fowler's position to facilitate drainage of lochia, provide a private room for the mother, instruct the mother in proper handwashing techniques, initiate contact precautions as necessary, monitor I&Os and encourage fluid intake, administer antibiotics as prescribed, administer comfort measures such as a backrub and position changes, pain medications as prescribed, administer oxytocic meds as prescribed to improve uterine tone psychological support

Modified BPP (mBPP)

NST quantity of amniotic fluid

laminaria tent

Natural cervical dilator made from desiccated seaweed.

Euglycemia

Normal blood glucose level with preprandial range of 65-95 and 1-hour postprandial range no higher than 130-140.

Late/Secondary PPH

Occurs more than 24 hours but less than 6 weeks after birth -result of subinvolution of the uterus, endometritis -retained placental fragments

Eclampsia

Onset of seizure activity or coma in the woman diagnosed with preeclampsia, with no history of preexisting pathology that can result in seizure activity.

Disseminated intravascular coagulation (DIC) or consumptive coagulopathy

Pathologic form of clotting that is diffuse and consumes large amounts of clotting factors, causing widespread external bleeding, internal bleeding, or both, and clotting.

What is fetal death in utero?

Refers to the death of a fetus after the twentieth week of gestation and before birth

What is a complication of DIC?

Renal failure

Phosphatidylglycero

Substance in amniotic fluid that is a better predictor of fetal lung maturity than is the lecithin/sphingomyelin (L/S) ratio in pregnancies complicated by diabetes; it should be at a level greater than 3% to confirm adequate lung maturity."

The nurse is caring for a patient who has just been prescribed intravenous (IV) oxytocin for the induction of labor. The nurse's subsequent assessments should address the risk for which complication of induction? Tachysystole Shoulder dystocia Fluid volume deficit Maternal hypertensive crisis

Tachysystole Oxytocin directly stimulates contractions, creating a risk for uterine hyperstimulation and tachysystole. For this reason, the nurse must prioritize assessments of the frequency, intensity, and duration of contractions.

The nurse is assessing a pregnant client who has a history of migraine headaches. Which tocolytic agent is contraindicated in the client? Nifedipine (Adalat) Evening primrose oil Terbutaline (Brethine) Magnesium sulfate (Epsom salts)

Terbutaline (Brethine) Rationale Beta 2-adrenergic agonists like terbutaline (Brethine) are contraindicated in clients with migraine headaches because these drugs may increase the episodes of migraine. Nifedipine (Adalat) is contraindicated in hypertensive clients, but not in clients with migraine. Evening primrose oil and magnesium sulfate (Epsom salts) are tocolytic agents used to reduce oxytocin (Pitocin) usage, and are not contraindicated in clients with migraine headaches.

A woman in labor at 34 weeks of gestation is hospitalized and treated with intravenous magnesium sulfate for 18 to 20 hours. When the magnesium sulfate is discontinued, which oral drug will be prescribed for continuation of the tocolytic effect? Buccal oxytocin (Pitocin) Terbutaline sulfate (Brethine) Calcium gluconate (Calgonate) Magnesium sulfate (Magnesium sulfate)

Terbutaline sulfate (Brethine) Rationale The woman receiving decreasing doses of magnesium sulfate often is switched to oral terbutaline to maintain tocolysis. Buccal oxytocin increases the strength of contractions and is used to augment or stimulate labor. Buccal oxytocin dosing is uncontrollable. Calcium gluconate reverses magnesium sulfate toxicity. The drug should be available for complications of magnesium sulfate therapy. Magnesium sulfate usually is given intravenously or intramuscularly. The client must be hospitalized for magnesium therapy because of the serious side effects of this drug.

Complete hydatidiform mole

Term used to refer to the result of fertilization of an egg with a lost or inactivated nucleus; it resembles a bunch of white grapes; there is an increased risk for persistent GTD.

Partial hydatidiform mole

Term used to refer to the result of two or more sperm fertilizing an apparently normal ovum; embryonic or fetal parts and an amniotic sac are often present in the uterus.

Uterine Atony

The greatest risk for early postpartum hemorrhage (esp. within first hour after birth) -occurs when uterus is flaccid after detachment of all or part of the placenta -hypotonia of the uterus -uterus feels boggy (soft leading cause of earlu PPH associated with high parity polyhydromnios fetal macrosomia multiple gestations traumatic birth magnesium sulfate rapid or prolonged labot chromniamniosis oxytocin

Diabetes in pregnancy puts the fetus at risk in several ways. Of what should the nurse be aware regarding this? With good control of maternal glucose levels, sudden and unexplained stillbirth is no longer a major concern The most important cause of perinatal loss in diabetic pregnancy is congenital malformations Infants of mothers with diabetes have the same risks for respiratory distress syndrome because of the careful monitoring At birth, the neonate of a diabetic mother is no longer in any greater risk

The most important cause of perinatal loss in diabetic pregnancy is congenital malformations Rationale Congenital malformations account for 30% to 50% of perinatal deaths. Even with good control, sudden and unexplained stillbirth remains a major concern. Infants of diabetic mothers are at increased risk for respiratory distress syndrome. The transition to extrauterine life often is marked by hypoglycemia and other metabolic abnormalities.

How do estrogen and progesterone affect insulin production in the first trimester?

They increase insulin production which promotes increased peripheral use of glucose and decreased blood glucose -fasting glucose levels decrease by 10%

What is hepatitis B?

Transmitted through blood, saliva, vaginal secretions, semen, and breast milk and across the placental barrier. The risks of prematurity, low birth weight, and neonatal death increase if the mother has HepB infection

Which assessment findings would be expected for a newborn whose mother was diagnosed with gestational diabetes? Select all that apply. Hyperactive reflexes Weight of 4850 g Serum glucose of 37 mg/dL Calcium concentration of 8.5 mg/dL Axillary temperature of 97.2° F

Weight of 4850 g Fetal size greater than 4000 g is an indication of macrosomia, an effect in fetuses related to gestational diabetes. Correct Serum glucose of 37 mg/dL Serum glucose levels are likely to be decreased (hypoglycemia) in a fetus of a mother with gestational diabetes. Normal newborn glucose levels are between 45-60 mg/dL. Axillary temperature of 97.2° F An axillary temperature of 97.2° F is low and a symptom of hypoglycemia in newborns. Because this infant has low blood glucose, energy is consumed to maintain normal metabolic function, causing temperatures to decrease.

diabetic exercise

When would regular exercise be contraindicated for pregnant diabetics? -When they have uncontrolled hypertension, advanced retinopathy, or severe autonomic or peripheral neuropathy -if she has positive urine ketones or a blood glucose level >250 mg/dl because hyperglycemia and ketosis can worsen with physical activity When is the best time for diabetics to exercise?After meals when the blood glucose level is rising What should a woman be told to do if she experiences uterine contractions during exercise?-stop exercising immediately -drink 2-3 glasses of water -lie on her side for an hour -contact HCP if the contractions continue

With regard to dysfunctional labor, nurses should be aware of what? Women who are underweight are more at risk. Women experiencing precipitous labor have a labor that lasts less than 3 hours. Hypertonic uterine dysfunction is more common than hypotonic dysfunction. Abnormal labor patterns are most common in younger women.

Women experiencing precipitous labor have a labor that lasts less than 3 hours. Rationale Precipitous labor lasts less than 3 hours. Short women who are more than 30 pounds overweight are more at risk for dysfunctional labor. Hypotonic uterine dysfunction, in which the contractions become weaker, is more common. Abnormal labor patterns are more common in older women.

The nurse is caring for a newborn after a vacuum-assisted birth. What changes should the nurse monitor in the newborn? Inability to pass urine Yellow discoloration of skin Listlessness Poor sucking patterns Difficulty breathing

Yellow discoloration of skin Listlessness Poor sucking patterns Rationale After a vacuum-assisted birth, the newborn might be at the risk of hyperbilirubinemia as the bruising resolves, which may cause neonatal jaundice. So, the nurse should monitor the newborn for yellow discoloration of the skin. A vacuum-assisted birth may cause cerebral irritation in the newborn, which manifests as listlessness and poor sucking. Inability to pass urine may indicate structural anomalies and may not be due to vacuum delivery. Difficulty in breathing can be caused by many factors and not necessarily due to vacuum delivery.

spurts of bloods

partial placental separation

External Cephalic Version (ECV)

used as an attempt to turn the fetus to a vertex presentation from a shoulder of breech presentation -attempted late in gestation (usually in labor and delivery) -if unsuccessful, woman typically will have a c-section

The nurse is caring for a patient in the post-anesthesia care unit (PACU) immediately following cesarean delivery. During fundal assessment, the patient reports discomfort and exhaustion and asks the nurse, "Why do you keep poking at me?" Which response from the nurse is appropriate? "I'll wait until you're on the postpartum floor to assess you again." "I need to do this assessment to make sure your incision is healing correctly." "This assessment must be done every 30 minutes until you transfer out of the recovery room." "A cesarean delivery puts you at risk for excessive bleeding, so it's important to assess you frequently."

"A cesarean delivery puts you at risk for excessive bleeding, so it's important to assess you frequently." Fundal assessment must be performed frequently following cesarean delivery to assess for uterine atony, which can lead to hemorrhage. Postpartum hemorrhage and uterine atony are risks of cesarean delivery.

A 38-year-old nulliparous woman bearing twins asks the nurse why she is at risk for PROM. Which statement by the nurse is appropriate? "Gaining weight progressively predisposes a woman to PROM." "Since this is your first pregnancy, you have an increased risk for PROM." "Being pregnant at an advanced maternal age could precipitate PROM." "Bearing multiple fetuses in one pregnancy can lead to overdistention of the uterus, which could result in PROM."

"Bearing multiple fetuses in one pregnancy can lead to overdistention of the uterus, which could result in PROM." Overdistention of the uterus can be caused by multiple fetuses and may be a factor in an increased risk for PROM.

A nurse is caring for a patient who has experienced a fetal demise. Which statement by the nurse appropriately acknowledges the loss of the patient? "I am so sorry for your loss." "Everything happens for a reason." "It will be okay; you can become pregnant again." "Focus on your other children; your sadness will soon pass."

"I am so sorry for your loss." A simple statement of "I'm sorry" acknowledges the loss.

Which statement does the nurse use to describe PROM or PPROM to the pregnant woman? "PROM could occur at 38 weeks." "PROM occurs in full-term infants." "PPROM does not pose a risk to the infant." "PROM is a greater risk to the mother than is PPROM."

"PROM could occur at 38 weeks." Describing PROM as occurring before the onset of true labor is an appropriate response.

A patient carrying twins desires an elective cesarean delivery. Which statement indicates a need for further teaching regarding cesarean delivery? "I might need a blood transfusion after the surgery." "Having a c-section might make my hospital stay longer." "Since I am not having a vaginal birth, my babies can't be harmed during delivery." "Depending on how they do the incision, I may be able to have a vaginal delivery if I become pregnant again."

"Since I am not having a vaginal birth, my babies can't be harmed during delivery." Cesarean delivery carries risk of injury to the newborn, such as bruising, fractures, or other trauma.

Fetal complications of abruptio placentae

-IUGR -preterm birth -death of fetus -neurological defects -cerebral palsy -death from SIDS

Indomethacin (Indocin)

-NSAID shown to suppress preterm labor by blocking production of prostaglandins

Meconium-Aspiration Syndrome (MAS)

-When baby breathes in meconium -causes a asevere form in aspiration pneumonia

Clinical Manifestations of Hydatidiform Mole

-early signs- cannot be distinguished from normal pregnancy -vaginal bleeding (dark brown resembling prune juice or bright red) -uterus larger than expected -anemia from blood loss -excessive nausea and vomiting (hyperemesis gravidarum) -abdominal cramps caused by uterine distention -hyperthyoridism

Rare complications of hyperemesis gravidarum

-esophageal rupture -pneumomediastinum -deficiencies of vitamin K and thiamine resulting in Wernicke encephalopathy

Risk for fetal deformity in abdominal ectopic pregnancy

-result of pressure deformities caused by oligohydramnios -facial or cranial asymmetry -joint deformities -limb deficiency -CNS anomalies

Incomplete rupture of the uterus

-rupture into the peritoneum covering the uterus, but not into the peritoneal cavity

Diagnosis of Placenta Previa

-transvaginal ultrasound

Abnormal Labor Patterns

1) Prolonged latent phase 2) Protracted active phase dilation 3) Secondary arrest, no change 4) Protracted descent 5) Arrest of descent 6) Failure of descent

Assessment of Postpartum Infection

1. Fever 2. Chills 3. Anorexia 4. Pelvic discomfort or pain 5. Vaginal discharge that is malodorous; normal vaginal discharge has a fleshy odor or an odor similar to a menstrual period. 6. Elevated white blood cell coun

Assessment of Mastitis

1. Localized heat and swelling 2. Pain; tender axillary lymph nodes 3. Elevated temperature 4. Complaints of flulike symptoms

A client with a fetal demise is admitted to labor and delivery in the latent phase of labor. Which of the following behaviors would the nurse expect this client to exhibit? 1. Crying and sad. 2. Talkative and excited. 3. Quietly doing rapid breathing. 4. Loudly chanting songs.

1. The nurse would expect the client to be crying and sad. TEST-TAKING TIP: A client in the latent phase of labor who is carrying a healthy fetus is likely to be talkative and excited, but a woman whose fetus has died is likely to be crying and sad throughout her labor. Clients in the latent phase usually are performing slow chest breathing.

Assessment of Subinvolution

1. Uterine pain on palpation 2. Uterus larger than expected 3. More than normal vaginal bleeding

Types of Vasa Previa

1. Velamentous Insertion of cord -when the cord vessels begin to branch at the membranes and then course onto the placenta 2. Succenturiate Placenta -when the placenta has divided into 2+ lobes rather than remaining as a single mass 3. Battledore (marginal) insertion of the cord -increases risk of fetal hemorrhage

Iron deficiency anemia predisposes the patient to postpartum infection. The nurse should watch out of which signs and symptoms?

1. fatigue 2. headache 3. pallor 4. tachycardia 5. hemoglobin less than 10g/dL (100 mmol/L) 6. hematocrit value less than 30%

abortion assessment

1. spontaneous vaginal bleeding 2. low uterine cramping or contractions 3. blood clots or tissue through the vagina 4. hemorrhage and shock can result if bleeding is excessive

When can the systolic/diastolic ratio be first detected in the fetus? 2nd week of pregnancy 15th week of pregnancy 24th week of pregnancy 32nd week of pregnancy

15th week of pregnancy Rationale The first systolic/diastolic ratio can be determined at 15 weeks of pregnancy, because the resistance to the blood flow in the umbilical and uterine arteries is high. The heart is not well developed during the 2nd week of pregnancy, so the systolic/diastolic ratio cannot be determined. As the pregnancy progresses the resistance to the blood flow decreases in the umbilical and uterine arteries. Therefore, the systolic/diastolic ratio decreases as the pregnancy progresses and cannot be determined during the 24th and 32nd weeks of pregnancy.

A mother, G4 P4004, is 15 minutes postpartum. Her baby weighed 4,595 grams at birth. For which of the following complications should the nurse monitor this client? 1. Seizures. 2. Hemorrhage. 3. Infection. 4. Thrombosis

2. Hemorrhage.

A client is 3 days post-cesarean delivery for eclampsia. The client is receiving hydralazine (Apresoline) 10 mg 4 times a day by mouth. Which of the following findings would indicate that the medication is effective? 1. The client has had no seizures since delivery. 2. The client's blood pressure has dropped from 160/120 to 130/90. 3. The client's postoperative weight has dropped from 154 to 144 lb. 4. The client states that her headache is gone.

2. The client's blood pressure has dropped from 160/120 to 130/90.

The blood glucose of a client with type 1 diabetes 12 hours after delivery is 96 mg/dL. The client has received no insulin since delivery. The drop in serum levels of which of the following hormones of pregnancy is responsible for the glucose level? 1. Estrogen. 2. Progesterone. 3. Human placental lactogen (hPL). 4. Human chorionic gonadotropin (hCG).

3. Human placental lactogen (hPL).

A nurse is working on the postpartum unit. Which of the following patients should the nurse assess first? 1. PP1 from vaginal delivery with complaints of burning on urination. 2. PP2 from forceps delivery with blood loss of 500 mL at time of delivery. 3. PP3 from vacuum delivery with hemoglobin of 7.2 g/dL. 4. PO4 from cesarean delivery with complaints of firm and painful breasts.

3. PP3 from vacuum delivery with hemoglobin of 7.2 g/dL.

The nurse caring for a type 1 diabetic client who wishes to become pregnant notes that the client's glycohemoglobin, or glycosylated hemoglobin (HgbA,,), result was 15% today and the fasting blood glucose result was 100 mg/dL. Which of the following interpretations by the nurse is correct in relation to these data? 1. The client has been hyperglycemic for the last 3 months and is hyperglycemic today. 2. The client has been normoglycemic for the last 3 months and normoglycemic today. 3. The client has been hyperglycemic for the last 3 months and normoglycemic today. 4. The client has been normoglycemic for the last 3 months and hyperglycemic today.

3. The client has been hyperglycemic for 3 months but is normoglycemic today. TEST-TAKING TIP: It is very important for a glycohemoglobin test to be performed at the same time that a fasting glucose is done in order to have an idea of a diabetic client's glucose control over the past 3 months in comparison to the results of the fasting test. The red blood cell (RBC), when in a hyperglycemic environment, becomes a compound molecule with a glucose group attached to it. Because the RBC lives for approximately 120 days, the health care practitioner can estimate the glucose control of the client over the preceding 3 months time by analyzing the glycohemoglobin. Up to 5% glycohemoglobin is considered normal. An HgbAlc level of 15%, therefore, indicates that the client has been hyperglycemic for the past 3 months. Since her fasting blood glucose level of 100 mg/dL is normal, however, she is normoglycemic today.

The nurse suspects that a client is third spacing fluid. Which of the following signs will provide the nurse with the best evidence of this fact? 1. Client's blood pressure. 2. Client's appearance. 3. Client's weight. 4. Client's pulse rate.

3. Weight is the most important sign for the nurse to assess. When clients third space, they are retaining fluids. Fluid is very heavy. A sudden weight increase is, therefore, the most important assessment the nurse can make to determine whether or not a client is third spacing. Clients who are being assessed for preeclampsia, therefore, should be weighed daily.

Which ratio would be used to restore effective circulating volume in a postpartum patient who is experiencing hypovolemic shock? 4:1 2:1 1:1 3:1

3:1 A 3:1 ratio, of 3 ml fluid infused for every 1 ml of estimated blood loss, is recommended to restore circulating volume.

An anesthesiologist informs the nurse that a woman for cesarean section will have the procedure under general anesthesia with postoperative patient-controlled analgesia rather than under continuous epidural infusion. Which of the following would warrant this decision? 1. The woman has a history of drug addiction. 2. The woman is allergic to morphine sulfate. 3. The woman is a thirteen-year-old adolescent. 4. The woman has had surgery for scoliosis.

4. A history of scoliosis surgery is a contraindication for epidural anesthesia. TEST-TAKING TIP: Scoliosis is a defect in the growth of the thoracic and lumbar spine. The surgery is, therefore, performed on the vertebrae of the spinal column. Any spinal surgery is a contraindication to the administration of regional anesthesia

A home care nurse is visiting a breastfeeding client who is 2 weeks postdelivery of a 7-lb baby girl over a midline episiotomy. Which of the following findings should take priority? 1. Lochia is serosa. 2. Client cries throughout the visit. 3. Nipples are cracked. 4. Client yells at the baby for crying.

4. Client yells at the baby for crying.

The nurse is caring for a couple who are in the labor/delivery room immediately after the delivery of a dead baby who exhibited visible birth defects. Which of the following actions by the nurse is appropriate? 1. Discourage the parents from naming the baby. 2. Advise the parents that the baby's defects would be too upsetting for them to see. 3. Transport the baby to the morgue as soon as possible. 4. Give the parents a lock of the baby's hair and a copy of the footprint sheet.

4. Give the parents a lock of the baby's hair and a copy of the footprint sheet.

rubella nonimmune, breastfeeding client has just received the rubella vaccine. Which of the following side effects should the nurse warn the client about? 1. The baby may develop a rash a week after the shot. 2. The baby may temporarily reject the breast milk. 3. The mother's milk supply may decrease precipitously. 4. The mother's joints may become painful and stiff.

4. The mother's joints may become painful and stiff.

A woman who wishes to breastfeed advises the nurse that she has had breast augmentation surgery. Which of the following responses by the nurse is appropriate? 1. Breast implants often contaminate the milk with toxins. 2. The glandular tissue of women who need implants is often deficient. 3. Babies often have difficulty latching to the nipples of women with breast implants. 4. Women who have implants are often able exclusively to breastfeed.

4. Women who have implants are often able exclusively to breastfeed.

Fetal distress is occurring with a laboring client. As the nurse prepares the client for a cesarean birth, what is the most important nursing action? 1. slow the intravenous flow rate 2. continue the oxytocin drip if infusing 3. place the client in a high Fowler's position 4. administer oxygen, 8 to 10 L/min via face mask

4. administer oxygen, 8 to 10 L/min via face mask

The nurse responsible for the care of postpartum woman recognizes that the first sign of puerperal infection most likely is A Temperature elevation to 38C or higher after the first 24 hours following birth B Increases white blood cell count C Foul smelling profuse lochia D Bradycardia

A Temperature elevation to 38C or higher after the first 24 hours following birth

Pelvic Hematoma

A collection of blood in the connective tissue that can be vulvar, vaginal, or retroperitoneal -pain and feeling of pressure are most common symptoms Treatment: -surgical evacuation -attention to pain relief, replacing fluids, and reviewing lab results What is the most common hematoma? vulvar, usually visible. What is the most common symptom with hematomas?pain. What are the least common hematomas? retroperitoneal hematomas; life threatening, usually associated with rupture of a cesarean scar during labor, initial symptoms can be signs of shock.

chromosomal abnormalities

A fluid collection greater than 3 mm is considered abnormal+ low maternal serum marker levels - tisomies 13, 18, 21 elevated nuchal translucency alone - increased risk of fetal cardiac disease when NT is abnormal - genetic testing other findings could be - nasal bone, short femur/humerus, echogenic bowel, intracardiac focus they are markers not diagnostic

What is the most common reason for late postpartum hemorrhage (PPH)? a. Subinvolution of the uterus b. Defective vascularity of the decidua c. Cervical lacerations d. Coagulation disorders

ANS: A Late PPH may be the result of subinvolution of the uterus. Recognized causes of subinvolution include retained placental fragments and pelvic infection. Although defective vascularity, cervical lacerations, and coagulation disorders of the decidua may also cause PPH, late PPH typically results from subinvolution of the uterus, pelvic infection, or retained placental fragments.

What is the most common medical complication of pregnancy? a. Hypertension b. Hyperemesis gravidarum c. Hemorrhagic complications d. Infections

ANS: A Preeclampsia and eclampsia are two noted deadly forms of hypertension. A large percentage of pregnant women will have nausea and vomiting, but a relatively few will have the severe form called hyperemesis gravidarum. Hemorrhagic complications are the second most common medical complication of pregnancy; hypertension is the most common. Infection is a risk factor for preeclampsia.

In caring for an immediate postpartum client, the nurse notes petechiae and oozing from her intravenous (IV) site. The client would be closely monitored for which clotting disorder? a. DIC b. Amniotic fluid embolism (AFE) c. Hemorrhage d. HELLP syndrome

ANS: A The diagnosis of DIC is made according to clinical findings and laboratory markers. A physical examination reveals unusual bleeding. Petechiae may appear around a blood pressure cuff on the woman's arm. Excessive bleeding may occur from the site of slight trauma such as venipuncture sites. These symptoms are not associated with AFE, nor is AFE a bleeding disorder. Hemorrhage occurs for a variety of reasons in the postpartum client. These symptoms are associated with DIC. Hemorrhage would be a finding associated with DIC and is not a clotting disorder in and of itself. HELLP syndrome is not a clotting disorder, but it may contribute to the clotting disorder DIC.

Which nursing intervention should be immediately performed after the forceps-assisted birth of an infant? a. Assessing the infant for signs of trauma b. Administering prophylactic antibiotic agents to the infant c. Applying a cold pack to the infant's scalp d. Measuring the circumference of the infant's head

ANS: A The infant should be assessed for bruising or abrasions at the site of application, facial palsy, and subdural hematoma. Prophylactic antibiotics are not necessary with a forceps delivery. A cold pack would place the infant at risk for cold stress and is contraindicated. Measuring the circumference of the head is part of the initial nursing assessment.

Cell-free deoxyribonucleic acid (DNA) screening is a new method of noninvasive prenatal testing (NIPT) that has recently become available in the clinical setting. This technology can provide a definitive diagnosis of which findings? (Select all that apply.) a. Fetal Rh status b. Fetal gender c. Maternally transmitted gene disorder d. Paternally transmitted gene disorder e. Trisomy 21

ANS: A, B, D, E The NIPT cannot actually distinguish fetal from maternal DNA. It can determine fetal Rh status, gender, trisomies 13, 18, and 21, as well as paternally transmitted gene disorders. The test can be performed any time after 10 weeks of gestation and is recommended for women who have previously given birth to a child with chromosomal abnormalities.

Lacerations of the cervix, vagina, or perineum are also causes of PPH. Which factors influence the causes and incidence of obstetric lacerations of the lower genital tract? (Select all that apply.) a. Operative and precipitate births b. Adherent retained placenta c. Abnormal presentation of the fetus d. Congenital abnormalities of the maternal soft tissue e. Previous scarring from infection

ANS: A, C, D, E Abnormal adherence of the placenta occurs for unknown reasons. Attempts to remove the placenta in the usual manner can be unsuccessful, and lacerations or a perforation of the uterine wall may result. However, attempts to remove the placenta do not influence lower genital tract lacerations. Lacerations of the perineum are the most common of all lower genital tract injuries and often occur with both precipitate and operative births and are classified as first-, second-, third-, and fourth-degree lacerations. An abnormal presentation or position of the fetus, the relative size of the presenting part, and the birth canal may contribute to lacerations of the lower genital tract. Congenital abnormalities, previous scarring from infection or injury, and a contracted pelvis may also influence injury to the lower genital tract, followed by hemorrhage.

A client who has undergone a D&C for early pregnancy loss is likely to be discharged the same day. The nurse must ensure that her vital signs are stable, that bleeding has been controlled, and that the woman has adequately recovered from the administration of anesthesia. To promote an optimal recovery, what information should discharge teaching include? (Select all that apply.) a. Iron supplementation b. Resumption of intercourse at 6 weeks postprocedure c. Referral to a support group, if necessary d. Expectation of heavy bleeding for at least 2 weeks e. Emphasizing the need for rest

ANS: A, C, E The woman should be advised to consume a diet high in iron and protein. For many women, iron supplementation also is necessary. The nurse should acknowledge that the client has experienced a loss, however early. She can be taught to expect mood swings and possibly depression. Referral to a support group, clergy, or professional counseling may be necessary. Discharge teaching should emphasize the need for rest. Nothing should be placed in the vagina for 2 weeks after the procedure, including tampons and vaginal intercourse. The purpose of this recommendation is to prevent infection. Should infection occur, antibiotics may be prescribed. The client should expect a scant, dark discharge for 1 to 2 weeks. Should heavy, profuse, or bright bleeding occur, she should be instructed to contact her health care provider.

Which clinical finding is a major use of ultrasonography in the first trimester? a. Amniotic fluid volume b. Presence of maternal abnormalities c. Placental location and maturity d. Cervical length

ANS: B Ultrasonography can detect certain uterine abnormalities such as bicornuate uterus, fibroids, and ovarian cysts. Amniotic fluid volume, placental location and maturity, and cervical length are not available via ultrasonography until the second or third trimester.

25. The indirect Coombs' test is a screening tool for Rh incompatibility. If the titer is greater than ______, amniocentesis may be a necessary next step. a. 1:2 b. 1:4 c. 1:8 d. 1:12

ANS: C If the maternal titer for Rh antibodies is greater 1:8, then an amniocentesis is indicated to determine the level of bilirubin in the amniotic fluid. This testing will determine the severity of fetal hemolytic anemia.

A pregnant client is diagnosed with abruptio placentae. What signs and symptoms would the nurse find in the client? Select all that apply. Hypoglycemia Abdominal pain Vaginal bleeding Delayed menses Uterine tenderness

Abdominal pain Vaginal bleeding Uterine tenderness Rationale Premature separation of the placenta from the uterus is called as abruptio placentae. Vaginal bleeding, abdominal pain, and uterine tenderness are signs and symptoms of abruptio placentae. Abruptio placentae does not affect blood glucose level; therefore, it does not cause hypoglycemia. Delayed menses is a sign of an ectopic pregnancy.

Pelvic dystocia

Abnormal labor caused by contractures of the pelvic diameters that reduce the capacity of the bony pelvis, including the inlet, midpelvis, outlet, or any combination of these planes.

Cephalopelvic disproportion (CPD); fetopelvic disproportion (FPD)

Abnormal labor caused by excessive fetal size in relation to the size of the maternal pelvis.

Fetal dystocia

Abnormal labor caused by fetal anomalies, excessive fetal size, malpresentation, malposition, or multifetal pregnancy.

Major placental complications involving abnormal attachment to the uterus that inhibits the placental separation process after birth and can result in hemorrhage.

Accreta; increta; precreta

Which parameter is measured to detect neural tube defects in a fetus? Acetylcholinesterase Phosphatidylglycerol (PG) Lecithin-to-sphingomyelin (L/S) ratio Surfactant-to-albumin (S/A) ratio

Acetylcholinesterase Rationale Acetylcholinesterase and amniotic fluid alpha-fetoprotein (AFP) measurement are used to diagnose neural tube defects. The presence of Phosphatidylglycerol (PG) is used in amniocentesis to determine fetal lung maturity. The lecithin-to-sphingomyelin (L/S) ratio is used in amniocentesis to determine fetal lung maturity. The surfactant-to-albumin (S/A) ratio helps determine fetal lung maturity in the TDx FLM assay.

Vasospasm

Activity in blood vessels that diminishes their diameter, thereby impeding tissue perfusion in all organ systems, increasing peripheral vascular resistance and blood pressure, and increasing endothe- lial cell permeability.

Are pregnant diabetic women more likely to develop hyperglycemia or hypoglycemia?

Actually hypoglycemia

The postpartum nurse is caring for a patient who experienced a cesarean delivery and whose vertical skin incision is closed with sutures and surgical glue. Which step should the nurse take to ensure the patient's comfort during the assessment? Assist the patient to extend her legs prior to palpating the fundus. Avoid palpating the area around the incision during assessment. Decrease the frequency of assessments to allow for patient rest. Administer the patient's pain medication before fundal assessment.

Administer the patient's pain medication before fundal assessment. Patients with vertical skin incisions often experience greater discomfort during fundal assessment than patients with Pfannenstiel incisions. Therefore, administering pain medication before fundal assessment helps ensure patient comfort and is a step the nurse should take during assessment.

The majority of ectopic pregnancies are located in the: Uterine fundus. Cervical os. Ampulla. Fimbriae.

Ampulla. A pregnancy within the uterus would be considered a normal pregnancy. Implantation of the pregnancy at the cervical os would be a significant abnormality. The majority of ectopic pregnancies, approximately 80%, are located in the ampulla or largest portion of the tube.

Mechanical dilation

Balloon catheter inserted through intracervical canal to ripen and dilate cervix -stimulates the release of endogenous prostaglandins -reduced risk of uterine tachysystole and fetal heart rate changes -low cost compared with prostaglandins -balloon usually falls out on its own in 8-12 hours

Nsg interventions for a neonate born to an HIV-positive mother?

Bathe the neonate carefully before any invasive procedure, clean the umbillical cord stump meticulously every day until healed, newborn can room with mother, administer zidovudine to the newborn as prescribed for the 1st 6 weeks of life, all HIV exposed newborns should be treated with medication to prevent infeciton by Pneumocystis jiroveci and HIV culture is recommended at 1 & 4 months after birth should be seen by HCP at 1 week, 2 weeks, 1 month, 2 months, 4 months can be asymptomatic the first year

Forceps-assisted

Birth method in which an instrument with two curved blades is used to assist the birth of the fetal head.

Cesarean birth

Birth of the fetus through a transabdominal incision of the uterus.

Late preterm birth

Birth that occurs between 34 and 36 weeks of gestation.

Glycosylated hemoglobin (hemoglobin A1c)

Blood test that evaluates glycemic control over the previous 4 to 6 weeks."

If placenta lies within 2 cm of cervix as documented by TVUS,

C-section must be performed

Gestational diabetes

Carbohydrate intolerance with onset or first recognition occurring during pregnancy.

Overweight BMI

Characterized as BMI of 25 or greater

Obesity- BMI

Characterized as BMI of 30 or greater

The primary expected outcome for nursing care associated with the administration of magnesium sulfate would be met if which assessment finding is present? A. The woman exhibits a decrease in both systolic and diastolic blood pressure. B. The woman experiences no seizures. C. The woman states that she feels more relaxed and calm. D. The woman urinates more frequently, resulting in a decrease in pathologic edema.

Choice B is correct; magnesium sulfate acts as a CNS depressant and is given to prevent seizures. B. The woman experiences no seizures.

Which manifestation does the nurse relate to hypoglycemia in a diabetic client? Clammy skin Rapid breathing Nausea or vomiting Increased urination

Clammy skin Rationale A client with hypoglycemia experiences sweating, pallor, and clammy skin. The hypoglycemic client has shallow respirations, whereas the hyperglycemic client takes rapid breaths. Hyperglycemia causes nausea and vomiting. The client with hypoglycemia experiences hunger. In cases of hyperglycemia, the kidneys excrete large volumes of urine in an attempt to regulate excess vascular volume.

What is disseminated intravascular coagulation (DIC)?

DIC is a maternal condition in which the clotting cascade is activated, resulting in the formation of clots in the microcirculation

Euglycemia

Desired blood glucose levels Should be in the range of 60-99 mg/dl (fasting) and 100-129 mg/dl when measured 1 hour after a meal 2 hrs after a meal, should be no higher than 120 mg/dl

A pregnant client is treated with prostaglandins after a miscarriage. What side effects of prostaglandins will the nurse monitor in the patient? Diarrhea Vomiting Skin rashes Blurred vision Yellow-colored sputum

Diarrhea Vomiting Rationale Prostaglandins help expel the products of conception completely. Diarrhea and vomiting are side effects of prostaglandins. Prostaglandins do not affect skin integrity; therefore, they do not cause skin rashes. Prostaglandins do not affect vision; therefore, they do not cause blurred vision. Yellow-colored sputum is not a side effect of prostaglandins. Yellow-colored sputum is a sign of infection.

Couvelaire uterus

Disorder of the uterus in which blood accumulates between a separating placenta and the uterine wall; the uterus appears purple or blue and contractility is lost.

What are the signs and symptoms associated with urinary tract infections (UTIs)? Dysuria Oligouria Erythema Hematuria Low grade feve

Dysuria Hematuria Low grade fever Rationale UTIs are associated with dysuria, low grade fever, and hematuria. Dysuria is caused by the growth of bacteria in the urinary tract. Bacterial infection of the urinary tract also causes low grade fever. Hematuria is caused by ruptured uterine layers due to a bacterial infection. Urinary tract infection may manifest as increased frequency and urgency of micturition. It is not associated with oligouria, which refers to the decreased production of urine. Erythema is the redness of the skin, which is a sign of wound infection after childbirth.

A woman with a BMI of 30 delivered a 3402 g infant with a vacuum extractor and an episiotomy. The patient is at greatest risk for which infection? Mastitis Endometritis Laceration infection Episiotomy infection

Endometritis Maternal obesity and use of a vacuum extractor are risk factors for endometritis.

Hydramnios (polyhydramnios)

Excessive amount of amniotic fluid. "

Macrosomia

Excessive fetal size.

Fetal fibronectins

Glycoprotein found in plasma and produced during fetal life; its reappearance in the cervical mucus during the late second and early third trimesters of pregnancy may be related to placental inflam- mation, which is thought to be one possible cause of spontaneous preterm labor.

For which woman should a nurse schedule a 3-hour glucose tolerance test? Fasting glucose of 90 mg/dL 3-hour fasting glucose of 95 mg/dL Greater than 140 mg/dL reading from a glucose challenge test Less than 120 mg/dL reading from a glucose challenge test

Greater than 140 mg/dL reading from a glucose challenge test A reading greater than 140 mg/dL is considered abnormal and does warrant a 3-hour glucose tolerance test.

Gestational HTN

HTN can be mild or severe leading to preeclampsia then eclampsia (seizures).

Major maternal complication associated with placenta previa

Hemorrhage Other complications: -abnormal placental attachment (placenta accreta, increta, percreta) -blood transfusion reactions -anemia -thrombophlebitis -infection

What is an ectopic pregnancy?

Implantation of the fertilized ovum outside of the uterine cavity, the most common location is the ampulla of the fallopian tube

Placenta Previa

Improperly implanted placenta in lower uterine segment over internal cervical. S/S Sudden onset of PAINLESS, Bright Red vaginal bleeding, Uterus soft, relaxed, NONtender, fungal height may be more than expected for gestational age Interventions- monitor vs, FHR, prepare for US, avoid vaginal exam, bed rest in side lying position, monitor amount of bleeding, admin IV fluids, possible RhoGAM, tocolytic meds if bleedin gis heavy - c-section

Polyhydraminos

Increase or exvessive amount of aminotic fluid

What side effects should the nurse be aware of in a pregnant client who had been administered betamethasone (Diprolene)? Hot flushes Hypotension Hypoglycemia Increased white blood cells (WBCs)

Increased white blood cells (WBCs) Rationale Betamethasone (Diprolene) is a glucocorticoid administered to a client who is at risk of preterm birth. The client will have a transient increase in WBCs and elevated blood glucose levels as a side effect of the medication. Hot flushes are observed in clients who are administered magnesium sulfate (Epsom Salt), and nifedipine (Adalat) causes hypotension. These effects are not related to the administration of betamethasone (Diprolene). Betamethasone (Diprolene) causes hyperglycemia (increased blood glucose levels) as a side effect.

Indirect Coombs

Indirect Coombs Screening test for Rh incompatibility by examining the serum of Rh-negative women for Rh antibodies

Mastitis

Inflammation of the breast as a result of infection. Occurs primarily in breast-feeding mothers 2 to 3 weeks after delivery but may occur at any time during lactation.

The nurse is preparing to administer dexamethasone (Decadron) to a pregnant client. Which nursing intervention should the nurse perform for safe administration of the drug? Monitor blood pressure of the client. Inform the client that it will be painful. Assess blood glucose levels in the client. Administer the drug by intramuscular injection. Administer the oral form if the client refuses injection.

Inform the client that it will be painful. Assess blood glucose levels in the client. Administer the drug by intramuscular injection. Rationale Dexamethsone (Decadron) is a glucocorticoid used to promote fetal lung maturation. The drug can also increase blood sugar levels in the client. Therefore the nurse should monitor the blood sugar levels to assess the need for an increased insulin dose. The drug should be given by intramuscular injection in the ventral gluteal or vastus lateralis muscle for better absorption. The client should be informed that the injection will be painful, because this type of truthfulness promotes client cooperation. The drug does not affect blood pressure levels and it does not need to be monitored. The oral form is not beneficial in promoting fetal lung maturation and should not be administered.

Vacuum-Assisted Birth

Involves the attachment of a vacuum cup to the fetal head -generally not used to assist birth before 34 weeks of gestation -cervix must be completely dilated -rupture of membranes must have occurred -engaged head -vertex presentation -no suspicion of CPD Advantages of Vacuum-Assisted birth over Forceps-Assisted Birth -ease with which the vacuum can be placed -need for less anesthesia Risks to newborn and mother of Vacuum-Assisted Birth Mother -perineal, vaginal, or cervical lacerations -soft-tissue hematomas Baby -cephalhematoma -scalp lacerations -subdural hematoma -hyperbilirubinemia -Caput succedaneum

Nurses should be aware that HELLP syndrome: Is a mild form of preeclampsia. Can be diagnosed by a nurse alert to its symptoms. Is characterized by hemolysis, elevated liver enzymes, and low platelets. Is associated with preterm labor but not perinatal mortality.

Is characterized by hemolysis, elevated liver enzymes, and low platelets. The acronym HELLP stands for hemolysis (H), elevated liver enzymes (EL), and low platelets (LP). HELLP syndrome is a variant of severe preeclampsia. It is difficult to identify, because the symptoms often are not obvious. It must be diagnosed in the laboratory. Preterm labor is greatly increased with HELLP syndrome, and so is perinatal mortality.

Hypoglycemia "

Less than the normal amount of glucose in the blood. "

The nurse is assessing a patient in the post-anesthesia care unit (PACU) following a cesarean delivery under general anesthesia. After confirming respiratory status, which is the next assessment the nurse should complete? Fundal height Color of lochia Blood pressure (BP) Level of consciousness (LOC)

Level of consciousness (LOC) Following confirmation of an adequate airway and respiratory status, LOC is next assessed. The patient needs to be alert and oriented in order to continue respiratory (breathing) effort. Consider the ABC principle of prioritization.

Dysfunctional labor (dystocia)

Long, difficult, or abnormal labor caused by various conditions associated with the five factors affecting labor.

Which of the following findings is not likely to be seen in a pregnant client who has hypothyroidism? Miscarriage Macrosomia Gestational hypertension Placental abruption

Macrosomia Rationale Infants born to mothers with hypothyroidism are more likely to be of low birth weight or preterm. These outcomes can be improved with early diagnosis and treatment. Hypothyroidism is often associated with both infertility and an increased risk of miscarriage. Pregnant women with hypothyroidism are more likely to experience both preeclampsia and gestational hypertension. Placental abruption and stillbirth are risks associated with hypothyroidism.

A pregnant client has preterm labor. Which medication would be helpful in preventing preterm labor in the client? Oxytocin (Pitocin) Methotrexate (MTX) Misoprostol (Cytotec) Magnesium sulfate (Sulfamag)

Magnesium sulfate (Sulfamag) Rationale Magnesium sulfate (Sulfamag) is a tocolytic agent used to slow down uterine contractions. Oxytocin (Pitocin) is used to prevent uterine hemorrhage. MTX is used for the treatment of ectopic pregnancy. Misoprostol (Cytotec) is used to expel the products of conception completely after a miscarriage.

After massaging the boggy fundus of a client who delivered a large baby after a prolonged labor with a forceps-assisted birth, the nurse is unable to obtain a firm fundus. What nursing action is indicated at this time? Increase the rate of the intravenous infusion. Massage the fundus while another nurse notifies the PCP. Change the peripad, replacing it with a double pad. Administer a half-dose of a uterine contracting medication

Massage the fundus while another nurse notifies the PCP. Rationale Fragments of the placenta can remain in the uterus after spontaneous separation of the placenta during the third stage of labor. In this case, the woman will have excessive bleeding and the uterus will feel boggy (soft) because of uterine atony. Ultrasonography can be used to detect placental fragments. The client should not be left alone because of the boggy uterus, which can lead to hemorrhage. There is an absolute need for further medical intervention.

Type 2 diabetes mellitus

Metabolic disease resulting in insulin resistance and relative rather than absolute insulin deficiency; it is the most prevalent form of the disease."

Type 1 diabetes mellitus"

Metabolic disease that occurs when pancreatic islet beta cells are destroyed, resulting in an absolute insulin deficiency; an autoimmune process may be involved in its etiology.

Should women receiving methotrexate to treat ectopic pregnancy take analgesics?

None stronger than acetaminophen because stronger analgesics can mask symptoms of tubal rupture

Biophysical profile

Noninvasive dynamic assessment of the fetus and its environment that is based on acute and chronic markers of fetal disease. It uses both real-time ultrasounds and external electronic fetal monitoring. This test includes assessment of 5 variables, namely fetal breathing movements, fetal body movements, fetal tone, nonstress test, and amniotic fluid volume

Trial of labor

Observance of a woman and her fetus for a reasonable period of spontaneous active labor (e.g., 4 to 6 hours) to assess the safety of a vaginal birth for both.

Early/Acute/Primary PPH

Occurs within 24 hours of birth

Which drug is used for treating a client with severe postpartum bleeding? Nifedipine (Adalat) Oxytocin (Pitocin) Propranolol (Inderal) Metronidazole (Flagyl)

Oxytocin (Pitocin) Rationale Oxytocin (Pitocin) is a synthetic hormone used to induce labor and to control severe postpartum bleeding by making the uterus contract. Nifedipine (Adalat) is a calcium channel blocker that is used intocolytic therapy for preterm labor. Propranolol (Inderal) is used to reverse intolerable cardiovascular effects of terbutaline (Brethine). Metronidazole (Flagyl) is a broad-spectrum antibiotic that is used to treat chorioamnionitis after cesarean birth.

The nurse is caring for a patient recovering from a postpartum hemorrhage. The patient's bleeding is under control, and vital signs are stable. Which action should the nurse take? Discourage the patient from eating seafood. Leave the room while the patient feeds her newborn to allow for privacy. Perform shift assessments and medication administration at the same time. Reassure the patient that a blood transfusion will most likely not be necessary at this point in her care.

Perform shift assessments and medication administration at the same time. Patients recovering from a postpartum hemorrhage are often exhausted. Organizing care so that some interventions are done together allows the patient to conserve energy.

A woman at 34 weeks' gestation presents with painful uterine contractions, lower back pain, and bloody vaginal discharge. Additionally, the woman has a temperature of 102° F and is noted to be at 2 cm dilated. Which action should the nurse take first? Assess for infection. Provide emotional support. Place the woman on bed rest. Evaluate fetus for distress, size, and maturity.

Place the woman on bed rest. Placing the woman on bed rest in side-lying position is the initial nursing intervention that is appropriate in preterm labor. This action helps prevent further uterine contractions.

PPH

Postpartum Hemorrhage -leading cause of maternal morbidity and mortality -loss of 500 mL or more of blood after vaginal birth or 1000 mL or more of blood after C-section -Defined also by a 10% decrease in hematocrit between admission for labor and postpartum

Which condition can be predicted in a pregnant client if uterine artery Doppler measurements in the second trimester of pregnancy are abnormal? Preeclampsia HELLP syndrome Molar pregnancy Gestational hypertension

Preeclampsia Rationale Preeclampsia is a condition in which clients develop hypertension and proteinuria after 20 weeks of gestation. It can be predicted if uterine artery Doppler measurements in the second trimester of pregnancy are abnormal. HELLP syndrome is characterized by hemolysis (H), elevated liver enzymes (EL), and low platelet count (LP) in a patient with preeclampsia. Molar pregnancy refers to the growth of the placental trophoblast due to abnormal fertilization. Gestational hypertension is a condition in which hypertension develops in a client after 20 weeks of gestation

A patient who is pregnant already has Type 2 diabetes with a hemoglobin A1c value of 7. The nurse would categorize this patient as having: Gestational diabetes. Insulin-dependent diabetes complicated by pregnancy. Pregestational diabetes mellitus. Non-insulin-dependent diabetes with complications

Pregestational diabetes mellitus. Pregestational diabetes mellitus is a term used to describe patients with type 1 or type 2 diabetes in whom diabetes existed prior to pregnancy. Gestational diabetes occurs when a woman becomes diabetic during the pregnancy state. Type 2 diabetes is non-insulin-dependent. None of the information presented indicates complications at this point, because the hemoglobin A1c is within normal range, signifying adequate glycemic control.

Nsg interventions for fetal death in utero?

Prepare the client for delivery of the fetus, support the client's decision about laborm bith, and the postpartum period, facilitate the grieving process as appropriate considering cultural practices and beliefs, accept behaviors such as anger and hostility from the parents, refer the parents to an appropriate support group

Which tests help detect the presence of Down syndrome in the fetus of a patient in the second trimester of pregnancy? Quad-screen Fetal nasal bone assessment Triple-marker screen Nuchal translucency screening Pregnancy-associated placental protein (PAPP-A) measurement

Quad-screen Triple-marker screen Rationale The quad-screen is performed in the second trimester using four different serum markers: maternal serum alpha-fetoprotein (MSAFP), unconjugated estriol, human chorionic gonadotropin (hCG), and inhibin A. Low levels of MSAFP, unconjugated estriol, and inhibin A, and high levels of hCG, indicate Down syndrome. The triple-marker screen is similar to the quad-screen but does not include assessment of inhibin A. The triple-marker screen is performed at 16 to 18 weeks of gestation and consists of three maternal serum markers: MSAFP, unconjugated estriol, and hCG. The fetal nasal bone is assessed in the first trimester of pregnancy. Nuchal translucency would be used in combination with different serum markers to identify Down syndrome. Nuchal translucency is measured in the first trimester of pregnancy. Pregnancy-associated placental protein (PAPP-A) is measured in the first trimester of pregnancy to detect Down syndrome.

Preterm labor

Regular contractions along with a change in cervical effacement or dilation or both or presentation with regular uterine contractions and cervical dilation of at least 2 cm

Preterm labor

Regular uterine contractions along with cervical changes (dilation) occurring between 20 and 37 weeks of pregnancy

The nurse is caring for a patient who has just been admitted to the labor and delivery unit. The patient suddenly grips the side rails and yells loudly, and the nurse can see the infant rapidly crowning. Which action should the nurse take first? Contact the health care provider. Reposition the patient to a side-lying position. Administer oxygen to the patient through a face mask. Reposition the patient so her hips are elevated above the head.

Reposition the patient to a side-lying position. A patient experiencing precipitate birth should remain in a side-lying position to slow the descent of the infant and prevent perineal lacerations. Repositioning the patient to a side-lying position is the first action the nurse should take because this is an independent nursing action that directly addresses the patient's priority health problem. This intervention can be performed rapidly, ensuring that other aspects of care are not delayed.

A woman whose labor was induced has a heart rate of 95 bpm. Which other sign could indicate that the woman might have an infection? Dark red lochia Generalized muscle aching Fundus that is firm on palpation Respiratory rate of 22 breaths per minute

Respiratory rate of 22 breaths per minute Respiratory rate of 22 breaths per minute is considered elevated and may indicate possible infection.

The nurse is caring for a client with preeclampsia who is receiving an intravenous (IV) magnesium sulfate (Sulfamag) infusion. The nurse assesses the client every 20 minutes. Which maternal findings require immediate intervention by the nurse? Deep tendon reflex of 2+ Urinary output of 30 ml/hr Blood pressure of 130/90 mm Hg Respiratory rate of 9 breaths/minute

Respiratory rate of 9 breaths/minute Rationale While caring for a pregnant client with preeclampsia on IV magnesium sulfate infusion therapy, the nurse should report a respiratory rate below 12 breaths per minute to the primary health care provider. Magnesium sulfate (Sulfamag) has the potential to decrease the respiratory rate. Any respiratory rate less than 12 breaths/minute in this client indicates decreased respiration and should be reported to the primary health care provider. A deep tendon reflex of 2+ is an expected response and a normal finding in the client. Urinary output of 30 ml/hr and blood pressure of 130/90 mm Hg are also normal findings. These conditions need not to be reported to the primary health care provider.

A pregnant woman at 14 weeks of gestation is admitted to the hospital with a diagnosis of hyperemesis gravidarum. What is the primary goal of her treatment at this time? Rest the gastrointestinal (GI) tract by restricting all oral intake for 48 hours Reduce emotional distress by encouraging the woman to discuss her feelings Reverse fluid, electrolyte, and acid-base imbalances Restore the woman's ability to take and retain oral fluid and foods

Reverse fluid, electrolyte, and acid-base imbalances Rationale Fluid, electrolyte, and acid-base imbalances present the greatest immediate danger to the well-being of the mother and fetus and should be corrected as soon as possible. Resting the GI tract is a component of treatment; however, it is not an immediate goal for this client. Discussing her feelings is also a component of treatment but not an immediate goal at this time. The ability to retain oral fluid and foods is a longer-term goal of treatment for this condition.

What is the characteristic sign of a hydatidiform mole?

Snowstorm pattern shown on ultrasound

Premature rupture of membranes (PROM)

Spontaneous rupture of the amniotic sac and leakage of amniotic fluid before the onset of labor at any gestational age.

The nurse is assessing a client who is 18 weeks pregnant. The client reports heavy bleeding, infection, and excessive cramping. Which treatment strategy should be included in the treatment plan? Suction curettage McDonald technique Administration of methotrexate (MTX) Administration of misoprostol (Cytotec)

Suction curettage Rationale Heavy bleeding, infection, and excessive cramping are signs and symptoms of miscarriage. In this situation, fetal or placental tissue must be removed from the uterus by suction curettage. McDonald technique is used in case of cervical insufficiency. In this technique, suture is placed around the cervix beneath the mucosa to constrict the internal os of the cervix. MTX is used in the treatment of ectopic pregnancy. Misoprostol (Cytotec) is prostaglandin drug. If bleeding and infection are absent, then misoprostol (Cytotec) is used for miscarriage.

Vibroacoustic stimulation test

Test used to determine fetal response to sound; the expected response is acceleration of fetal heart rate

Which assessment finding does the nurse recognize as an indicator for early screening for gestational diabetes mellitus (GDM)?> The client is 24 years old. The client's body mass index (BMI) is 22. The client does not have diabetes. The client had a previous stillbirth.

The client had a previous stillbirth. Rationale Screening for GDM usually takes place between 24 and 28 weeks of gestation. However, the client is screened for GDM earlier if there is a history of stillbirth or birth of a malformed or macrosomic infant. A 24-year-old client would not be considered for earlier screening for GDM; maternal age older than 25 years is a risk factor for GDM. A client with a BMI of 22 is not obese and is not considered for earlier screening for GDM. If the client does not have diabetes before gestation, the client need not be screened early for GDM.

The nurse is caring for a client who reports abdominal pain and abnormal vaginal bleeding after the menstrual cycle. The nurse tells the client, "You should not consume foods rich in folic acid, and you should stay indoors." What is the reason for these instructions? The client is taking oxytocin (Pitocin). The client is taking methotrexate (MTX). The client is taking misoprostol (Cytotec). The client is taking ergonovine (Methergine).

The client is taking methotrexate (MTX). Rationale Abdominal pain and abnormal vaginal bleeding after the menstrual cycle indicate that the client has ectopic pregnancy. MTX therapy is used in the treatment of ectopic pregnancy. Folic-rich, gas-forming foods and sun exposure decreas the effectiveness of MTX. These factors do not affect the effectiveness of oxytocin (Pitocin), misoprostol (Cytotec), and ergonovine (Methergine). Hence, the client does not need to avoid folic-rich, gas-forming foods, or sun exposure, while taking oxytocin (Pitocin), misoprostol (Cytotec), and ergonovine (Methergine).

A pregnant woman has maternal phenylketonuria (PKU) and is interested in whether or not she will be able to breastfeed her baby. Which statement made by the nurse indicates accurate information? The client can breastfeed the baby as long as she continues to maintain a PKU-restricted diet. The client should alternate breastfeedings with bottle feedings in order to reduce PKU levels provided to the baby. The client should be advised to not breastfeed the infant, because her breast milk will contain large amounts of phenylalanine The client can breasteed for the first three months without any untoward effects on the infant

The client should be advised to not breastfeed the infant, because her breast milk will contain large amounts of phenylalanine Rationale Breastfeeding is not advised for a client who has maternal PKU, because phenylalanine levels will be high in the client's breast milk. Dietary restriction will not limit the amount of this substance in breast milk. Alternating feeding sources is not advised.

A nurse is caring for a patient following cesarean delivery. According to the operative report, the patient had a low transverse uterine incision. Which interpretation of this data by the nurse is accurate? The patient had minimal blood loss during the surgery. The patient is no longer a candidate for subsequent vaginal delivery. The patient is at risk for a uterine tear that could extend down into the cervix. The patient is at risk for uterine incisional rupture during a subsequent birth.

The patient had minimal blood loss during the surgery. A low transverse uterine incision is preferred since one advantage is that there is typically little blood loss during the procedure. This is related to its small size and placement on the uterus.

What is classification of placental abruption based on?

Type and severity

Velamentous insertion of the cord

Type of vasa previa when the cord vessels begin to branch at the membranes and then course into the placenta.

Succenturiate placenta

Type of vasa previa when the placenta is divided into two or more separate lobes; fetal vessels run between the lobes.

Internal Version

Used by HCP who inserts a hand into the uterus to change the position of the baby -usually in twin gestations to deliver the 2nd fetus

When is a pregnant woman at greatest risk for hypoglycemia?

When is a pregnant woman at greatest risk for hypoglycemia? During the first trimester when hepatic production of glucose is decreased and peripheral uptake of glucose is increased

thrombocytopenia

abnormally low platelet level

alpha-fetoprotein (AFP)

amniotic fluid levels used as a screening tool for NTDs in pregnancy, no known cause, but risks are - folic acid deficiency, pregestational diabetes, teratogen exposure, carbamazepine

postpartum (puerperal) infection

any clinical infection of the genital canal that occurs within 28 days after miscarriage, induced abortion or childbirth

Coagulopathies or clotting disorders

are associated with many obstetric complications; they include idiopathic or immune thrombocytopenic purpura, von Willebrand disease, and DIC

Postpartum UTIs

are often related to cesarean births, urinary catheterization, frequent pelvic examinations, and regional anesthesia.

daily fetal movement count (DFMC)

assessment of the fetal activity by the mother also called kick count; the presence of movement is generally a reassuring sign of fetal health

What is preeclampsia?

before they seize, HT and proteinuria develops after 20th week and had previously no condition, reduced organ perfusion w/ presence of hyper tension & protein in urine, associated w/ obesity. MF pregnancy, may be caused by endothelial cell dysfunction or disruption in placental perfusion if no proteiuria - hypertension + thrombocytopenia, impaired liver function, renal insufficiency, pulmonary edema, new onset cerebral or visual disturbances progressive, placenta is the cause, resolves after it expelled

Incomplete inversion of uterus

cannot be seen, a smooth mass can be palpated through dilated cervix.

In the pregnancy complicated by diabetes mellitus,

careful glucose monitoring, insulin administration when necessary, and dietary counseling are used to create a normal intrauterine environment for fetal growth and development All women who are not known to have pregestational diabetes should undergo routine screening through history, clinical risk factor assessment, or laboratory assessment of blood glucose levels during pregnancy.

A postpartum woman in the fourth stage of labor received prostaglandin F2a (Hemabate) 0.25mg intramuscularly. The expected outcome of care for the administration of this medication would be which of the following? a. Relief from the pain of uterine cramping b. Prevention of intrauterine infection c. Reduction in the blood's clotting ability d. Limitation of excessive blood loss that is occurring after birth

d. Limitation of excessive blood loss that is occurring after birth

fetal growth

fetal growth is determined by intrinsic growth potential and environmental factors it has to be done if: - poor maternal weight gain - previous pregnancy with intrauterine growth restriction - chronic infection - drugs - maternal diabetes - hypertension - multifral pregnancy Reduced fetal growth is still one of the most frequent conditions associated with stillbirth

venous thromboembolism (VTE)

formation of a blot clot or clots inside a blood vessel

thrombophlebitis

formation of a clot in a blood vessel as result of inflammation

symmetric intrauterine growth restrictionlow genetic

growth potential intrauterine infection chromosomal anamaly maternal undernutrition heavy smoker

Discharge instructions of PPH

imit physical activity to conserve strength, increase iron and protein intake to rebuild lost RBC, and get assistance with infant care. mother should be informed may have problems with delayed lactogenesis, and insufficient milk production or PPD. -Teach her how to increase dietary iron and protein intake -Iron supplementation -May experience delayed lactogenesis, insufficient milk production -May experience PPD -Tell her she will eel fatigued and will need to limit her physical activities to conserve her strength

control of BP

indicated when BP >160/110 need assessment of VS, I/O, DTR, level of consciousness continued 12-24 hrs postpartum resolution of preclampsia diuresis and decreased edema NSAID use carefully for pain monitor BP 72 hrs pospatrum + 7-10 days postpatum

endometritis

infection located in the lining the uterus most common postpartum infection usually begins as a localized infection at the placental side but can spread to involve the entire lining

Postpartum infection

is any clinical infection of the genital tract that occurs within 28 days after miscarriage, induced abortion, or birth. It continues to be a major cause of maternal morbidity and mortality throughout the world.

High maternal phenylalaline levels are associated with what

microcephaly, cognitive impairment, congenital heart defects

perineal lacerations

most common of all injuries in the low portion of the genital tract; they are classified as 1st, 2nd, 3rd, and 4th degree

gestational hypertension

onset of HT without proteinuria or other symptoms of preeclampsia after week 20 previously normal blood pressure systolic >140 diastolic >90 only one needs to be elevated to meet criteria doesnt persists longer than 12 weeks postpartum

teaching during the prenatal period.

operative obstetrics (e.g., forceps-assisted, vacuum-assisted, and cesarean births)

The benefits of a transvaginal ultrasonography allows the

pelvic anatomic features to be evaluated in greater detail AND intrauterine pregnancy to be diagnosed earlier no need for full bladder

placenta percreta

placenta goes all the way through the uterus

future health care preeclampsia

possibility in future pregnancies low dose aspirin have risk developing chronic hypertension, cardiovascular disease

early (acute, primary) late (secondary) postpartum hemorrhage (PPH)

postpartum hemorrhage can be classified as early if it occurs within 24 hours of the birth or late if it occurs more than 24 hours but less than 6 weeks after the birth

Complications of diabetes

premature atherosclerosis -retinopathy -nephropathy -neuropathy

predisposing conditions gestational hypertension

primigravida women younger than 19 and oldern than 40 chronic renal disease chronic hypertension diabetes milletus Rh incompatibility history or a family history of gestational hypertension

Risk factors for a second-trimester loss

race -ethnicity -poor outcomes in previous pregnancies -extremes of maternal age -severe dietary deficiencies -morbid obesity -regular or heavy alcohol use -excessive caffeine intake (about 500 mg daily) - bleeding in 1st trimester

What hemorrhagic disorders have the highest incidence of maternal mortality?

ruptured ectopic pregnancy and abrutpito placentae

placenta percreta

term used to refer to perforation of the uterus by the placenta

vulvar hematomas

the most common type of hematoma following a birth

specialized (detailed or targeted)

ultrasound examinations performed if a woman is suspected of carrying an anatomically or physiologically abnormal fetus

immediate birth indication in severe gestational hypertension with severe features

uncontrollable severe hypertension eclampsia pulmonary edema placental abruption DIC nonreassuring fetal status intrapartum fetal demise

cause of preeclapsia

unknown

forced cesarean birth

when medically needed and forced according to the court order

Dysfunctional labor is suspected

when there is an alteration in the characteristics of uterine contractions, a lack of progress in the rate of cervical dilation, or a lack of progress in fetal descent and expulsion.

What are early signs of hypovolemic shock?

woman may report "seeing stars," feeling dizzy or nauseated, she can become restless and orthopneic. As blood loss continues and woman has even further decreased blood flow, what symptoms might she have? Confusion, slow reaction to stimuli, headaches What is the most objective and least invasive assessment of adequate organ perfusion and oxygenation? Urine output of at least 30 ml/hr -

Causes of Postpartum Hemorrhage

■ Uterine atony ■ Laceration of the cervix or vagina ■ Hematoma development in the cervix, perineum, or labia ■ Retained placental fragments

Which statement by the nursing student about the management of reduced cervical competence (premature dilation of the cervix) in a pregnant patient indicates effective learning? "Progesterone supplementation is the only effective treatment." "An abdominal cerclage is performed at the first week of gestation." "Surgical treatment is ineffective in patients with extremely short cervix." "A prophylactic cerclage is used to constrict the internal os of the cervix."

"A prophylactic cerclage is used to constrict the internal os of the cervix." Rationale The best treatment option for premature dilation of the cervix is to surgically place a prophylactic cerclage to constrict the internal os of the cervix. It is usually placed at 11 to 15 weeks of gestation. Progesterone supplementation may not be effective in constricting the cervix and surgical intervention may be necessary. An abdominal cerclage is necessary in case of a failed vaginal cerclage and is usually placed at 11 to 13 weeks of gestation in patients by means of a laparotomy. In patients with an extremely short cervix, an abdominal cerclage is used, which is followed by a cesarean birth.

The antepartum nurse is caring for a patient who is scheduled for cesarean delivery. The patient reports anxiety about the procedure because she is unsure of her exact due date and whether the fetus is mature enough for cesarean delivery. Which is the best response made by the nurse to this patient? "The exact due date can be determined with a gynecological assessment." "Cesarean delivery can be performed after spontaneous onset of labor to verify gestational age." "Amniocentesis can be performed prior to cesarean delivery to verify that delivery is safe for your infant." "Ultrasound can be performed prior to cesarean delivery to verify that cesarean delivery on the scheduled date is safe for the infant."

"Amniocentesis can be performed prior to cesarean delivery to verify that delivery is safe for your infant." Amniocentesis may be done prior to cesarean delivery to verify fetal lung maturity for patients with questionable due dates.

The postpartum nurse is talking with the patient following a cesarean delivery with a Pfannenstiel skin incision and low transverse uterine incision. The patient asks the nurse to describe the procedure, since the surgical draping prevented her from seeing the operation. Which response from the nurse is appropriate? "An incision was made on the uterus that goes up toward your navel." "The layer of skin and fat is cut open and then a uterine incision is made." "Are you sure you're comfortable hearing details about the procedure?" "An incision was made on the uterus that looks a lot like the scar on your abdomen."

"An incision was made on the uterus that looks a lot like the scar on your abdomen." Low transverse uterine incision and Pfannenstiel skin incision are similar in appearance and location.

The nurse is caring for a client with premature rupture of membranes (PROM). How should the nurse instruct the client to manage the situation? "Consume excess amounts of fluids." "Assess fetal movement on a daily basis." "Monitor the skin for any discoloration." "Place yourself in Trendelenburg position."

"Assess fetal movement on a daily basis." Rationale The nurse should instruct a pregnant client with PROM to perform daily fetal movement counts. Reduction in fetal movements indicates fetal dysfunction. Clients who are administered tocolytic agents, such as nifedipine (Adalat), are instructed to consume excess fluids to prevent effects of vasodilatation. Consumption of excess fluids is unrelated to the management of PROM. Skin discoloration is observed in conditions like jaundice, but not in clients with PROM. The nurse places the client in Trendelenburg position if the client has symptoms of umbilical cord prolapse.

A woman at 36 weeks' gestation and in preterm labor is being sent home. Which statements about home management would the nurse include in the patient education? Select all that apply. "Avoid eating spicy foods." "Avoid sex during this time." "Only move around your house when necessary." "When lying down, make sure your feet are raised." "Notify the health care provider if your temperature is greater than 100° F."

"Avoid sex during this time." The woman experiencing preterm labor should avoid sexual intercourse and orgasm, both of which could speed labor. "Only move around your house when necessary." The nurse would advise the woman experiencing preterm labor to restrict her activity. "Notify the health care provider if your temperature is greater than 100° F." The woman experiencing preterm labor should be taught to take her temperature at least four times a day and report temperatures greater than 100° F to the health care provider, as a fever may indicate infection.

The quantitative human chorionic gonadotropin (β-hCG) levels are high in a client who is on methotrexate therapy for dissolving abdominal pregnancy. Which instruction does the nurse give to this client? "Avoid sexual activity." "Avoid becoming pregnant again." "Avoid feeling sad and low." "Take folic acid without fail."

"Avoid sexual activity." Rationale High β-hCG levels indicate that the abdominal pregnancy is not yet dissolved. Therefore the nurse advises the client to avoid sexual activity until the β-hCG levels drop and the pregnancy is dissolved completely. If the client engages in vaginal intercourse, the pelvic pressure may rupture the mass and cause pain. Abdominal pregnancy increases the chances of infertility or recurrent ectopic pregnancy in clients. However, the nurse need not instruct the client to avoid further pregnancy, because it may increase the feelings of sadness and guilt in the client. The nurse encourages the client to share feelings of guilt or sadness related to pregnancy loss. Folic acid is contraindicated with methotrexate therapy, because it may exacerbate ectopic rupture.

The nurse is educating a postpartum woman who is ready for discharge. Which instructions are appropriate regarding infection prevention? Select all that apply. "Change perineal pads frequently." "When you use the bathroom, wipe from front to back." "When your symptoms are gone, you can stop taking your antibiotics." "Dispose of used dressings in a container your infant cannot reach." "If you continue to feel nauseated or your fever comes back, call the health care provider."

"Change perineal pads frequently." Perineal pads should be changed frequently in order to prevent continued infection. Correct "When you use the bathroom, wipe from front to back." The patient should be instructed to wipe from front to back to prevent transfer of bacteria to the vaginal area. "Dispose of used dressings in a container your infant cannot reach." The patient should be educated to prevent infant contact with contaminated articles such as used dressings. Correct "If you continue to feel nauseated or your fever comes back, call the health care provider." Nausea and fever are symptoms of continued or worsening infection and should be reported to the health care provider

A woman is admitted to the Labor and Delivery unit with premature rupture of membranes at 31 weeks' gestation and is experiencing symptoms of preterm labor. The nurse has assessed the woman for signs of infection and is administering corticosteroids following an order from the health care provider. Which statement should the nurse make to explain why corticosteroids have been ordered? "Corticosteroids are administered to help delay labor." "Corticosteroids are administered to help "quiet" uterine activity." "Corticosteroids are administered to help fight infection in the mother." "Corticosteroids are administered to help speed your baby's lung maturity."

"Corticosteroids are administered to help speed your baby's lung maturity." The nurse should make this statement, as corticosteroids are administered to speed fetal lung maturity.

Which instructions does the nurse give to a client who is prescribed methotrexate therapy for dissolving the tubal pregnancy? "Discontinue folic acid supplements." "Get adequate exposure to sunlight." "Take stronger analgesics for severe pain." "Vaginal intercourse is safe during the therapy."

"Discontinue folic acid supplements." Rationale The nurse advises the client to discontinue folic acid supplements, because they interact with methotrexate and may exacerbate ectopic rupture in the patient. Exposure to sunlight is avoided, because the therapy makes the patient photosensitive. Analgesics stronger than acetaminophen are avoided, because they may mask symptoms of tubal rupture. Vaginal intercourse is avoided until the pregnancy is dissolved completely.

The nurse is preparing to administer terbutaline (Brethine) to a pregnant client who is in preterm labor. What questions should the nurse ask the patient before drug administration to promote drug safety? Select all that apply. "Do you experience urinary frequency?" "Do you ever have hyperthyroidism?" "Do you suffer from any cardiac disease?" "Do you suffer from nausea and vomiting?" "Do you have pregnancy-induced diabetes?"

"Do you ever have hyperthyroidism?" "Do you suffer from any cardiac disease?"" "Do you have pregnancy-induced diabetes?" Rationale Terbutaline (Brethine) is a beta-adrenergic agonist that relaxes the smooth muscles of the body. It adversely affects cardiac function. Therefore the nurse should ask about the client's history of heart disease. Terbutaline (Brethine) may cause hyperglycemia and should be avoided in clients with gestational diabetes. It should also be avoided in clients with hyperthyroidism. The drug does not affect the urinary function or the gastrointestinal function. Therefore, history related to urine frequency and nausea and vomiting are unrelated

What dietary instructions does the nurse give a client who has experienced miscarriage? "Increase folic acid and zinc in your diet." "Eat foods that are high in iron and protein." "Restrict fluid intake to 125 milliliters per hour." "Reduce dietary fat intake by 40 to 50 grams per day."

"Eat foods that are high in iron and protein." Rationale The nurse advises the client to eat foods that are high in iron and protein to help in tissue repair and red blood cell (RBC) replacement. Folic acid and zinc are more helpful in maintaining a pregnancy and are not needed after a miscarriage. Fluid intake is restricted in clients who are at risk for pulmonary edema. Dietary fat is reduced if the client has cholecystitis or cholelithiasis, because it may worsen epigastric pain.

In planning for an expected cesarean birth for a woman who has given birth by cesarean previously and who has a fetus in the transverse presentation, the nurse includes which information? "Because this is a repeat procedure, you are at the lowest risk for complications." "Even though this is your second cesarean birth, you may wish to review the preoperative and postoperative procedures." "Because this is your second cesarean birth, you will recover faster." "You will not need preoperative teaching because this is your second cesarean birth."

"Even though this is your second cesarean birth, you may wish to review the preoperative and postoperative procedures." Rationale "Even though this is your second cesarean birth, you may wish to review the preoperative and postoperative procedures." is the most appropriate response. "Because this is a repeat procedure, you are at the lowest risk for complications." is not accurate. Maternal and fetal risks are associated with every cesarean section. "Because this is your second cesarean birth, you will recover faster." is not accurate. Physiologic and psychologic recovery from a cesarean section are multifactorial and individual to each client each time. Preoperative teaching should always be performed regardless of whether the client has already had this procedure.

In planning for an expected cesarean birth for a woman who has given birth by cesarean section previously and who has a fetus in the transverse presentation, the nurse includes which information? "Because this is a repeat procedure, you are at the lowest risk for complications." "Even though this is your second cesarean birth, you may wish to review the preoperative and postoperative procedures." "Because this is your second cesarean birth, you will recover faster." "You will not need preoperative teaching because this is your second cesarean birth."

"Even though this is your second cesarean birth, you may wish to review the preoperative and postoperative procedures." The statement in B is most appropriate. The statements in A, C, and D are not accurate. Maternal and fetal risks are associated with every cesarean section. Physiologic and psychologic recovery from a cesarean section is multifactorial and individual to each client each time. Preoperative teaching should always be performed regardless of whether the client has already had this procedure.

The nurse is providing discharge teaching to a woman going home on anticoagulant therapy. Which instructions regarding medication use are appropriate? Correct "Every time you take this medicine, put a mark on your calendar." "If you miss a dose, take two doses at the next scheduled time." "You should be careful not to miss a lab appointment while taking this drug." "Write down the dose you are taking every time you get a new prescription." "If you have a headache or joint pain while taking this drug, you should take aspirin."

"Every time you take this medicine, put a mark on your calendar." The woman on anticoagulant therapy should be encouraged to develop a method of remembering when the last dose was taken. "You should be careful not to miss a lab appointment while taking this drug." The importance of repeated laboratory testing to regulate the dose of the anticoagulant should be included in discharge education. Correct "Write down the dose you are taking every time you get a new prescription." Encouraging the woman to keep track of the dose of medication they are taking is important in maintaining consistent care and should be included in discharge education.

What instruction does the nurse provide to a pregnant client with mild preeclampsia? "You need to be hospitalized for fetal evaluation." "Nonstress testing can be done once every month." "Fetal movement counts need to be evaluated daily." "Take complete bed rest during the entire pregnancy."

"Fetal movement counts need to be evaluated daily." Rationale Preeclampsia can affect the fetus and may cause fetal growth restrictions, decreased amniotic fluid volume, abnormal fetal oxygenation, low birth weight, and preterm birth. Therefore the fetal movements need to be evaluated daily. Clients with mild preeclampsia can be managed at home effectively and need not be hospitalized. Nonstress testing is performed once or twice per week to determine fetal wellbeing. Clients need to restrict activity, but complete bed rest is not advised, because it may cause cardiovascular deconditioning, muscle atrophy, and psychologic stress.

The nursing instructor is teaching nursing students about the daily fetal movement count. Which statement by a student indicates a need for further teaching? "Fetal movements decrease in clients who consume alcohol." "Fetal movements cannot be detected during the fetus sleep cycle." "Fetal movements cannot be easily detected in obese clients." "Fetal movements decrease 1 week before the expected date of delivery."

"Fetal movements decrease 1 week before the expected date of delivery." Rationale Fetal movements do not decrease one week before the expected date of delivery; rather they increase due to the development of limbs and muscles. Therefore, normal fetal movements can be detected as the pregnancy progresses to the expected date of delivery. Alcohol affects fetal growth and development and may reduce fetal movement. The fetus has regular sleep and wake cycles. During the fetal sleep cycle, fetal activity decreases, and due to this, fetal movements may not be detected. Due to excess deposition of subcutaneous fat near the abdomen in obese patients, fetal movements may be difficult to detect.

Place the patient statements in the order in which they would be made following fetal demise, based on the Miles and Demi Model of Parental Grief.

"I can't believe my baby has died." "It is my fault that my baby died; I work too many hours." "I just want to be alone; I am so sad that my baby died." "I look forward to getting pregnant again." Correct Answer Exactly! According to Miles and Demi Model of Parental Grief the first phase of grief of "acute distress" would align with the statement "I can't believe my baby has died." In the second stage of grief "intense grief" parents may desire to talk about the loss of their child or may simply want to be left alone to grieve by themselves. Thus, the statements "It is my fault that my baby died; I work too many hours" and "I just want to be alone; I am so sad that my baby died" both exemplify the "intense grief" stage of the Miles and Demi Model of Parental Grief. The final statement "I look forward to getting pregnant again" demonstrates "reorganization" because it demonstrates the family is ready to take an important step forward in their lives which is a trademark trait of a parent in the "reorganization" phase of the Miles and Demi Model of Parental Grief.

The patient is discussing with the nurse her preference for a vaginal delivery for her current pregnancy. Which statement by the patient indicates the need for further teaching? "I had general anesthesia during my last c-section." "Before my last c-section, the doctor tried to induce labor." "My ultrasound showed a lot of scar tissue from my last c-section." "I had a c-section with my last pregnancy because of placenta previa."

"I had a c-section with my last pregnancy because of placenta previa." Cesarean delivery due to placenta previa is typically done using a classic uterine incision, which eliminates the option for vaginal birth after cesarean (VBAC).

The nurse is teaching a client in the second trimester about fetal kick count. Which statement by the client needs correction? "I should count the fetal kicks once a day for 60 minutes." "I may not feel any movements when the fetus is sleeping." "I should count the fetal kicks either after meals or before bedtime." "I should consult an obstetrician if the fetal movements are less than 10 in one hour."

"I should consult an obstetrician if the fetal movements are less than 10 in one hour." Rationale The fetal movement count helps monitor the condition of the fetus. If the fetal movements are less than 3 in one hour, it indicates a risk of abnormality. Therefore, less than 10 fetal movements in an hour may not be abnormal and the client may not need to consult the obstetrician. The client should count fetal kicks at least once a day for a period of 60 minutes because it helps to assess fetal activity. Fetal movements are usually not present during the fetal sleep cycle. The client can count the fetal kicks either after meals or before bedtime when fetal movements are typically highest.

The nurse is teaching care precautions to a pregnant client who underwent cervical cerclage during the 2nd trimester. Which statement by the client needs correction? "I should avoid exercise and take complete bed rest for a few days." "I should avoid sexual intercourse until my next postoperative check up." "I should regularly check for the rupture of membrane and signs of infection." "I should consult the obstetrician if I have contractions 20 minutes apart."

"I should consult the obstetrician if I have contractions 20 minutes apart." Rationale The nurse should inform the client to consult the obstetrician if she has strong contractions that are less than 5 minutes apart. This instruction helps to provide immediate treatment and prevent the risk of miscarriage. Contractions that occur less than 5 minutes apart may not cause severe complications in the client. The client should avoid physical activity and take bed rest for few days after the placement of cerclage, because this helps prevent the rupture of sutures. The client should avoid sexual intercourse until the postoperative check up in order to prevent a risk of infection. The client should regularly check for the rupture of the membrane and signs of infection to prevent complications.

The nurse is preparing to discharge a 30-year-old woman who has experienced a miscarriage at 10 weeks of gestation. Which statement by the woman indicates a correct understanding of the discharge instructions? "I will not experience mood swings, because I was only at 10 weeks of gestation." "I will avoid sexual intercourse for 6 weeks and pregnancy for 6 months." "I should eat foods that are high in iron and protein to help my body heal." "I should expect the bleeding to be heavy and bright red for at least 1 week."

"I should eat foods that are high in iron and protein to help my body heal." Rationale A woman who has experienced a miscarriage should be advised to eat foods that are high in iron and protein to help replenish her body after the loss. After a miscarriage a woman may experience mood swings and depression from the reduction of hormones and the grieving process. Sexual intercourse should be avoided for 2 weeks or until the bleeding has stopped, and pregnancy should be avoided for 2 months. The woman should not experience bright red, heavy, profuse bleeding; this should be reported to the health care provider.

The nurse is teaching a pregnant client how to recognize signs of preeclampsia and when to report to the primary health care provider. Which statements by the client indicate effective learning? Select all that apply. "I should report if I see an increase in urinary output." "I should report if a dipstick test shows proteinuria less than 1+." "I should report if I experience blurred vision or headache." "I should report if I feel a decrease in the baby's movements." "I should sit and use my right arm to accurately measure my blood pressure."

"I should report if I experience blurred vision or headache." "I should report if I feel a decrease in the baby's movements." "I should sit and use my right arm to accurately measure my blood pressure." Rationale The pregnant client should report to the primary health care provider if she experiences blurred vision, dizziness, and headache. These are the common clinical signs of preeclampsia. The client should report to the primary health care provider if she observes fewer fetal movements per hour, because it may be indicative of fetal compromise due to preeclampsia. To obtain accurate recordings, the pregnant client should use her right arm while in a sitting position to measure her blood pressure. The pregnant client should report to the primary health care provider in case of decreased urinary output, because a decrease in the glomerular filtration rate leads to degenerative glomerular changes and oliguria. The pregnant client should inform the primary health care provider if a dipstick test shows the value of 1+ or more, because it indicates proteinuria, an important sign of preeclampsia.

The nurse is teaching a pregnant client how to recognize signs of preeclampsia and when to report to the primary health care provider. Which statements by the client indicate effective learning? Select all that apply. "I should report if I see an increase in urinary output." "I should report if a dipstick test shows proteinuria less than 1+." "I should report if I experience blurred vision or headache." "I should report if I feel a decrease in the baby's movements." "I should sit and use my right arm to accurately measure my blood pressure."

"I should report if I experience blurred vision or headache." "I should report if I feel a decrease in the baby's movements." "I should sit and use my right arm to accurately measure my blood pressure." Rationale The pregnant client should report to the primary health care provider if she experiences blurred vision, dizziness, and headache. These are the common clinical signs of preeclampsia. The client should report to the primary health care provider if she observes fewer fetal movements per hour, because it may be indicative of fetal compromise due to preeclampsia. To obtain accurate recordings, the pregnant client should use her right arm while in a sitting position to measure her blood pressure. The pregnant client should report to the primary health care provider in case of decreased urinary output, because a decrease in the glomerular filtration rate leads to degenerative glomerular changes and oliguria. The pregnant client should inform the primary health care provider if a dipstick test shows the value of 1+ or more, because it indicates proteinuria, an important sign of preeclampsia.

A patient who has experienced a fetal demise declines seeing the infant after birth. Which response by the nurse is appropriate? "You will regret not seeing your baby." "I agree; it is best if you do not see your baby." "I understand. If you would like change your mind, please let me know." "For you to grieve properly, you must see your baby as soon as possible."

"I understand. If you would like change your mind, please let me know." Nurses must accept that each family needs to go through this difficult experience in their own way, and this may include not seeing the infant. This is the appropriate response by the nurse.

The nurse is teaching a group of pregnant clients about preterm labor and the actions to take if the signs and symptoms of preterm labor develop. Which patient statement indicates the need for further teaching? "I will empty my bladder immediately." "I will drink 2 to 3 glasses of water or juice." "I will lie in the supine position for 1 hour." "I will go to hospital if symptoms continue."

"I will lie in the supine position for 1 hour." Rationale If there are signs and symptoms of preterm labor, the client should lie down on her side for 1 hour, because it helps improve placental and fetal circulation. The client should empty her bladder immediately, because a full bladder may sometimes irritate the uterus. Dehydration may also irritate the uterus. Therefore, the client should drink 2 to 3 glasses of water or juices. The patient should go to the hospital if the symptoms of preterm labor do not subside.

The nurse is teaching a group of pregnant clients about preterm labor and the actions to take if the signs and symptoms of preterm labor develop. Which patient statement indicates the need for further teaching? "I will empty my bladder immediately." "I will drink 2 to 3 glasses of water or juice." "I will lie in the supine position for 1 hour." "I will go to hospital if symptoms continue."

"I will lie in the supine position for 1 hour." Rationale If there are signs and symptoms of preterm labor, the client should lie down on her side for 1 hour, because it helps improve placental and fetal circulation. The client should empty her bladder immediately, because a full bladder may sometimes irritate the uterus. Dehydration may also irritate the uterus. Therefore, the client should drink 2 to 3 glasses of water or juices. The patient should go to the hospital if the symptoms of preterm labor do not subside.

A woman with deep vein thrombosis complains of soreness in the affected leg and asks for assistance ambulating. Which response from the nurse is appropriate? "I will go get another person to assist us." "I will hold you steady while you stand up on your own." "I will need to look at your leg before you get out of bed." "I will get you a dose of your pain medicine before you get up."

"I will need to look at your leg before you get out of bed." A woman with deep vein thrombosis should remain on bed rest until symptoms have disappeared to avoid dislodging the thrombus.

Once home, the first-time parents of a premature neonate find the neonate running a fever. The parents also have a feeling the neonate isn't doing well, so they call the nurse for guidance. How would the nurse address the parent's concerns? "I will notify the health care provider immediately." "Dress your baby in fewer layers to avoid overheating." "It's not uncommon for first-time parents to get nervous." "You can give the infant an anti-pyretic to reduce the fever."

"I will notify the health care provider immediately." An abnormal temperature and a general feeling of the infant being unwell could be signs of sepsis and should be immediately reported to the health care provider.

The nurse is caring for a patient who sustained a third-degree perineal laceration during delivery. The patient reports pain with defecation and asks if anything can be done. Which response from the nurse is appropriate? "Increasing your fluid intake will make your stool much softer." "I will obtain an order for an enema to make things easier for you." "I will request an order for stool softeners to help you pass stools more comfortably." "I can give you some of the hydrocodone bitartrate and acetaminophen the health care provider ordered to help with the pain."

"I will request an order for stool softeners to help you pass stools more comfortably." Stool softeners are recommended for patients with perineal lacerations to help relieve pain with defecation and ensure the patient maintains continence.

Which statements indicate to the nurse that a pregnant woman is at risk for development of ketoacidosis? Select all that apply. "I avoid eating any type of red meat." "I've had a cold for the last 2 weeks." "My mother has had diabetes for 20 years." "I can never remember to take my insulin." "I only drink water with lemon when I'm thirsty."

"I've had a cold for the last 2 weeks." Infection predisposes women to ketoacidosis. "I can never remember to take my insulin." Missing doses of insulin predisposes women to ketoacidosis.

A neonate is being discharged home after a surgical repair of a cardiac defect. The nurse is teaching the parents about signs of post-surgical pain in the newborn. What statement by the parents indicates effective teaching by the nurse? "A low, deep cry is a sign of pain." "Not making eye contact can be a sign of pain." "Neonates do not experience or remember pain." "If my baby is showing weak or tense muscles, it may be a sign of pain."

"If my baby is showing weak or tense muscles, it may be a sign of pain." Tense, rigid, or flaccid muscles can be a sign of neonate pain, as well as crying, whimpering, or moaning.

The nursing instructor is teaching a group of students about the Magnetic Resonance Imaging (MRI) scan. Which statement made by a student indicates the need for further teaching? "The client should stay completely still during the scan." "MRI shows accurate fetal anatomic details." "MRI provides both a biophysical and a biochemical profile of the mother and fetus." "The patient will not be exposed to any ionizing radiation or injected iodine dye during the scan."

"MRI shows accurate fetal anatomic details." Rationale MRI takes 20 to 60 minutes, depending on the reason for study. Accurate anatomic details would not be obtained, because there is a possibility of fetal movement. Theclient should be completely still during the procedure, with short breaks, because physical movement would alter the effectiveness of the scan. MRI helps evaluate the fetal structure, overall growth, biochemical status, and the placenta. MRI is a noninvasive procedure. Unlike with a computed tomography (CT) scan, iodine dye would not be injected, and MRI does not use ionizing radiation.

The partner of a woman experiencing postpartum anxiety disorder asks how to best help the woman and newborn after discharge. Which nursing education is appropriate? "Be careful letting the woman hold the newborn until you're sure it's safe." "Make sure the woman is taking all of the prescribed medications on time." "You should let the woman prepare meals, so it will be something the woman enjoys." "You should let the woman handle the majority of newborn care to help build confidence."

"Make sure the woman is taking all of the prescribed medications on time." Adhering to the prescribed medication dosage and schedule is important in treating postpartum anxiety and is appropriate teaching for the woman's partner.

Which statement made by the woman to the nurse demonstrates her understanding of her gestational diabetes mellitus (GDM) diagnosis? Select all that apply. "I can only control my GDM through insulin." "My diagnosis of GDM will last through the baby's delivery." "GDM can occur because pregnancy reduces my glucose sensitivity." "A diagnosis of GDM means I could develop type 2 diabetes in 10-20 years." "Because I have been diagnosed with GDM, I will be diagnosed with preeclampsia too."

"My diagnosis of GDM will last through the baby's delivery." This woman understands that a diagnosis of GDM is related to the baby's gestation and will end when the gestation ends. Correct "GDM can occur because pregnancy reduces my glucose sensitivity." This woman understands that her baby is drawing glucose from her blood and is thus lowering her glucose sensitivity. Correct "A diagnosis of GDM means I could develop type 2 diabetes in 10-20 years." This woman understands that a diagnosis of GDM increases her risk of developing type 2 diabetes mellitus later in life.

Which statements by a woman about personal health history indicate an increased risk for postpartum depression? Select all that apply. "This is my first pregnancy." "I had a lot of trouble conceiving." "My father has depression, but I don't." "I take medication for anxiety sometimes." "I plan on giving up my baby for adoption." "I was diagnosed with diabetes 2 years ago."

"My father has depression, but I don't." Family history of depression increases a woman's risk for postpartum depression. Correct "I take medication for anxiety sometimes." Personal history of mental illness increases a woman's risk for postpartum depression. "This is my first pregnancy." A nullipara is at an increased risk for postpartum depression. "I was diagnosed with diabetes 2 years ago." Women who experience medical problems during pregnancy, such as preexisting diabetes mellitus, are at increased risk for postpartum depression.

What does the nurse instruct a pregnant client who is diabetic to do? "Perform exercises just before meals." "Perform exercises with a partner." "Stop exercising if contractions occur." "Lie supine if contractions occur." "Exercise at the same time every day."

"Perform exercises with a partner." "Stop exercising if contractions occur." "Exercise at the same time every day." Rationale The nurse should instruct the client to exercise with a partner as a safety measure. The client should be instructed to stop exercising immediately if contractions occur. The client should perform exercises, eat meals, and take insulin at the same time every day. It is best to perform exercises after meals, when the blood glucose levels are rising. If contractions occur, the client should drink two to three glasses of water and lie down on her side for an hour. If contractions do not cease, the client should contact the primary health care provider.

A nurse is caring for a patient that recently experienced an ectopic pregnancy. Which is the most appropriate statement made by the nurse? "You can have other children." "These things happen for the best." "You are so lucky it happened early." "Please tell me how you are feeling about your loss."

"Please tell me how you are feeling about your loss." Acknowledging that the patient had a loss and encouraging the patient to discuss feelings is part of supportive nursing care.

The nurse is assessing the fundus of a patient who had a cesarean delivery. The patient reports abdominal discomfort during palpation. Which instructions should the nurse give the patient regarding abdominal pain during fundal assessment? "Turn to your side." "Empty your bladder before the fundal assessment." "Relax and breathe slowly during the fundal assessment." "Take deep breaths and hold them during the fundal assessment."

"Relax and breathe slowly during the fundal assessment." Relaxing the abdominal muscles and taking slow, deep breaths can help prevent discomfort during abdominal assessment.

While performing a contraction stress test in a pregnant client, the nurse finds that the client has three uterine contractions in a 10-minute period with no significant variable decelerations. The nurse communicates the test findings to the primary health care provider. Which instruction would the nurse expect to receive from the primary health care provider? "Repeat the test in the client the next day." "Administer intravenous fluids to the client." "Immediately admit the client to the hospital." "Resume a routine weekly testing schedule for the client."

"Resume a routine weekly testing schedule for the client." Rationale A contraction stress test helps determine fetal heart rate during uterine contraction. If the client has three uterine contractions within 10 minutes, with no significant deceleration, it indicates normal or negative results and the fetus is doing well. Therefore, the primary health care provider would resume a routine weekly testing schedule for the client. If three contractions are not produced within 10 minutes and no fetal heart rate is heard, then the test will be repeated the next day. Presence of uterine contractions is a normal finding, and it does not indicate risk of dehydration. Therefore, the primary health care provider will not instruct the nurse to administer intravenous fluids to the client. If the client has late decelerations with 50% or more of contractions, then the primary health care provider would instruct the nurse to admit the client for further evaluation.

The nurse is caring for a postpartum client with preeclampsia. What teaching does the nurse give the client before discharge from the hospital? "Return to the hospital if you have epigastric pain." "Take an analgesic if you have epigastric pain." "If you get a headache, relax. It will subside in a while." "Get an eye exam done if you have blurred vision."

"Return to the hospital if you have epigastric pain." Rationale A postpartum client with preeclampsia may have headache, visual disturbances, and epigastric pain. The epigastric pain may worsen the client's condition if left untreated, and therefore, the nurse should ask the client to return to the hospital or contact the health care provider. The client should not take analgesics unassisted because delaying the appropriate clinical treatment for the epigastric pain may worsen the symptoms of preeclampsia. The client should immediately report to the primary health care provider if she experiences headache and blurred vision, because these conditions may indicate worsening of the preeclampsia or indicate other reasons for concern.

Which statement by the nursing student about the management of molar pregnancy indicates effective learning? "Methotrexate therapy is prescribed to abort molar pregnancy." "Expectant management is initiated as per the amount of bleeding." "Suction curettage is the safest way of terminating molar pregnancy." "Induction of labor with oxytocic agents is one of the treatment options."

"Suction curettage is the safest way of terminating molar pregnancy." Rationale In molar pregnancy the avascular transparent vesicles in the uterus may cause uterine distention. Therefore suction curettage is used for rapid and effective evacuation of the hydatidiform mole. Methotrexate therapy is prescribed to dissolve an ectopic pregnancy. Expectant management is initiated in case of a normal fetus, not in the case of a molar pregnancy. Induction of labor with oxytocic agents is not a safe method, because it has a risk of embolization of trophoblastic tissue.

The nurse is assessing a patient who had a cesarean delivery. The patient is silent during the physical assessment, but when asked how she feels responds, "I feel like I can't do anything right. How am I supposed to raise a child?" Which response from the nurse is most appropriate? "Tell me why you feel this way." "I can bring your baby in from the nursery so the two of you can bond." "Everyone is concerned about their first child. I can give you some printed material that can help." "I can give you contact information for a support group for new parents, if you feel overwhelmed."

"Tell me why you feel this way." The nurse should open a dialogue to understand the patient's concerns. Often, patients can feel a sense of inadequacy following cesarean delivery due to failed expectations of having a normal vaginal birth.

Which patient statement indicates symptoms of a wound infection? "The cut inside my vagina hurts constantly." "My episiotomy hurts when I use the bathroom." "My cesarean scar forms a raised line on my skin." "My cesarean scar is a lighter color than the rest of my skin."

"The cut inside my vagina hurts constantly." Constant pain around a wound is a potential sign of infection and should be reported to the health care provider.

The antepartum nurse is caring for a patient with a body mass index (BMI) of 30 who is scheduled for a planned cesarean delivery. The patient asks the nurse why the provider has told her that a midline vertical incision will be done for the procedure. Which explanation by the nurse is accurate? "The midline vertical incision is less likely to lead to hernia formation later in life." "The midline vertical incision will heal more quickly than a Pfannenstiel incision." "The midline vertical incision will cause less postoperative pain and allow for earlier ambulation." "The midline vertical incision is a better option for larger patients because more of the uterus is exposed."

"The midline vertical incision is a better option for larger patients because more of the uterus is exposed." For obese patients (BMI ≥ 30), a midline vertical incision is optimal because it exposes more of the uterus and allows for safer delivery.

A patient in labor is in visible distress and reports fear of pushing too hard and "tearing something." Which response from the nurse is appropriate? "Cervical ripening may help ensure the tissue does not tear." "If you feel tearing sensations, you should not push any harder." "If you feel like you might tear something, I may need to request the health care provider schedule you for cesarean delivery." "The team can't guarantee that you won't have some tearing, but your body will stretch to accommodate the baby."

"The team can't guarantee that you won't have some tearing, but your body will stretch to accommodate the baby." It is not certain that stretching will prevent tearing, but stretching is sure to occur and will greatly reduce the tearing that would occur if the patient's perineal tissues could not stretch.

The antepartum nurse is caring for a patient carrying twins. While educating the patient about what to expect during labor, the patient states, "I thought all twins were delivered by C-section." Which response from the nurse is appropriate? "Cesarean delivery is usually started after labor has begun." "There is a very realistic possibility that the twins may be delivered vaginally." "Many health care providers prefer to deliver one baby vaginally and one by cesarean section." "That's correct. The latest evidence shows that cesarean delivery is the safest option for multiple births."

"There is a very realistic possibility that the twins may be delivered vaginally." Although twins are often delivered by cesarean delivery, it is possible that delivery may be vaginal. It is also important that the nurse know the plan of care and history of the patient so that he or she does not contradict a patient-centered plan that was already in place.

A patient having a scheduled cesarean delivery asks the nurse why the provider plans to use a Pfannenstiel incision (transverse skin incision). Which is the most accurate response by the nurse? "There is usually a longer recovery time with a Pfannenstiel incision." "There likely will be less post-operative pain with a transverse incision." "The transverse incision allows for more space and better access to the fetus." "The transverse incision allows for faster uterine access, thus speeding up the delivery."

"There likely will be less post-operative pain with a transverse incision." A low transverse incision causes less disruption of abdominal wall musculature, and thus is less painful than a vertical, midline incision.

The parents of a neonate who had meconium-stained fluid and respiratory distress tell the nurse "We don't feel prepared to care for our baby at home". Which statement by the nurse demonstrates appropriate knowledge of how to respond? "Don't worry. Your baby will be fine." "We can review basic infant care before you go home." "This hospital will allow you to spend the night here and care independently for your baby." "Parents often feel this way. You will eventually become comfortable caring for your baby."

"This hospital will allow you to spend the night here and care independently for your baby." Some hospitals allow parents to care for their neonate independently while still in the hospital to prepare them for the care they will provide at home. This would be an appropriate response if the hospital had such a policy.

After delivery with a vacuum extractor, a patient asks the nurse about the red, swollen area on the side of the infant's head. Which response from the nurse is appropriate? "This is most likely the spot where the vacuum extractor was placed." "This is where the baby's head pressed up against the pelvis during extraction." "This may be a sign of a serious complication. I will notify the health care provider." "This occurs naturally because the newborn's head is compressed during delivery."

"This is most likely the spot where the vacuum extractor was placed." The vacuum extractor forms a round, edematous spot on the infant's scalp, called "a chignon," which resolves quickly after birth.

Which statements by the nurse are the most appropriate to make to a patient who has recently had a miscarriage? Select all that apply. "When it is meant to happen, it will happen." "Miscarriage is very common. You can try again." "Don't worry; nature has a way of taking care of mistakes. It is for the best." "This is often a consequence of unavoidable factors and it is important to realize this is not your fault." "I am very sorry for your loss. Having a miscarriage must be so emotionally painful. How are you feeling?"

"This is often a consequence of unavoidable factors and it is important to realize this is not your fault." The nurse can help the patient by emphasizing that most spontaneous abortions are due to unavoidable factors or abnormalities. "I am very sorry for your loss. Having a miscarriage must be so emotionally painful. How are you feeling?" Acknowledging the patient's pregnancy loss and associated emotional pain is part of supportive nursing care.

A woman who is at 36 weeks of gestation is having a nonstress test. Which statement indicates her correct understanding of the test? "I will need to have a full bladder for the test to be done accurately." "I should have my husband drive me home after the test because I may be nauseated." "This test will help to determine whether the baby has Down syndrome or a neural tube defect." "This test observes for fetal activity and an acceleration of the fetal heart rate to determine the well-being of the baby."

"This test observes for fetal activity and an acceleration of the fetal heart rate to determine the well-being of the baby." The nonstress test is one of the most widely used techniques to determine fetal well-being and is accomplished by monitoring fetal heart rate in conjunction with fetal activity and movements. An ultrasound requires a full bladder. An amniocentesis is the test after which a pregnant woman should be driven home. A maternal serum alpha-fetoprotein test is used in conjunction with unconjugated estriol levels and human chorionic gonadotropin helps to detect Down syndrome.

A woman who is at 36 weeks of gestation is having a nonstress test. Which statement by the woman indicates a correct understanding of the test? "I will need to have a full bladder for the test to be done accurately." "I should have my husband drive me home after the test because I may be nauseous." "This test will help to determine if the baby has Down syndrome or a neural tube defect." "This test will observe for fetal activity and an acceleration of the fetal heart rate to determine the well-being of the baby."

"This test will observe for fetal activity and an acceleration of the fetal heart rate to determine the well-being of the baby." Rationale The nonstress test is one of the most widely used techniques to determine fetal well-being and is accomplished by monitoring fetal heart rate in conjunction with fetal activity and movements. An ultrasound is the test that requires a full bladder. An amniocentesis is the test after which a pregnant woman should be driven home. A maternal alpha-fetoprotein test is used in conjunction with unconjugated estriol levels, and human chorionic gonadotropin helps to determine Down syndrome.

A nurse is teaching the parents of a neonate with a high-risk condition to distinguish between overhydration and dehydration. Which statement by the parents indicates that they understand the signs of dehydration? "Three wet diapers a day could mean dehydration." "Swelling in my baby's face could mean dehydration." "A greater than expected weight gain could indicate dehydration." "If my baby has difficulty breathing, this could be a sign of dehydration."

"Three wet diapers a day could mean dehydration." After the first week of life, it is expected that a neonate will have 5-6 wet diapers per day. Fewer than that could mean dehydration.

The nurse is assessing a woman with a deep vein thrombosis. While the nurse measures the circumference of both legs, the woman asks why the measurements are necessary. Which response is appropriate? "We measure where the blood vessels are in each leg." "The measurements let us know where the clot is located." "We need to know what size compression stockings you would need." "We use measurements to determine the amount of swelling in the affected leg."

"We use measurements to determine the amount of swelling in the affected leg." Leg measurements for women with deep vein thrombosis are used to compare the affected leg to the unaffected leg, in order to determine the level of swelling in the affected leg.

A nurse is teaching the parents of a high-risk neonate to monitor during feedings for nipple readiness. Which statement by the parents indicates that the parents need more teaching? "Drooling is a sign my baby is feeding intolerant." "Coughing is a sign my baby is feeding intolerant." "Sucking on a pacifier is a sign my baby is ready to feed." "When my baby falls asleep early during feeding, it means he is already satisfied."

"When my baby falls asleep early during feeding, it means he is already satisfied." Falling asleep early during feeding is a sign of feeding intolerance. This statement by the parents indicates that more teaching is needed.

The parents of a premature newborn ask the nurse the age at which the newborn will start to turn over. Which statement by the nurse demonstrates appropriate knowledge of how to respond? "Most infants turn over between 4 and 6 months of age." "It will be difficult to know exactly when that will happen." "It will be earlier than a term infant because she was premature." "When your baby's corrected age is between 4 to 6 months of age."

"When your baby's corrected age is between 4 to 6 months of age." Parents should base the appearance of milestones on the corrected age of the neonate, not the chronological age.

The antepartum nurse is caring for a patient with a history of one cesarean birth via low transverse incision. The patient asks the nurse what her chances are of a successful vaginal birth for the current pregnancy. Which is the appropriate nursing education for this patient? "Because of the risk of cervical laceration with low transverse incisions, vaginal birth after cesarean is not recommended." "With low transverse incisions, vaginal birth after cesarean is a possibility, because of a decreased risk of incision rupture." "Because of the increased risk of infection, vaginal birth after cesarean is not indicated for patients with a history of low transverse incision." "Vaginal birth after cesarean is not a possibility for patients with a history of low vertical incision, because the uterine incision may lead to lots of scar tissue."

"With low transverse incisions, vaginal birth after cesarean is a possibility, because of a decreased risk of incision rupture." Vaginal birth after cesarean is a possibility for patients with a history of low transverse incisions due to a decreased risk of uterine rupture.

A client with gestational hypertension is prescribed labetalol hydrochloride (Normodyne) therapy, which is continued after giving birth. What does the nurse instruct the client about breastfeeding? "You may breastfeed the infant if you desire." "Breastfeeding may cause convulsions in the infant." "Breastfeed only once a day and use infant formulas." "There may be high levels of the drug in the breast milk."

"You may breastfeed the infant if you desire." Rationale Labetalol hydrochloride (Normodyne) has a low concentration in breast milk so the client can breastfeed the infant. Breastfeeding is safe and will not cause convulsions or any side effects in the infant. Infant formulas are used only if the mother is unable to breastfeed the infant or if the mother does not desire breastfeeding.

The partner of a patient who had a fetal demise is withdrawn and appears angry. Which statements by the nurse are appropriate? Select all that apply. "Anger is part of grief. It's normal." "You may feel like you must be strong." "You seem upset. Would you like to talk?" "I am so sorry about the loss of your baby." "It can be hard to talk about the loss of your baby."

"You may feel like you must be strong." Being sensitive to the feelings of the patient's partner provides an opportunity to share feelings. Correct "You seem upset. Would you like to talk?" Being sensitive and offering availability on the part of the nurse provides an opportunity for the partner to share feelings. Correct "I am so sorry about the loss of your baby." Being honest and truthful to your feelings about the situation provides an opportunity for the partner to share feelings. Correct "It can be hard to talk about the loss of your baby." Acknowledging emotions or feelings of the patient's partner is therapeutic and it provides an opportunity for to share feelings.

Which statements should be included in discharge education for a patient who has experienced a postpartum hemorrhage? Select all that apply. "You should start feeling better in a few days." "You should get out of bed slowly, especially if you feel dizzy." "You may need help caring for your infant while you recover." "You should not limit your activity, or it may take you longer to recover." "If your temperature is elevated, or you start feeling achy, you should call your health care provider immediately."

"You should get out of bed slowly, especially if you feel dizzy." Patients with postpartum hemorrhage are at risk for orthostatic hypotension and should be instructed to sit up slowly when getting out of bed. "You may need help caring for your infant while you recover." Patients with postpartum hemorrhage may need assistance caring for the infant, and attachment to the infant may be hindered by fatigue. "If your temperature is elevated, or you start feeling achy, you should call your health care provider immediately." Patients with postpartum hemorrhage should be instructed to monitor for signs of postpartum infection.

A patient whose labor was complicated by shoulder dystocia reports that one of her infant's arms is "floppy." The nurse examines the infant and finds flaccid muscle tone on one side. Which is the most appropriate nursing education for this patient? "This is a normal effect of shoulder dystocia and should resolve in a few days." "Your health care provider will likely recommend physical therapy to treat this injury." "This could be a symptom of a permanent neurologic condition. I will notify the health care provider immediately." "Your baby's collarbone may have fractured as a result of shoulder dystocia. I will notify the health care provider immediately."

"Your health care provider will likely recommend physical therapy to treat this injury." Flaccid muscle tone in one arm is a presentation of Erb's palsy, or nerve injury to the brachial plexus. Most cases resolve in a few weeks, but physical therapy may be started in the immediate postpartum period. The nurse is justified in providing this anticipatory guidance to the patient on the basis of the health care provider's most likely recommendation.

The parents of a 29 weeks' gestation neonate who is now 3 weeks old asked if they will be able to take their infant home next week. Which statement by the nurse demonstrates appropriate knowledge of how to respond? "Your infant will need to weigh 4 pounds before going home." "Your infant will need to be free of all monitors before going home." "Your infant will need to feed without difficulty before going home." "Your infant will need to only have 1-2 apneic episodes a day before going home."

"Your infant will need to feed without difficulty before going home." The premature neonate must be able to feed without cardiorespiratory compromise.

Management of Hemorrhagic (hypovolemic) Shock

*classic signs of shock may not occur until woman has already lost 30-40% of her blood volume Assess: -pulse- rapid, weak, and irregular -respirations- rapid and shallow -blood pressure- decreasing (late sign) -O2 sat -Skin (temperature, turgor, color)- cool, pale, clammy -Urinary output-decreasing -LOC--> lethargy and coma -mental status- anxiety and then coma -central venous pressure- decreased Interventions: -Summon assistance and equipment -Start IV infusion per standing orders to replace circulating fluid volume -Ensure patent airway -Administer O2 -Continue to monitor status

Nsg interventions for anemia?

- Monitor H&H levels every 2 weeks, - administer and instruct about iron and folic acid supplements, - instruct client to take iron with a source of vitamin C to increase its absorption and to avoid taking iron with tea,, milk, caffeine . best taken between meals - instruct client to eat foods high in iron, folic acid, and protein, - teach client to monitor for S/S of infection - Parenteral iron and possible blood transfusion, as prescribed for severe anemia - administer oxytiocin postpartum if excessive bleeding is a concern?

Common causes of early miscarriage

-25%- chromosomal abnormalities -endocrine imbalance (e.g. hypothyroidism, poorly controlled insulin-dependent diabetes), - systemic disorder- lupus - genetic fsctors - immunologic - antiphospholipid antibodies Are infections a common cause of early miscarriage? no, except varicella in 1st trimester

Grade 2 Placental Abruption

-Absent to moderate bleeding -1000-1500 mL of blood lost -dark red blood -milk shock -occasional DIC -increased uterine tone- uterus fails to relax between contractions -pain= present -normal location of placenta -abnormal fetal heart rate and pattern -20-50% separation

Grade 3 Placental Abruption

-Absent to moderate bleeding ->1500 mL of blood lost -dark red blood -shock is common, profound -frequent DIC -tetanic contractions- boardlike uterus -agonizing, unremitting uterin pain -normal location of placenta -engaged fetal part -abnormal heart rate and pattern -fetal death can occur ->50% separation

Contraindications for Labor Induction

-Acute, severe fetal distress -Shoulder presentation (transverse lie) -Floating fetal presenting part -Uncontrolled hemorrhage -Placenta previa -Previous uterine incision that prohibits a trial of labor Relative Contraindications (not really recommended) -5+ pregnancies that ended after 20 weeks -multiple gestation -suspected CPD -breech presentation -inability to adequately monitor FHR throughout labor

Insulin Therapy During Pregnancy

-Administer short-acting insulin before each meal and then longer-acting insulin at bedtime Weeks 3-7- insulin requirements are increased Weeks 7-15- insulin requirements are decreased Second and Third Trimesters- insulin must be increased (up until 36 weeks) Classifications of insulin -Rapid acting -short acting (regular) -intermediate acting -Long acting Where should unused insulin vials be stored?In the refrigerator -should never be left in a car or exposed to extreme heat What is the preferred injection site for insulin? why? what are other alternative sites? The abdomen because it is best absorbed there Other possible sites- upper outer arm, thighs, and buttocks Each insulin injection should be given at least ____ from the previous injection? 90 degrees angle 2 inches and then they should move to another quadrant for next injections

Fetal Risks Associated with Postterm Pregnancy

-Aging placenta- due to calcium deposits in tissue -decreased amniotic fluid (oligohydramnios) -meconium-stained amniotic fluid- increased risk of meconium aspiration -Shoulder dystocia Postmaturity Syndrome -Dry, cracked, peeling skin -Long nails -meconium staining of skin, nails, and umbilical cord -loss of subcutaneous fat and muscle mass

Measures of Cardiac Output in Postpartum Women with Excessive Bleeding

-Arterial pulse -Auscultation of heart sounds and breath sounds -Inspection of skin color, temperature, turgor -LOC -Capillary refill -Neck veins -Mucous membranes -Presence or absence of anxiety, apprehension, restlessness, disorientation -Blood pressure -Pulse ox -Urinary output

Assessment of Mom and Fetus during Pitocin administration

-Assess FHR tracing every 15 minutes and with every change of dose -Monitor blood pressure, pulse, and respirations every 30-60 minutes -Assess intake and output- limit IV intake to 1000 ml/8 hrs -Urine output should be 120 mL or more every 4 hours -monitor for side effects including nausea, vomiting, headahce, and hypotension

Management of a Newborn with Meconium-Stained Amniotic Fluid- immediately after birth

-Assess the baby's respiratory efforts, heart rate, and muscle tone -Suction only the baby's mouth and nose, using either a bulb syringe or a large bore suction catheter if the baby has strong respiratory efforts, good muscle tone, and heart rate > 100 bpm -suction the trachea using an ET tube connected to a meconium aspiration device

Nursing Interventions for DIC

-Assessment for signs of bleeding -Assessment for complications from the administration of blood and blood products -administering fluid or blood replacement as ordered -cardiac and hemodynamic monitoring -protecting the woman from injury -closely monitoring urinary output because renal failure can occur (must be 30 ml/hr) -assess vital signs frequently -keep woman in side-lying tilt to maximize blood flow to uterus DIC cured by birth?

Cervidil

-Avoid use in women with asthma, glaucoma, and hypotension or hypertension -Should not be used if patient has cardiac, renal, or hepatic disease

What to expect after placement of a cerclage

-Bed rest for a few days -Advise her to avoid sexual intercourse at least until a post-op check -Explain what she should watch for and report if preterm labor, rupture of membranes, or infection or suspected -make sure she knows the signs that would warrant an immediate return to the hospital- strong contractions <5 mins apart, preterm PROM, severe perineal pressure, and an urge to push

Hydatidiform Mole (Molar Pregnancy)

-Benign proliferative growth of the placental trophoblast in which the chorionic villi develop into edamatous, cystic, avascular transparent vesicles that hang in a grapelike clster -A type of gestational trophoblastic disease (GTD)

What to do if prolapsed umbilical cord is detected?

-Call for assistance- do not leave woman alone -Relieve pressure on cord by putting a sterile glove into the vagina and holding the presenting part off the umbilical cord- keep hand there! -Have mom get into a modified Sims position, Trendelenburg, or knee-chest position to take pressure of cord -if cord is protruding from vagina, wrap loosely in a sterile towel saturated with warm sterile normal saline. Do not attempt to replace cord into cervix -Administer oxygen to woman at 8-10 L/min until birth is accomplished -continue to monitor FHR -Explain to woman and support person what is happening and how it is being managed -Prepare for immediate vaginal birth if she is completely dilated or C-section if not

Indications for Cesarean Birth

-Cardiac Disease -Guillain Barre -Conditions associated with increased ICP -mechanical obstruction of lower uterine segment (e.g. tumors, fibroids) -mechanical vulvar obstruction -Abnormal FHR -Malpresentation (breech or transverse lie) -Active maternal herpes lesions -HIV -Congenital anomalies -Dysfunctional labor -Placental abruption -Placenta previa

glucose cross the placenta

-Carrier-mediated facilitated diffusion insulin does not

Teaching woman with VTE

-Change positions frequently but avoid placing knees in sharply flexed position (can lead to pooling of blood in lower extremities) -avoid rubbing affected area because this could cause clot to dislodge -explain heparin and warfarin -ok to breastfeed with these medications -manage pain by elevating leg, changing positions, or applying moist head -oral contraceptives are contraindicated -do not take aspirin while on anti-coagulant therapy What is an accurate diagnostic indicator of thrombosis? venous ultrasonography with or without color Doppler. Medical Management for superficial venous thrombosis NSAIDS, rest, elevation, and compression stockings. Medical Management for DVT anticoagulants (usually IV Heparin) for 3-5 days, bed rest, leg elevation, pain meds; Oral anticoagulants (Warfarin) for about 3 months. After symptoms decrease she is fitted for compression stockings. Medical Management for PE anticoagulant therapy until symptoms have resolved and IV intermittent or oral meds up to 6 month. What should be taught to woman on bed rest from VTD change positions frequently, avoid placing knees in sharply flexed position, avoid tubing affected area because could cause clots, and neither Heparin or Warfarin excrete significantly in breast milk. Pain management with VTD change positions, elevating affected leg, and applying moist heat.

Nursing Interventions for Hypoglycemia in Mother

-Check blood glucose levels when symptoms first appear -if blood glucose is <70 mg/dl, eat 2-4 glucose tablets or gel immediately -Recheck blood glucose level in 15 minutes. If blood glucose is still <70 mg/dl, give 2-4 additional glucose tablets -Recheck blood glucose level again in 15 minutes. If glucose level is still <70 mg/dl, contact provider immediately -if woman is unconscious, administer 50% dextrose IV push or 1 mg glucagon IM -obtain blood and urine specimens for laboratory testing

Discharge teaching for woman after early miscarriage

-Clean the perineum after voiding or bowel movement and change perineal pads frequently -Shower (avoid tub baths) for 2 weeks -Avoid tampon use, douching, and vaginal intercourse for 2 weeks -Notify HCP if an elevated temperature or fowl-smelling discharge develops -Eat foods high in iron and protein to promote tissue repair and RBC replacement -Seek assistance from support groups, clergy, or counseling -Allow yourself (and your partner) to grieve before becoming pregnant again

Cervical Insufficiency

-Common cause of late miscarriage -Passive and painless dilation of cervix leading to recurrent preterm births during the second trimester in absence of other causes -measurement of cervical length can detect this (short cervix = <25 mm)- measured via ultrasound -prevented with a cerclage- cervical suture placed to tie the cervix closed until it is clipped at term

Major causes of perinatal mortality for babies of diabetics

-Congenital malformations -Resp distress syndrome -extreme prematurity -IUFD What is the major cause of diabetes-associated birth defects? -hyperglycemia during the first trimester of pregnancy, when organs and organ systems are forming Most commonly, the defects occur in the CV system or CNS Diabetic mothers have a risk of what coexisting disease?Thyroid disease

Teaching after cerclage placement

-Couple should avoid intercourse until 1 week after placement and use condoms afterwards

Laboratory Findings with DIC

-Decreased platelets -decreased fibrinogen -decreased factor V and VIII -prolonged prothrombin time -increased fibrin degradation products -Increased d-dimer test

Fetal indications for labor induction

-Diabetes -Postterm pregnancy -Hypertensive complications of pregnancy -IUGR -isoimmunization -chorioamnionitis -PROM with established fetal maturity

Diet Recommendations for Diabetics

-Distribute total calories among 3 meals and 2-3 snacks each day -never skip meals -do not go more than 4 hours without food -Have a large bedtime snack of at least 25 g of complex carbs with some protein or fat (to prevent hypoglycemia and starvation ketosis) -avoid foods high in refined sugar -eat foods high in dietary fiber -avoid alcoho,nicotine, caffeine -Avoid excessive artificial sweeteners Ideal Diet- 55% carbohydrate, 20% protein, 25% fat

Follow-up care for hyperemesis gravidarum

-Eat small, frequent meals and foods that sound appealing E.g. nongreasy, dry, sweet, and salty foods) every 2-3 hours -eat dry, bland, low-fat, high-protein foods -cold foods may be better tolerated than warm foods -eat salty and sweet foods. in general, eat what sounds good rather than trying to have a balanced diet -eat protein after sweets -dairy products may sit better than other foods -if you vomit on an empty stomach, try sucking on a popsicle -try ginger tea -try warm gingerale -drink liquids from a cup with a lid -separate liquids from solids and alternate every 2-3 hours -warn her that other family members should cook because she may find exposure to cooking odors nauseating

What should client do if symptoms occur?

-Empty your bladder -Drink 2-3 glasses of water or juice -Lie down on your side for 1 hour -Palpate for contractions -If symptoms continue, call HCP -if symptoms go away, resume light activity but not what you were doing when symptoms began -if symptoms return, call HCP immediately -Call HCP or go to birthing center immediately if: -uterine contractions every 10 minutes or less for 1 hour or more -vaginal bleeding - smelly vaginal discharge -fluid leaking from vagina

Nursing interventions for PPH

-Evaluate the contractility of the uterus (first step) -If uterus is hypotnoic/boggy, firmly massage fundus and take measures to increase contractility and minimize blood loss -express clots -eliminate bladder distention -infuse continuous IV of oxytocin and LR -administer misoprostol (cytotec) if oxytocin is not working -Administer blood products to restore woman's intravascular volume -oxygen delivery -insert indwelling urinary catheter

Causes, onset, and sypmtoms of hypoglycemia/insulin shock

-Excess insulin -insufficient food -excessive exercise -indigestion, diarrhea, vomiting Onset: -rapid (regular insulin) -gradual (modified insulin) Symptoms: -irritability, hunger, sweating, nervousness, personality change, weakness, fatigue, blurred or double vision, dizziness, headache, palor, clammy skin, shallow respirations, rapid pulse, blood glucose <70 mg/dl

Treatment of Placenta Previa

-Expectant management (observation and bed rest)- implemented if the fetus is at < 36 weeks of gestation and has a normal FHR tracing, the bleeding is mild (<250 mL) and stops, and the woman is not in labor -woman will initially be hospitalized for continuous FHR and contraction monitoring -assess hemoglobin, hematocrit, platelet count, coagulation studies If woman is <34 weeks of gestation, corticosteroids should be administered -When bleeding stops, woman will be on bedrest with bathroom privileges -can move around hospital room 15-30 minutes 4x per day -no vaginal examinations should be performed. woman on pelvic rest -ultrasound exams every 2-3 weeks

What should be done before inserting cytotec to ripen the cervix?

-Explain the procedure -Assess the woman and fetus before each insertion and during treatment -Avoid giving magnesium-containing antacids -Use with caution in women with renal failure -Have the woman void before insertion -Assist the woman to maintain a supine position with a lateral tilt or side-lying position for 30-40 minutes after insertion -initiate oxytocin no sooner than 4 hours after the last dose of cytotec was administered

Ectopic pregnancy

-Fertilized ovum is implanted outside the uterine cavity -women are less likely to have a successful subsequent pregnancy after an ectopic pregnancy -leading cause of infertility -AKA tubal pregnancies because 90% are located in the uterine tube -can also occur in the abdominal cavity, on an ovary, or on the cervix -classifed according to site of implantation (tubal ovarian, abdominal)

Why are C-sections increasing in frequency?

-Fetal macrosomia -advanced maternal age -obesity -GDM -multifetal pregnancy -dystocia in nulliparous women -decrease in number of VBACs

What should the nurse do if a woman is refusing to get a C-Section when it is indicated for the fetus?

-Find out why she is refusing and provide information that may persuade her to change her mind

Fetal surveillance in diabetics

-Follow up ultrasounds are performed every 3-4 weeks to monitor fetal growth, estimate fetal weight, and detect hydramnios, macrosomia, and congenital abnormalities -Maternal serum alpha-fetoprotein is tested between weeks 15-20 because diabetic moms are at increased risks for fetuses with neural tube defects -Fetal nuchal translucency can be tested between 11 and 14 weeks to increase detection of heart defects

Medical Management of Ectopic Pregnancy

-Giving methotrexate to dissolve the tubal pregnancy which destroys rapidly dividing cells by depriving them of folic acid -helps avoid surgery -woman must be hemodynamically stable and have normal liver and kidney function to be given methotrexate -best results occur when mess is unruptured and small, no fetal cardiac activity is noted on US, and hCG levels < 1000

Predisposing Factors for Postpartum Infection

-History of VTE, UTI, mastitis, pneumonia -Diabetes mellitus -Alcoholism -Drug abuse -immunosuppression -anemia -malnutrition -C-section -operative vaginal birth -prolonged rupture of membranes -chorioamnionitis -prolonged labor -bladder catheterization -internal fetal monitoring -multiple vaginal exams after ROM -epidural -retained placental fragments -episiotomy or lacerations -hematoma

Oxytocin

-Hormone normally released by pituitary -stimulates uterine contractions and aids in milk let-down -Pitocin= synthetic oxytocin -High-alert medication because it has the potential to cause significant harm when used inappropriately -used for labor induction and augmentation -also used to control postpartum leeding

When should a C-section optimally be planned for a diabetic?

-In the early morning to facilitate glycemic control -no morning insulin is given on the day of the surgery and the woman is given nothing by mouth

Uterine Dehiscence

-Incomplete uterine rupture -Separation of prior uterine scar -does not always result in hemorrhage Signs and symptoms: -abnormal FHR tracing- variable or prolonged decelerations and bradycardia -loss of fetal station -constan t abdominal pain, uterine tenderness, change in uterine shape, and cessation of contractions

Risk of pregnancy associated with obesity

-Increased incidence of postterm pregnancy -Gestational diabetes -Hypertensive disorders -Increased risk of Cesarean birth -Increased risk of thromboembolic complications after surgery

Maternal Risks Associated with postterm pregnancy

-Increased risk of perineal injury due to macrosomia -increased risk for hemorrhage and infection -abnormal fetal growth

Gestational Diabetes Mellitus- considerations

-Increases the risk for development of overt diabetes later in life -likely to recur in future pregnancies Risk factors for GDM -A family history of diabetes -A previous pregnancy that resulted in an unexplained stillbirth -Previous birth of a malformed or macrosomic fetus -obesity -hypertension -glycosuria -maternal age older than 25 screening Between 24-28 weeks of gestation Those with strong risk factors for GDM should be screened earlier on in the pregnancy diagnosedf During the second half of the pregnancy

Nursing Interventions

-Initiate and monitor IV therapy for correction of fluid and electrolyte balance (first step after assessment) -administer pyridoxine (vitamin B6) etiher alone or in combination with Unisom (administering drugs and nutritional supplements) -monitoring woman's response to interventions -observing for complications such as metabolic acidosis, jaundice, or hemmorhage -accurately recording intake and output -may give Reglan or corticosteroids if the women is still not responsding to treatment (should be avoided during the first trimester) -Enteral or Parenteral nutrition

Causes and symptoms of Hyperglycemia/DKA

-Insufficient insulin -Excess of wrong kind of food -Infection, injuries, illness -Emotional stress -Insufficient Exercise Symptoms: -Thirst, nausea/vomiting, abdominal pain, constipation, drowsiness, dim vision, increased urination, headache, flushed dry skin, rapid breathing, weak/thready pulse, fruity breath, urine positive for sugar and acetone, blood glucose >200 mg/dL

Causes of PPH

-Lacerations of cervix, vagina, perineum -Uterine atony -hematomas

Fetal causes of detocia

-Macrosomia -Anomalies like large tumors, open neural tube defects, hydrocephalus -cephalopelvic disproportion -malposition (ex. occipitoposterior position) -Malpresentation (e.g. breech)

Priorities for Postoperative Care of a Woman who Had a C-Section

-Maintain a patent airway -Position the woman to prevent aspiration -Assess blood pressure and pulse at least every 15 minutes for 2 hours -Assess temperature every 4 hour for the first 8 hours after birth and then at least every 8 hours -Assess condition of the incision, IV intake, urine output, fundal height, and lochia -Help woman cough, deep breathe, turn, and perform leg exercises -Keep woman NPO until bowel sounds are heard Nonpharmacologic Postpartum Pain Relief After Cesarean Birth -Splint the incision with a pillow when moving or coughing -Use relaxation techniques such as music, breathing, and dim lights Intestinal Gas -walk often -do not eat or drink gas-forming foods (carbonated beverages, whole milk, or ice) --do not use straws for drinking fluids -lie on your left side to expel gas -rock in a rocking chair -take antiflatulence medication When is the foley catheter removed after a C-section? On the first postpartum day -she should be ambulated and get out of bed immediately after removal of foley How should woman get out of bed after a C-section? She should be taught to seek assistance initially when getting out of bed -She should sit on the side of the bed first to determine if dizziness will occur -Then stand for a minute -Then ambulate

What is the initial management of a retained placenta?

-Manual separation and removal by the HCP

Biggest risk factor for abruptio placentae

-Maternal hypertension -also cocaine use -blunt external abdominal trauma (motor vehicle accidents) -maternal battering -cigarette smoking -a hx of abruption in a previous pregnancy -PROM -more likely with twin gestations

Clinical Manifestations of Ectopic Pregnancies

-Most cases are diagnosed before rupture due to 3 classic symptoms: 1. Abdominal pain (progressed from dull, lower quadrant pain in one side to colicky pain to sharp, stabbing pain) 2. delayed menses 3. abnormal vaginal bleeding (spotting) that occurs 6-8 weeks after last normal period- mild to moderate dark red or brown intermittent vaginal bleeding Once rupture has occurred: -referred shoulder pain -generalized, one-sided, or deep lower quadrant acute abdominal pain -signs of shock, faintness, dizziness -ecchymotic blueness around the umbilicus (Cullen sign)

Partial Hydatidiform Mole

-Normal ovum is fertilized by 2+ sperm -often have embryonic or fetal parts and an amniotic sac -congenital anomalies are present

Nursing Interventions for DKA

-Notify primary care provider -Administer insulin in accordace with blood glucose levels -give IV fluids such as normal saline and potassium when urinary output is adequate -give bicarbonate when pH <7 -monitor lab testing of blood and urine

Interventions for woman with Amniotic Fluid Embolus

-Oxygenate- administer O2 through nonrebreather face mask at 8-10 L/min -prepare for intubation and mechanical ventilation -initiate or assist with CPR. Tilt pregnant woman 30 degrees to her side to displace uterus -Maintain cardiac output and replace fluid losses- administer IV fluids and blood products -Insert indwelling catheter and measure hourly urine output -Correct coagulation failure -Monitor fetal and maternal status -Prepare for emergency birth -Provide emotional support to woman, her partner, and family

Treatment of Hyperthyroidism during pregnancy

-PTU or methimazole -have potentially dangerous maternal and fetal side effects -PTU can cause hepatic toxicity -methimazole can cause choanal atresia or esophageal atresia, facial anomalies, and developmental delay in exposed fetuses -women should show clinical improvement in 2-6 weeks -the medication can often be discontinued by 32-36 weeks of gestation -medications cross the placenta easily and may cause fetal hypothyroidsim

Teaching Self-Management of Postterm Pregnancy

-Perform daily fetal movement counts -Assess for signs of labor -Call PCP if your membranes rupture or if you notice a decrease in or no fetal movement -keep appointment for fetal assessment test and cervical checks -go to the birthing facility soon after labor begins

Common causes of indicated preterm birth

-Preexisting or gestational diabetes -chronic hypertension -preeclampsia -obstetrical disorders -previous C-section via a classical uterine incision -placental disorders -medical disorders -seizures -thromboembolism -obesity -maternal HIV or active herpes infection -obesity -advanced maternal age -fetal disorders -chronic IUGR -abnormal NST or BPP- fetal compromise -excessive or inadequate amniotic fluid -congenital fetal abnormalities

Maternal indications for foceps-assisted birth

-Prolonged second stage of labor -Maternal exhaustion or maternal cardiopulmonary disease Fetal indications for forceps-assisted birth Fetus in distress -abnormal fetal presentations -arrest of rotation -extraction of the head in a breech presentation What conditions must be present for forceps-assisted birth to be successful? -Cervix must be fully dilated to prevent lacerations and hemorrhage -Bladder should be empty -Presenting part must be engaged -Membranes must be ruptured -Maternal pelvis must be able to support fetal passage What should the nurse assess for after a forceps-assisted delivery? In mom: Vaginal or cervical lacerations, urinary retention, hematoma formation in the pelvis soft tissues In baby: -bruising or abrasions at the site of the blade applications -facial palsy resulting from pressure ob blades on facial nerve -subdural hematoma

Screening Tools for Ectopic Pregnancy

-Quantitative hCG levels (>1500-2000 mU/ml indicate pregnancy so if levels are as such and no intrauterine pregnancy is observed on an ultrasound, ectopic pregnancy should be expected) -Transvaginal ultrasound exam -serum progesterone level >25 ng/mL rules out the presence of an ectopic pregnancy (if less than <5 ng/mL, may indiacte ectopic pregnancy) -vaginal exam should be performed only once and then with great caution -palpable mass on palpation (be gentle because mass can be ruptured during a bimanual examination

Hyperthyroidsim in pregnancy

-Rare -usually caused by Graves' disease Clinical Manifestations -heat intolerance -diaphoresis -fatigue -anxiety -emotional lability -tachycardia (all of these can also be normal signs of pregnancy) Differentiated by: -weight loss -goiter -pulse rate >100 bpm -elevated T3 and T4 -greatly suppressed TSH

Calcium Channel Blockers (Nifedipine [procardia])

-Relaxes smooth muscles including the uterus by blocking Ca2+ entry Maternal side effects: -hypotension -fatigue -flushing -dizziness -nausea Fetal side effects: -hypotension -should not be given simultaneously with mag sulfate -should not be given simultaneously with terbutaline because of effects on HR and blood pressure

Signs of Amniotic Fluid Embolus

-Respiratory Distress -Restlessness -Dyspnea -Cyanosis -Pulmonary edema -Respiratory arrest -Circulatory collapse -Hypotension -Tachycardia -Shock -Cardiac arrest -Hemorrhage -Coagulation failure- bleeding from incisions, venipuncture sites, laceartions, petechiae, ecchmoses -uterine atony

What symptoms would cause you to stop a magnesium sulfate infusion?

-Respiratory rate < 12 breaths/min -pulmonary edema -absent DTRs -chest pain -severe hypotension -altered level of consciousness -extreme muscle weakness -urine output < 25-30 ml/hr -serum magnesium level >/= 10 mEq/L What is therapeautic dose of magnesium sulfate? Between 4-7 mEq/L

Complete Hydatidiform Mole

-Results from fertilization of an egg in which the nucleus has been lost or inactivated -contains no fetus, placenta, amniotic membranes, or fluid -causes hemorrhage into the uterine cavity and vaginal bleeding -empty egg may be fertilized by two normal sperm -looks like a bunch of grapes

Causes of Ectopic pregnancy

-STIs -Tubal infection and damage -Contraceptive methods that predispose failures to be ectopic (e.g. IUD) -use of tubal sterilization methods -increased use of assisted reproductive techniques -tubal surgery

Which factors would lead to an increased likelihood of uterine rupture?

-Scarred uterus as the result of previous classical C-section incision -multiple prior cesarean births -no previous vaginal birth -induced or augmented labor -fetal macrosomia -Short interpregnancy interval

When does ketoacidosis most often occur in pregnancy?

-Second and third trimesters when the diabetogenic effect of pregnancy is the greatest Major complication of DKA -Intrauterine fetal death

Alternative Methods to Induce Labor

-Sexual intercourse- prostaglandins in the semen and stimulation of contractions with orgasm -Nipple stimulation- endogenous oxytocin released -gravity which applies pressure to the cervix and leads to secretion of endogenous oxytocin -Black cohosh -Castor oil -Accupuncture

Education of a Patient with an Ectopic Pregnancy

-She should be on a contraceptive for at least 3 menstrual cycles to allow time for her body to heal -She should contact her HCP as soon as she suspects she might be pregnant because of the increased risk of ectopic pregnancy

Clinical Manifestations of Hyperemesis Gravidarum

-Significant weight loss -dehydration -dry mucous membranes -increased pulse -decreased BP -poor skin turgor -unable to keep down even clear liquids by mouth -electrolyte imbalances seen on lab tests

Threatened Miscarriage- What is it? Clinical manifestations and management

-Spotting of blood but with the cervical os closed -mild uterine cramping present -no cervical dilation -no passage of tissue Management: -bed rest -repetitive transvaginal ultrasounds and assessment of hCG and progesterone levels to determine if the fetus is still alive

Surgical Management of Hydatidiform Mole

-Suction curettage is performed if mole does not spontaneously abort as it usually does -induction of labor and oxytocic agents or prostaglandin is not recommended because of the increased risk of embolization of trophoblastic tissue -administer rhogam

What signs of postoperative complications of a C-section should be reported to HCP?

-Temperature >38oC (100.4oF) -Urination- painful urination, urgency, cloudy urine -Lochia- heavier than a normal menstrual period, clots, odor -Cesarean incision- redness, swelling, bruising, foul-smelling discharge or bleeding, wound separation -severe, increasing abdominal pain

Complete Miscarriage- What is it? Clinical manifestations and management

-The cervix has already closed after all fetal tissue was expelled -slight bleeding may occur with mild uterine cramping -passage of fetal tissue -cervix is no longer dilated Management: - no further intervention may be needed if uterine contractions are adequate to prevent hemorrhage and no infection is present -suction curettage may be performed to ensure no retention of fetal or maternal tissue

What usually triggers DIC?

-The release of large amounts of tissue thromboplastin which occurs in placental abruption -retained dead fetus syndrome -amniotic fluid embolus -severe preeclampsia -HELLP syndrome -gram-negative sepsis

Preconception Counseling for Diabetics- why is it important?

-To establish glycemic control before conception -To diagnose any vascular complications of diabetes (which can significantly impact the fetus)

Diagnosis of a hydatidiform mole?

-Transvaginal ultrasound- most accurate -serum hCG levels- consistently high or rising even past the expected 10-12 week mark -snowstorm pattern on US- pattern of multiple diffuse intrauterine masses

Inversion of the Uteru

-Turning inside out of uterus after birth -can recur with a subsequent birth -can be complete, incomplete, or prolapsed Sings: -sudden hemorrhage, shock, pain, uterus is not palpable abdominally; Uterus must be replaced into proper position by physician. Factors that contribute to uterine inversion -fundal implantation of the placenta -vigorous fundal pressure -excessive traction applied to the cord -fetal macrosomia -short umbilical cord -tocoloysis -prolonged labor -uterine atony -nulliparity -abnormally adherent placental tissue treatment emergency situation; fluid replacement, and replacement of uterus within pelvic cavity.

Etiology of Hydatidiform Mole

-Unknown -may be related to an ovular defect or a nutritional deficiency Risk factors: -women who are in early teens or older than 40 -hx of prior molar pregnancy

Contraindications to ECV

-Uterine anomalies -3rd trimester bleeding -Multiple gestation -oligohydramnios -Evidence of uteroplacental insufficiency -Nuchal cord -Previous c-section

C-Section Post-surgery care in obese women

-Wash the incision with warm water and soap several times a day -Use a hair dryer on low setting to dry the area (to prevent moisture from being trapped by pannus, roll of fat)

Retained Placenta

-When the placenta has not been delivered within 30 minutes after birth despite gentle traction on umbilical cord detected by ultrasonography

Implications of PKU in pregnant women

-Women with PKU should avoid all high-protein foods (meat, milk, eggs, nuts, and wheat) before conception and through pregnancy -phenylalanine levels should be monitored at least 1-2 x per week -Maternal phenylalanine levels should be less than 6 mg/dl for at least 3 months before conception and during pregnancy No brEASTFEEDING because their milk contains a high concentration of phenylalanine

Treatment for mom with thyroid storm during birth

-administer IV fluids, oxygen, PTU, iodide, antipyretics, beta-blocker

Risk factors for placenta previa

-advanced maternal age >35 years -history of previous cesarean birth -multiparity -history of suctioning curettage -smoking -women carrying male fetuses -multiple gestations -endometrial damage and scarring

Maternal Phenylketonuria

-an inborn error of metabolism caused by an autosomal recessive trait -impairs the body's ability to metabolize the amino acid phenylalanine found in all protein foods -toxic accumulation occurs in the blood which interferes with brain development and function

Hypertonic uterine dysfunction

-anxious nullipara who is having frequent and painful contractions that are ineffective in causing cervical dilation or effacement to progress -therapeutic rest with administration of an analgesic to reduce pain and encourage sleep may be ordered to give her a 4-6 hr rest

Infants born to women who had poor pregnancy weight gain because of hyperemesis may

-be small for gestational age -have a low birth weight -be born prematurely -have a 5-minute Apgar score less than 7

The standard of care for monitoring blood glucose levels during pregnancy

-blood glucose testing at home -should be checked before breakfast, lunch, and dinner; 1 to 2 hours after each meal; at bedtime; and in the middle of the night if insulin level is being adjusted (or more frequently during infection, or episodes of vomiting/diarrhea)

Type 1 Diabetes

-caused by pancreatic islet beta cell destruction -prone to ketoacidosis -absolute insulin deficiency

Treatments for cervical insufficiency

-cerclage -pessaries -progesterone- given orally, vaginally, or IM- every day

What should a diabetic do when illness occurs?

-continue to take insulin even if unable to eat anything -increase oral intake of fluids to prevent dehydration -rest as much as possible Is urine testing for glucose reliable during pregnancy? No because the lowered renal threshold for glucose causes glycosuria which does not accurately reflect the blood glucose level

Advantages of Rapid-Acting Insulins?

-convenience because they are injected immediately before mealtime -they produce less hyperglycemia after meals -cause fewer hypoglycemic episodes

Elective Cesarean birth

-done without medical indication -woman must be NPO for 8 hours before Risks -longer hospital stay -increased risk of respiratory problems for the baby -greater complications in subsequent pregnancies including uterine rupture and placental implantation problems -should not be performed unless 39 weeks of gestation has been reached -should not be performed for women who desire several additional children because of risks for placenta previa, placenta accreta, and cesarean hysterectomy

Teaching woman regarding methotrexate therapy

-explain that methotrexate dissolves ectopic (tubal) pregnancies by destroying rapidly dividing cells -urine contains toxic levels for 72 hours after administration. teach her to avoid getting urine on the toilet seat and to double flush the toilet with lid down -feces may contain residual drug for up to 7 days -inform of possible side effects- nausea, vomiting, stomatitis, and dizziness -Tell the woman to avoid foods and vitamins containing folic acid, gas forming foods, sun exposure, sexual intercourse until hCG levels are undectable -tell her to contact HCP immediately if she has severe abdominal pain which may be a sign of actual or impending tubal rupture

Incomplete Miscarriage- What is it? Clinical manifestations and management

-expulsion of the fetus with retention of the placenta -heavy to profuse amount of bleeding with an open cervical os -tissue may be present with bleeding -severe uterine cramping -cervical dilation with tissue in cervix Management: -may or may not require additional cervical dilation before curettage

Nursing interventions once woman with hyperemesis gravidarum stops vomiting

-feedings are started in small amounts at frequent intervals -limited intake of oral fluids and bland foods such as crackers, toast, or baked chicken are offered

Steps for Self-Monitoring Blood Glucose Levels

-gather supplies, check expiration date -wash hands with warm water to increase circulation -select site on side of any finger (rotate fingers) -pierce site with lancet -drop hand down to side and gently squeeze finger from hand to fingertip with other hand -allow blood to be drawn into glucose test strip

Antenatal Glucocorticoids (e.g. Betamethasone)

-given as IM injection to mother to accelerate fetal lung maturity by stimulating fetal surfactant production -all women between 24-34 weeks of gestation should be given a single course of glucocorticoids when preterm birth is threatened within 7 days -optimal benefit for fetus occurs 24 hours after first injection

Signs of fetal hypothyroidism in moms being treated for hyperthyroidism

-goiter -bradycardia -IUGR

Shoulder dystocia

-head is born during labor but anterior shoulder cannot pass under the pubic arch -uncommon -can be caused by fetopelvic disproportion related to excessive fetal size or maternal pelvic abnormalities Risk factors: -maternal diabetes -history of shoulder dystocia in previous birth -prolonged second stage of labors Complications of Shoulder Dystocia for Baby -Asphyxia related to delay in completing birth -Trauma from maneuvers used to accomplish birth- brachial plexis and phrenic nerve injuries, fracture of humerus or clvicle -Brachial Plexis inury= Erb palsy- Major maternal complications of shoulder dystocia -Postpartum hemorrhage -rectal injuries

Risk factors for spontaneous preterm labor

-hx of previous spontaneous preterm birth -nonwhite race -genital tract colonization and infection -multifetal gestation -second-trimester bleeding -low prepregnancy weight -cervical length- >30 mm= unlikely to give birth prematurely

Risk factors for Hyperemesis Gravidarum

-hyperthyroid disorders -prepregnancy psychiatric disorders -previous pregnancies complicated by hyperemesis gravidarum -molar pregnancy -multiple gestation with a male and a female fetus -diabetes -GI disorders -women carrying a female fetus are more likely than women carrying a male fetus to develop it (unknown why)

Possible fetal adverse effects of oxytocin

-hypoxemia -acidosis -abnromal FHR

What causes hyperglycemia?

-increased hepatic glucose production and decreased periopheral glucose use

Risks for mothers with untreated hyperthyroidism

-increased risk of miscarriage, preterm birth, and giving birth to stillborn infants or infants with goiter, hyperthyroidism or hypothyroidism

Meconium-Stained Amniotic Fluid

-indicates that fetus has passed meconium before birth -green-colored amniotic fluid -consistency can be thin (light) or thick

Diabetes and infection

-infections are more common and more serious in pregnant women with diabetes the inflammatory response, leukocyte function, and vaginal pH are all affected -explains why vaginitis and UTis are more common in moms with diabetes -infections may lead to ketoacidosis

Medical Management of DIC

-involves correction of underlying cause -rapid replacement of blood products and clotting factors -optimization of oxygenation -achievement of normal body temp -continued reassessment of laboratory parameters -vitamin K administration

Evaluation of couples with recurrent pregnancy loss

-karyotyping of both partners and miscarriage specimens -assessment of the placenta -evaluating the woman's uterine cavity -testing the woman for antiphospholipid antibody syndrome -testing woman for thyroid disease -should be screened for depression and PTSD

Risks of untreated hypothyroidism

-miscarriage -preeclampsia -gestational hypertension -placental abruption -preterm birth -stillbirth

Inevitable Miscarriage- What is it? Clinical manifestations and management

-moderate to heavy mount of bleeding with an open cervical os -tissue may be present with bleeding -mild to severe uterine cramping -rupture of membranes cervical dilatio passage of products of conseption Management: -bed rest if no pain, bleeding, r infection -If ROM, pain, bleeding, or infection is present, then prompt termination of the pregnancy through dilation and curettage

Nursing interventions for women intrapartum (during birth)

-monitor them closely to prevent complications related to dehydration, hypoglycemia, or hyperglycemia -insert an IV -once active labor begins, piggyback a solution that contains 5% dextrose which can help manage the stress of labor and birth -maintain blood glucose levels at 80-110 mg/dl (check hourly)

Diabetes Mellitus

-most common endocrine disorder associated with pregnancy - a group of metabloic diseases characterized by hyperglycemia resulting from defects in insulin secretion, insulin action, or both -pregnancy with diabetes is considered a high risk pregnancy

Type 2 Diabetes

-most prevalent form of the disease -relative insulin deficiency Risk factors: -obesity -aging -sedentary lifestyle -family history and genetics -puberty -hypertension -prior gestational diabetes

Elective Induction of labor

-no medical reason Major risks associated with it: -increased rate of C-sections -neonatal morbidity -cost

Diagnosis of cervical insufficiency

-obstetric history -speculum and digital pelvic examinations -transvaginal ultrasound (cervix <25 mm) -short cervix may be accompanied by funneling (beaking), effacement of the cervical os

Methotrexate Administration

-obtain the woman's height and weight (to calculate body surface area) -usually 50 mg/m2 given IM -should be prepared in hospital pharmacy under a biologic safety cabinet -don two pairs of gloves before removing the syringe -do not expel air from the syringe or prime the needle because methotrexate can be aerosolized -check client's identity and dosage -have another nurse check -dispose of any items worn or used to prepare, dispense, or administer the methotrexate injection in a hazardous drug waste container -wash hands thoroughly after removing gloves

Hypothyroidism during pregnancy

-often associated with infertility and increased risk of miscarriage and is not often seen during pregnancy -usually caused by glandular destruction by autoantibodies (dypically due to Hashimoto's thyroiditis) Symptoms: -weight gain -lethargy -decrease in exercise capacity -cold intolerance -constipation -hoarseness -brittle nails -dry skin -elevated levels of TSH Treatment -thyroid hormone supplements -should not be taken at the same time as iron supplements (must be taken at least 4 hours apart)

Risk Factors for Dysfunctional Labor

-overweight -short stature -advanced maternal age -infertility -prior version -masculine characteristics -uterine abnormalities -malpresentations and positions of the fetus -cephalopelvic disproportion (CPD) -uterine overstimulation with oxytocin -maternal fatigue, dehydration, electrolyte imbalance, fear -administration of analgesic too early in labor

Maternal Hazards Associated with Oxytocin administration

-placental abruption -uterine rupture -unnecessary C-section due to abnormal FHR and patterns -postpartum hemorrhage -infection

What are some reasons to proceed with birth before term?

-poor metabolic control -coexisting hypertension -nonreassuring responses to fetal testing

Assessment of a woman reporting vaginal bleeding in early pregnancy

-pregnancy history -vital signs -type and location of pain -quality and nature of bleeding -emotional status Lab tests: -hCG levels -hemoglobin level (anemia) -white blood cell count (infection)

Greatest fetal risk associated with placenta previa

-preterm birth Other risks -still birth -malpresentation -fetal anemia -IUGR

Vaginal Birth after Cesarean

-prior vaginal birth -spontaneous (rather than induced or augmented) labor -clinically adequate pelvis -previous c-sections- low-transverse cut Women with these characteristics are less likely to have a successful VBAC: -Recurrent indication for C-section (e.g. labor dystocia) -Increased maternal age -Non-Caucasian race -Gestational age >40 weeks -Maternal obesity (BMI >30) -Preeclampsia -Short interpregnancy interval -Estimated fetal weight >4000 g -Giving birth at a rural or private hospital Major risk of VBAC Utterine Rupture

Candidate for home care of placenta previa

-should be willing to comply with restrictions (activity) -live within 20 minutes of hospital -have access to a telephone -constant access to transportation -have constant supervision by family or friends in the home

Possible physical examination findings with DIC

-spontaneous bleeding from gums, nose -oozing, excessive bleeding from venipuncture site -petecchiae -hematomas -signs of bruising -GI bleeding -tachycardia -diaphoresis

Pessaries

-used to support a prolapsed uterus -rings inserted into the vagina that distribute the weight of the uterus away from the cervix and obstruct the cervical os

Symptoms of Placental Abruption

-vaginal bleeding -abdominal pain -uterine tenderness -contractions -board-like abdomen

Follow-Up care for hydatidiform mole

-weekly measurements of hCG levels until level decreases to normal and remains normal for 3 consecutive weeks -follow-up assessments for a year

Hypotonic Uterine Dysfunction

-woman initially makes normal progress into the active first stage labor but then contractions become weak and inefficient or stop altogether -increased risk of infection -labor augmentation may be necessary

A client on the postpartum unit has been diagnosed with deep vein thrombosis. The following titration schedule is included in the client's orders: If INR is less than 1: administer 7,500 units heparin subcu If INR is 1.1 to 2: administer 5,000 units heparin subcu If INR is 2.1 to 3: administer 2,500 units heparin subcu If INR is greater than 3: administer 0 units heparin subcu The client's INR is 2.6. How many mL of heparin will the nurse administer if the available concentration of heparin is 5,000 units per 0.2 mL? (Calculate to the nearest tenth.) __________ mL.

0.1 mL

A stillborn baby was delivered in the birthing suite a few hours ago. After the delivery, the family remained together, holding and touching the baby. Which statement by the nurse would further assist the family in their initial period of grief? 1. "What can I do for you?" 2. "Now you have an angel in heaven." 3. "Don't worry, there is nothing you could have done to prevent this from happening." 4. "We will see to it that you have an early discharge so that you don't have to be reminded of this experience."

1 When a loss or death occurs, the nurse should ensure that parents have been honestly told about the situation by their health care provider or others on the health care team. It is important for the nurse to be with the parents at this time and to use therapeutic communication techniques. The nurse must also consider cultural and religious practices and beliefs. The correct option provides a supportive, giving, and caring response. Options 2, 3, and 4 are blocks to communication and devalue the parents' feelings.

A 39-year-old, 16-week-gravid woman has had an amniocentesis. Before discharge, the nurse teaches the woman to call her doctor if she experiences any of the following side effects? Select all that apply. 1. Fever or chills. 2. Lack of fetal movement. 3. Abdominal pain. 4. Rash or pruritus. 5. Vaginal bleeding.

1, 2, 3, and 5 are correct. 1. The client should call her practitioner if she experiences fever or chills. 2. Because the fetus can be injured during an amniocentesis, the client should report either a decrease or an increase in fetal movement. 3. The client should report abdominal pain or cramping. An amniocentesis can precipitate preterm labor. 4. Neither rash nor pruritus is associated with amniocentesis. 5. The client should report any vaginal loss-blood or amniotic fluid. The placenta may become injured or the membranes may rupture during an amniocentesis. During an amniocentesis, the amniotic sac is entered with a large needle. As a result of the procedure, a number of complications can develop, including infection, preterm labor, rupture of the membranes, and/or fetal injury. Although the incidence of complications is small, it is very important for the nurse to advise the client of the signs of each of these problems.

A nurse has been assigned to circulate during the cesarean section of triplets. Which of the following actions should the nurse take before the birth of the babies? Select all that apply. 1. Count the number of sterile sponges. 2. Document the time of the first incision. 3. Notify the pediatric staff. 4. Perform a sterile scrub. 5. Assemble the sterile instruments.

1, 2, and 3 are correct. 1. The circulating nurse should count the sterile sponges. This is done together with the scrub nurse. 2. The circulating nurse must document in the medical record all key events that occur during the surgery, including the time of the first incision. 3. The circulating nurse should notify the pediatric staff. There should be one resuscitation team assembled in the delivery room for each baby that will be delivered. TEST-TAKING TIP: The circulating nurse is responsible for coordinating the activity in the operating room. He or she is the only member of the team who is able to move freely throughout the room in order to make telephone calls, obtain needed supplies, maintain the documentation record, and so on. When multiple babies are being birthed, he or she is especially important. The more babies who are birthed at once—e.g., twins, triplets—the more vulnerable the babies are at birth. Multiple-gestation babies are often born preterm and small-forgestational age. There must be a resuscitation team available for each baby in case emergent care is needed.

A client, 32 weeks' gestation with placenta previa, is on total bed rest. The physician expects her to be hospitalized on bed rest until her cesarean section, which is scheduled for 38 weeks' gestation. To prevent complications while in the hospital, the nurse should do which of the following? Select all that apply. 1. Perform passive range of motion exercises. 2. Restrict the fluid intake of the client. 3. Decorate the room with pictures of family. 4. Encourage the client to eat a high-fiber diet. 5. Teach the client deep breathing exercises.

1, 3, 4, and 5 are correct. 1. Passive range of motion will help to decrease the potential for muscle atrophy. 3. This client is separated from family. The separation can lead to depression. Decorating the room and enabling family to visit freely is very important. 4. A high-fiber diet will help to maintain normal bowel function. 5. Deep breathing exercises are important in order to maintain the client's respiratory function. TEST-TAKING TIP: Although bed rest is often used as therapy for antenatal clients, it does not come without its complications-constipation, depression, respiratory compromise, muscle atrophy, to name but a few. The nurse must provide preventative care to maintain the health and well-being of the client as much as possible.

A woman, G1P0000, is 40 weeks' gestation. Her Bishop score is 4. Which of the following complementary therapies do midwives frequently recommend to clients in similar situations? Select all that apply. 1. Sexual intercourse. 2. Aromatherapy. 3. Breast stimulation. 4. Ingestion of castor oil. 5. Aerobic exercise.

1, 3, and 4 are correct. 1. Sexual intercourse has been recommended to women as a means of increasing their Bishop score. 3. Midwives have recommended that women employ breast stimulation as a means of stimulating labor. 4. Midwives have recommended that women ingest castor oil as a means of increasing their Bishop score. There are a variety of interventions that have been used to increase women's Bishop scores and/or to stimulate labor. Because oxytocin is produced during orgasm and when the breasts are stimulated, intercourse and breast stimulation both can be used as complementary methods of stimulating labor. Castor oil stimulates the bowels. Prostaglandins, which ripen the cervix, are produced as a result of gastrointestinal stimulation. If there is any indication that the baby may be unable to withstand labor, however, these means should not be employed.

A client enters the labor and delivery suite. It is essential that the nurse note the woman's status in relation to which of the following infectious diseases? Select all that apply. 1. Hepatitis B. 2. Rubeola. 3. Varicella. 4. Group B streptococcus. 5. HIV/AIDS.

1, 4, and 5 are correct. 1. The client's hepatitis B status should be assessed. 4. The client's group B streptococcus status should be assessed. 5. The client's HIV/AIDS status should be assessed. TEST-TAKING TIP: There are a number of infectious diseases that impact care given during pregnancy, labor and delivery, postpartum, and in the newborn nursery. The hepatitis B status must be assessed in order to notify the nursery for care needed by the baby. Group B strep status must be assessed in order to administer needed antibiotics to the mother during labor and to monitor the baby's status in the newborn nursery. The HIV/AIDS status must be assessed in order to administer needed antiviral medications to the mother in labor and/or to the baby postdelivery. HIV/AIDS is also an indication for cesarean section delivery.

A woman is to receive methotrexate IM for an ectopic pregnancy. The nurse should teach the woman about which of the following common side effects of the therapy? Select all that apply. 1. Nausea and vomiting. 2. Abdominal pain. 3. Fatigue. 4. Lightheadedness. 5. Breast tenderness.

1,2,3,4 are correct. 1. Nausea and vomiting is a common side effect. 2. Abdominal pain is a common side effect. The pain associated with the medication needs to be carefully monitored to differentiate it from the pain caused by the ectopic pregnancy itself. 3. Fatigue is a common side effect. 4. Lightheadedness is a common side effect. Because methotrexate is an antineoplastic agent, the nurse would expect to see the same types of complaints that he or she would see with a patient receiving chemotherapy for cancer. It is very important that the abdominal pain seen with the medication not be dismissed since the most common complaint of women with ectopic pregnancies is pain. The source of the pain, therefore, must be clearly identified.

A 12-week-gravid client presents in the emergency department with abdominal cramps and scant dark red bleeding. What should the nurse assess this client for? 1. tachycardia 2. referred shoulder pain 3. headache 4. Fetal heart dysrhythmias. 5 hypertension

1,3,4,5 1. tachycardia 3. headache 4. Fetal heart dysrhythmias. 5 hypertension

A client is receiving a blood transfusion after the delivery of a placenta accreta and hysterectomy. Which of the following complaints by the client would warrant immediately discontinuing the infusion? 1. "My lower back hurts all of a sudden." 2. "My hands feel so cold." 3. "I feel like my heart is beating fast." 4. "I feel like I need to have a bowel movement."

1. "My lower back hurts all of a sudden."

Thrombophlebitis

1. A clot forms in a vessel wall as a result of inflammation of the vessel wall. 2. A partial obstruction of the vessel can occur. 3. Increased blood-clotting factors in the postpartum period place the client at risk. 4. Early ambulation in the postoperative period after cesarean section is a preventive measure.

A woman has received Prepidil (dinoprostone gel) for cervical ripening. For which of the following signs/symptoms should the nurse carefully monitor the client? 1. Diarrhea and back pain. 2. Hypothermia and rectal pressure. 3. Urinary retention and rash. 4. Tinnitus and respiratory distress.

1. A common side effect of Prepidil is diarrhea and labor contractions are often first felt in the back. TEST-TAKING TIP: Prepidil (dinoprostone) is a prostaglandin medication used to ripen the cervix for induction. Gastrointestinal side effects are commonly seen when prostaglandin is used since the gastrointestinal system is adjacent to the vagina where the medication is inserted. In addition, the nurse must be watchful for signs of labor.

A 32-weeks' gestation client states that she "thinks" she is leaking amniotic fluid. Which of the following tests could be performed to determine whether the membranes had ruptured? 1. Fern test. 2. Biophysical profile. 3. Amniocentesis. 4. Kernig sign.

1. A fern test is performed to assess for the presence of amniotic fluid. The fern test was so named because when amniotic fluid is viewed under a microscope, it appears as a fern-like image. The image is a reflection of the high estrogen levels in the fluid that create a crystalline pattern. When the fern appears, the nurse can be assured that amniotic fluid is leaking from the amniotic sac.

A 38-week-gestation woman is in labor and delivery with a painful, board-like abdomen and progressively larger serial girth measurements. Which of the following assessments is appropriate at this time? 1. Fetal heart rate. 2. Cervical dilation. 3. White blood cell count. 4. Maternal lung sounds

1. A fetal heart check is the appropriate assessment TEST-TAKING TIP: The clinical scenario is indicative of a placental abruption. Since the only oxygenation available to the fetus is via the placenta, the appropriate action by the nurse at this time is to determine the well-being of the fetus.

A 32-week-gestation client was last seen in the prenatal client at 28 weeks' gestation. Which of the following changes should the nurse bring to the attention of the certified nurse midwife? 1. Weight change from 128 pounds to 138 pounds. 2. Pulse rate change from 88 bpm to 92 bpm. 3. Blood pressure change from 120/80 to 118/78. 4. Respiratory rate change from 16 rpm to 20 rpm.

1. A weight gain of 10 pounds in a 4-week period is worrisome. The recommended weight gain during the second and third trimesters is approximately 1 pound per week. A weight gain above that which is recommended can be related to a few things, including preeclampsia, excessive food intake, or multiple gestations. The midwife should be advised of the weight gain in order to identify the reason for the increase and to intervene accordingly.

Hematoma Assessment

1. Abnormal, severe pain 2. Pressure in the perineal area 3. Sensitive, bulging mass in the perineal area with discolored skin 4. Inability to void 5. Decreased hemoglobin and hematocrit levels 6. Signs of shock, such as pallor, tachycardia, and hypotension, if significant blood loss has occurred

The nurse is discharging five Rh-negative clients from the maternity unit. The nurse knows that the teaching was successful when the client who had which of the following deliveries asks why she must receive a RhoGAM injection? Select all that apply. 1. Abortion at 10 weeks' gestation. 2. Amniocentesis at 16 weeks' gestation. 3. Fetal demise at 24 weeks' gestation. 4. Birth of Rh-negative twins at 35 weeks' gestation. 5. Delivery of a 40-week-gestation, Rh-positive baby1. Abortion at 10 weeks' gestation.

1. Abortion at 10 weeks' gestation. 2. Amniocentesis at 16 weeks' gestation. 3. Fetal demise at 24 weeks' gestation. 5. Delivery of a 40-week-gestation, Rh-positive baby

Interventions for Pulmonary Embolism

1. Administer oxygen. 2. Position client with the head of the bed elevated. 3. Monitor vital signs frequently, especially respiratory and heart rate and breath sounds. 4. Monitor for signs of respiratory distress and for signs of increasing hypoxemia. 5. Administer intravenous fluids as prescribed. 6. Administer anticoagulants as prescribed. 7. Prepare to assist the HCP to administer medications to dissolve the clot if prescribed

A nurse works in a clinic with a high adolescent pregnancy population. The nurse provides teaching to the young women in order to prevent which of the following high risk complications of pregnancy? 1. Preterm birth. 2. Gestational diabetes. 3. Macrosomic babies. 4. Polycythemia.

1. Adolescents are at high risk for preterm labor. TEST-TAKING TIP: It is very important that pregnant teens learn the telltale signs of preterm labor, such as intermittent backache, cramping, discomfort low in the pelvic area, and the like. Because of their life style choices, pregnant teens are at high risk for low birth weight, preterm births.

A woman is receiving Paxil (paroxetine) for postpartum depression. To prevent a drug/food interaction, the client must be advised to refrain from consuming which of the following? 1. Alcohol. 2. Grapefruit. 3. Milk. 4. Cabbage

1. Alcohol.

A nurse remarks to a 38-week-gravid client, "It looks like your face and hands are swollen." The client responds, "Yes, you're right. Why do you ask?" The nurse's response is based on the fact that the changes may be caused by which of the following? 1. Altered glomerular filtration. 2. Cardiac failure. 3. Hepatic insufficiency. 4. Altered splenic circulation

1. Altered glomerular filtration leads to protein loss and, subsequently, to fluid retention, which can lead to swelling in the face and hands. The hypertension associated with preeclampsia results in poor perfusion of the kidneys. When the kidneys are poorly perfused, the glomerlular filtration is altered, allowing large molecules, most notably the protein albumin, to be lost through the urine. With the loss of protein, the colloidal pressure drops in the vascular tree, allowing fluid to third space. The body gets the message to retain fluids, exacerbating the problem. One of the early signs of the third spacing is the swelling of a client's hands and face.

A nurse is about to inject RhoGAM into an Rh-negative mother. Which of the following is the preferred site for the injection? 1. Deltoid. 2. Dorsogluteal. 3. Vastus lateralis. 4. Ventrogluteal.

1. Although the dosage can be administered in the gluteal muscles, the deltoid is the preferred site of the RhoGAM injection. TEST-TAKING TIP: Whenever possible, it is preferable to inject the antibodies into the recommended injection site. The antibodies are absorbed optimally from that site and, therefore, are more apt to suppress the mother's immune response.

A labor nurse is caring for a client, 38 weeks' gestation, who has been diagnosed with symptomatic placenta previa. Which of the following physician orders should the nurse question? 1. Begin oxytocin drip rate at 0.5 millunits/min. 2. Assess fetal heart rate every 10 minutes. 3. Weigh all vaginal pads. 4. Assess hematocrit and hemoglobin

1. An order for oxytocin administration should be questioned. TEST-TAKING TIP: Since the stem states that this woman has symptomatic placenta previa, the test taker can conclude that the woman is bleeding vaginally. It would be appropriate to monitor the fetal heart for any signs of distress, to weigh pads to determine the amount of blood loss, and to assess the hematocrit and hemoglobin to check for anemia. Labor, however, is contraindicated since vaginal delivery is contraindicated.

o reduce possible side effects from a cesarean section under general anesthesia, clients are routinely given which of the following medications? 1. Antacids. 2. Tranquilizers. 3. Antihypertensives. 4. Anticonvulsants.

1. Antacids are routinely administered presurgically to cesarean section clients. TEST-TAKING TIP: Progesterone is a muscle relaxant. Because pregnant women have elevated levels of progesterone, their cardiac sphincters are relaxed. They are especially at high risk, therefore, for vomiting during surgery. To decrease the acidity of the vomitus in case of aspiration, gravid women are routinely given antacids presurgically.

Interventions for Subinvolution

1. Assess vital signs. 2. Assess uterus and fundus. 3. Monitor for uterine pain and vaginal bleeding. 4. Elevate legs to promote venous return. 5. Encourage frequent voiding. 6. Monitor hemoglobin and hematocrit. 7. Prepare to administer methylergonovine maleate (Methergine), which provides sustained contraction of the uterus, as prescribed.

A client with a complete placenta previa is on the antepartum clinical unit in preparation for delivery. Which of the following should the nurse include in a teaching session for this client? 1. Coughing and deep breathing. 2. Phases of the first stage of labor. 3. Lamaze labor techniques. 4. Leboyer hydrobirthing

1. Because the client will have a cesarean section with anesthesia, the woman should be taught coughing and deep breathing exercises for the postoperative period TEST-TAKING TIP: When a client has a complete placenta previa, the placenta has attached to the uterine lining so that it fully covers the internal cervical os. If the woman were to go through labor, during dilation and effacement the villi of the placenta would incrementally be exposed, leading the client to bleed profusely. The baby would exsanguinate and die. The only safe way to deliver the baby, therefore, is via cesarean section

he physician declares after delivering the placenta of a client during a cesarean section that it appears that the client has a placenta accreta. Which of the following maternal complications would be consistent with this diagnosis? 1. Blood loss of 2,000 mL. 2. Blood pressure of 160/110. 3. Jaundiced skin color. 4. Shortened prothrombin time.

1. Blood loss of 2,000 mL.

The nurse is admitting four full term primigravid clients to the labor and delivery unit. The nurse requests pre-cesarean section orders from the health care practitioner for which of the clients? The client who has: 1. cervical cerclage. 2. FH 156 with beat-to-beat variability. 3. Maternal blood pressure of 90/60. 4. Full effacement.

1. Cervical cerclage, a stitch encircling the cervix, is incompatible with vaginal delivery. TEST-TAKING TIP: The test taker must be able to differentiate in which circumstances a full-term, otherwise healthy woman, would be unable to deliver vaginally. There are a few absolute indicators for cesarean section: maternal infection with active herpes simplex 2 and HIV/AIDS (http://www.cdc.gov/hiv/ topics/perinatal/overview_partner.htm# strategies); malpresentation—for example, horizontal lie and breech; previous maternal surgery—e.g., myomectomy; a vertical cesarean scar; some congenital anomalies—e.g., hydrocephalus and meningomyelocele; and other physical conditions, including cervical cerclage in place, obstructive lesions in the lower gynecological system, and complete placenta previa. The test taker should become familiar with each of these.

The nurse is monitoring a postpartum client who received epidural anesthesia for delivery for the presence of a vulvar hematoma. Which assessment finding would best indicate the presence of a hematoma? 1. Changes in vital signs 2. Signs of heavy bruising 3. Complaints of intense pain 4. Complaints of a tearing sensation

1. Changes in vital signs Because the client has had epidural anesthesia and is anesthetized, she cannot feel pain, pressure, or a tearing sensation. Changes in vital signs indicate hypovolemia in an anesthetized postpartum client with vulvar hematoma. Option 2 (heavy bruising) may be seen, but vital sign changes indicate hematoma caused by blood collection in the perineal tissues.

A client with 4+ protein and 4+ reflexes is admitted to the hospital with severe preeclampsia. The nurse must closely monitor the woman for which of the following? 1. Grand mal seizure. 2. High platelet count. 3. Explosive diarrhea. 4. Fractured pelvis.

1. Clients with severe preeclampsia are high risk for seizure. A client who is diagnosed with severe preeclampsia is high risk for becoming eclamptic. Clients who become eclamptic have had at least one seizure.

Interventions for Perinatal Loss

1. Communicate therapeutically and actively listen, providing parents time to grieve. 2. Notify the hospital chaplain or other religious person. 3. Discuss with the parents options such as seeing, holding, bathing, or dressing the deceased infant; visitation by other family members or friends; religious or cultural rituals; and funeral arrangements. 4. Prepare a special memories box with keepsakes such as footprints, handprints, locks of hair, and pictures, if appropriate. 5. Admit the mother to a private room; if possible, mark the door to the room with a special card (per agency procedure and maintaining confidentiality) that denotes to hospital staff that this family has experienced a loss.

Steps to take if umbilical cord prolapse is suspected

1. Elevate fetal presenting part that is lying on cord by applying finger pressure 2. Place client into extreme Trendelenburg's or modified SIm's position or knee chest position. 3. Admin oxygen face mask 8-10L/min 4. Monitor FHR and assess for hypoxia 5. IV fluids 6. Prepare for immediate birth

A nurse administered RhoGAM to a client whose blood type is A+ (positive). Which of the following responses would the nurse expect to see? Select all that apply. 1. Fever. 2. Flank pain. 3. Dark-colored urine. 4. Swelling at the injection site. 5. Polycythemia.

1. Fever. 2. Flank pain. 3. Dark-colored urine.

Rubella (German Measles)

1. First trimester - teratogenic 2. causes Congenital defects of the eyes, heart, ears, and brain 3. Postpartum period, 1-3mo before getting pregnant again after shot cross placenta not immune titer less than 1: 8

A client is postpartum 24 hours from a spontaneous vaginal delivery with rupture of membranes for 42 hours. Which of the following signs/symptoms should the nurse report to the client's health care practitioner? 1. Foul-smelling lochia. 2. Engorged breasts. 3. Cracked nipples. 4. Cluster of hemorrhoids.

1. Foul-smelling lochia.

Which of the following signs/symptoms would the nurse expect to see in a woman with abruptio placentae? 1. Increasing fundal height measurements. 2. Pain-free vaginal bleeding. 3. Fetal heart accelerations. 4. Hyperthermia with leukocytosis.

1. Fundal heights increase during pregnancy approximately 1 cm per week. When a placental abruption occurs, the height increases hour by hour TEST-TAKING TIP: When a placenta abrupts, it separates from the uterine wall. As a result, a pool of blood appears behind the placenta. The pool of blood takes up space leading to an increase in the size of the uterus. The fundal height increases as the uterine size increases.

The nurse is providing discharge counseling to a woman who is breastfeeding her baby. What should the nurse advise the woman to do if she should palpate tender, hard nodules in her breasts? 1. Gently massage the areas toward the nipple, especially during feedings. 2. Apply ice to the areas between feedings. 3. Bottle feed for the next twenty-four hours. 4. Apply lanolin ointment to the areas after each and every breastfeeding

1. Gently massage the areas toward the nipple, especially during feedings.

Diagnosis of Preterm Labor is based on 3 diagnostic criteria

1. Gestational age between 20 0/7 and 36 6/7 weeks 2. Uterine activity (regular contractions) 3. Progressive cervical change (effacement of 80% or cervical dilation of 2 cm or greater)

A client, who had no prenatal care, delivers a 10-lb 10-oz baby boy whose serum glucose result 1 hour after delivery was 20 mg/dL. Based on these data, which of the following tests should the mother have at her 6-week postpartum checkup? 1. Glucose tolerance test. 2. Indirect Coombs' test. 3. Blood urea nitrogen (BUN). 4. Complete blood count (CBC).

1. Glucose tolerance test.

A postpartum woman has been diagnosed with postpartum psychosis. Which of the following signs/symptoms would the client exhibit? 1. Hallucinations. 2. Polyphagia. 3. Induced vomiting. 4. Weepy sadness.

1. Hallucinations.

In anticipation of a complication that may develop in the second half of pregnancy, the nurse teaches an 18-week gravid client to call the office if she experiences which of the following? 1. Headache and decreased output. 2. Puffy feet. 3. Hemorrhoids and vaginal discharge. 4. Backache.

1. Headache and decreased output are signs of preeclampsia. TEST-TAKING TIP: It is important for the test taker to realize that, although some symptoms like puffy feet may seem significant, they are normal in pregnancy, while other symptoms like headache, which in a nonpregnant woman would be considered benign, may be potentially very important in a pregnant woman.

3. A client has just received Hemabate (carboprost) because of uterine atony not controlled by IV oxytocin. For which of the following side effects of the medication will the nurse monitor this patient? Select all that apply. 1. Hyperthermia. 2. Diarrhea. 3. Hypotension. 4. Palpitations. 5. Anasarca.

1. Hyperthermia. 2. Diarrhea.

The physician has ordered oxytocin (Pitocin) for induction for 4 gravidas. In which of the following situations should the nurse refuse to comply with the order? 1. Primigravida with a transverse lie. 2. Multigravida with cerebral palsy. 3. Primigravida who is 14 years old. 4. Multigravida who has type 1 diabetes.

1. Induction is contraindicated in transverse lie. TEST-TAKING TIP: A baby in the transverse lie is in a scapular presentation. The baby is incapable of being birthed vaginally. Whenever a vaginal birth is contraindicated, induction is also contraindicated

The nurse administers RhoGAM to a postpartum client. Which of the following is the goal of the medication? 1. Inhibit the mother's active immune response. 2. Aggressively destroy the Rh antibodies produced by the mother. 3. Prevent fetal cells from migrating throughout the mother's circulation. 4. Change the maternal blood type to Rh-positive.

1. Inhibit the mother's active immune response.

Interventions for Mastitis

1. Instruct the client in good hand washing and breast hygiene techniques. 2. Promote comfort. 3. Apply heat or cold to the site as prescribed. 4. Maintain lactation in breast-feeding mothers. 5. Encourage manual expression of breast milk or use of a breast pump every 4 hours. 6. Encourage the client to support the breasts by wearing a supportive bra; avoid wearing an underwire bra. 7. Administer analgesics as prescribed. 8. Administer antibiotics as prescribed.

Reasons for passage of meconium in amniotic fluid

1. It is a normal physiologicl function that occurs with maturity 2. Breech presentation 3. Result of hypoxia-induced peristalsis and sphincter relaxation 3. Umbilical cord compression-induced vaginal stimulation

A client has just entered the labor and delivery suite with ruptured membranes for 2 hours, fetal heart rate of 146, contractions every 5 minutes 60 seconds, and a history of herpes simplex type 2. She has no observable lesions. After notifying the doctor of the admission, which of the following is the appropriate action for the nurse to take? 1. Check dilation and effacement. 2. Prepare the client for surgery. 3. Place the bed in Trendelenburg position. 4. Check the biophysical profile results.

1. It is appropriate for the nurse to assess the client's dilation and effacement. TEST-TAKING TIP: Although cesarean deliveries are recommended to be performed when a client has an active case of herpes simplex, surgical delivery is not indicated when no lesions are present. Clients who have histories of herpes with no current outbreak, therefore, are considered to be healthy laboring clients who may deliver vaginally.

A baby is entering the pelvis in the vertex presentation and in the extended attitude. The nurse determines that which of the following positions is consistent with this situation? 1. LMA (left mentum anterior). 2. LSP (left sacral posterior). 3. RScT (right scapular transverse). 4. ROP (right occiput posterior).

1. LMA position is consistent with that information. TEST-TAKING TIP: In order to conceptualize the relationship between attitude, presentation, and position, the test taker must first thoroughly understand the three concepts. The vertex presentation is a head-down presentation; both occipital and mentum presentations are vertex presentations. When the attitude is extended in a head-down presentation, the front of the head or the face is the presenting part, while when the head is flexed, the back of the head, or occiput, is presenting. When the scapula is presenting, the baby is lying sideways in utero, called transverse lie.

A client is to receive a blood transfusion after significant blood loss following a placenta previa delivery. Which of the following actions by the nurse is critical prior to starting the infusion? Select all that apply. 1. Look up the client's blood type in the chart. 2. Check the client's arm bracelet. 3. Check the blood type on the infusion bag. 4. Obtain an infusion bag of dextrose and water. 5. Document the time the infusion begins.

1. Look up the client's blood type in the chart. 2. Check the client's arm bracelet. 3. Check the blood type on the infusion bag. 5. Document the time the infusion begins.

A postoperative cesarean client, who was diagnosed with severe preeclampsia in labor and delivery, is transferred to the postpartum unit. The nurse is reviewing the client's doctor's orders. Which of the following medications that were ordered by the doctor should the nurse question? 1. Methergine 2. Magnesium sulfate. 3. Advil (ibuprofen). 4. Morphine sulfate.

1. Methergine

A nurse massages the atonic uterus of a woman who delivered 1 hour earlier. The nurse identifies the nursing diagnosis: Risk for injury related to uterine atony. Which of the following outcomes indicates that the client's condition has improved? 1. Moderate lochia flow. 2. Decreased pain level. 3. Stable blood pressure. 4. Fundus above the umbilicus

1. Moderate lochia flow.

Interventions for Postpartum Infection

1. Monitor vital signs and temperature every 2 to 4 hours. 2. Make the client as comfortable as possible; position the client to promote vaginal drainage. 3. Keep the client warm if chilled. 4. Isolate the newborn from the client only if the client can infect the newborn. 5. Provide a nutritious, high-calorie, high-protein diet. 6. Encourage fluids to 3000 to 4000 mL/day, if not contraindicated. 7. Encourage frequent voiding and monitor intake and output. 8. Monitor culture results if cultures were prescribed. 9. Administer antibiotics according to identified organism, as prescribed

Hematoma Interventions

1. Monitor vital signs. 2. Monitor client for abnormal pain or perineal pressure, especially when forceps delivery has occurred. 3. Place ice at the hematoma site. 4. Administer analgesics as prescribed. 5. Monitor intake and output. 6. Encourage fluids and voiding. 7. Prepare for urinary catheterization if the client is unable to void. 8. Administer blood products as prescribed. 9. Monitor for signs of infection, such as increased temperature, pulse rate, and white blood cell count. 10. Administer antibiotics as prescribed because infection is common after hematoma formation. 11. Prepare for incision and evacuation of hematoma if necessary.

Which symptom would the nurse expect to observe in a postpartum client with a vaginal hematoma? 1. Pain. 2. Bleeding. 3. Warmth. 4. Redness.

1. Pain.

Placenta Accreta, Percreta, and Increta ranked from least severe to most severe degree of attachment:

1. Placenta accreta 2. Placenta increta 3. Placenta percreta Each one can be either partial or complete Placenta Accreta Slight penetration of myometrium by placenta -most common Placenta Increta Deep penetration of myometrium by placenta Placenta Percreta -Perforation of the uterus by placenta

Intermittent positive pressure boots have been ordered for a client who had an emergency cesarean section. Which of the following is the rationale for that order? 1. Postpartum clients are at high risk for thrombus formation. 2. Post-cesarean clients are at high risk for fluid volume deficit. 3. Postpartum clients are at high risk for varicose vein development. 4. Post-cesarean clients are at high risk for poor milk ejection reflex.

1. Postpartum clients are at high risk for thrombus formation.

A nurse who is caring for a pregnant diabetic should carefully monitor the client for which of the following? 1. Urinary tract infection. 2. Multiple gestation. 3. Metabolic acidocic 4. Pathological hypotension. 5. hypolipidimia

1. Pregnant diabetic clients are particularly at high risk for urinary tract infections. 3. Metabolic acidocic It is very important for the test taker to read each response carefully. If the test taker were to read the responses to the preceding question very quickly, he or she might choose incorrect answers. For example, the test taker might pick pathological hypotension assuming that it says hypertension. Pregnant diabetics are high risk for UTIs because they often excrete glucose in their urine. The glucose is an excellent medium for bacterial growth.

A nurse is assessing a 1-day postpartum client who had a spontaneous vaginal delivery over an intact perineum. The fundus is firm at the umbilicus, lochia moderate, and perineum edematous. One hour after receiving ibuprofen 600 mg po, the client is complaining of perineal pain at level 9 on a 10-point scale. Based on this information, which of the following is an appropriate conclusion for the nurse to make about the client? 1. She should be assessed by her doctor. 2. She should have a sitz bath. 3. She may have a hidden laceration. 4. She needs a narcotic analgesic.

1. She should be assessed by her doctor.

A client has just done a fetal kick count assessment. She noted 6 movements during the last hour. If taught correctly, what should be her next action? 1. Nothing because further action is not warranted. 2. Call the doctor to set up a nonstress test. 3. Redo the test during the next one-half hour. 4. Drink a glass of orange juice and redo the test.

1. She should do nothing because the woman should feel 3 or more counts in 1 hour. Fetal kick counting is a valuable, noninvasive means of monitoring fetal well-being. Mothers are taught to consciously count the numbers of times they feel their baby kick during a 60-minute period, one or more times during the day. If the baby kicks 3 or more times, the woman can be reassured that the baby is healthy. If the baby kicks fewer times, the woman should notify her health care practitioner who will likely perform either a nonstress test or, in some situations, a more sophisticated fetal assessment tes

Interventions for Thrombophlebitis

1. Specific therapies may depend on the location of thrombophlebitis. 2. Assess the lower extremities for edema, tenderness, varices, and increased skin temperature. 3. Maintain bed rest. 4. Elevate affected leg. 5. Apply a bed cradle and keep bedclothes off affected leg. 6. Never massage the leg. 7. Monitor for manifestations of pulmonary embolism. 8. Apply hot packs or moist heat to the affected site as prescribed to alleviate discomfort. 9. Apply elastic stockings (support hose) if prescribed. 10. Administer analgesics and antibiotics as prescribed. 11. Heparin sodium intravenously may be prescribed for femoral or pelvic thrombophlebitis to prevent further thrombus formation.

A couple has delivered a 28-week fetal demise. Which of the following nursing actions are appropriate to take? Select all that apply. 1. Swaddle the baby in a baby blanket. 2. Discuss funeral options for the baby. 3. Encourage the couple to try to get pregnant again in the near future. 4. Ask the couple whether they would like to hold the baby. 5. Advise the couple that the baby's death was probably for the best.

1. Swaddle the baby in a baby blanket. 2. Discuss funeral options for the baby. 4. Ask the couple whether they would like to hold the baby.

A woman's glucose challenge test (GCT) results are 155 mg/dL at 1 hour post-glucose ingestion. Which of the following actions, as ordered by the physician, is appropriate? 1. Send the woman for a glucose tolerance test. 2. Teach the woman how to inject herself with insulin. 3. Notify the woman of the normal results. 4. Provide the woman with oral hypoglycemic agents.

1. The 1-hour GCT results are above normal. She needs a 3-hour glucose tolerance test (GTT). The glucose challenge test (GCT) is a nonfasting test performed on the vast majority of pregnant clients at or about 24 weeks' gestation. The test is performed to assess for gestational diabetes. Clients with test results of 130 mg/dL or higher are referred for a 3-hour glucose tolerance test in order to make a definitive diagnosis.

The nurse is caring for 4 women who are in labor. The nurse is aware that he or she will likely prepare which of the women for cesarean delivery? 1. Fetus is in the left sacral posterior position. 2. Placenta is attached to the posterior portion of the uterine wall. 3. Client is hepatitis B-surface antigen positive. 4. The lecithin/sphingomyelin ratio in the amniotic fluid is 1.5 to 1.

1. The baby in the LSP position is in a breech presentation. Most breech babies are delivered by cesarean section. TEST-TAKING TIP: Although it is recommended that cesarean section be performed when a mother is affected by two viral illnesses—herpes simplex type 2 (only when active lesions are present) and HIV/AIDS—it is not recommended in the presence of other viral diseases. Hepatitis B is a very serious viral disease, but vertical transmission rates are not significantly different between those babies who are born vaginally and those babies who are born by cesarean section

The laboratory reported the L/S ratio results from an amniocentesis as 1:1. How should the nurse interpret the result? 1. The baby is premature. 2. The mother is high risk for hemorrhage. 3. The infant has kernicterus. 4. The mother is high risk for eclampsia.

1. The baby is preterm. TEST-TAKING TIP: The amount of lecithin must be 2 times the amount of sphingomyelin before the practitioner can be assured that the fetal lungs are mature. The ratio in the scenario-1:1-indicates that the surfactant is insufficient for extrauterine respirations.

A type 1 diabetic client has developed polyhydramnios. The client should be taught to report which of the following? 1. Uterine contractions. 2. Reduced urinary output. 3. Marked fatigue. 4. Puerperal rash.

1. The client should be taught to observe for signs of preterm labor. TEST-TAKING TIP: Clients with hydramnios have excessive quantities of amniotic fluid in their uterine cavities. The excessive quantities likely result from increased fetal urine production resulting from the mother's having periods of hyperglycemia. When the uterus is overextended from the large quantities of fluid, these women are at high risk for preterm labor.

Immediately after a woman spontaneously ruptures her membranes, the nurse notes a loop of the umbilical cord protruding from the woman's vagina. Which of the following actions should the nurse perform first? 1. Put the client in the knee chest position. 2. Assess the fetal heart rate. 3. Administer oxygen by tight face mask. 4. Telephone the obstetrician with the findings

1. The first action the nurse should take is to place the woman in the knee chest position. TEST-TAKING TIP: The weight of the fetus on the prolapsed cord can rapidly result in fetal death. Therefore, the nurse must act quickly to relieve the pressure on the cord. Additional actions that can take pressure off the cord are placing the client in the Trendelenburg position and pushing the head off the cord with a gloved hand. This situation is an obstetric emergency.

A client who works as a waitress and is 35 weeks' gestation, telephones the labor suite after getting home from work and states, "I am feeling tightening in my groin about every 5 to 6 minutes." Which of the following comments by the nurse is appropriate at this time? 1. "Please lie down and drink about four full glasses of water or juice." 2. "You are having false labor pains so you need not worry about them." 3. "It is essential that you get to the hospital immediately." 4. "That is very normal for someone who is on her feet all day."

1. The first intervention for preterm labor is hydration. Clients who are dehydrated are at high risk for preterm labor. TEST-TAKING TIP: Preterm cramping should never be ignored. To assess whether or not a client is in true labor, clients are encouraged to improve their hydration. The client is encouraged to drink about 1 quart of fluid and to lie on her side. If the contractions do not stop, she should proceed to the hospital to have her cervix assessed. If the cervix begins to dilate or efface, a diagnosis of preterm labor would be made. If the contractions stop, clients are usually allowed to begin light exercise. But if the contractions restart, the woman should proceed to the hospital to be assessed.

A pregnant diabetic has been diagnosed with hydramnios. Which of the following would explain this finding? 1. Excessive fetal urination. 2. Recurring hypoglycemic episodes. 3. Fetal sacral agenesis. 4. Placental vascular damage.

1. The hydramnios is likely a result of excessive fetal urination. TEST-TAKING TIP: The majority of amniotic fluid is created as urine by the fetal kidneys. Fetuses of diabetic mothers often experience polyuria as a result of hyperglycemia. If the mother's diabetes is out of control, excess glucose diffuses across the placental membrane, resulting in the fetus becoming hyperglycemic. As a result the fetus exhibits the classic sign of diabetes-polyuria. If the mother's serum glucose levels are very high during the first trimester, it is likely that the fetus will develop structural congenital defects. Sacral agenesis is one of the most severe of these defects.

A client has given birth to a baby girl with a visible birth defect. Which of the following maternal responses would lead the nurse to suspect poor mother-infant bonding? 1. The mother states, "I'm so tired. Please feed the baby in the nursery for me." 2. The mother states, "Her eyes look like mine, but her chin is her Dad's." 3. The mother says, "We have decided to name her Sarah after my mother." 4. The mother says, "I breastfed her. I still need help swaddling her, though."

1. The mother states, "I'm so tired. Please feed the baby in the nursery for me."

A woman states that all of a sudden her 4-day-old baby is having trouble feeding. On assessment, the nurse notes that the mother's breasts are firm, red, and warm to the touch. The nurse teaches the mother manually to express a small amount of breast milk from each breast. Which observation indicates that the nurse's intervention has been successful? 1. The mother's nipples are soft to the touch. 2. The baby swallows after every 5th suck. 3. The baby's pre- and postfeed weight change is 20 milliliters. 4. The mother squeezes her nipples during manual expression.

1. The mother's nipples are soft to the touch.

A client who has been diagnosed with severe preeclampsia is being administered magnesium sulfate via IV pump. Which of the following medications must the nurse have immediately available in the client's room? 1. Calcium gluconate. 2. Morphine sulfate. 3. Naloxone (Narcan). 4. Meperidine (Demerol).

1. The nurse must have calcium gluconate in the client's room. TEST-TAKING TIP: Calcium gluconate is the antidote for magnesium sulfate toxicity. It is very important the test taker know that, if needed, calcium gluconate must be administered very slowly. If administered rapidly, the client may experience sudden convulsions

A client is on terbutaline (Brethine) via subcutaneous pump for preterm labor. The nurse auscultates the fetal heart rate at 100 beats per minute via Doppler. Which of the following actions should the nurse perform next? 1. Assess the maternal pulse while listening to the fetal heart rate. 2. Notify the health care provider. 3. Stop the terbutaline infusion. 4. Administer oxygen to the mother via face mask.

1. The nurse should assess the fetal heart and the maternal pulse simultaneously. TEST-TAKING TIP: Because the medication should increase both the mother's pulse and fetal heart rates, it is likely that the fetal monitor is mistakenly registering the maternal pulse rather than the fetal heart rate. If the pulsations are the same when the radial pulse of the mother and the fetal heart are monitored simultaneously, the nurse can determine that, indeed, the mother's pulse rate is being monitored

client, 42 weeks' gestation, is admitted to the labor and delivery suite with a diagnosis of acute oligohydramnios. The nurse must carefully observe this client for signs of which of the following? 1. Fetal distress. 2. Dehydration. 3. Oliguria. 4. Jaundice

1. The nurse should carefully monitor the client for fetal distress TEST-TAKING TIP: Oligohydramnios is often seen in postterm pregnancies. When the placenta begins to deteriorate, the hydration of the baby drops. Since the predominant component of amniotic fluid is fetal urine, when the baby is dehydrated, the quantity of amniotic fluid drops. Fetal distress can occur because of two factors: cord compression because there is insufficient fluid to cushion the umbilical cord and uteroplacental insufficiency because the placenta is functioning suboptimally

A patient, 32 weeks pregnant with severe headache, is admitted to the hospital with preeclampsia. In addition to obtaining baseline vital signs and placing the client on bed rest, the physician ordered the following four items. Which of the orders should the nurse perform first? 1. Assess deep tendon reflexes. 2. Obtain complete blood count. 3. Assess baseline weight. 4. Obtain routine urinalysis.

1. The nurse should check the client's patellar reflexes. The most common way to assess the deep tendon reflexes is-to-assess the patellar reflexes. Preeclampsia is a very serious complication of pregnancy. The nurse must assess for changes in the blood count, for evidence of marked weight gain, and for changes in the urinalysis. By assessing the patellar reflexes first, however, the nurse can make a preliminary assessment of the severity of the preeclampsia. For example, if the reflexes are +2, the client would be much less likely to become eclamptic than a client who has +4 reflexes with clonus.

The physician has ordered a nonstress test (NST) to be done on a 41-week-gestation client. During the 'h hour test, the nurse observed three periods of fetal heart accelerations that were 15 beats per minute above the baseline and that lasted 15 seconds each. No contractions were observed. Based on these results, what should the nurse do next? 1. Send the client home and report positive results to the MD. 2. Perform a nipple stimulation test to assess the fetal heart in response to contractions. 3. Prepare the client for induction with IV oxytocin or endocervical Prostaglandins. 4. Place the client on her side with oxygen via face mask.

1. The nurse should report the positive results to the doctor. This client is postdates. The NST is being performed to assess the well-being of the fetus. The results of the test-reactive NST results-are evidence that the fetus is well and will likely be well for another few days. There is no need to provide emergent care.

An ultrasound has identified that a client's pregnancy is complicated by oligohydramnios. The nurse would expect that an ultrasound may show that the baby has which of the following structural defects? 1. Dysplastic kidneys. 2. Coarctation of the aorta. 3. Hydrocephalus. 4. Hepatic cirrhosis.

1. The nurse would expect that the baby has dysplastic kidneys. TEST-TAKING TIP: The majority of amniotic fluid is produced by the fetal kidneys. When a pregnancy is complicated by oligohydramnios, ultrasounds may be performed to check for defects in the fetal renal system.

A woman in active labor received Nubain (nalbuphine hydrochloride) 14 mg IV for pain relief. One-half hour later her respirations are 8 rpm. The nurse reports the respiratory rate to the physician. Which of the following medications would be appropriate for the physician to order at this time? 1. Narcan (naloxone). 2. Reglan (declopramide). 3. Benadryl (diphenhydramine). 4. Vistaril (hydroxyzine).

1. The nurse would expect to administer Narcan to the client. TEST-TAKING TIP: Nubain is an opioid analgesia. It has markedly depressed the client's respiratory response. Narcan is an opioid antagonist. It is likely that the physician will order Narcan to be administered at this time.

A gravid woman has been diagnosed with listeriosis. She eats rare meat and raw smoked seafood. Which of the following signs/symptoms would this woman exhibit? 1. Fever and muscle aches. 2. Rash and throinbocytopenia. 3. Petechiae and anemia. 4. Amnionitis and epistaxis.

1. The symptoms of listeriosis are similar to symptoms of the flu and include fever and muscle aches. TEST-TAKING TIP: Even though the adult disease is relatively mild, if listeriosis is contracted during pregnancy, it can lead to serious fetal and neonatal complications. It is important for the nurse to provide the client with needed dietary education in order to prevent antepartal disease.

A woman, 32 weeks' gestation, contracting every 3 min X 60 sec, is receiving magnesium sulfate. For which of the following maternal assessments is it critical for the nurse to monitor the client? 1. Low urinary output. 2. Temperature elevation. 3. Absent pedal pulses. 4. Retinal edema

1. The urinary output should be carefully monitored TEST-TAKING TIP: Even though this client is receiving magnesium sulfate to treat preterm labor and not preeclampsia, the medication still has the same side effects. Magnesium sulfate is excreted through the kidneys. If the urinary output drops, the concentration of magnesium sulfate can rise in the blood stream. Since, at toxic levels, the client can experience respiratory depression and cardiac compromise, it is very important for the nurse carefully to monitor the client's urinary output.

postpartum woman has been diagnosed with postpartum psychosis. Which of the following is essential to be included in the family teaching for this client? 1. The woman should never be left alone with her infant. 2. Symptoms rarely last more than one week. 3. Clinical response to medications is usually poor. 4. The woman must have her vitals assessed every two days.

1. The woman should never be left alone with her infant.

A gravid woman, 36 weeks' gestation with type 1 diabetes, has just had a biophysical profile (BPP). Which of the following results should be reported to the obstetrician? 1. One fetal heart acceleration in 20 minutes. 2. Three episodes of fetal rhythmic breathing in 30 minutes. 3. Two episodes of fetal extension and flexion of 1 arm. 4. One amniotic fluid pocket measuring 3 cm.

1. There should be a minimum of 2 fetal heart accelerations in 20 minutes (approximately 1 every 10 minutes). The biophysical profile is a comprehensive assessment geared to evaluate fetal health. In addition to the four items mentioned above, the fetus should exhibit 3 or more discrete body or limb movements in 30 minutes.

A client has been diagnosed with water intoxication after having received IV oxytocin (Pitocin) for over 24 hours. Which of the following signs/symptoms would the nurse expect to see? 1. Confusion, drowsiness, and vomiting. 2. Hypernatremia and hyperkalemia. 3. Thrombocytopenia and neutropenia. 4. Paresthesias, myalgias, and anemia.

1. These are the classic signs of water intoxication. TEST-TAKING TIP: Clients who receive oxytocin over a long period of time are at high risk for water intoxication. The oxytocin molecule is similar in structure to the antidiuretic hormone (ADH) molecule. The body retains fluids in response to the medication much the same way it would in response to ADH. The nurse, therefore, should carefully monitor intake and output when clients are induced with oxytocin

The nurse is caring for a laboring gravida who is 43 weeks pregnant. For which of the following should the nurse carefully monitor this client and fetus? 1. Late decelerations. 2. Hyperthermia. 3. Hypotension. 4. Early decelerations.

1. This baby is high risk for the development of late fetal heart decelerations. TEST-TAKING TIP: The test taker must attend to all important information in the question. The gestational age of this fetus is 43 weeks. The baby and placenta, therefore, are both postdates. Placental function usually deteriorates after 40 weeks' gestation. Since late decelerations result from poor uteroplacental blood flow, the nurse should monitor this client carefully for late decelerations.

A client is in labor and delivery with a diagnosis of HELLP syndrome. The nurse notes the following blood values: PT (prothrombin time) 99 sec (normal 60 to 85 sec). PTT (partial thromboplastin time) 30 sec (normal 11 to 15 sec). For which of the following signs/symptoms would the nurse monitor the client? 1. Pink-tinged urine. 2. Early decelerations. 3. Patellar reflexes +1. 4. Blood pressure 140/90.

1. This client has likely developed disseminated intravascular coagulation (DIC). The nurse should watch for pink-tinged urine. TEST-TAKING TIP: The test taker must be familiar with the implications of standard blood tests like PT and PTT. Even if the nurse did not know that clients who are diagnosed with HELLP syndrome are at high risk for DIC, he or she should know that clients with prolonged PT and PTT times are at high risk for spontaneous bleeds.

The nurse in the obstetrician's office is caring for four 25-week-gestation white prenatal clients who are carrying singleton pregnancies. With which of the following clients should the nurse carefully review the signs and symptoms of preterm labor (PTL)? 1. 38-year-old registered nurse in an abusive relationship. 2. 32-year-old secretary whose first child was born at 42 weeks' gestation. 3. 26-year-old attorney whose baby has a two-vessel cord. 4. 20-year-old college student with a history of long menstrual periods.

1. This client is high risk for PTL because she is over 35 years of age and in an abusive relationship. TEST-TAKING TIP: Even though medical and psychosocial histories are not absolute predictors of preterm labor, there are a number of factors that have been shown to place clients at risk, including pregnancy history of multiple gestations; previous preterm deliveries; cigarette smoking and/or illicit drug use; a number of medical histories like diabetes and hypertension; and social issues like adolescent pregnancy and domestic violence.

The nurse is admitting a 38-week-gestation client in labor. The nurse is unable to find the fetal heart beat with a Doppler. Which of the following comments by the nurse would indicate that the nurse is in denial? 1. "I'll keep trying until I find the heart beat." 2. "I am sure it is the machine. If I change the battery, I'm sure it will work." 3. "I am so sorry. I am not able to find your baby's heart beat." 4. "Sometimes I really hate these machines."

1. This is an example of the stage of denial. TEST-TAKING TIP: It is essential that the test taker be familiar with the concepts of grief and mourning. Everyone who is caring for a couple who experiences a fetal or neonatal loss, as well as the couple themselves, will progress through the stages of grief. It is very important that the nurse realize that grieving is individual and that the stages of grief are never experienced in a linear fashion. Health staff progress rapidly through the stages, while the couple's grief is likely to be delayed.

A 30-week-gestation multigravida, G3P1011, is admitted to the labor suite. She is contracting every 5 minutes X 40 seconds. Which of the comments by the client would be most informative regarding the etiology of the client's present condition? 1. "For the past day I have felt burning when I urinate." 2. "I have a daughter who is 2 years old." 3. "I jogged 1 1/2 miles this morning." 4. "My miscarriage happened a year ago today."

1. This is the most important statement made by the client. Preterm labor is strongly associated with the presence of a urinary tract infection. Whenever an infection is present in the body, the body produces Prostaglandins. Prostaglandins ripen the cervix and the number of oxytocin receptor sites on the uterine body increase. Preterm labor can then develop.

Which of the following long-term goals is appropriate for a client, 10 weeks' gestation, who is diagnosed with gestational trophoblastic disease (hydatiform mole)? 1. Client will be cancer-free 1 year from diagnosis. 2. Client will deliver her baby at full term without complications. 3. Client will be pain-free 3 months after diagnosis. 4. Client will have normal hemoglobin and hematocrit at delivery.

1. This long-term goal is appropriate. TEST-TAKING TIP: When nurses plan care, they have in mind short-term and long-term goals that their clients will achieve. Short-term goals usually have a time frame of a week or two and often are specific to the client's current hospitalization. Long-term goals are expectations of client achievement over extended periods of time. It is important for nurses to develop goals in order to implement appropriate nursing interventions.

A hospitalized gravida's blood work is hematocrit 30% and hemoglobin 10 gm/dL. In light of the laboratory data, which of the following meal choices should the nurse recommend to this patient? 1. Chicken livers, sliced tomatoes, and dried apricots. 2. Cheese sandwich, tossed salad, and rice pudding. 3. Veggie burger, cucumber salad, and wedge of cantaloupe. 4. Bagel with cream cheese, pear, and hearts of lettuce.

1. This meal choice is high in iron and ascorbic acid. It would be an excellent lunch choice for this client who has a below normal hematocrit level. The client in the scenario is anemic. Although a hematocrit of 32% in pregnancy is acceptable, it is recommended that the value not drop below that level. The nurse, having evaluated the lab statement, should choose foods that are high in iron. Liver and dried fruits are good iron sources. Tomatoes are high in vitamin C, which promotes the absorption of iron.

Which of the following clients is at highest risk for developing a hypertensive illness of pregnancy? 1. G1P0000, age 41 with history of diabetes mellitus. 2. G2P0101, age 34 with history of rheumatic fever. 3. G3P1102, age 27 with history of scoliosis. 4. G3P1011, age 20 with history of celiac disease.

1. This primigravid client-age 44 and with a history of diabetes-is very high risk for preeclampsia. Preeclampsia is a vascular disease of pregnancy. Although any woman can develop the syndrome, women who are highest risk for the disease are primigravidas, those with multiple gestations, women who are younger than 17 or older than 34, those who had preeclampsia with their first pregnancy, and women who have been diagnosed with a vascular disease like diabetes mellitus or chronic hypertension.

A type 1 diabetic is being seen for preconception counseling. The nurse should emphasize that during the first trimester the woman may experience which of the following? 1. Need for less insulin than she normally injects. 2. An increased risk for hyperglycemic episodes. 3. Signs and symptoms of hydramnios. 4. A need to be hospitalized for fetal testing.

1. Type 1 diabetics often need less insulin during the first trimester than they did preconception. TEST-TAKING TIP: Nausea and vomiting are common complaints of gravid clients during the first trimester. As a result, women, including diabetic women, consume fewer calories than before becoming pregnant. Their need for insulin drops commensurately. Therefore, it is very important that the women monitor their blood glucose regularly upon awakening and throughout the day.

A client, 1 day postpartum (PP), is being monitored carefully after a significant postpartum hemorrhage. Which of the following should the nurse report to the obstetrician? 1. Urine output 200 mL for the past 8 hours. 2. Weight decrease of 2 pounds since delivery. 3. Drop in hematocrit of 2% since admission. 4. Pulse rate of 68 beats per minute.

1. Urine output 200 mL for the past 8 hours.

A gravid woman has sickle cell anemia. Which of the following situations could precipitate a vaso-occlusive crisis in this woman? 1. Hypoxia. 2. Alkalosis. 3. Fluid overload. 4. Hyperglycemia.

1. Vaso-occlusive crises are precipitated by hypoxia in pregnant as well as nonpregnant sickle cell clients. TEST-TAKING TIP: Sickle cell anemia is an autosomal recessive disease. The hemoglobin of sickle cell clients become misshapen when the clients are hypoxic, acidotic, and/or dehydrated. This is a very serious state for the pregnant woman and her fetus. These clients must be cared for immediately with intravenous fluids and methods to reverse the hypoxia and acidosis.

The nurse is preparing a list of self-care instructions for a postpartum client who was diagnosed with mastitis. Which instructions should be included on the list? Select all that apply. 1. Wear a supportive bra. 2. Rest during the acute phase. 3. Maintain a fluid intake of at least 3000 mL. 4. Continue to breast-feed if the breasts are not too sore. 5. Take the prescribed antibiotics until the soreness subsides. 6. Avoid decompression of the breasts by breast-feeding or breast pump.

1. Wear a supportive bra. 2. Rest during the acute phase. 3. Maintain a fluid intake of at least 3000 mL. 4. Continue to breast-feed if the breasts are not too sore. Mastitis is an infection of the lactating breast. Client instructions include resting during the acute phase, maintaining a fluid intake of at least 3000 mL/day (if not contraindicated), and taking analgesics to relieve discomfort. Antibiotics may be prescribed and are taken until the complete prescribed course is finished. They are not stopped when the soreness subsides. Additional supportive measures include the use of moist heat or ice packs and wearing a supportive bra. Continued decompression of the breast by breast-feeding or breast pump is important to empty the breast and prevent the formation of an abscess.

The nurse is evaluating the effectiveness of bed rest for a client with mild preeclampsia. Which of the following signs/symptoms would the nurse determine is a positive finding? 1. Weight loss. 2. 2+ proteinuria. 3. Decrease in plasma protein. 4. 3 + patellar reflexes.

1. Weight loss is a positive sign. The key to answering this question is the test taker's ability to interpret the meaning of mild preeclampsia and to realize that this is an evaluation question. There are two levels of preeclampsia. Mild preeclampsia is characterized by the following signs/symptoms: blood pressure 140/90, urine protein +2, patellar reflexes + 3, and weight gain. As can be seen, the values included in answers 2 and 4 are the same as those in the diagnosis. They, therefore, are not signs that the preeclampsia is resolving. Similarly, loss of protein is not a sign of resolution of the disease.

A client just spontaneously ruptured membranes. Which of the following factors makes her especially at high risk for having a prolapsed cord? 1. Breech presentation. 2. Station +3. 3. Oligohydramnios. 4. Dilation 2 cm.

1. When a baby is in the breech presentation, there is increased risk of prolapsed cord. TEST-TAKING TIP: When babies are in the breech presentation, the cord sometimes slips past the baby and becomes the presenting part. The weight of the baby then compresses the cord, preventing the baby from being oxygenated. Additional situations that are at high risk for cord prolapse are hydramnios, premature rupture of membranes, and negative fetal station.

A client is receiving terbutaline (Brethine) for preterm labor. Which of the following findings would warrant stopping the infusion? 1. Fetal heart rate 210 bpm. 2. Maternal heart rate 60 bpm. 3. Early decelerations. 4. Beat-to-beat variability

1. When the FH is >200 bpm, the terbutaline should be stopped TEST-TAKING TIP: Terbutaline, a beta agonist, stimulates the "fight or flight" response in the mother and in the fetus. The fetal heart rate, therefore, increases in response to the medication. When the rate is too high, however, there is insufficient time for the blood to enter the heart, which leads to a drop in cardiac output.

A client, 38 weeks' gestation, is being induced with IV oxytocin (Pitocin) for hypertension and oligohydramnios. She is contracting q 3 min 60 to 90 seconds. She suddenly complains of abdominal pain accompanied by significant fetal heart bradycardia. Which of the following interventions should the nurse perform first? 1. Turn off the oxytocin infusion. 2. Administer oxygen via face mask. 3. Reposition the patient. 4. Call the obstetrician

1. Whenever there is marked fetal bradycardia and oxytocin is running, the nurse should immediately turn off the oxytocin drip. TEST-TAKING TIP: Oxytocin stimulates the contractility of the uterine muscle. When the muscle is contracted, the blood flow to the placenta is reduced. Whenever there is evidence of fetal compromise and oxytocin is being infused, the intravenous should be stopped

The nurse is monitoring a client who is in the active stage of labor. The nurse documents that the client is experiencing labor dystocia. The nurse determines that which risk factors in the client's history placed her at risk for this complication? Select all that apply. 1. age 54 2. BMI 28 3. previous difficulty with fertility 4. administration of oxytocin for induction 5. potassium level of 3.6 mEq/L

1. age 54 2. BMI 28 3. previous difficulty with fertility

An ultrasound is performed on a client at term gestation who is experiencing moderate vaginal bleeding. The results of the ultrasound indicate that abruptio placentae is present. On the basis of these findings, the nurse should prepare the client for which anticipated prescription? 1. delivery of the fetus 2. strict monitoring of intake and output 3. complete bed rest for the remainder of the pregnancy 4. the need for weekly monitoring of coagulation studies until the time of delivery

1. delivery of the fetus

A pregnant client with diabetes mellitus asks the nurse about insulin needs during pregnancy. What information should the nurse provide to the client?

1. explain to the client that pregnancy places demands on carbohydrate metabolism + causes insulin requirements to change 2. maternal glucose crosses the placenta, but insulin does not - fetus produces its own insulin + pulls glucose from mom, which predisposes mom for hypoglycemic reactions 3. 1st trimester: maternal insulin needs decrease 4. 2nd + 3rd trimester: increases in placental hormones cause an insulin-resistant state, requiring an increase in the client's insulin dose. 5. after placental delivery, placental hormone levels abruptly decrease + insulin requirements decrease

Name the two goals of antepartum testing

1. identify fetuses at risk for injury caused by acute or chronic interruption of oxygenation so permanent injury or death might be prevented 2. identify appropriately oxygenated fetuses so unnecessary intervention can be avoided

Interventions related to abortion

1. maintain bed rest as prescribed 2. monitor vital signs 3. monitor for cramping and bleeding 4. count perineal pads to evaluate blood loss, and save expelled tissues and clots 5. maintain IV fluids as prescribed; monitor for signs of hemorrhage or shock 6. prepare the client for dialtion and curettage as prescribed for incomplete abortion 7. administer Rh D immune globulin as prescribed for and RH negative woman 8. provide psychological support

The nurse in a labor room is preparing to care for a client with hypertonic uterine contractions. The nurse is told that the client is experiencing uncoordinated contractions that are erratic in their frequency, duration, and intensity. What is the priority nursing action? 1. provide pain relief measures 2. prepare the client for an amniotomy 3. promote ambulation every 30 min 4. monitor the oxytocin infusion closely

1. provide pain relief measures

The nurse administered magnesium sulfate to a woman experiencing preterm labor following a health care provider order. Which assessment would concern the nurse? Urine output of 50 mL Oxygen saturation of 95% 10 respirations per minute Bronchial sounds heard over the body of the sternum

10 respirations per minute The nurse assesses the woman who has been administered magnesium sulfate for respirations. Ten respirations per minute is a common hospital criteria to discontinue magnesium sulfate therapy.

What is the normal range of amniotic fluid index? 1 to 5 cm 10 to 25 cm 25 to 40 cm 40 to 65 cm

10 to 25 cm Rationale Amniotic fluid index is a biophysical profile that helps to estimate the amitotic fluid volume and fetal wellbeing. An amniotic fluid index of 10 to 25 cm is a normal finding. An amniotic fluid value of 1 to 5 cm is indicative of oligohydramnios. An amniotic fluid index of greater than 25 cm is indicative of polyhydramnios. Therefore, 25 to 40 cm or 40 to 65 cm is not a normal range of amniotic fluid index.

A client who has been diagnosed with deep vein thrombosis has been ordered to receive 12 units heparin/min. The nurse receives a 500-mL bag of D5W with 20,000 units of heparin added from the pharmacy. At what rate in mL/hr should the nurse set the infusion pump? (Calculate to the nearest whole.) __________ mL/hr.

18 mL/hr

Changing Insulin Needs During Pregnancy

1st Trimester- insulin need is reduced because of increased insulin production by pancreas 2nd Trimester-insulin needs begin to increase as placental hormones, cortisol, and insulinase act as insulin agonists 3rd Trimester- insulin needs may double or quadruple but usually level off after 36 weeks of gestation Day of birth- maternal insulin requirements decrease dramatically Breastfeeding mother maintains lower insulin requirements than prepregnancy Non-breastfeeding mother returns to prepregnancy levels in 7-10 days

The nurse is caring for a 32-week G8P7007 with placenta previa. Which of the following interventions would the nurse expect to perform? Select all that apply. 1. Daily contraction stress tests. 2. Blood type and cross match. 3. Bed rest with passive range of motion exercises. 4. Daily serum electrolyte assessments. 5. Weekly biophysical profiles.

2, 3, and 5 are correct. 1. It would be inappropriate to perform contraction stress tests. 2. There should be blood available in the blood bank in case the woman begins to bleed. 3. The nurse would expect to keep the woman on bed rest with bathroom privileges only. 4. Although important to monitor, it would be unnecessary to assess the electrolytes daily. The client is able to eat a normal diet. 5. The nurse would expect that weekly biophysical profiles would be done to assess fetal well-being. TEST-TAKING TIP: Because clients with placenta previa are at high risk for bleeding from the placental site, it is essential that they be limited in their activity and have blood on hand in case of hemorrhage. In addition, their babies must be monitored carefully for signs of fetal well-being. It would be inappropriate to stimulate contractions since dilation of the cervix would stimulate bleeding.

Methods for Screening for GDM

2-Step Screening Method -Step 1- a screen consisting of a 50 g oral glucose load followed by a plasma glucose measurement 1 hour later - a glucose value of >130-140 mg/dl is considered + - if step 1 is positive, a 3hr glucose tolerance test will be done on a different day 3 hr glucose tolerance test (100 g) -administered after an overnight fast and at least 3 days of unrestricted diet and physical activity -woman told to avoid caffeine -blood glucose levels are checked 1, 2, and 3 hours after Positive for GDM if 2+ values are met or exceeded: Fasting- 95 mg/dl or 105 mg/dl 1-hour- 180 mg/dl or 190 mg/dl 2-hour 155 mg/dl or 165 mg/dl 3 hour 140 mg/dl or 145 mg/dl 1-step method- 2-hr 75 g oral glucose tolerance test

A woman with a history of congestive heart disease is 36 weeks pregnant. Which of the following findings should the nurse report to the primary health care practitioner? 1. Presence of striae gravidarum. 2. Dyspnea on exertion. 3. 4-pound weight gain in a month. 4. Patellar reflexes +2.

2. A client who is complaining of dyspnea on exertion is likely going into left-sided congestive heart failure. TEST-TAKING TIP: It is important for the test taker to know that pregnancy is a significant stressor on the cardiac system. Women who enter the pregnancy with a history of cardiac problems must be monitored very carefully not only by the obstetric practitioner but also by an internist or cardiologist. The nurse must be vigilant in observing for signs of cardiac failure, including respiratory and systemic congestion.

he nurse turns off the oxytocin (Pitocin) infusion after a period of hyperstimulation. Which of the following outcomes indicates that the nurse's action was effective? 1. Intensity moderate. 2. Frequency every 3 minutes. 3. Duration 120 seconds. 4. Attitude flexed.

2. A frequency pattern of every 3 minutes is ideal. TEST-TAKING TIP: This question is asking the test taker to evaluate an expected outcome. When a nurse intervenes, he or she is expecting a positive outcome. In this situation, the nurse is determining whether or not the action has reversed the hyperstimulation that developed from oxytocin administration. The normal contraction frequency is evidence of a positive outcome.

A gravid client, 27 weeks' gestation, has been diagnosed with gestational diabetes. Which of the following therapies will most likely be ordered for this client? 1. Oral hypoglycemic agents. 2. Diet control with exercise. 3. Regular insulin injections. 4. Inhaled insulin.

2. About 95% of gestational diabetic clients are managed with diet and exercise alone. Gestational diabetic clients are first counseled regarding proper diet and exercise as well as blood glucose assessments. The vast majority of women are able to regulate their glucose levels with this intervention. If the glucose levels do not stabilize, the obstetrician will determine whether to order oral hypoglycemics or injectable insulin.

An insulin-dependent diabetic, G3P0200, 38 weeks' gestation, is being seen in the labor and delivery suite in metabolic dysequillibrium. The nurse knows that which of the following maternal blood values is most high risk to her unborn baby? 1. Glucose 150 mg/dL. 2. pH 7.25. 3. pCO2 34 mmHg. 4. Hemoglobin A, 10%.

2. Acidosis is fatal to the fetus. This is the most important finding. Acidosis is life threatening to the fetus. It is essential that the nurse monitor clients for situations that would put the fetus in jeopardy of being in an acidotic environment, including maternal hypoxia and diabetic ketoacidosis.

The nurse is monitoring a client in the immediate postpartum period for signs of hemorrhage. Which sign, if noted, would be an early sign of excessive blood loss? 1. A temperature of 100.4 ° F 2. An increase in the pulse rate from 88 to 102 beats/minute 3. A blood pressure change from 130/88 to 124/80 mm Hg 4. An increase in the respiratory rate from 18 to 22 breaths/minute

2. An increase in the pulse rate from 88 to 102 beats/minute During the fourth stage of labor, the maternal blood pressure, pulse, and respiration should be checked every 15 minutes during the first hour. An increasing pulse is an early sign of excessive blood loss because the heart pumps faster to compensate for reduced blood volume. A slight increase in temperature is normal. The blood pressure decreases as the blood volume diminishes, but a decreased blood pressure would not be the earliest sign of hemorrhage. The respiratory rate is slightly increased from normal.

A pregnant woman mentions to the clinic nurse that she and her husband enjoy working together on projects around the house and, "I always wear protective gloves when I work." The nurse should advise the woman that, even when wearing gloves, which of the following projects could be high risk to the baby's health? 1. Replacing a light fixture in the nursery. 2. Sanding the paint from an antique crib. 3. Planting tulip bulbs in the side garden. 4. Shoveling snow from the driveway.

2. Antique cribs are often painted with lead-based paint. This is a dangerous activity. It is very important that clients stay away from aerosolized lead that can develop when lead paint is being sanded. Lead can enter the body through the respiratory tract as well as through the gastrointestinal tract. Once it is ingested, the lead enters the vascular tree and is transported across the placenta to the unborn baby. The baby, especially the baby's central nervous system, can be severely adversely affected by the lead.

A client is being discharged on Coumadin (warfarin) post-pulmonary embolism after a cesarean delivery. Which of the following should be included in the patient teaching? 1. Take only ibuprofen for pain. 2. Avoid overeating dark green, leafy vegetables. 3. Drink grapefruit juice daily. 4. Report any decrease in urinary output.

2. Avoid overeating dark green, leafy vegetables.

A physician writes the following order—administer ampicillin 1 Gm IV q 4 h until delivery—for a newly admitted laboring client with ruptured membranes. The client had positive vaginal and rectal cultures for group B streptococcal bacteria at 36 weeks' gestation. Which of the following is a rationale for this order? 1. The client is at high risk for chorioamnionitis. 2. The baby is at high risk for neonatal sepsis. 3. The bacterium is sexually transmitted. 4. The bacterium causes puerperal sepsis

2. Babies are susceptible to neonatal sepsis from vertical transmission of the bacteria. TEST-TAKING TIP: At approximately 36 weeks' gestation, pregnant women are cultured for group B strep. If they culture positive, standard protocol is to administer a broad-spectrum antibiotic IV q 4 hours from the time her membranes rupture until delivery. That action markedly decreases the vertical transmission of the bacteria to neonates.

A nurse is interviewing a prenatal client. Which of the following factors in the client's history should the nurse highlight for the health care practitioner? 1. That she is eighteen years old. 2. That she owns a cat and a dog. 3. That she eats peanut butter every day. 4. That she works as a surgeon.

2. Cat feces are a potential source of toxoplasmosis. The nurse must be familiar with any possible circumstances that place antepartal clients and their fetuses at high risk. Toxoplasmosis is an illness caused by a protozoan. The organism can be contracted in a number of ways, including eating rare or raw meat, drinking unpasteurized goat milk, and coming in contact with cat feces. When contracted by the mother during pregnancy, it can cause serious fetal and neonatal disease.

Which of the following pregnant clients is most high risk for preterm premature rupture of the membranes (PPROM)? 1. 30-week gestation with prolapsed mitral valve (PMV). 2. 32-week gestation with urinary tract infection (UTI). 3. 33 weeks gestation with twins post-in vitro fertilization (IVF) 3. 34-week gestation with gestational diabetes (GDM). 4. 36-week gestation with deep vein thrombosis (DVT).

2. Clients with UTIs are high risk for PPROM. 3. 33 weeks gestation with twins post-in vitro fertilization (IVF) Although the exact mechanism is not well understood, clients who have urinary tract infections are high risk for PPROM. This is particularly important since pregnant clients often have urinary tract infections that present either with no symptoms at all or only with urinary frequency, a complaint of many pregnant clients.

A physician has given a nurse a verbal order to apply cricoid pressure. Which of the following is the likely indication for the action? 1. Forceps delivery. 2. Endotracheal tube insertion. 3. Epidural insertion. 4. Third stage of labor

2. Cricoid pressure is indicated during endotracheal intubation TEST-TAKING TIP: When a client is being intubated, there is a possibility that the stomach contents will be regurgitated. When the vomiting occurs, the client may aspirate the contents. Since the contents are highly acidic, the trachea and lung fields can become damaged. Cricoid pressure helps to reduce the potential for respiratory aspiration of the stomach contents.

A woman, 26 weeks' gestation, has just delivered a fetal demise. Which of the following nursing actions is appropriate at this time? 1. Remind the mother that she will be able to have another baby in the future. 2. Dress the baby in a tee shirt and swaddle the baby in a receiving blanket. 3. Ask the woman if she would like the doctor to prescribe a sedative for her. 4. Remove the baby from the delivery room as quickly as possible.

2. Dress the baby in a tee shirt and swaddle the baby in a receiving blanket.

A breastfeeding mother calls the obstetrician's office with a complaint of pain in one breast. Upon inspection, a diagnosis of mastitis is made. Which of the following nursing interventions is appropriate? 1. Advise the woman to apply ice packs to her breasts. 2. Encourage the woman to breastfeed frequently. 3. Inform the woman that she should wean immediately. 4. Direct the woman to notify her pediatrician as soon as possible.

2. Encourage the woman to breastfeed frequently.

A postpartum client is diagnosed with cystitis. The nurse should plan for which priority nursing action in the care of the client? 1. Providing sitz baths 2. Encouraging fluid intake 3. Placing ice on the perineum 4. Monitoring hemoglobin and hematocrit levels

2. Encouraging fluid intake Cystitis is an infection of the bladder. The client should consume 3000 mL of fluids per day if not contraindicated. Sitz baths and ice would be appropriate interventions for perineal discomfort. Hemoglobin and hematocrit levels would be monitored with hemorrhage.

Which of the following physical findings would lead the nurse to suspect that a client with severe preeclampsia has developed HELLP syndrome? 1. +3 pitting edema and pulmonary edema. 2. Epigastric pain and systemic jaundice. 3. +4 deep tendon reflexes and clonus. 4. Oliguria and elevated specific gravity.

2. Epigastric pain and jaundice are reflective of hemolysis of red blood cells and of severe liver damage. These symptoms should make the nurse suspect HELLP syndrome. TEST-TAKING TIP: The test taker must be able to discriminate between symptoms of severe preeclampsia and HELLP syndrome. If the nurse remembers what each of the letters in HELLP stands for, he or she can determine which of the responses is correct. Bilirubin is the byproduct of RBC breakdown and bilirubin is metabolized in the liver. Based on this information, the nurse can deduce the correct answer.

During a vaginal delivery, the obstetrician declares that a shoulder dystocia has occurred. Which of the following actions by the nurse is appropriate at this time? 1. Administer oxytocin intravenously per doctor's orders. 2. Flex the woman's thighs sharply toward her hips. 3. Apply oxygen using a tight-fitting face mask. 4. Apply downward pressure on the woman's fundus.

2. Flexing the woman's hips sharply toward her shoulders, called McRoberts' maneuver, is appropriate. TEST-TAKING TIP: Flexing the woman's hips sharply toward her shoulders increases slightly the diameter of the pelvic outlet which often enables the practitioner to successfully deliver the baby. It is especially important to note that fundal pressure is contraindicated because it may actually magnify the problem by wedging the shoulders into the pelvis even more deeply. Suprapubic pressure, on the other hand, is often helpful in assisting with the delivery

A client is 10 minutes postpartum from a forceps delivery of a 4,500-gram neonate with a cleft lip. The physician performed a right mediolateral episiotomy during the delivery. The client is at risk for each of the following nursing diagnoses. Which of the diagnoses is highest priority at this time? 1. Ineffective breastfeeding. 2. Fluid volume deficit. 3. Infection. 4. Pain.

2. Fluid volume deficit.

A nurse is caring for the following four laboring patients. Which client should the nurse be prepared to monitor closely for signs of postpartum hemorrhage (PPH)? Select all that apply. 1. G1 P0000, delivered a fetal demise at 29 weeks' gestation. 2. G2 P1001, prolonged first stage of labor. 3. G2 P0010, delivered by cesarean section for failure to progress. 4. G3 P0200, delivered vaginally a 42-week, 2,200-gram neonate. 5. G4 P3003, with a succenturiate placenta..

2. G2 P1001, prolonged first stage of labor. 5. G4 P3003, with a succenturiate placenta.

The fetal monitor tracing of a laboring woman who is 9 cm dilated shows recurring late decelerations to 100 bpm. The nurse notes a moderate amount of greenish colored amniotic fluid gush from the vagina after a practitioner performs an amniotomy. Which of the following nursing diagnoses is appropriate at this time? 1. Risk for infection related to rupture of membranes. 2. Risk for fetal injury related to possible intrauterine hypoxia. 3. Risk for impaired tissue integrity related to vaginal irritation. 4. Risk for maternal injury related to possible uterine rupture.

2. Green amniotic fluid in the presence of late decelerations is indicative of fetal distress. TEST-TAKING TIP: Late decelerations are related to poor uteroplacental blood flow. As a result of the poor blood flow, the fetus is being poorly oxygenated and nourished. Amniotic fluid becomes greentinged in the presence of meconium. Meconium is expelled in utero when the fetal anal sphincter relaxes. Sphincters relax when the body is hypoxic. The nurse, therefore, must conclude that the fetus is at high risk for injury related to intrauterine hypoxia.

A breastfeeding woman, 6 weeks postdelivery, must go into the hospital for a hemorrhoidectomy. Which of the following is the best intervention regarding infant feeding? 1. Have the woman wean the baby to formula. 2. Have the baby stay in the hospital room with the mother. 3. Have the woman pump and dump her milk for two weeks. 4. Have the baby bottle fed milk that the mother has stored.

2. Have the baby stay in the hospital room with the mother.

A client is being discharged on Coumadin (warfarin) post-pulmonary embolism after a cesarean delivery. Which of the following laboratory values indicates that the medication is effective? 1. PT (prothrombin time): 12 sec (normal is 10-13 seconds). 2. INR (international normalized ratio): 2.5 (normal is 1.0-1.4). 3. Hematocrit 55%. 4. Hemoglobin 10 g/dL.

2. INR (international normalized ratio): 2.5

A breastfeeding client is being seen in the emergency department with a hard, red, warm nodule in the upper outer quadrant of her left breast. Her vital signs are: T 104.6°F, P 100, R 20, and BP 110/60. She has a recent history of mastitis and is crying in pain. Which of the following nursing diagnoses is highest priority? 1. Ineffective breastfeeding. 2. Infection. 3. Ineffective individual coping. 4. Pain

2. Infection.

In which of the following clinical situations would it be appropriate for an obstetrician to order a labor nurse to perform amnioinfusion? 1. Placental abruption. 2. Meconium-stained fluid. 3. Polyhydramnios. 4. Late decelerations

2. It would be appropriate for a health care practitioner to order an amnioinfusion when a client's amniotic fluid is meconium stained. TEST-TAKING TIP: Amnioinfusion is the instillation of intravenous fluid into the uterine cavity through intravenous tubing inserted via the vagina. It may be ordered if the amniotic fluid is meconium stained. The infusion would dilute the concentration of meconium in order to decrease the potential of the baby aspirating large quantities of meconium at birth.

A physician has notified the labor and delivery suite that four clients will be admitted to the unit. The client with which of the following clinical findings would be a candidate for an external version? 1. +3 station. 2. Left sacral posterior position. 3. Flexed attitude. 4. Rupture of membranes for 24 hours.

2. LSP position is a breech presentation. It may be appropriate for a physician to perform an external version prior to this delivery. TEST-TAKING TIP: If a baby is in the breech presentation, the version would have to be performed before the baby had engaged. Once the baby is well established in the true pelvis, it is at high risk for the baby to be moved.

A doctor orders a narcotic analgesic for a laboring client. Which of the following situations would lead a nurse to hold the medication? 1. Contraction pattern is every 3 min 60 sec. 2. Fetal monitor tracing shows late decelerations. 3. Client sleeps between contractions. 4. The blood pressure is 150/90.

2. Late decelerations are indicative of uteroplacental insufficiency and indicate fetal distress. It is inappropriate to administer a central nervous system (CNS) depressant to the mother at this time TEST-TAKING TIP: Analgesics are central nervous system (CNS) depressants. They not only depress the CNS of the mother, reducing her pain, but also depress the CNS of the baby. It is inappropriate to administer a depressant to a mother whose fetus is already exhibiting signs of distress. First, the variability of the baseline would be diminished, preventing the nurse from assessing that very important indicator of fetal well-being. And if the baby were to be delivered via cesarean section, the baby would likely be depressed and in need of resuscitation.

nurse is caring for a client, PP2, who is preparing to go home with her infant. The nurse notes that the client's blood type is O− (negative), the baby's type is A+ (positive), and the direct Coombs' test is negative. Which of the following actions by the nurse is appropriate? 1. Advise the client to keep her physician appointment at the end of the week to receive her RhoGAM injection. 2. Make sure that the client receives a RhoGAM injection before she is discharged from the hospital. 3. Notify the client that because her baby's Coombs' test was negative she will not receive an injection of RhoGAM. 4. Inform the client's physician that because the woman is being discharged on the second day, the RhoGAM could not be given.

2. Make sure that the client receives a RhoGAM injection before she is discharged from the hospital.

A client is admitted in labor with spontaneous rupture of membranes 24 hours earlier. The fluid is clear and the fetal heart rate is 124 with moderate variability. Which assessment is most important for the nurse to make at this time? 1. Contraction frequency and duration. 2. Maternal temperature. 3. Cervical dilatation and effacement. 4. Maternal pulse rate.

2. Maternal temperature is the highest priority. TEST-TAKING TIP: The test taker must remember that the uterine cavity is a sterile space while the vaginal vault is an unsterile space. When membranes have ruptured over 24 hours, there is potential for pathogens to ascend into the uterine cavity and infection to result. Elevated temperature is a sign of infection.

A woman with a diagnosis of ectopic pregnancy is to receive medical intervention rather than a surgical interruption. Which of the following intramuscular medications would the nurse expect to administer? 1. Decadron (desamethasone). 2. Arnethopterin (methotrexate). 3. Pergonal (menotropins). 4. Prometrium (progesterone).

2. Methotrexate is the likely medication. TEST-TAKING TIP: Methotrexate is an antineoplastic agent. Even if the test taker were unfamiliar with its use in ectopic pregnancy but was aware of the action of methotrexate, he or she could deduce its efficacy here. Methotrexate is a folic acid antagonist that interferes with DNA synthesis and cell multiplication. The conceptus is a ball of rapidly multiplying cells. Methotrexate interferes with that multiplication, killing the conceptus and, therefore, precluding the need for the client to undergo surgery.

client's vital signs and reflexes were normal throughout pregnancy, labor, and delivery. Four hours after delivery the client's vitals are 98.6°F, P 72, R 20, BP 150/100, and her reflexes are 4+. She has an intravenous infusion running with 20 units of Pitocin (oxytocin) added. Which of the following actions by the nurse is appropriate? 1. Nothing, because the results are normal. 2. Notify the obstetrician of the findings. 3. Discontinue the intravenous immediately. 4. Reassess the client after fifteen minutes.

2. Notify the obstetrician of the findings.

The nurse is performing a postpartum assessment on a client who delivered 4 hours ago. The nurse notes a firm uterus at the umbilicus with heavy lochial flow. Which of the following nursing actions is appropriate? 1. Massage the uterus. 2. Notify the obstetrician. 3. Administer an oxytocic as ordered. 4. Assist the client to the bathroom.

2. Notify the obstetrician.

A nurse who is called to a client's room notes that the client's cesarean incision has separated. Which of the following actions is the highest priority for the nurse to perform? 1. Cover the wound with sterile wet dressings. 2. Notify the surgeon. 3. Elevate the head of the client's bed slightly. 4. Flex the client's knees.

2. Notify the surgeon.

An obese gravid woman is being seen in the prenatal clinic. The nurse will monitor this client carefully throughout her pregnancy because she is high risk for which of the following complications of pregnancy? 1. Placenta previa. 2. Gestational diabetes. 3. DVT 4. Preeclampsia 5. Chromosomal defects.

2. Obese clients are at high risk for gestational diabetes. 3. DVT 4. Preeclampsia TEST-TAKING TIP: Because clients who enter pregnancy obese are at such high risk for gestational diabetes, many obstetricians skip the glucose challenge test and automatically schedule a glucose tolerance test at approximately 24 weeks' gestation. As a result, the complication is discovered much earlier and intervention can begin much earlier.

The nurse is caring for a 30-week-gestation client whose fetal fibronectin (fFN) levels are positive. It is essential that she be taught about which of the following? 1. How to use a blood glucose monitor. 2. Signs of preterm labor. 3. Signs of preeclampsia. 4. How to do fetal kick counts.

2. Positive fetal fibronectin levels are seen in clients who deliver preterm. TEST-TAKING TIP: Fetal fibronectin (fFN) is a substance that is metabolized by the chorion. Although positive during the first half of pregnancy, it is very rare to see positive results between 24 and 34 weeks' gestation unless the client's cervix begins to efface and dilate. It is an excellent predictor of preterm labor (PTL); therefore, many practitioners assess the cervical and vaginal secretions of women at high risk for PTL for the presence of fFN.

A client just delivered the placenta pictured below. For which of the following complications should the nurse carefully observe the woman? 374 MATERNAL AND NEWBORN SUCCESS 1. Endometrial ischemia. 2. Postpartum hemorrhage. 3. Prolapsed uterus. 4. Vaginal hematoma

2. Postpartum hemorrhage

The nurse assesses a 2-day postpartum, breastfeeding client. The nurse notes blood on the mother's breast pad and a crack on the mother's nipple. Which of the following actions should the nurse perform at this time? 1. Advise the woman to wash the area with soap to prevent mastitis. 2. Provide the woman with a tube of topical lanolin. 3. Remind the woman that the baby can become sick if he drinks the blood. 4. Get the woman an order for a topical anesthetic.

2. Provide the woman with a tube of topical lanolin.

A client is 1 day post-cesarean section with spinal anesthesia. Even though the nurse advised against it, the client has had the head of her bed in high Fowler's position since delivery. Which of the following complications would the nurse expect to see in relation to the client's action? 1. Postpartum hemorrhage. 2. Severe postural headache. 3. Pruritic skin rash. 4. Paralytic ileus.

2. Severe postural headache.

The nurse is monitoring a woman, G2P1001, 41 weeks' gestation, in labor. A 12 p.m. assessment revealed: cervix, 4 cm; 80% effaced; -3 station; and FH 124 with moderate variability. A 5 p.m. assessment: cervix, 6 cm; 90% effaced; -3 station; and FH 120 with moderate variability. A 10 p.m. assessment: cervix, 8 cm; 100% effaced; -3 station; and FH 124 with moderate variability. Based on the assessments, which of the following should the nurse conclude? 1. Labor is progressing well. 2. The woman is likely carrying a macrosomic fetus. 3. The baby is in fetal distress. 4. The woman will be in second stage in about five hours.

2. Since the presenting part is not descending into the birth canal, the nurse can logically conclude that the baby is macrosomic. TEST-TAKING TIP: The test taker must carefully analyze the results of the three vaginal examinations. The fetal heart is virtually unchanged: the rate is within normal limits and the variability is normal. There is no sign of fetal distress. The dilation and effacement are changing, but the lack of progressive descent of the presenting part is unexpected. When babies are too big to fit through a client's pelvis, they fail to descend. That is the conclusion that the nurse must make from the findings.

A nurse notes a sinusoidal fetal heart pattern while analyzing a fetal heart tracing of a newly admitted client. Which of the following actions should the nurse take at this time? 1. Encourage the client to breathe with contractions. 2. Notify the practitioner. 3. Increase the intravenous infusion. 4. Encourage the client to push with contractions.

2. Sinusoidal patterns are related to Rh isoimmunization, fetal anemia, severe fetal hypoxia, or a chronic fetal bleed. They also may occur transiently as a result of Demerol (meperidine) or Stadol (butorphanol) administration. Since this client has just been admitted, medication administration is not a likely cause. The health care practitioner should be notified. TEST-TAKING TIP: Sinusoidal fetal heart patterns exhibit no variability and have a uniform wave-like pattern (see below). The nurse would note no periods when the heart rate appears normal. The fetus is in imminent danger. The practitioner must be notified as soon as possible so that he or she can determine the appropriate intervention.

A couple, accompanied by their 5-year-old daughter, have been notified that their 32-week-gestation fetus is dead. The father is yelling at the staff. The mother is crying uncontrollably. The 5-year-old is banging the head of her doll on the floor. Which of the following nursing actions is appropriate at this time? 1. Tell the father that his behavior is inappropriate. 2. Sit with the family and quietly communicate sorrow at their loss. 3. Help the couple to understand that their daughter is acting inappropriately. 4. Encourage the couple to send their daughter to her grandparents.

2. Sit with the family and quietly communicate sorrow at their loss.

A woman, 39 weeks' gestation, is admitted to the delivery unit with vaginal warts from human papillomavirus. Which of the following actions by the nurse is appropriate? 1. Notify the health care practitioner for a surgical delivery. 2. Follow standard infectious disease precautions. 3. Notify the nursery of the imminent delivery of an infected neonate. 4. Wear a mask whenever the perineum is exposed

2. Standard precautions are indicated in this situation. TEST-TAKING TIP: Although HPV is a sexually transmitted infection and it can, in rare instances, be contracted by the neonate from the mother, the Centers for Disease Control and Prevention do not recommend that cesarean section be performed merely to prevent vertical transmission of HPV (http://www.cdc. gov/std/HPV/common-infection/CDC_ HPV_ClinicianBro_LR.pdf, p 10)

A client has been admitted with a diagnosis of hyperemesis gravidarum. Which of the following orders written by the primary health care provider is highest priority for the nurse to complete? 1. Obtain complete blood count. 2. Start intravenous with multivitamins. 3. Check admission weight. 4. Obtain urine for urinalysis.

2. Starting an intravenous with multivitamins takes priority. TEST-TAKING TIP: Clients who are vomiting repeatedly are energy depleted, vitamin depleted, and electrolyte depleted and are often dehydrated. It is essential that the client receive her IV therapy as quickly as possible. The other orders should be completed soon after the IV is started.

A postpartum client has been diagnosed with deep vein thrombosis. For which of the following additional complications is this client high risk? 1. Hemorrhage. 2. Stroke. 3. Endometritis. 4. Hematoma.

2. Stroke.

The nurse has administered Benadryl (diphenhydramine) to a post-cesarean client who is experiencing side effects from the parenteral morphine sulfate that was administered 30 minutes earlier. Which of the following actions should the nurse perform following the administration of the drug? 1. Monitor the urinary output hourly. 2. Supervise while the woman holds her newborn. 3. Position the woman slightly elevated on her left side. 4. Ask any visitors to leave the room.

2. Supervise while the woman holds her newborn.

At 28 weeks' gestation, an Rh-negative woman receives RhoGAM. Which of the following would indicate that the medication is effective? 1. The baby's Rh status changes to Rh negative. 2. The mother produces no Rh antibodies. 3. The baby produces no Rh antibodies. 4. The mother's Rh status changes to Rh positive.

2. That the mother produces no Rh antibodies is the goal of RhoGAM administration. The test taker should review the immune response to an antigen. In this situation, the antigen is the baby's Rh(+) blood. It can leak into the maternal blood stream from the fetal blood stream at various times during the pregnancy. Most commonly it happens at the time of placental delivery. Because the mother is antigen negative-that is, Rh(-), when exposed to Rh(+) blood, her immune system develops antibodies. RhoGAM is composed of Rh(+) antibodies. It acts as passive immunity. Because antibodies are already present in the mother's bloodstream, her immune system is suppressed and fails to develop antibodies via the active immune response.

A lecithin:sphingomyelin (L/S) ratio has been ordered by a pregnant woman's obstetrician. Which of the following data will the nurse learn from this test? 1. Coagulability of maternal blood. 2. Maturation of the fetal lungs. 3. Potential for fetal development of erythroblastosis fetalis. 4. Potential for maternal development of gestational diabetes.

2. The L/S ratio indicates the maturity of the fetal lungs. Lecithin and sphingomyelin are two components of surfactant, the slippery substance that lines the alveoli. The fetal lungs have usually reached maturation when the ratio of the substances is 2:1 or higher. In order to perform the test, the obstetrician must obtain amniotic fluid during an amniocentesis. A quick test, called a shake or foam test, can also be performed on the amniotic fluid to assess fetal lung maturation. (It is important to note that even with an L/S ratio above 2:1, the lungs of fetuses of diabetic mothers are often immature.)

Which of the following is the appropriate nursing care outcome for a woman who suddenly develops an amniotic fluid embolism during her labor? 1. Client will be infection-free at discharge. 2. Client will exhibit normal breathing function at discharge. 3. Client will exhibit normal gastrointestinal function at discharge. 4. Client will void without pain at discharge.

2. The appropriate nursing care outcome is that the client survives and is breathing normally at discharge TEST-TAKING TIP: At the time of placental separation or sometimes during stage 1 of labor, a small amount of amniotic fluid sometimes seeps into the mother's bloodstream via the chorionic villi. With the contraction of the uterus, the fluid is shunted into the peripheral circulation and forced into the woman's lung fields. If there is meconium or other foreign material in the fluid, the woman's prognosis declines. Women who experience forceful, rapid labors are especially at risk for this life-threatening complication.

A nurse is monitoring the labor of a client who is receiving IV oxytocin (Pitocin) at 6 mL per hour. Which of the following clinical signs would lead the nurse to stop the infusion? 1. Change in maternal pulse rate from 76 to 98 bpm. 2. Change in fetal heart rate from 128 to 102 bpm. 3. Maternal blood pressure of 150/100. 4. Maternal temperature of 102.4ºF.

2. The baseline fetal heart rate has dropped over 20 bpm. This finding warrants that the oxytocin be stopped. TEST-TAKING TIP: The test taker must determine which of the vital signs is unsafe in the presence of oxytocin. Oxytocin increases the contractility of the uterine muscle. When the muscle contracts, the blood supply to the fetus is diminished. A drop in fetal heart rate, therefore, is indicative of poor oxygenation to the fetus and is unsafe in the presence of oxytocin.

A gestational diabetic, who requires insulin therapy to control her blood glucose levels, telephones the triage nurse complaining of dizziness and headache. Which of the following actions should the nurse take at this time? 1. Have the client proceed to the office to see her physician. 2. Advise the client to drink a glass of milk and then call back. 3. Instruct the client to inject herself with regular insulin. 4. Tell the client immediately to telephone her medical doctor.

2. The client should drink a full glass of milk. TEST-TAKING TIP: Because the signs and symptoms of hyperglycemia and hypoglycemia are very similar, it is important for the nurse to err on the side of caution. If the client should be hypoglycemic, this is a medical emergency. Drinking a glass of milk will stabilize the glucose in the woman's body. If she is hyperglycemic, the milk may increase the glucose levels, but not significantly. A blood glucose assessment can be done and insulin can be administered, if needed, shortly after consuming the milk.

A client, G2P1001, telephones the gynecology office complaining of left-sided pain. Which of the following questions by the triage nurse would help to determine whether the one-sided pain is due to an ectopic pregnancy? 1. "When did you have your pregnancy test done?" 2. "When was the first day of your last menstrual period?" 3. "Did you have any complications with your first pregnancy?" 4. "How old were you when you first got your period?"

2. The date of the last menstrual period will assist the nurse in determining how many weeks pregnant the client is. The date of the last menstrual period is important for the nurse to know. Ectopic pregnancies are usually diagnosed between the 8th and the 9th week of gestation because, at that gestational age, the conceptus has reached a size that is too large for the fallopian tube to hold.

In analyzing the need for health teaching in a client, G5P4004 with gestational diabetes, the nurse should ask which of the following questions? 1. "How old were you at your first pregnancy?" 2. "Do you exercise regularly?" 3. "Is your partner diabetic?" 4. "Do you work outside of the home?"

2. The likelihood of developing either gestational or type 2 diabetes is reduced when clients exercise regularly. There are a number of issues that the nurse should discuss with a client who has been diagnosed with gestational diabetes. The need for exercise is one of those topics. Other topics are diet, blood glucose testing, treatment for hypoglycemic episodes, and the like.

A 24-week-gravid client is being seen in the prenatal clinic. She states, "I have had a terrible headache for the past 2 days." Which of the following is the most appropriate action for the nurse to perform next? 1. Inquire whether or not the client has allergies. 2. Take the woman's blood pressure. 3. Assess the woman's fundal height. 4. Ask the woman about stressors at work.

2. The nurse should assess the client's blood pressure. Headache is a symptom of preeclampsia. Preeclampsia, a serious complication, is a hypertensive disease of pregnancy. In order to determine whether or not the client is preeclamptic, the next action by the nurse would be to assess the woman's blood pressure.

A client, G8P3406, 14 weeks' gestation, is being seen in the prenatal clinic. During the nurse's prenatal teaching session, the nurse will emphasize that the woman should notify the obstetric office immediately if she notes which of the following? 1. Change in fetal movement. 2. Signs and symptoms of labor. 3. Swelling of feet and ankles. 4. Appearance of spider veins.

2. The nurse should emphasize the need for the client to notify the office of signs of preterm labor. The test taker must be able to interpret a client's gravidity and parity. The letter "G" stands for gravid, or the number of pregnancies. The letter "P" stands for para, or the number of deliveries. The delivery information is further distinguished by 4 separate numbers: the first refers to the number of full-term pregnancies the client has had, the second refers to the number of preterm pregnancies the client has had, the third refers to the number of abortions the client has had (any pregnancy loss before 20 weeks' gestation), and the fourth refers to the number of living children that the client currently has. The client in the scenario, therefore, has had 8 pregnancies (she is currently pregnant) with 3 full-term deliveries, 4 preterm deliveries, no abortions, and she currently has 6 living children.

The nurse identifies the following nursing diagnosis for a client undergoing an emergency cesarean section: Risk for ineffective individual coping related to emergency procedure. Which of the following nursing interventions would be appropriate in relation to this diagnosis? 1. Apply antiembolic boots bilaterally. 2. Explain all procedures slowly and carefully. 3. Administer an antacid per MD orders. 4. Monitor the FH and maternal vital signs.

2. The nurse should explain all procedures slowly and carefully. TEST-TAKING TIP: Whenever a question is asked, the test taker must attend to the content of the question. All of the responses are appropriate in relation to cesarean deliveries but only response "2" is related to the diagnosis of risk for ineffective individual coping.

A known drug addict is in active labor. She requests pain medication. Which of the following actions by the nurse is appropriate? 1. Encourage the woman to refrain from taking medication to protect the fetus. 2. Notify the physician of her request. 3. Advise the woman that she can receive only an epidural because of her history. 4. Assist the woman to do labor breathing.

2. The nurse should notify the health care practitioner of the client's request TEST-TAKING TIP: The test taker should be aware of two important facts: pain is the fifth vital sign as identified by the Joint Commission on Accreditation of Healthcare Organizations (JCAHO) and actions must be taken to reduce drug abusers' pain in the same manner that non-drug abusers' pain is managed. Although it is strongly discouraged for women to take illicit drugs when pregnant, the nurse must maintain his or her caring philosophy and provide unbiased care to addicted clients.

An obstetrician declares at the conclusion of the third stage of labor that a woman is diagnosed with placenta accreta. The nurse would expect to see which of the following signs/symptoms? 1. Hypertension. 2. Hemorrhage. 3. Bradycardia. 4. Hyperthermia.

2. The nurse would expect the woman to hemorrhage TEST-TAKING TIP: A placenta accreta is present when the chorionic villi attach directly to or invade through the myometrium of the uterus. There is no way, therefore, for the placenta to separate from the uterine wall. Hemorrhage results. It is not uncommon for a hysterectomy to have to be performed in order to save the woman's life.

An ultrasound has identified that a client's pregnancy is complicated by hydramnios. The nurse would expect that an ultrasound may show that the baby has which of the following structural defects? 1. Pulmonic stenosis. 2. Tracheoesophageal fistula. 3. Ventriculoseptal defect. 4. Developmental hip dysplasia.

2. The nurse would expect to find that the baby has tracheoesophageal fistula. Babies swallow the amniotic fluid while in utero. When there is a surplus of fluid, ultrasounds may be performed to check for defects in the fetal gastrointestinal system

A 29-week-gestation woman diagnosed with severe preeclampsia is noted to have blood pressure of 170/112, 4+ proteinuria, and a weight gain of 10 pounds over the last 2 days. Which of the following signs/symptoms would the nurse also expect to see? 1. Fundal height of 32 cm. 2. Papilledema. 3. Patellar reflexes of +2. 4. Nystagmus.

2. The nurse would expect to see papilledema. TEST-TAKING TIP: Intracranial pressure (ICP) is present in a client with severe preeclampsia because she is third spacing large quantities of fluid. As a result of the ICP, the optic disk swells and papilledema is seen when the disk is viewed through an ophthalmoscope.

A client's assessments reveal that she is 4 cm dilated and 80% effaced with a fetal heart tracing showing frequent late decelerations and strong contractions every 3 minutes, each lasting 90 seconds. The nursing management of the client should be directed toward which of the following goals? 1. Completion of the first stage of labor. 2. Delivery of a healthy baby. 3. Safe pain medication management. 4. Prevention of a vaginal laceration.

2. The nurse's goal at this point must be the delivery of a healthy baby. TEST-TAKING TIP: Nursing goals may change repeatedly during a client's labor. The nurse must assess the woman's progress in relation to the health and well-being of the fetus. As long as the baby is responding well, the nurse's focus should relate to maternal comfort and care. Once fetal compromise is noted, however, nursing actions often shift.

An induction of a 42-week gravida with IV oxytocin (Pitocin) is begun at 0900 at a rate of 0.5 milliunits per minute. The woman's primary physician orders: Increase the oxytocin drip by 0.5 milliunits per minute every 10 minutes until contractions are every 3 minutes 60 seconds. The nurse refuses to comply with the order. Which of the following is the rationale for the nurse's action? 1. Fetal distress has been noted in labors when oxytocin dosages greater than 2 milliunits per minute are administered. 2. The relatively long half-life of oxytocin can result in unsafe intravascular concentrations of the drug. 3. It is unsafe practice to administer oxytocin intravenously to a woman who is carrying a postdates fetus. 4. A contraction duration of 60 seconds can lead to fetal compromise in a baby that is postmature

2. The practitioner should increase the dosage of oxytocin at a minimum time interval of every 30 minutes. TEST-TAKING TIP: The half-life (the time it takes half of a medication to be metabolized by the body) of oxytocin is relatively long—about 15 minutes. And at least 3 half lives usually elapse before therapeutic responses are noted. Increasing the infusion rate too rapidly, therefore, can lead to hyperstimulation of the uterine muscle and consequent fetal distress.

A gravid client is admitted with a diagnosis of third-trimester bleeding. The nurse must carefully monitor for a change in which of the following vital signs? 1. Temperature. 2. Pulse. 3. Respirations. 4. Blood pressure.

2. The pulse is the highest priority in this situation. The key to answering this question is the fact that the nursing care plan is for a client with third-trimester bleeding. By the end of the second trimester, pregnant women have almost doubled their blood volume. Because of this, if they bleed, they are able to maintain their blood pressures for a relatively long period of time. Their pulse rates, however, do rise. Nurses, therefore, must carefully attend to the pulse rate of pregnant women who have been injured or who are being observed for third-trimester bleeding. A drop in blood pressure is a very late and ominous sign.

The nurse is caring for an eclamptic client. Which of the following is an important action for the nurse to perform? 1. Check each urine for presence of ketones. 2. Pad the client's bed rails and head board. 3. Provide visual and auditory stimulation. 4. Place the bed in the high Fowler's position.

2. The side rails of eclamptic clients' beds should be padded TEST-TAKING TIP: Eclamptic clients have had at least one seizure. To protect them from injury during any potential subsequent seizures, the nurse should pad the head board and the side rails of the client's bed.

A woman with postpartum depression has been prescribed Zoloft (sertraline) 50 mg daily. Which of the following should the client be taught about the medication? 1. Chamomile tea can potentiate the affect of the drug. 2. Therapeutic effect may be delayed a week or more. 3. The medication should only be taken whole. 4. A weight gain of up to ten pounds is commonly seen

2. Therapeutic effect may be delayed a week or more.

A client's 32-week clinic assessment was: BP 90/60; TPR 98.6°F, P 92, R 20; weight 145 lb; and urine negative for protein. Which of the following findings at the 34-week appointment should the nurse highlight for the certified nurse midwife? 1. BP 110/70; TPR 99.2°F, 88, 20. 2. Weight 155 lb; urine protein +2. 3. Urine protein trace; BP 88/56. 4. Weight 147 lb; TPR 99.0°F, 76,18

2. There has been a 10-lb weight gain in 2 weeks and a significant amount of protein is being spilled in the urine. This client should be brought to the attention of the primary caregiver. There is a great deal of information included in this question. The test taker must methodically assess each of the pieces of data. Important things to attend to are the timing of the appointments-2 weeks apart; changes in vital signs-it is normal for pulse and respiratory rate to increase slightly and BP to drop slightly; changes in urinary protein-trace is normal, +2 is not normal; and changes in weight-2-lb increase over 2 weeks is normal, a 10-lb increase is not normal.

A woman, G4P0210 and 12 weeks' gestation, has been admitted to the labor and delivery suite for a cerclage procedure. Which of the following long-term outcomes is appropriate for this client? 1. The client will gain less than 25 pounds during the pregnancy. 2. The client will deliver after 37 weeks' gestation. 3. The client will have a normal blood glucose throughout the pregnancy. 4. The client will deliver a baby that is appropriate for gestational age.

2. This client is at high risk for pregnancy loss. This is an appropriate long-term goal. 2. This client is at high risk for pregnancy loss. This is an appropriate long-term goal.

A 30-year-old gravida, G3P1101, 6 weeks' gestation, states that her premature baby boy, born 8 years ago, died shortly after delivery from an infection secondary to spina bifida. Which of the following interventions is most important for this client? 1. Grief counseling. 2. Nutrition counseling. 3. Infection control counseling. 4. Genetic counseling.

2. This client is in need of nutrition counseling. There is a strong association between low folic acid intake during the first trimester of pregnancy and spina bifida, a neural tube defect. It is very important that all clients, and especially clients with a family or personal history of a neural tube defect, consume adequate amounts of folic acid during their pregnancies. It is recommended that all women consume at least 600 micrograms of the vitamin per day. To that end, to prevent neural tube defects, it is recommended that pregnant women with no family history take a supplement of 400 micrograms per day, while pregnant women with a family history take a supplement that is 10 times the standard dose, or 4 mg per day.

A client's admitting medical diagnosis is third-trimester bleeding: rule out placenta previa. Each time the nurse enters the client's room, the woman asks: "Please tell me, do you think the baby will be all right?" Which of the following is an appropriate nursing diagnosis for this client? 1. Hopelessness related to possible fetal loss. 2. Anxiety related to unidentified diagnosis. 3. Situational low self-esteem related to blood loss. 4. Potential for altered parenting related to inexperience.

2. This client is very anxious. TEST-TAKING TIP: Situational crises arise when problems occur unexpectedly. And crises are often intensified when information is lacking. In this situation, the exact diagnosis is unknown. The client is exhibiting her fright and concern by repeatedly asking the nurse his or her opinion of the baby's health.

The physician has ordered Prepidil (dinoprostone) for 4 gravidas at term. The nurse should question the order for which of the women? 1. Primigravida with Bishop score of 4. 2. Multigravida with late decelerations. 3. G1P0000 contracting every 20 minutes 30 seconds. 4. G6P3202 with blood pressure 140/90 and pulse 92.

2. This client's fetus is already showing signs of fetal distress. Induction increases the risk of fetal injury. TEST-TAKING TIP: It is important to remember that although the fetus of a pregnant woman may be at term, it is not always safe for labor contractions to be stimulated. Although Prepedil is not directly used for induction, it is an agent that promotes cervical ripening in preparation for labor. A baby who is exhibiting signs of poor uteroplacental blood flow is likely to be compromised further by the addition of the medication

A client is being admitted to the labor suite with a diagnosis of eclampsia. Which of the following actions by the nurse is appropriate at this time? 1. Tape a tongue blade to the head of the bed. 2. Pad the side rails and head of the bed. 3. Provide the client with needed stimulation. 4. Provide the client with grief counseling.

2. This is appropriate. The side rails and the head board should be padded. When a client has been diagnosed with eclampsia, she has already had at least one seizure. The nurse, therefore, must be prepared to care for the client during another seizure. The most important action during the seizure is to protect the client from injury. Padding the side rails and head board will provide that protection.

A diabetic client is to receive 5 units regular and 15 units NPH insulin at 0800. In order to administer the medication appropriately, what should the nurse do? 1. Draw 5 units regular in one syringe and 15 units NPH in a second syringe and inject in different locations. 2. Draw 5 units regular first and 15 units NPH second into the same syringe and inject. 3. Draw 15 units NPH first and 5 units regular second into the same syringe and inject. 4. Mix 5 units regular and 15 units NPH in a vial before drawing the full 20 units into a syringe and inject.

2. This is the appropriate method. The regular insulin should be drawn up first and then the NPH insulin in the same syringe. TEST-TAKING TIP: The nurse must be familiar with the appropriate method for administering medications. Insulin must be drawn up in the correct sequence: regular insulin first and NPH insulin second.

A client is being taught fetal kick counting. Which of the following should be included in the patient teaching? 1. The woman should choose a time when her baby is least active. 2. The woman should lie on her side with her head elevated about 30°. 3. The woman should report fetal kick counts of greater than 10 in an hour. 4. The woman should refrain from eating immediately before counting.

2. This is the best position for perfusing the placenta. TEST-TAKING TIP: Since the goal of fetal kick counting is to monitor fetal well-being, it is best to do the test when the baby is most active and is most likely to be well nourished and well oxygenated. Many women find that the best time for the assessment is immediately after a meal.

A client has been receiving magnesium sulfate for severe preeclampsia for 12 hours. Her reflexes are 0 and her respiratory rate is 10. Which of the following situations could be a precipitating factor in these findings? 1. Apical heart rate 104 bpm. 2. Urinary output 240 mL/12 hr. 3. Blood pressure 160/120. 4. Temperature 100°F.

2. Urinary output 240 mL/12 hr.

full-term client, contracting every 15 min X 30 sec, has had ruptured membranes for 20 hours. Which of the following nursing interventions is contraindicated at this time? 1. Intermittent fetal heart auscultation. 2. Vaginal examination. 3. Intravenous fluid administration. 4. Nipple stimulation..

2. Vaginal examination is contraindicated. TEST-TAKING TIP: The client in this scenario is at risk of an ascending infection from the vagina to the uterine body because she has prolonged rupture of membranes. Any time a vaginal examination is performed, the chance of infection rises. Nipple stimulation is appropriate since endogenous oxytocin will be released which would augment the client's weak labor pattern

Which of the following findings should the nurse expect when assessing a client, first trimester, with gestational trophoblastic disease (hydatiform mole) that the nurse would not expect to see when assessing a first-trimester gravida with a normal pregnancy? 1. hematocrit 39% 2. grape-like clusters passed from the vagina 3. markedly elevated blood pressure 4. white blood cell count 8000 5. hypertrophied brest tissue

2. grape-like clusters passed from the vagina 3. markedly elevated blood pressure

The nurse in the postpartum unit is caring for a client who has just delivered a newborn infant following a pregnancy with placenta previa. The nurse reviews the plan of care and prepares to monitor the client for which risk associated with placenta previa? 1. infection 2. hemorrhage 3. chronic hypertension 4. disseminated intravascular coagulation

2. hemorrhage

The nurse has created a plan of care for a client experiencing dystocia and includes several nursing actions in the plan of care. What is the priority nursing action? 1. providing comfort measures 2. monitoring the fetal HR 3. changing the client's position frequently 4. keeping the significant other informed of the progress of labor

2. monitoring the fetal HR

The maternity nurse is preparing for the admission of a client in the third trimester of pregnancy who is experiencing vaginal bleeding and has a suspected diagnosis of placenta previa. The nurse reviews the HCP prescriptions and should question which prescription? 1. prepare the client for an ultrasound 2. obtain equipment for a manual pelvic exam 3. prepare to draw a hemoglobin and hematocrit blood sample 4. obtain equipment for external electronic fetal heart rate monitoring

2. obtain equipment for a manual pelvic exam

The nurse in a labor room is performing a vaginal assessment on a pregnant client in labor. The nurse notes the presence of the umbilical cord protruding from the vagina. What is the first nursing action with this finding? 1. gently push the cord into the vagina 2. place the client in Trendelenburg position 3. find the closest telephone and page the health care provider stat 4. call the delivery room to notify the staff that the client will be transported immediately

2. place the client in Trendelenburg position

A nurse is caring for four prenatal clients in the clinic. Which of the clients is high risk for placenta previa? 1. Jogger with low body mass index. 2. primagravida who smokes 1 pack of cigarettes per day 3. infectility client who is carrying in vitro triplets. 4. Registered professional nurse who works 12 hrs shifts. 5. Police officer on foot patrol.

2. primagravida who smokes 1 pack of cigarettes per day 3. infectility client who is carrying in vitro triplets.

The nurse is performing an assessment on a client who has just been told that a pregnancy test is positive. Which assessment finding indicates that the client is at risk for preterm labor? 1. the client is a 35 yr old primigravida 2. the client has a history of cardiac disease 3. the client's hemoglobin level is 13.5 g/dL 4. the client is a 20 yr old primigravida of average age weight and height

2. the client has a history of cardiac disease

The nurse is assessing a pregnant client in the second trimester of pregnancy who was admitted to the maternity unit with a suspected diagnosis of abruptio placentae. Which assessment finding should the nurse expect to note if this condition is present? 1. soft abdomen 2. uterine tenderness 3. absence of abdominal pain 4. painless, bright red vaginal bleeding

2. uterine tenderness

Cloxacillin 500 mg by mouth four times per day for 10 days has been ordered for a client with a breast abscess. The client states that she is unable to swallow pills. The oral solution is available as 125 mg/5 mL. How many mL of medicine should the woman take per dose? (Calculate to the nearest whole.) ______ mL per dose.

20 mL

The doctor writes the following order for a 31-week-gravid client with symptomatic placenta previa: weigh all vaginal pads and estimate blood loss. The nurse weighs one of the client's saturated pads at 24 grams and a dry pad at 4 grams. How many milliliters (mL) of blood can the nurse estimate the client has bled? __________ mL

20 mL of blood

A gestational diabetic client, who delivered yesterday, is currently on the postpartum unit. Which of the following statements is appropriate for the nurse to make at this time? 1. "Monitor your blood glucose five times a day until your 6-week checkup." 2. "I will teach you how to inject insulin before you are discharged." 3. "Daily exercise will help to prevent you from becoming diabetic in the future." 4. "Your baby should be assessed every 6 months for signs of juvenile diabetes."

3. "Daily exercise will help to prevent you from becoming diabetic in the future."

A breastfeeding woman calls the pediatric nurse with the following complaint: "I woke up this morning with a terrible cold. I don't want my baby to get sick. Which kind of formula should I have my husband feed the baby until I get better?" Which of the following replies by the nurse is appropriate at this time? 1. "Any formula brand is satisfactory, but it is essential that it be mixed with water that has been boiled for at least 5 minutes." 2. "Don't forget to pump your breasts every 3 hours while the baby is being fed the prescribed formula." 3. "The best way to keep your baby from getting sick is for you to keep breastfeeding him rather than switching him to formula." 4. "In addition to giving the baby formula, you should wear a surgical face mask when you are around him."

3. "The best way to keep your baby from getting sick is for you to keep breastfeeding him rather than switching him to formula."

The nurse is grading a woman's reflexes. Which of the following grades would indicate reflexes that are slightly brisker than normal? 1. + 1. 2. +2. 3. +3. 4. +4.

3. +3 reflexes are defined as slightly brisker than normal or slightly hyperreflexic. Although, as seen above, a clear categorization of reflex assessment exists, the value assigned to a reflex by a clinician does have a subjective component. Therefore, it is recommended that at the change of shift both the new and departing nurses together assess the reflexes of a client who has suspected abnormal reflexes. A common understanding of the reflex assessment can then be determined.

During the delivery of a macrosomic baby, the woman develops a fourth-degree laceration. How should the nurse document the extent of the laceration in the woman's medical record? 1. Into the musculature of the buttock. 2. Through the urinary meatus. 3. Through the rectal sphincter. 4. Into the head of the clitoris.

3. A fourth-degree laceration extends through the rectal sphincter. TEST-TAKING TIP: One of the many complications that can occur with the delivery of a macrosomic baby is a perineal laceration. If the laceration is extensive and it progresses through the rectal sphincter it is defined as a fourth degree. As a result, this client is at high risk for the development of a vaginal-rectal fistula.

After a multiparous woman has been in active labor for 15 hours, an ultrasound is done. The results state that the obstetric conjugate is 10 cm and the suboccipitobregmatic diameter is 10.5 cm. Which of the following labor findings is related to these results? 1. Full dilation of the cervix. 2. Full effacement of the cervix. 3. Station of -3. 4. Frequency every 5 minutes.

3. A high station is consistent with the data in the scenario. TEST-TAKING TIP: The dimensions noted in the stem are consistent with a diagnosis of cephalopelvic disproportion since the anterior-posterior diameter of the pelvis (obstetric conjugate) is smaller than the diameter of the baby's head (suboccipitobregmatic). When the fetal head is larger than the maternal pelvis, the baby is unable to descend

Three 30-week-gestation clients are on the labor and delivery unit in preterm labor. For which of the clients should the nurse question a doctor's order for beta agonist tocolytics? 1. A client with hypothyroidism. 2. A client with breast cancer. 3. A client with cardiac disease. 4. A client with asthma.

3. A history of cardiac disease would place a client who is to receive a beta agonist medication at risk. The nurse should question this order. TEST-TAKING TIP: The test taker should remember that beta agonists stimulate the "fight or flight" response. The client's heart rate will increase precipitously and there is a possibility that the potassium levels of the client may fall. These side effects place the client with heart disease at risk of heart failure and/or dysrhythmias. The client is also at high risk for pulmonary edema and congestive heart failure so lung field assessments should be done regularly.

The nurse is preparing to care for four assigned clients. Which client is at highest risk for hemorrhage? 1. A primiparous client who delivered 4 hours ago 2. A multiparous client who delivered 6 hours ago 3. A multiparous client who delivered a large baby after oxytocin (Pitocin) induction 4. A primiparous client who delivered 6 hours ago and had epidural anesthesia

3. A multiparous client who delivered a large baby after oxytocin (Pitocin) induction A multiparous client who delivered a large baby after oxytocin (Pitocin) induction The causes of postpartum hemorrhage include uterine atony; laceration of the vagina; hematoma development in the cervix, perineum, or labia; and retained placental fragments. Predisposing factors for hemorrhage include a previous history of postpartum hemorrhage, placenta previa, abruptio placentae, overdistention of the uterus from polyhydramnios, multiple gestation, a large neonate, infection, multiparity, dystocia or labor that is prolonged, operative delivery such as a cesarean or forceps delivery, and intrauterine manipulation. The multiparous client who delivered a large fetus after oxytocin induction has more risk factors associated with postpartum hemorrhage than the other clients. In addition, there are no specific data in the client descriptions in options 1, 2, and 3 that present the risk for hemorrhage.

The labor nurse has just received shift report on four gravid patients. Which of the patients should the nurse assess first? 1. G5P2202, 32 weeks, placenta previa, today's hemoglobin 11.6 gm/dL. 2. G2P0101, 39 weeks, type 2 diabetic, blood glucose (15 minutes ago) 85 mg/dL. 3. G1P0000, 32 weeks, placental abruption, fetal heart (15 minutes ago) 120 bpm. 4. G2P1001, 39 weeks, Rh negative, today's hematocrit 31%.

3. A placental abruption is a lifethreatening situation for the fetus. It has been 15 minutes since the fetal heart was assessed. This is the nurse's priority. TEST-TAKING TIP: In this question, the test taker must discriminate among four situations to discern which is the highest priority. Although a client with placenta previa is at high risk for bleeding, it is very likely that if she did start to bleed spontaneously that she would notify the nurse. The fetus of a client who has a placental abruption, however, is already in a life-threatening situation.

There are four clients in active labor in the labor suite. Which of the women should the nurse monitor carefully for the potential of uterine rupture? 1. Age 15, G3P0020, in active labor. 2. Age 22, G1P0000, eclampsia. 3. Age 25, G4P3003, last delivery by cesarean section. 4. Age 32, G2P0100, first baby died during labor.

3. A woman, no matter what age, who has had a previous cesarean section is at risk for uterine rupture. TEST-TAKING TIP: When babies are birthed via cesarean section, the surgeon must create an incision through the uterine body. The muscles of the uterus have, therefore, been ligated and a scar has formed at the incision site. Scars are not elastic and do not contract and relax the way muscle tissue does. A vaginal birth after cesarean (VBAC) section can only be performed if the woman had a low flap (Pfannenstiel) incision in the uterus during her previous cesarean section.

A client with an internal fetal monitor catheter in place has just received IV butorphanol (Stadol) for pain relief. Which of the following monitor tracing changes should the nurse anticipate? 1. Early decelerations. 2. Late decelerations. 3. Diminished short- and long-term variability. 4. Accelerations after contractions.

3. Absent variability would be expected as a result of Stadol administration. TEST-TAKING TIP: Variability is an indicator of fetal well-being. It reflects the competition between the sympathetic and the parasympathetic nervous systems' affects on the fetal heart rate. When the CNS is depressed from the administration of a narcotic analgesic, therefore, the nurse should expect to see diminished variability

A woman is to receive RhoGAM at 28 weeks' gestation. What action must the nurse take before giving the injection? 1. Validate that the baby is Rh negative. 2. Assess that the direct Coombs' test is positive. 3. Verify the identity of the woman. 4. Reconstitute the globulin with sterile water.

3. Although this is an important action that must be taken before the administration of any medication, it is especially critical in this situation. TEST-TAKING TIP: When RhoGAM is given, the nurse is administering Rh antibodies to Rh(-) mothers. If the nurse should make a mistake and administer the dosage to an Rh(+) mother, the client would then have been injected with antibodies that would act to destroy her own blood.

labor nurse is caring for a client, 30 weeks' gestation, who is symptomatic from a complete placenta previa. Which of the following physician orders should the nurse question? 1. Administer bethamethasone (Celestone) 12 mg IM daily times 2. 2. Maintain strict bed rest. 3. Assess cervical dilation. 4. Regulate intravenous (Ringer's lactate: drip rate to 150 cc/hr).

3. An order to assess the woman's cervical dilation should be questioned. TEST-TAKING TIP: If the nurse were to assess the cervical dilation of a client with complete previa, he or she could puncture the placenta. Vaginal examinations are absolutely contraindicated with a diagnosis of complete placenta previa. Betamethasone is administered to promote maturation of the baby's lungs.

A 30-year-old G2P0010 in preterm labor is receiving nifedipine (Procardia). Which of the following maternal assessments noted by the nurse must be reported to the health care practitioner immediately? 1. Heart rate of 100 bpm. 2. Wakefulness. 3. Audible rales. 4. Daily output of 2000 cc.

3. Audible rales should be reported to the health care practitioner TEST-TAKING TIP: The presence of audible rales is indicative of pulmonary edema, a serious side effect related to the medication. The pulmonary edema may be caused by the development of congestive heart failure. Whenever a client is on nifedipine, the nurse should regularly monitor the client's lung fields.

A client received general anesthesia during her cesarean section 4 hours ago. Which of the following postpartum nursing interventions is important for the nurse to make? 1. Place the client flat in bed. 2. Assess for dependent edema. 3. Auscultate lung fields. 4. Check patellar reflexes.

3. Auscultate lung fields.

A client with mild preeclampsia, who has been advised to be on bed rest at home, asks why it is necessary. Which of the following is the best response for the nurse to give the client? 1. "Bed rest will help you to conserve energy for your labor." 2. "Bed rest will help to relieve your nausea and anorexia." 3. "Reclining will increase the amount of oxygen that your baby gets." 4. "The position change will prevent the placenta from separating."

3. Bed rest, especially side-lying, helps to improve perfusion to the placenta. TEST-TAKING TIP: This question requires the nurse to have a clear understanding of the pathology of preeclampsia. Only with an understanding of the underlying disease, can the test taker be able to remember the rationale for many aspects of client care. The vital organs of preeclamptic clients are being poorly perfused as a result of the abnormally high blood pressure. When a woman lies on her side, blood return to the heart is improved and the cardiac output is also improved. With improved cardiac output, perfusion to the placenta and other organs is improved.

A client who is post-cesarean section for severe preeclampsia is receiving magnesium sulfate via IV pump and morphine sulfate via patient-controlled anesthesia (PCA) pump. The nurse enters the room on rounds and notes that the client is not breathing. Which of the following actions should the nurse perform first? 1. Give two breaths. 2. Discontinue medications. 3. Call a code. 4. Check carotid pulse.

3. Call a code.

A nurse is caring for four laboring women. Which of the women will the nurse carefully monitor for signs of abruptio placentae? 1. G3P0020, 17 years of age. 2. G4P2101, cancer survivor. 3. G5P1211, cocaine abuser. 4. G6P0323, 27 weeks' gestation

3. Cocaine is a powerful vasoconstrictive agent. It places pregnant clients at high risk for placental abruption. TEST-TAKING TIP: It is very important that the test taker not read into any question or response. In the preceding question, all four of the women have had complicated pregnancies. The test taker should not presume the cause of the complications when they are not stated but rather look for the answer that does absolutely place the client at high risk for the abruption

A preterm labor client, 30 weeks' gestation who ruptured membranes 4 hours ago, is being given IM dexamethasone (Decadron). When she asks why she is receiving the drug, the nurse replies: 1. "To help to stop your labor contractions." 2. "To prevent an infection in your uterus." 3. "To help to mature your baby's lungs." 4. "To decrease the pain from the contractions."

3. Decadron is a steroid that hastens the maturation of the fetal lung fields. TEST-TAKING TIP: Steroids (either IM betamethasone or IM dexamethasone) are given over a 2-day period to mothers in preterm labor. The medications have been shown to hasten the development of surfactant in the lung fields of fetuses. Babies whose mothers have received one of the medications experience fewer respiratory complications

The postpartum nurse is assessing a client who delivered a healthy infant by cesarean section for signs and symptoms of superficial venous thrombosis. Which sign would the nurse note if superficial venous thrombosis were present? 1. Paleness of the calf area 2. Coolness of the calf area 3. Enlarged, hardened veins 4. Palpable dorsalis pedis pulses

3. Enlarged, hardened veins Thrombosis of superficial veins usually is accompanied by signs and symptoms of inflammation, including swelling, redness, tenderness, and warmth of the involved extremity. It also may be possible to palpate the enlarged, hard vein. Clients sometimes experience pain when they walk. Palpable dorsalis pedis pulses is a normal finding

The nurse is caring for two post-cesarean section clients in the postanesthesia suite. One of the clients had her surgery under spinal anesthesia, while the other client had her surgery under epidural anesthesia. Which of the following is an important difference between the two types of anesthesia that the nurse should be aware of? 1. The level of the pain relief is lower in spinals. 2. Placement of the needle is higher in epidurals. 3. Epidurals do not fully sedate motor nerves. 4. Spinal clients complain of nausea and vomiting.

3. Epidurals do not fully sedate the motor nerves of the client. Epidural clients are capable of moving their lower extremities even when fully pain free. TEST-TAKING TIP: The single most important difference between epidural and spinal anesthesia is the depth of needle insertion. Epidural anesthesia is administered into the epidural space. This is outside of the spinal canal. The anesthesia, therefore, is not in direct contact with the spinal nerves. In contrast, spinal anesthesia, instilled into the spinal canal, is in direct contact with the spinal nerves. All of the spinal nerves of spinal anesthesia clients are anesthetized, including motor nerves. Spinal anesthesia clients are paralyzed until the anesthesia is metabolized by the body.

The nurse has assessed four primigravid clients in the prenatal clinic. Which of the women would the nurse refer to the nurse midwife for further assessment? 1. 10 weeks' gestation, complains of fatigue with nausea and vomiting. 2. 26 weeks' gestation, complains of ankle edema and chloasma. 3. 32 weeks' gestation, complains of epigastric pain and facial edema. 4. 37 weeks' gestation, complains of bleeding gums and urinary frequency.

3. Epigastric pain and facial edema are not normal. This client should be referred to the nurse midwife. The nurse must be prepared to identify clients with symptoms that are unexpected. This question requires the test taker to differentiate between normal signs and symptoms of pregnancy at a variety of gestational ages and those that could indicate a serious complication of pregnancy.

Which of the following is a priority nursing diagnosis for a woman, G10 P6226, who is PP1 from a spontaneous vaginal delivery with a significant postpartum hemorrhage? 1. Alteration is comfort related to afterbirth pains. 2. Risk for altered parenting related to grand multiparity. 3. Fluid volume deficit related to blood loss. 4. Risk for sleep deprivation related to mothering role.

3. Fluid volume deficit related to blood loss.

The results from a fetal blood sampling test are reported as pH 7.22. The nurse interprets the results as: 1. The baby is severely acidotic. 2. The baby must be delivered as soon as possible. 3. The results are equivocal warranting further sampling. 4. The results are within normal limits.

3. Further testing is indicated. TEST-TAKING TIP: Some practitioners perform fetal scalp sampling when there is a decrease in fetal heart variability. A normal fetal pH is defined as 7.25 to 7.35. An acidotic fetus has a pH that is less than 7.20. When the pH is between 7.20 to 7.25, the value is considered to be equivocal with a need for further testing. Usually interventions are instituted— oxygen applied, position changed, IV fluid increased—and another sampling is done in 10 to 15 minutes.

A woman has just had a low forceps delivery. For which of the following should the nurse assess the woman during the immediate postpartum period? 1. Infection. 2. Bloody urine. 3. Heavy lochia. 4. Rectal abrasions.

3. Heavy lochia.

A woman is recovering at the gynecologist's office following a late first-trimester spontaneous abortion. At this time, it is essential for the nurse to check which of the following? 1. Maternal rubella titer. 2. Past obstetric history. 3. Maternal blood type. 4. Cervical patency

3. It is essential that the woman's blood type be assessed. TEST-TAKING TIP: If the woman is found to be Rh(-), even though the fetal blood type is unknown, the woman must receive a dose of RhoGAM within 72 hours of the abortion. If the fetus were Rh(+) and the woman were not to receive RhoGAM, the woman's immune system might be stimulated to produce antibodies against Rh(+) blood. Any future Rh(+) fetus would be in danger of developing erythroblastosis fetalis.

When counseling a preeclamptic client about her diet, what should the nurse encourage the woman to do? 1. Restrict sodium intake. 2. Increase intake of fluids. 3. Eat a well-balanced diet. 4. Avoid simple sugars.

3. It is important for the client to eat a well-balanced diet. Clients with preeclampsia are losing albumin through their urine. They should eat a well-balanced diet with sufficient protein to replace the lost protein. Even though preeclamptic clients are hypertensive, it is not recommended that they restrict salt-they should have a normal salt intake-because during pregnancy the kidney is salt sparing. When salt is restricted, the kidneys become stressed.

The health care practitioner performed an amniotomy 5 minutes ago on a client, G3P1011, 41 weeks' gestation, 4 station, and ROP position. The fetal heart rate is 140 with variable decelerations. The fluid is green tinged and smells musty. The nurse concludes that which of the following situations is present at this time? 1. The fetus is postterm. 2. The presentation is breech. 3. The cord is prolapsed. 4. The amniotic fluid is infected.

3. It is likely that the cord is prolapsed because the amniotomy was performed when the presenting part was not yet engaged and because variable decelerations are seen on the FH monitor. TEST-TAKING TIP: The likelihood of a prolapsed cord occurring during an amniotomy increases when the fetal presenting part is in negative station. As the amniotic fluid is released from the uterus during the rupture of membranes, the cord can slip and precede the fetus. At that time, variable decelerations are seen on the electronic fetal monitor tracing since the cord is being compressed by the presenting part.

A nurse is performing an assessment on four 22-week-pregnant clients. The nurse reports to the obstetrician that which of the clients may be carrying twins? 1. The client whose progesterone levels are elevated 2. The client with a weight gain of 13 pounds. 3. The client whose fundal height measurement is 26 cm. 4. The client whose alpha-fetoprotein level is one-half normal.

3. It is possible that this client is carrying twins. TEST-TAKING TIP: After 20 weeks' gestation, the nurse would expect the fundal height to be equal to the number of weeks of the woman's gestation. Since the fundal height is 4 cm above the expected 22 cm, it is likely that the woman is either having twins or has polyhydramnios.

Which of the following lab values should the nurse report to the physician as being consistent with the diagnosis of HELLP syndrome? 1. Hematocrit 48%. 2. Potassium 5.5 mEq/L. 3. Platelets 75,000. 4. Sodium 130 mEq/L.

3. Low platelets are consistent with the diagnosis of HELLP syndrome. TEST-TAKING TIP: HELLP is the acronym for a serious complication of pregnancy and labor and delivery. The letters represent the following information: H, hemolysis; EL, elevated liver enzymes; LP, low platelets. When a client has HELLP syndrome, the nurse would, therefore, expect to see low hemoglobin and hematocrit levels, high aspartate aminotransferase (AST) and alanine aminotransferase (ALT) levels, and low platelets, as seen in the scenario.

A 28-week-gestation client with intact membranes is admitted with the following findings: Contractions every 5 min X 60 sec, 3 cm dilated, 80% effaced. Which of the following medications will the obstetrician likely order? 1. Oxytocin (Pitocin). 2. Ergonovine (Methergine). 3. Magnesium sulfate. 4. Morphine sulfate.

3. Magnesium sulfate is a tocolytic agent. It would be appropriate for this medication to be administered at this time. TEST-TAKING TIP: The client in the scenario is exhibiting signs that meet the criteria for preterm labor. The test taker should deduce, therefore, that a tocolytic agent may be ordered in this situation. The only tocolytic agent included in the choices is magnesium sulfate.

On assessment of a postpartum client, the nurse notes that the uterus feels soft and boggy. The nurse should take which initial action? 1. Document the findings. 2. Elevate the client's legs. 3. Massage the fundus until it is firm. 4. Push on the uterus to assist in expressing clots

3. Massage the fundus until it is firm Massage the fundus until it is firm. If the uterus is not contracted firmly (i.e., it is soft and boggy), the initial intervention is to massage the fundus until it is firm and to express clots that may have accumulated in the uterus. Elevating the client's legs would not assist in managing uterine atony. Documenting the findings is an appropriate action but is not the initial action. Pushing on an uncontracted uterus can invert the uterus and cause massive hemorrhage.

A woman who wishes to breastfeed advises the nurse that she had a breast reduction one year earlier. Which of the following responses by the nurse is appropriate? 1. Advise the woman that unfortunately she will be unable to breastfeed. 2. Examine the woman's breasts to see where the incision was placed. 3. Monitor the baby's daily weights for excessive weight loss. 4. Inform the woman that reduction surgery rarely affects milk transfer

3. Monitor the baby's daily weights for excessive weight loss.

The nurse is assessing a client in the fourth stage of labor and notes that the fundus is firm, but that bleeding is excessive. Which should be the initial nursing action? 1. Record the findings. 2. Massage the fundus. 3. Notify the health care provider (HCP). 4. Place the client in Trendelenburg's position.

3. Notify the health care provider (HCP). If bleeding is excessive, the cause may be laceration of the cervix or birth canal. Massaging the fundus if it is firm would not assist in controlling the bleeding. Trendelenburg's position should be avoided because it may interfere with cardiac and respiratory function. Although the nurse would record the findings, the initial nursing action would be to notify the HCP.

In which of the following situations should a nurse report a possible deep vein thrombosis (DVT)? 1. The woman complains of numbness in the toes and heel of one foot. 2. The woman has cramping pain in a calf that is relieved when the foot is dorsiflexed. 3. One of the woman's calves is swollen, red, and warm to the touch. 4. The veins in the ankle of one of the woman's legs are spider-like and purple.

3. One of the woman's calves is swollen, red, and warm to the touch.

The nurse is developing a standard care plan for the post-cesarean client. Which of the following should the nurse plan to implement? 1. Maintain client in left lateral recumbent position. 2. Teach sitz bath use on second postoperative day. 3. Perform active range-of-motion exercises until ambulating. 4. Assess central venous pressure during first postoperative day

3. Perform active range-of-motion exercises until ambulating.

A delirious patient is admitted to the hospital in labor. She has had no prenatal care and vials of crack cocaine are found in her pockets. The nurse monitors this client carefully for which of the following intrapartal complications? 1. Prolonged labor. 2. Prolapsed cord. 3. Abruptio placentae. 4. Retained placenta

3. Placental abruption is associated with maternal illicit drug use. TEST-TAKING TIP: Crack cocaine is a powerful vasoconstrictive agent. The chorionic villi atrophy as a result of the vasoconstrictive effects of the drug. Placental abruption, when the placenta detaches from the decidual lining of the uterus, is therefore of particular concern

A woman has just been admitted to the emergency department subsequent to a head-on automobile accident. Her body appears to be uninjured. The nurse carefully monitors the woman for which of the following complications of pregnancy? 1. Placenta previa. 2. Transverse fetal lie. 3. Placental abruption. 4. Severe preeclampsia. 5 preterm labor

3. Placental abruption may develop as a result of the auto accident. 5 preterm labor TEST-TAKING TIP: The fetus is well protected within the uterine body. The musculature of the uterus and the amniotic fluid provide the baby with enough cushioning to withstand minor bumps and falls. A major automobile accident, however, can cause anything from preterm premature rupture of the membranes, to a ruptured uterus, to placental abruption. The nurse should especially monitor the fetal heart beat for any variations.

A serum electrolyte report for a client, 1 day post-cesarean delivery for eclampsia, has just been received by the nurse. The client is receiving 5% dextrose in 1 /2 normal saline IV at 125 mL/hr and magnesium sulfate 2 G/hr IV via infusion pump. Which of the following values should the nurse report to the surgeon? 1. Magnesium 7 mg/dL. 2. Sodium 136 mg/dL. 3. Potassium 3.0 mg/dL. 4. Calcium 9 mg/dL.

3. Potassium 3.0 mg/dL.

woman is to receive Prepidil (dinoprostone gel) for labor induction. The nurse should be prepared to administer the medication via which of the following routes? 1. Intravenously. 2. Orally. 3. Endocervically. 4. Intrathecally.

3. Prepidil is administered endocervically. TEST-TAKING TIP: Prostaglandins, hormone-like substances that mediate a wide range of physiological functions, do so locally. Prepidil, therefore, is administered adjacent to the cervix where it acts to soften the cervix in preparation for dilation and effacement

A woman, G5P0311, is in the postanesthesia care unit (PACU) after a cervical cerclage procedure. During the immediate postprocedure period, what should the nurse carefully monitor this client for? 1. Hyperthermia. 2. Hypotension. 3. Uterine contractions. 4. Fetal heart dysrhythmias.

3. Preterm labor is a complication in the immediate postprocedure period. Cerclages are inserted when clients have a history of recurring pregnancy loss related to a cervical insufficiency. This client has had 5 pregnancies but only one living child. Unfortunately, with the manipulation of the cervix at the time of the cerclage, the clients may develop preterm labor. The clients should be monitored carefully with a tocometer to assess for labor contractions.

The nurse notes the following vital signs of a postoperative cesarean client during the immediate postpartum period: 100.0°F, P 68, R 12, BP 130/80. Which of the following is a correct interpretation of the findings? 1. Temperature is elevated, a sign of infection. 2. Pulse is too low, a sign of vagal pathology. 3. Respirations are too low, a sign of medication toxicity. 4. Blood pressure is elevated, a sign of preeclampsia.

3. Respirations are too low, a sign of medication toxicity.

A gravid client, G6P5005, 24 weeks' gestation, has been admitted to the hospital for placenta previa. Which of the following is an appropriate long-term goal for this client? 1. The client will state an understanding of need for complete bedrest. 2. The client will have a reactive nonstress test on day 2 of hospitalization. 3. The client will be symptom-free until at least 37 weeks' gestation. 4. The client will call her children shortly after admission.

3. That the client be symptom-free until at least 37 weeks' gestation is a long-term goal. At that time, the baby will be full term. Each and every one of the goals is appropriate for a client with placenta previa. Only the statement that projects the client's response into the future is, however, a long-term goal.

A woman has been in the second stage of labor for 21⁄2 hours. The fetal head is at +4 station and the fetal heart is showing mild late decelerations. The obstetrician advises the woman that the baby will be delivered with forceps. Which of the following actions should the nurse take at this time? 1. Obtain a consent for the use of forceps. 2. Encourage the woman to push between contractions. 3. Assess the fetal heart rate after each contraction. 4. Advise the woman to refuse the use of forceps

3. The FH should always be assessed after each contraction during stage 2. Plus, this baby is especially at risk because the stage is prolonged and the physician is using forceps for delivery TEST-TAKING TIP: This is an excellent example of a medically indicated use of forceps. The woman is likely fatigued from pushing for over 2 hours, the presenting part is at the pelvic floor, and the baby is showing signs of fetal distress. It would be advisable to deliver this baby in a timely fashion. The use of forceps should result in a speedy delivery.

A client had an epidural inserted 2 hours ago. It is functioning well, the client is hemodynamically stable, and the client's labor is progressing as expected. Which of the following assessments is highest priority at this time? 1. Assess blood pressure every 15 minutes. 2. Assess pulse rate every 1 hour. 3. Palpate bladder. 4. Ausculate lungs.

3. The client's bladder should be palpated. TEST-TAKING TIP: There are three very important reasons why the client's bladder should be assessed. First, clients receive at least 1 liter of fluid immediately before the insertion of an epidural. Within a 2-hour period, it is likely that the woman's bladder has become full. Second, clients are unable to feel when they need to urinate with an epidural in place. Third, a full bladder can impede fetal descent.

A 14-year-old woman is seeking obstetric care. Which of the following vital signs must be monitored very carefully during this woman's pregnancy? 1. Heart rate. 2. Respiratory rate. 3. Blood pressure. 4. Temperature.

3. The client's blood pressure is the most important vital sign. Adolescents who are 16 years old or younger are particularly high risk for hypertensive illnesses of pregnancy. It is especially important for the nurse and the client's primary health care practitioner to determine the client's baseline blood pressure in order to identify any elevations as early as possible.

When monitoring a fetal heart rate with moderate variability, the nurse notes V-shaped decelerations to 80 from a baseline of 120. One occurred during a contraction while another occurred 10 seconds after the contraction and a third occurred 40 seconds after yet another contraction. The nurse interprets these findings as resulting from which of the following? 1. Metabolic acidosis. 2. Head compression. 3. Cord compression. 4. Insufficient uteroplacental blood flow.

3. The contractions described in the scenario result from cord compression (variable decelerations) TEST-TAKING TIP: First, the test taker should be able to interpret fetal heart tracings both visually and verbally. This includes baseline data as well as acceleration and deceleration changes. Second, the test taker should know the etiology of each of the tracings. Third, the test taker should know the appropriate nursing intervention related to each tracing

A nurse is caring for a gravid client who is G1P0000, 35 weeks' gestation. Which of the following would warrant the nurse to notify the woman's health care practitioner that the client is in preterm labor? 1. Contraction frequency every 15 minutes. 2. Effacement 10%. 3. Dilation 3 cm. 4. Contraction duration of 30 seconds.

3. The dilation of 3 cm is indicative of preterm labor. fined as labor before 38 weeks' gestation with 3 or more contractions occurring within a 30-minute period PLUS cervical change of one of the following: cervical effacement >80%, cervical dilation >1 cm, or cervical length of < 2.5 cm. The change in cervical length is diagnosed by transvaginal ultrasound

Immediately prior to an amniotomy, the external fetal heart monitor tracing shows 145 bpm with early decelerations. Immediately following the procedure, an internal tracing shows a fetal heart rate of 120 with variable decelerations. A moderate amount of clear, amniotic fluid is seen on the bed linens. The nurse concludes that which of the following has occurred? 1. Placental abruption. 2. Eclampsia. 3. Prolapsed cord. 4. Succenturiate placenta.

3. The drop in fetal heart rate with variable decelerations indicates that the cord has likely prolapsed TEST-TAKING TIP: The test taker must remember that variable decelerations are caused by cord compression. The fact that variables are seen in the scenario as well as a precipitous drop in the fetal heart baseline is an indirect indication that the cord is being compressed, resulting in decreased oxygenation to the fetus

A woman with severe preeclampsia, 38 weeks' gestation, is being induced with IV oxytocin (Pitocin). Which of the following would warrant the nurse to stop the infusion? 1. Blood pressure 160/110. 2. Frequency of contractions every 3 minutes. 3. Duration of contractions of 120 seconds. 4. Fetal heart rate 156 with early decelerations

3. The duration of the contractions is prolonged. The baby will be deprived of oxygen. TEST-TAKING TIP: The test taker should consider that not only is this client receiving oxytocin, but she is also preeclamptic. Preeclampsia is a vasoconstrictive disease state. The likelihood of poor placental perfusion is already high. When the contraction duration is also prolonged, the fetus is at high risk of becoming hypoxic.

A client, 39 weeks' gestation, fetal heart baseline at 144 bpm, tells the admitting labor and delivery room nurse that she has had to wear a pad for the past 4 days, "because I keep leaking urine." Which of the following is an appropriate action for the nurse to perform at this time? 1. Palpate the woman's bladder to check for urinary retention. 2. Obtain a urine culture to check for a urinary tract infection. 3. Assess the fluid with nitrazine and see if the paper turns blue. 4. Percuss the woman's uterus and monitor for ballottement.

3. The fluid should be assessed with nitrazine paper. TEST-TAKING TIP: Nitrazine paper is another name for litmus paper. It detects the pH of fluid. Amniotic fluid is alkaline while urine is acidic. If the paper turns a dark blue, the nurse can conclude that the membranes have ruptured and that the woman is leaking amniotic fluid.

A laboring woman, who has developed an apparent amniotic fluid embolism, is not breathing and has no pulse. In addition to calling for assistance, which of the following actions by the nurse, who is alone with the patient, is appropriate at this time? 1. Perform cardiac compressions and breaths in a 15 to 2 ratio. 2. Provide chest compressions at a depth of 3 inches. 3. Compress the chest at the lower 1⁄2 of the sternum. 4. Provide rescue breaths over a 10-second time frame.

3. The lower half of the sternum should be compressed when delivering cardiac compressions. TEST-TAKING TIP: The American Heart Association frequently revises cardiopulmonary resuscitation (CPR) guidelines. The responses above reflect the 2005 guidelines. The test taker should make sure that he or she is familiar with current protocols. In addition to the responses above, it is important for the rescuer to tilt the woman slightly toward the left to decrease the compression of the gravid uterus on the aorta and vena cava.

A 29-week-gravid client is admitted to the labor and delivery unit with vaginal bleeding. To differentiate between placenta previa and abruptio placentae, the nurse should assess which of the following? 1. Leopold's maneuver results. 2. Quantity of vaginal bleeding. 3. Presence of abdominal pain. 4. Maternal blood pressure.

3. The most common difference between placenta previa and placenta abruption is the absence or presence of abdominal pain. TEST-TAKING TIP: Because at least some of the blood from a placental abruption is trapped behind the placenta, women with that complication usually complain of intense, unrelenting pain. But because the blood from a symptomatic placenta previa flows freely through the vagina, the bleeding from that complication is virtually pain free.

A 16-year-old woman is being seen for the first time in the obstetric office. Which of the following comments by the young woman is highest priority for the nurse to respond to? 1. "My favorite lunch is a burger with fries." 2. "I've been dating my new boyfriend for 2 weeks." 3. "On weekends we go out and drink a few beers." 4. "I dropped out of school about 3 months ago."

3. The nurse must respond to this comment. This young woman is repeatedly exposing her fetus to alcohol. TEST-TAKING TIP: The nurse must prioritize her care with teen clients as well as with mature clients. This young woman will eventually need to be counseled regarding diet, infection control, and her education, but the fetus is at highest risk at the present time from repeated alcohol exposure. Indeed, alcohol exposure is injurious for the unborn child throughout the entire pregnancy. The nurse must discuss this with the young woman at this time.

A woman being induced with oxytocin (Pitocin) is contracting every 3 min 30 seconds. Suddenly the woman becomes dypsneic, cyanotic, and begins to cough up bloody sputum. Which of the following nursing interventions is of highest priority? 1. Check blood pressure. 2. Assess fetal heart rate. 3. Administer oxygen. 4. Stop oxytocin infusion.

3. The nurse's priority action is to administer oxygen. TEST-TAKING TIP: This client is exhibiting the classic signs of an amniotic fluid embolism. At this point, the baby's health is secondary because the mother is in a life-threatening situation. The nurse must apply oxygen and call a code immediately.

A client, 37 weeks' gestation, has been advised that she is positive for group B streptococci. Which of the following comments by the nurse is appropriate at this time? 1. "The doctor will prescribe intravenous antibiotics for you. A visiting nurse will administer them to you in your home." 2. "You are very high risk for an intrauterine infection. It is very important for you to check your temperature every day." 3. "The bacteria are living in your vagina. They will not hurt you but we will give you medicine in labor to protect your baby from getting sick." 4. "This bacteria causes scarlet fever. If you notice that your tongue becomes very red and that you feel feverish you should call the doctor immediately."

3. This answer is correct. Exposure to group B strep is very dangerous for neonates. TEST-TAKING TIP: Group B strep can cause serious neonatal disease. Babies are at high risk for meningitis, sepsis, pneumonia, and even death. IV antibiotics are administered to the laboring mother every 4 hours to decrease the colonization in the mother's vagina and rectum. In addition, the antibiotics cross the placenta and act as a prophylaxis for the baby.

It is discovered that a 28-week-gestation gravid is leaking amniotic fluid. Before the client is sent home on bed rest, the nurse teaches her which of the following? 1. Perform a nitrazine test every morning upon awakening. 2. Immediately report any breast tenderness to the primary health care practitioner. 3. Abstain from engaging in vaginal intercourse for the rest of the pregnancy. 4. Carefully weigh all of her saturated peripads.

3. This client must abstain from vaginal intercourse for the remainder of the pregnancy. Once the membranes are ruptured, the barrier between the vagina and the uterus is broken. As a result, the pathogens in the vagina and the external environment are potentially able to ascend into the sterile uterine body. In addition, once the membranes are ruptured, the client is at high risk for preterm labor. Intercourse must be curtailed for both of these reasons

A gravid woman, who is 42 weeks' gestation, has just had a 20-minute nonstress test (NST). Which of the following results would the nurse interpret as a reactive test? 1. Moderate fetal heart baseline variability. 2. Maternal heart rate accelerations to 140 bpm lasting at least 20 seconds. 3. Two fetal heart accelerations of 15 bpm lasting at least 15 seconds. 4. Absence of maternal premature ventricular contractions.

3. This is the definition of a reactive nonstress test-there are two fetal heart accelerations of 15 bpm lasting 15 or more seconds during a 20-minute period. When a practitioner notes a reactive nonstress test, he or she can be fairly confident that the fetus is well and will probably remain well for at least 3 to 4 days. NSTs, therefore, are usually performed twice weekly. A nonreactive nonstress test, when the fetal heart fails to show 2 accelerations of 15 bpm lasting 15 or more seconds during a 20-minute period, is very hard to interpret. Usually practitioners order more extensive testing to determine the well-being of the baby after a nonreactive NST.

A client, G3P2002, 40 weeks' gestation, who has vaginal candidiasis, has just been admitted in early labor. Which of the following should the nurse advise the woman? 1. She may need a cesarean delivery. 2. She will be treated with antibiotics during labor. 3. The baby may develop thrush after delivery. 4. The baby will be isolated for at least one day.

3. Thrush is the term given to oral candidiasis, which the baby may develop after delivery. TEST-TAKING TIP: Candida can be transmitted to a baby during delivery as well as postdelivery via the mother's hands. Initially, the baby will develop thrush, but eventually the mother may notice a bright pink diaper rash on the baby. Also, if she is breastfeeding her baby, she may develop a yeast infection of the breast that is very painful. The mother with candidiasis should be advised to wash her hands carefully after toileting.

A client is admitted to the hospital with severe preeclampsia. The nurse is assessing for clonus. Which of the following actions should the nurse perform? 1. Strike the woman's patellar tendon. 2. Palpate the woman's ankle. 3. Dorsiflex the woman's foot. 4. Position the woman's feet flat on the floor.

3. To assess clonus, the nurse should dorsiflex the woman's foot. When clients have severe preeclampsia, they are often hyperreflexic and develop clonus. To assess for clonus, the nurse should dorsiflex the foot and then let the foot go. The nurse should observe for and count any pulsations of the foot. The number of pulsations is documented. The higher the number of pulsations, the more irritable the woman's central nervous system.

A woman who is hepatitis B-surface antigen positive is in active labor. Which action by the nurse is appropriate at this time? 1. Obtain an order from the obstetrician to prepare the client for cesarean delivery. 2. Obtain an order from the obstetrician to administer intravenous ampicillin during labor and the immediate postpartum. 3. Obtain an order from the pediatrician to administer hepatitis B immune globulin and hepatitis B vaccine to the baby after birth. 4. Obtain an order from the pediatrician to place the baby in isolation after delivery.

3. Within 12 hours of birth, the baby should receive both the first injection of hepatitis B vaccine and HBIG TEST-TAKING TIP: Although this is a woman who is in labor, the nurse must anticipate the needs of the neonate after delivery. Since it is recommended that the baby receive the medication within a restricted time frame, it is especially important for the nurse to be proactive and obtain the physician's order.

The nurse is reviewing the biophysical profile (BPP) of a client with the technician. The nurse observes no fetal breathing movements for 30 minutes, fewer than three episodes of limb movements, an amniotic fluid index (AFI) greater than 5 cm, and a reactive non-stress test. The fetal limb has no extension or flexion movements. What is the client's BPP score? Record your answer using a whole number.

4 Rationale The nurse observes no fetal breathing movements for 30 minutes. Therefore, the score for fetal breathing movements would be 0. Fewer than three episodes of limb movements also equals a fetal movements score of 0. An amniotic fluid index (AFI) greater than5cm indicates a score of 2. A reactive non-stress test receives a score of 2. No movement of the fetal limb is scored 0. Therefore, the total score would be 0 + 0 + 2 + 2 + 0 = 4.

The nurse is providing instructions about measures to prevent postpartum mastitis to a client who is breast-feeding her newborn. Which client statement would indicate a need for further instruction? 1. "I should breast-feed every 2 to 3 hours." 2. "I should change the breast pads frequently." 3. "I should wash my hands well before breast- feeding." 4. "I should wash my nipples daily with soap and water."

4. "I should wash my nipples daily with soap and water." Mastitis is inflammation of the breast as a result of infection. It generally is caused by an organism that enters through an injured area of the nipples, such as a crack or blister. Measures to prevent the development of mastitis include changing nursing pads when they are wet and avoiding continuous pressure on the breasts. Soap is drying and could lead to cracking of the nipples, and the client should be instructed to avoid using soap on the nipples. The mother is taught about the importance of hand-washing and that she should breast-feed every 2 to 3 hours.

A client, who is 2 weeks postpartum, calls her obstetrician's nurse and states that she has had a whitish discharge for 1 week but today she is "bleeding and saturating a pad about every 1/2 hour." Which of the following is an appropriate response by the nurse? 1. "That is normal. You are starting to menstruate again." 2. "You should stay on complete bed rest until the bleeding subsides." 3. "Pushing during a bowel movement may have loosened your stitches." 4. "The physician should see you. Please go to the emergency department."

4. "The physician should see you. Please go to the emergency department."

A client, G4P1021, has been admitted to the labor and delivery suite for induction of labor. The following assessments have been made: Bishop score of 3, fetal heart rate of 156 with good variability and no decelerations, TPR 98.6ºF, P 88, R 20, BP 120/80, negative obstetric history. Cervidil (dinoprostone) has been inserted. Which of the following findings would warrant the removal of the prostaglandin suppository? 1. Bishop score of 10. 2. Fetal heart of 152. 3. Respiratory rate of 24. 4. Contraction frequency of 1 minute.

4. A contraction frequency of 1 minute, even with a short duration, would warrant the removal of the medication. TEST-TAKING TIP: A frequency of 1 minute, even if the duration were 30 seconds, would mean that there were only 30 seconds when the uterine muscle was relaxed. This short amount of time would not provide the placenta with enough time to be sufficiently perfused. Fetal bradycardia is a likely response to such a short frequency period.

A client is in active labor. Which of the following assessments would warrant immediate intervention? 1. Maternal PaCO2 of 40 mm Hg. 2. Alpha-fetoprotein values of 2 times normal. 3. 3 fetal heart accelerations during contractions. 4. Fetal scalp sampling pH of 7.19.

4. A fetal scalp pH of 7.19 is indicative of an acidotic fetus. TEST-TAKING TIP: The test taker must read all four responses before choosing the best response. Although "2" includes a value that is not normal, it does not describe a situation that requires the nurse to take immediate action. A fetal scalp sampling pH below 7.20, however, is of immediate concern.

A client is on magnesium sulfate for severe preeclampsia. The nurse must notify the attending physician regarding which of the following findings? 1. Patellar and biceps reflexes of +3. 2. Urinary output of 30 cc/hr. 3. Respiratory rate of 16 rpm. 4. Serum magnesium level of 9 gm/dL.

4. A serum magnesium level of 9 gm/dL is dangerously high. The health care practitioner should be notified. TEST-TAKING TIP: When magnesium sulfate is being administered, the nurse should monitor the client for adverse side effects including respiratory depression, oliguria, and depressed reflexes. When the magnesium level is above 7 gm/dL, toxic effects can be seen.

Four women request to labor in the hospital bathtub. Which of the women is at increased risk from the procedure? 1. Woman during transition. 2. Woman during second stage of labor. 3. Woman receiving oxytocin for induction. 4. Woman with prolonged rupture of the membranes.

4. A woman with prolonged rupture of the membranes should be discouraged from laboring in the water bath. TEST-TAKING TIP: Hydrotherapy is an excellent complementary therapy for the laboring woman. The warm water is relaxing and many women find that their pain is minimized. Women with prolonged rupture of the membranes, however, are at high risk for intrauterine infection and, if they labor in the tub, pathogens can ascend into the uterine cavity more easily.

A client in a postpartum unit complains of sudden sharp chest pain and dyspnea. The nurse notes that the client is tachycardic and the respiratory rate is elevated. The nurse suspects a pulmonary embolism. Which should be the initial nursing action? 1. Initiate an intravenous line. 2. Assess the client's blood pressure. 3. Prepare to administer morphine sulfate. 4. Administer oxygen, 8 to 10 L/minute, by face mask.

4. Administer oxygen, 8 to 10 L/minute, by face mask. If pulmonary embolism is suspected, oxygen should be administered, 8 to 10 L/minute, by face mask. Oxygen is used to decrease hypoxia. The client also is kept on bed rest with the head of the bed slightly elevated to reduce dyspnea. Morphine sulfate may be prescribed for the client, but this would not be the initial nursing action. An intravenous line also will be required, and vital signs need to be monitored, but these actions would follow the administration of oxygen.

A gravid woman with sickle cell anemia is admitted in vaso-occlusive crisis. Which of the following is the priority intervention that the nurse must perform? 1. Administer narcotic analgesics. 2. Apply heat to swollen joints. 3. Place on strict bed rest. 4. Infuse intravenous solution.

4. Administering intravenous fluids is the priority action. TEST-TAKING TIP: Although this question is not directly related to pregnancy, the nurse must be able to translate information from another medical discipline into the obstetric area. The priority action is to improve perfusion to the client's organs. By providing intravenous fluids, the blood can flow through the vessels and perfuse the organs, including the placenta. When the client is dehydrated, the sickled red blood cells clump together inhibiting perfusion.

A woman, G3P2002, is 6 cm dilated. The fetal monitor tracing shows recurring deep late decelerations. The woman's doctor informs her that the baby must be delivered by cesarean section. The woman refuses to sign the informed consent. Which of the following actions by the nurse is appropriate? 1. Strongly encourage the woman to sign the informed consent. 2. Prepare the woman for the cesarean section. 3. Inform the woman that the baby will likely die without the surgery. 4. Provide the woman with ongoing labor support.

4. At this point the appropriate action for the nurse to take is to continue providing labor support. If accepted, emergency interventions, like providing oxygen by face mask and repositioning the client, would also be indicated. TEST-TAKING TIP: If the client's practitioner is convinced that surgery is the only appropriate intervention, he or she could get a court order to mandate the woman to accept surgery. The nurse's role at this point, however, is to provide the client with care in a nonthreatening, compassionate manner. The nurse must acknowledge and accept the client's legal right to refuse the surgery.

A client just delivered the placenta pictured below. The nurse will document that the woman delivered which of following placentas? 1. Circumvallate placenta. 2. Succenturiate placenta. 3. Placenta with velamentous cord insertion. 4. Battledore placenta.

4. Battledore placenta.

client who received a spinal for her cesarean delivery is complaining of pruritus and has a macular rash on her face and arms. Which of the following medications ordered by the anesthesiologist should the nurse administer at this time? 1. Reglan (metoclopramide). 2. Zofran (ondansetron). 3. Compazine (prochlorperazine). 4. Benadryl (diphenhydramine).

4. Benadryl (diphenhydramine).

A breastfeeding client calls her obstetrician stating that her baby was diagnosed with thrush and that her breasts have become infected as well. Which of the following organisms has caused the baby's and mother's infection? 1. Staphylococcus aureus. 2. Streptococcus pneumoniae. 3. Escherichia coli. 4. Candida albicans.

4. Candida albicans.

A 25-year-old client is admitted with the following history: 12 weeks pregnant, vaginal bleeding, no fetal heart beat seen on ultrasound. The nurse would expect the doctor to write an order to prepare the client for which of the following? 1. Cervical cerclage. 2. Amniocentesis. 3. Nonstress testing. 4. Dilation and curettage.

4. Dilation and curettage will be performed on a client with an incomplete abortion. This client is experiencing an incomplete abortion. The baby has died-there is no fetal heart beat-and she has expelled some of the products of conception, as evidenced by frank vaginal bleeding. It is important for the remaining products of conception to be removed in order to prevent hemorrhage and infection. A dilation and curettage (D&C) in which the physician dilates the cervix and scrapes the lining of the uterus with a curette is one means of completing the abortion. Another method of completing the abortion is by administering an abortifacient medication.

A 26-week-gestation woman is diagnosed with severe preeclampsia with HELLP syndrome. The nurse will assess for which of the following signs/symptoms? 1. Low serum creatinine. 2. High serum protein. 3. Bloody stools. 4. Epigastric pain.

4. Epigastric pain is associated with the liver involvement of HELLP syndrome. TEST-TAKING TIP: When the liver is deprived of sufficient blood supply, as can occur with severe preeclampsia, the organ becomes ischemic. The client experiences pain at the site of the liver as a result of the hypoxia in the liver

An insulin-dependent diabetic woman will require higher doses of insulin as which of the following pregnancy hormones increases in her body? 1. Estrogen. 2. Progesterone. 3. Human chorionic gonadotropin. 4. Human placental lactogen.

4. Human placental lactogen is an insulin antagonist so the client will require higher doses of insulin as the level of placental lactogen increases. TEST-TAKING TIP: During the first trimester, the insulin needs of a woman with type 1 diabetes are usually low. Once the diabetic client enters the second trimester, however, insulin demands increase. One of the most important reasons why the insulin demands increase is the increasingly higher levels of human placental lactogen that are found in the mother's blood stream.

The nurse is to intervene when caring for a laboring client whose baby is exhibiting signs of fetal distress. Which of the following actions should the nurse take? 1. Administer oxygen via nasal cannula. 2. Place the client in high Fowler's position. 3. Remove the internal fetal monitor electrode. 4. Increase the intravenous infusion rate.

4. Increasing the IV rate helps to improve perfusion to the placenta. TEST-TAKING TIP: Because the fetus is being oxygenated via the placenta, it is essential that in cases of fetal distress, the amount of oxygen perfusing the placenta be maximized. That requires high concentrations of oxygen being administered via mask, blood volume being increased by increasing the IV drip rate, and cardiac blood return being maximized by positioning the client in order to remove pressure from the aorta and the vena cava.

Which of the following situations in a fully dilated client would warrant the need for a forceps delivery? 1. Maternal history of asthma. 2. Right mentum posterior position and -1 station. 3. Transverse fetal lie. 4. Fetal heart rate of 60 beats per minute.

4. It is appropriate for a baby in severe fetal distress to be delivered with forceps. TEST-TAKING TIP: It is unsafe to use forceps to deliver a baby when the baby's station is above 2. When the baby is above that station, it is unknown whether or not there is sufficient room in the pelvis for the baby to pass. If there should be too little space, very serious fetal complications could arise, including fractured skull and subdural hematoma.

A woman, G3P1010, is receiving oxytocin (Pitocin) via IV pump at 3 milliunits/min. Her current contraction pattern is every 3 minutes 45 seconds with moderate intensity. The fetal heart rate is 150 to 160 bpm with moderate variability. Which of the following interventions should the nur se take at this time? 1. Stop her infusion. 2. Give her oxygen. 3. Change her position. 4. Monitor her labor.

4. It is appropriate to monitor the woman's labor. TEST-TAKING TIP: Even if the test taker were unfamiliar with a normal contraction pattern—as seen in the stem of the question—if he or she knew that the fetal heart pattern is normal, he or she could deduce the correct answer. Three of the responses infer that the nurse should take action because of a complication. Only "4" indicates that the nurse should continue monitoring the labor. In this situation, the one response that is different from the others is the correct answer.

The nurse is caring for a client who was just admitted to the hospital to rule out ectopic pregnancy. Which of the following orders is the most important for the nurse to perform? 1. Assess the client's temperature. 2. Document the time of the client's last meal. 3. Obtain urine for urinalysis and culture. 4. Report complaints of dizziness or weakness.

4. It is most important for the nurse to report complaints of dizziness or weakness. The nurse must prioritize care. When the question asks the test taker to decide which action is most important, all four possible responses are plausible actions. The test taker must determine which is the one action that cannot be delayed. In this situation, the most important action for the nurse to perform is to report complaints of dizziness or weakness. These symptoms are seen when clients develop hypovolemia from internal bleeding.

A client has just been diagnosed with gestation diabetes. She cries, "Oh no! I will never be able to give myself shots!!" Which of the following responses by the nurse is appropriate at this time? 1. "I am sure you can learn for your baby." 2. "I will work with you until you feel comfortable giving yourself the insulin." 3. "We will be giving you pills for the diabetes." 4. "If you follow your diet and exercise you will probably need no insulin."

4. It is unlikely that this client will need any medication. Diet and exercise will probably control the diabetes. TEST-TAKING TIP: The client should be reminded that if she follows her diet and exercises regularly that she will likely be able to manage her diabetes without medication. She should also be encouraged to continue the diet and exercise after delivery in order to prevent the development of type 2 diabetes later in life.

A breastfeeding woman has been diagnosed with retained placental fragments 4 days postdelivery. Which of the following breastfeeding complications would the nurse expect to see? 1. Engorgement. 2. Mastitis. 3. Blocked milk duct. 4. Low milk supply.

4. Low milk supply.

Which finding should the nurse expect when assessing a client with placenta previa? 1. Severe occipital headache. 2. History of thyroid cancer. 3. Previous premature delivery. 4. Painless vaginal bleeding.

4. Painless vaginal bleeding is often the only symptom of placenta previa. There are three different forms of placenta previa: low-lying placenta-one that lies adjacent to, but not over, the internal cervical os; partial-one that partially covers the internal cervical os; and complete-a placenta that completely covers the internal cervical os. There is no way to deliver a live baby vaginally when a client has a complete previa, although there are cases when live babies have been delivered when the clients had low-lying or partial previas.

The nurse is developing a plan of care for a postpartum client with a small vulvar hematoma. The nurse should include which specific action during the first 12 hours after delivery? 1. encourage ambulation hourly 2. Assess vital signs every 4 hours. 3. Measure fundal height every 4 hours. 4. Prepare an ice pack for application to the area..

4. Prepare an ice pack for application to the area.. Prepare an ice pack for application to the area. A hematoma is a localized collection of blood into the tissues of the reproductive sac after delivery. Vulvar hematoma is the most common. Application of ice reduces swelling caused by hematoma formation in the vulvar area. Options 1, 2, and 4 are not interventions that are specific to the plan of care for a client with a small vulvar hematoma.

A woman is scheduled to have an external version for a breech presentation. The nurse carefully assesses the client's chart knowing that which of the following is a contraindication to this procedure? 1. Station -2. 2. 38 weeks' gestation. 3. Reactive NST. 4. Previous cesarean section

4. Previous cesarean section is a contraindication for external version. TEST-TAKING TIP: During external version, the health care practitioner moves the fetus from a malpresentation— usually breech—to a vertex presentation. To accomplish the movement, the physician manually palpates the fetus externally through the mother's abdominal and uterine walls. Because significant stress is placed on the uterine body, the presence of a cesarean scar is a contraindication to the procedure.

A client, G1 P0000, is PP1 from a normal spontaneous delivery of a baby boy, Apgar 5/6. Because the client exhibited addictive behaviors, a toxicology assessment was performed; the results were positive for alcohol and cocaine. Which of the following interventions is appropriate for the nurse to perform for this postpartum client? 1. Strongly advise the client to breastfeed her baby. 2. Perform hourly incentive spirometer respiratory assessments. 3. Suggest that the nursery nurse feed the baby in the nursery. 4. Provide the client with supervised instruction on baby-care skills.

4. Provide the client with supervised instruction on baby-care skills.

A woman, 8 weeks pregnant, is admitted to the obstetric unit with a diagnosis of threatened abortion. Which of the following tests would help to determine whether the woman is carrying a viable or a nonviable pregnancy? 1. Luteinizing hormone level. 2. Endometrial biopsy. 3. Hysterosalpinogram. 4. Serum progesterone level

4. Serum progesterone will provide information on the viability of a pregnancy. TEST-TAKING TIP: When a previously gravid client is seen by her health care practitioner with a complaint of vaginal bleeding, it is very important to determine the viability of the pregnancy as soon as possible. Situational crises are often exacerbated when clients face the unknown. One relatively easy way to determine the viability of the conceptus is by performing a serum progesterone test; high levels indicate a viable baby while low levels indicate a pregnancy loss. Ultrasonography to assess for a beating heart may also be performed.

Which of the following situations is considered a vaginal delivery emergency? 1. Third stage of labor lasting 20 minutes. 2. Fetal heart dropping during contractions. 3. Three-vessel cord. 4. Shoulder dystocia.

4. Shoulder dystocia is an obstetric emergency. TEST-TAKING TIP: Dystocia means difficult delivery. A shoulder dystocia, therefore, refers to difficulty in delivering a baby's shoulders. This is an obstetric emergency since the dystocia occurs in the middle of the delivery when the head has been delivered but the shoulders remain wedged in the pelvis. The most common complications are related to nerve palsies from traction placed on the baby's head in attempts to deliver the shoulder. In addition, the baby's life is threatened since the baby is unable to breathe and umbilical cord flow is often dramatically reduced during this phase of the delivery.

The nurse is providing health teaching to a group of women of childbearing age. One woman, who states that she is a smoker, asks about its impact on the pregnancy. The nurse responds that which of the following fetal complications can develop if the mother smokes? 1. Genetic changes in the fetal reproductive system. 2. Extensive central nervous system damage. 3. Addiction to the nicotine inhaled from the cigarette. 4. Fetal intrauterine growth restriction.

4. Smoking in pregnancy does cause fetal intrauterine growth restriction. When someone smokes, there is a vasoconstrictive effect that occurs in the body. This vasoconstrictive effect is also seen at the placental site. Placentas of women who smoke are much smaller than those of nonsmoking women and, because of this, babies receive less oxygen and nutrients via the placenta. As a result, their growth is restricted.

Which of the following statements is appropriate for the nurse to say to a patient with a complete placenta previa? 1. "During the first phase of labor you will do slow chest breathing." 2. "You should ambulate in the halls at least two times each day." 3. "The doctor will deliver you once you reach 25 weeks' gestation." 4. "Please remember to tell me if you become constipated."

4. Straining at stool can result in enough pressure to result in placental bleeding. Clients diagnosed with complete placenta previa are usually maintained on bed rest. Because one of the many complications of bed rest is constipation, these clients must be monitored carefully. Many physicians order Colace (docusate sodium), a stool softener, to prevent this complication.

Nurses working in obstetric clinics know that, in general, teen pregnancies are high risk because of which of the following? 1. High probability of chromosomal anomalies. 2. High oral intake of manganese and zinc. 3. High numbers of postterm deliveries. 4. High number of late prenatal care registrants

4. Teens are likely to delay entry into the health care system. Late entry into prenatal care is particularly problematic for teen pregnancies. Because organogenesis occurs during the first trimester, by the time many teens acknowledge that they are pregnant and seek care they are already past this critical period. They are likely to have consumed damaging substances or, at the very least, consumed inadequate quantities of essential nutrients, like folic acid.

A client is scheduled for an external version. The nurse would expect to prepare which of the following medications to be administered prior to the procedure? 1. Oxytocin (Pitocin). 2. Ergonovine (Methergine). 3. Betamethasone (Celestone). 4. Terbutaline (Brethine).

4. Terbutaline (Brethine) is a smooth muscle relaxing agent. It would be administered prior to an external version. TEST-TAKING TIP: It is important that the uterine muscle not impede the physician's manipulations during an external version. In order to facilitate the movement, therefore, a muscle relaxant is administered. Terbutaline is one relaxing agent that is used by obstetricians.

A nurse administers magnesium sulfate via infusion pump to an eclamptic woman in labor. Which of the following outcomes indicates that the medication is effective? 1. Client has no patellar reflex response. 2. Urinary output 30 cc/hr. 3. Respiratory rate 16 rpm. 4. Client has no grand mal seizures.

4. The absence of seizures is an expected outcome related to magnesium sulfate administration. TEST-TAKING TIP: Eclamptic clients have seized. Magnesium sulfate is ordered and administered to these clients because it is an anticonvulsant. An expected outcome of its administration, therefore, is that the client will have no more seizures.

woman, G3P2002, 42 weeks' gestation, is admitted to the labor suite for induction. A biophysical profile (BPP) report on the client's chart states BPP score of 6 of 10. The nurse should monitor this client carefully for which of the following? 1. Maternal hypertension. 2. Maternal hyperglycemia. 3. Increased fetal heart variability. 4. Late fetal heart decelerations

4. The baby is at high risk for late fetal heart decelerations secondary to a postmature placenta. TEST-TAKING TIP: A BPP of 8 or lower indicates that the fetus is in jeopardy. The five assessments that constitute the BPP are nonstress test (NST), fetal movement, fetal breathing, amniotic fluid volume, and fetal tone. Each assessment is given a score of 0 or 2.

A pregnant Latina is being seen in the prenatal clinic with diarrhea, fever, stiff neck, and headache. Upon inquiry, the nurse learns that the woman drinks unpasteurized milk and eats soft cheese daily. For which of the following bacterial infections should this woman be assessed? 1. Staphylococcus aureus. 2. Streptococcus albicans. 3. Pseudomonas aeruginosa. 4. Listeria monocytogenes

4. The client is likely suffering from listeriosis, an infection caused by Listeria naonocytogenes bacteria. Latin women are especially at high risk for listeriosis because of their dietary patterns. They often eat soft cheeses and are unlikely to fear drinking unpasteurized milk. It is important that the nurse communicate to all pregnant women the need to refrain from consuming those substances with a clear rationale for the warning.

The home health nurse is visiting a client with HIV who is 6 weeks postdelivery. Which of the following findings would indicate that patient teaching by the nurse in the hospital was successful? 1. The client is breastfeeding her baby every two hours. 2. The client is using a diaphragm for family planning. 3. The client is taking her temperature every morning. 4. The client is seeking care for a recent weight loss.

4. The client is seeking care for a recent weight loss.

A client has severe preeclampsia. The nurse would expect the primary health care practitioner to order tests to assess the fetus for which of the following? 1. Severe anemia. 2. Hypoprothrombinemia. 3. Craniosynostosis. 4. Intrauterine growth restriction.

4. The fetus should be assessed for intrauterine growth restriction. TIP: Perfusion to the placenta drops when clients are preeclamptic because the client's hypertension impairs adequate blood flow. When the placenta is poorly perfused, the baby is poorly nourished. Without the nourishment provided by the mother through the umbilical vein, the fetus' growth is affected.

A client telephones the labor and delivery suite and states, "My bag of waters just broke and it smells funny." Which of the following responses should the nurse make at this time? 1. "Have you notified your doctor of the smell?" 2. "The bag of waters always has an unusual smell." 3. "Your labor should start pretty soon." 4. "Have you felt the baby move since the membranes broke?"

4. The most important information is the health and well-being of the fetus. Fetal movement indicates that the baby is alive. TEST-TAKING TIP: There are two concerns in this scenario: the fact that the membranes just ruptured and the smell of the fluid. The nurse should, therefore, consider two possible problems: possible prolapsed cord, which may occur as a result of the rupture of the amniotic sac, and possible infection, which may be indicated by the smell. Normal fetal movement will give the nurse some confidence that the cord is not prolapsed. This is the first question that should be asked. Then, the client should be encouraged to go to the hospital to be assessed for possible infection and signs of labor.

A client, G3P2002, is immediately postexternal version. The nurse monitors this client carefully for which of the following? 1. Decreased urinary output. 2. Elevated blood pressure. 3. Severe occipital headache. 4. Variable fetal heart decelerations.

4. The nurse should monitor the client carefully for variable fetal heart decelerations TEST-TAKING TIP: The umbilical cord can become compressed during an external version. Variable decelerations are caused by umbilical cord compression. If the cord were to become compressed, the nurse would note variable decelerations on the fetal heart monitor tracing.

A primigravid client received Prepidil (dinoprostone) for induction 8 hours ago. The Bishop score is now 10. Which of the following actions by the nurse is appropriate? 1. Perform nitrazine analysis of amniotic fluid. 2. Report abnormal findings to the obstetrician. 3. Place woman on her side. 4. Monitor for onset of labor.

4. The nurse should monitor this client for the onset of labor. TEST-TAKING TIP: The Bishop score indicates the inducibility of the cervix of a client. Five signs are assessed—cervical position, cervical dilation, cervical effacement, cervical station, and cervical consistency. A total score is calculated. A primigravid cervix is considered inducible when the Bishop score is 9 or higher. A multigravid cervix is considered inducible when the Bishop score is 5 or higher.

A 40-week-gestation client has an admitting platelet count of 90,000 mm3 and a hematocrit of 29%. Her lab values 1 week earlier were platelet count 200,000 mm3 and hematocrit 37%. Which additional abnormal lab value would the nurse expect to see? 1. Decreased serum creatinine level. 2. Elevated red blood count (RBC). 3. Decreased alkaline phosphatase. 4. Elevated alanine transaminase (ALT).

4. The nurse would expect to see an elevated ALT. TEST-TAKING TIP: This is a difficult, critical thinking question. This client is exhibiting signs of HELLP syndrome (low platelets and hemolysis). Even though not a part of the HELLP constellation, a client in severe preeclampsia would have poor renal function (elevated serum creatinine level). With hemolysis, the nurse would expect to see a drop in the RBC count, and with a damaged liver, an elevated alkaline phosphotase level as well as an elevated ALT level.

A pregnant woman, G3P2002, had her two other children by cesarean section. Which of the following situations would mandate that this delivery also be by cesarean? 1. The woman refuses to have a regional anesthesia. 2. The woman is postdates with intact membranes. 3. The baby is in the occiput posterior position. 4. The previous uterine incisions were vertical

4. The presence of vertical incisions in the uterine wall is an absolute indication for a cesarean delivery. TEST-TAKING TIP: The muscle tissue that contracts during labor is located in the fundal region of the uterus. A vertical incision into the uterus ligates fundal tissue. The scar that forms from the incision is nonelastic, putting the client at risk of uterine rupture. Having had a previous vertical uterine incision, therefore, is an absolute indicator for future cesarean delivery. In addition, some physicians also encourage clients who have had low-flap (Pfannenstiel) incisions into the uterus to have all subsequent children delivered via cesarean section. (It is important to note that the type of incision that the surgeon used to open the skin is not necessarily the type of incision used to open the uterus.)

A nurse is caring for four clients on the labor and delivery unit. Which of the following actions should the nurse take first? 1. Check the blood sugar of a gestational diabetic. 2. Assess the vaginal blood loss of a client who is post-spontaneous abortion. 3. Assess the patellar reflexes of a client with mild preeclampsia. 4. Check the fetal heart rate of a client who just ruptured membranes.

4. The priority action for this nurse is to assess the fetal heart rate of a client who has just ruptured membranes. The nurse is assessing for prolapsed cord, which is an obstetric emergency. TEST-TAKING TIP: Identifying the priority action is the most difficult thing that nurses must do. The nurse must determine which of the situations is most life threatening. Of the four choices above, prolapsed cord is life threatening to the fetus. None of the other situations, as stated in the question, is life threatening to either the mother or the fetus.

A nurse is monitoring a client who is receiving an amnioinfusion. Which of the following assessments is critical for the nurse to make in order to prevent a serious complication related to the procedure? 1. Color of the amniotic fluid. 2. Maternal blood pressure. 3. Cervical effacement. 4. Uterine resting tone

4. The uterine resting tone should be carefully monitored with an internal pressure electrode during amnioinfusion. TEST-TAKING TIP: Because fluid is being instilled into the uterine cavity, there is potential for the fluid to overload the space. As a result, the uterine resting tone will increase dramatically with the potential that the uterus could rupture. It is critically important, therefore, that the nurse monitor the resting tone frequently throughout the procedure.

A client has been admitted with a diagnosis of hyperemesis gravidarum. Which of the following lab values would be consistent with this diagnosis? 1. pO, 90, pCO2 35, HCO3 19 mEq/L, pH 7.30. 2. pO, 100, pCO2 30, HCO3 21 mEq/L, pH 7.50. 3. pO, 60, pCO, 50, HCO3 28 mEq/L, pH 7.30. 4. pO, 90, pCO, 45, HCO3 30 mEq/L, pH 7.50.

4. This client is in metabolic alkalosis. This is consistent with a diagnosis of hyperemesis gravidarum. TEST-TAKING TIP: The test taker must not panic when confronted with blood gas data. If assessed methodically, the test taker should have little trouble determining the correct answer. The first action is to determine what the results should show. If a woman is vomiting repeatedly, one would expect her to have lost acid from the stomach. She would, therefore, be in metabolic alkalosis. The test taker should then look at the pH levels-they should be elevated-and the 02 levels-they should be normal-to begin to determine which response is correct.

During a prenatal interview, a client tells the nurse, "My mother told me she had toxemia during her pregnancy and almost died!" Which of the following questions should the nurse ask in response to this statement? 1. "Does your mother have a family history of cancer?" 2. "Did your mother tell you what she was toxic from?" 3. "Does your mother have diabetes now?" 4. "Did your mother say whether she had a seizure or not?

4. This is the appropriate question. The nurse is asking whether or not the client's mother developed eclampsia. TEST-TAKING TIP: The hypertensive illnesses of pregnancy used to be called toxemia of pregnancy as well as pregnancy-induced hypertension (PIH). That term is still heard in the community because the mothers and grandmothers of clients were told that they had toxemia of pregnancy. Because daughters of clients who have had preeclampsia are high risk for hypertensive illness, it is important to find out whether or not the client's mother had developed eclampsia.

A nurse is caring for a 25-year-old client who has just had a spontaneous first trimester abortion. Which of the following comments by the nurse is appropriate? 1. "You can try again very soon." 2. "It is probably better this way." 3. "At least you weren't very far along." 4. "I'm here to talk if you would like."

4. This statement is appropriate. The nurse is offering his or her assistance to the client. Clients during the first trimester are often ambivalent about pregnancy. Those who abort at this time express a variety of feelings from intense sorrow to joy. The nurse should offer assistance to the client without making any assumptions about the client's feelings toward the pregnancy loss. Speaking platitudes is completely inappropriate

An obese client is being seen by the nurse during her prenatal visit. Which of the following comments by the nurse is appropriate at this time? 1. "We will want you to gain the same amount of weight we would encourage any pregnant woman to gain." 2. "To have a healthy baby we suggest that you go on a weight reduction diet right 3. "To prevent birth defects we suggest that you gain weight during the first trimester and then maintain your weight for the rest of the pregnancy." 4. "We suggest that you gain weight throughout your pregnancy but not quite as much as other women."

4. This statement is true. Normal weight clients are encouraged to gain between 25 and 35 pounds. TEST-TAKING TIP: It is not appropriate for an obese client to lose weight or to refrain from gaining weight during her pregnancy. When clients lose weight, they begin to break down fats and ketones develop. An acidic environment is unsafe for the unborn baby.

A gravid woman is carrying monochorionic twins. For which of the following complications should this pregnancy be monitored? 1. Oligohydramnios. 2. Placenta previa. 3. Cephalopelvic disproportion. 4. Twin-to-twin transfusion.

4. Twin-to-twin transfusion is a relatively common complication of monozygotic twin pregnancies. TEST-TAKING TIP: The key to answering this question is the fact that the twins originate from the same egg-that is, they are monozygotic twins. They share a placenta and a chorion. Because their blood supply is originating from the same source, the twins' circulations are connected. As a result, one twin may become the donor twin while the second twin may become the recipient. The donor grows poorly and develops severe anemia. The recipient becomes polycythemic and large.

The nurse is performing an assessment on a client diagnosed with placenta previa. Which assessment findings should the nurse expect to note? Select all that apply. 1. uterine rigidity 2. uterine tenderness 3. severe abdominal pain 4. bright red vaginal bleeding 5. soft, relaxed, nontender uterus 6. fundal height may be greater than expected for gestational age

4. bright red vaginal bleeding 5. soft, relaxed, nontender uterus 6. fundal height may be greater than expected for gestational age

A client is receiving an IV heparin drip at 16 mL/hr via an infusion pump for a diagnosis of deep vein thrombosis. The label on the 1 /2 liter bag of D5W indicates 25,000 units of heparin have been added. How many units of heparin is the client receiving per hour? (Calculate to the nearest whole.) __________ units per hour.

800 units/hr

A woman who has been diagnosed with an ectopic pregnancy is to receive methotrexate 50 mg/m' IM. The woman weighs 136 lb and is 5 ft 4 inches tall. What is the maximum safe dose, in mg, of methotrexate that this woman can receive? ________mg

83.5 mg Because the recommended dosage is written per square meters, the nurse must calculate a safe dosage level for this medication using a body surface area formula. The formula for determining the body surface area (BSA) of a client, using the English system, is: BSA =weight (lb) X height (in) 3131 The nurse first calculates the BSA. (The test taker must remember that there are 12 inches in 1 foot) The calculation in this situation is: 13 6 x 64 3131 8704 3131 2.779 BSA = 2.78 BSA = 1.67mz Second, a ratio and proportion equation must be created and solved: Reconunended dosage _ Safe dosage 1 m2 Client's BSA 50 _ x 1 1.67 x = 83.5 mg The nurse now knows that the maximum dosage of methotrexate that this client can safely receive is 83.5 mg. 35. 73

While assessing a postpartum client the nurse finds that the client has excessive foul smelling lochia. What medication would be helpful in treating the condition? A broad-spectrum antibiotic A diuretic to induce urination Intravenous oxytocin agents Intravenous fluids

A broad-spectrum antibiotic Rationale Profuse lochia with a foul smell indicates that the client has a bacterial infection, such as endometritis. Therefore, the nurse anticipates that the PHP will prescribe a broad-spectrum antibiotic. Diuretics are prescribed when the client shows urinary retention, characterized by decreased urinary output. Intravenous oxytocin agents are prescribedto induce uterine contractions in case of uterine inversion. The nurse should ensure adequate fluid intake; however, administration of intravenous fluids is likely not necessary.

While assessing a postpartum client the nurse finds that the client has excessive foul smelling lochia. What medication would be helpful in treating the condition? A broad-spectrum antibiotic A diuretic to induce urination Intravenous oxytocin agents Intravenous fluids

A broad-spectrum antibiotic Rationale Profuse lochia with a foul smell indicates that the client has a bacterial infection, such as endometritis. Therefore, the nurse anticipates that the PHP will prescribe a broad-spectrum antibiotic. Diuretics are prescribed when the client shows urinary retention, characterized by decreased urinary output. Intravenous oxytocin agents are prescribedto induce uterine contractions in case of uterine inversion. The nurse should ensure adequate fluid intake; however, administration of intravenous fluids is likely not necessary.

Premature dilation of the cervix (reduced cervical competence)

A cause of late miscarriage; it is traditionally defined as passive and painless dilation of the cervix during the second trimester.

magnesium sulfate

A central nervous system (CNS) depressant used during preterm labor for its ability to relax smooth muscles, including those of the uterus; it is administered intravenously.

In which pregnant client does the nurse identify the need to screen for undiagnosed homozygous maternal phenylketonuria (PKU)? A client who had a macrosomic fetus in a previous pregnancy A client who had obstructed labor in a previous pregnancy A client who has given birth to a microcephalic infant A patient who had placental insufficiency in a previous pregnancy

A client who has given birth to a microcephalic infant Rationale A client who has previously had a microcephalic infant must be screened for undiagnosed homozygous maternal PKU in the first prenatal visit. Toxic accumulation of phenylalanine in the blood due to a lack of the enzyme phenylalanine hydrolase interferes with brain development. The client who had a previous macrosomic fetus must be screened for hyperglycemia. A client with a macrosomic fetus may have obstructed labor. Placental insufficiency may lead to stillbirth. The client who had placental insufficiency in a previous pregnancy need not be screened for PKU.

Which client may need a cesarean delivery because of complications related to gestational diabetes? A client with a big fetus A client with uterine growth A client with reduced fetal movement A client with less than normal pelvic brim

A client with a big fetus Rationale Fetal macrosomia is a common complication associated with gestational diabetes. Hypersecretion of fetal insulin hormone as a response to maternal hyperglycemia results in an increased size of the fetus. Maternal hyperglycemia does not cause the development of uterine growths. Fetal movements may be cause for a cesarean, but they are not associated with the client's gestational diabetes. The size of the pelvic brim is not altered by maternal hyperglycemia. p. 690

Which patients with diabetes may develop complications if they perform exercises? Select all that apply. A client who is on insulin A client with diabetic ketoacidosis A client with uncontrolled hypertension A client with severe peripheral neuropathy A client who has lost 5 kg weight after diagnosis

A client with diabetic ketoacidosis A client with uncontrolled hypertension A client with severe peripheral neuropathy Rationale Exercise is usually prescribed for the prevention of complications in diabetic clients. However, a client with ketoacidosis should not do exercises, because exercises burn more fats and proteins, thereby increasing the blood ketone levels. Blood pressure increases with exercise; therefore, a client with uncontrolled blood pressure should not perform exercises. A client with severe peripheral neuropathy is susceptible to injury because of loss of sensations. Therefore, exercise is contraindicated in this client. A client with insulin dependent diabetes mellitus can perform exercises under supervision. Weight loss is a common complication associated with diabetes. A client who has lost weight can perform exercises to prevent complications of diabetes. p. 694

The nurse is working in an obstetric ward. Which client in the ward is at the highest risk of developing hydatidiform mole? A client with hypothyroidism A client with diabetes mellitus A client with lupus erythematosus A client with prior molar pregnancy

A client with prior molar pregnancy Rationale Hydatidiform mole is a benign proliferative growth of the placental trophoblast. In this condition the chorionic villi develop into edematous or avascular transparent vesicles, which hang in a grapelike cluster. A client with prior molar pregnancy is at a higher risk of developing hydatidiform mole. The presence of growing tissue in a molar pregnancy increases the risk of hydatidiform mole. Clients with hypothyroidism, diabetes mellitus, and lupus erythematosus are not at a higher risk of developing hydatidiform mole.

A nurse is caring for a client whose labor is being augmented with oxytocin. The nurse recognizes that the oxytocin should be discontinued immediately if there is evidence of what? Uterine contractions occurring every 8 to 10 minutes A fetal heart rate (FHR) of 180 with absence of variability The client needing to void Rupture of the client's amniotic membrane

A fetal heart rate (FHR) of 180 with absence of variability Rationale A fetal heart rate (FHR) of 180 with absence of variability is nonreassuring. The oxytocin should be immediately discontinued and the physician should be notified. Uterine contractions that occur every 8 to 10 minutes do not qualify as hyperstimulation. The oxytocin should be discontinued if uterine hyperstimulation occurs. The client needing to void is not an indication to discontinue the oxytocin induction immediately or to call the physician. Unless a change occurs in the FHR pattern that is nonreassuring or the client experiences uterine hyperstimulation, the oxytocin does not need to be discontinued. The physician should be notified that the client's membranes have ruptured.

A nurse is caring for a woman whose labor is being augmented with oxytocin. The nurse recognizes that the oxytocin should be discontinued immediately if there is evidence of what? Uterine contractions occurring every 8 to 10 minutes. A fetal heart rate (FHR) of 180 with absence of variability. The woman needing to void. Rupture of the woman's amniotic membranes.

A fetal heart rate (FHR) of 180 with absence of variability. Rationale An FHR of 180 with absence of variability is nonreassuring. The oxytocin should be discontinued immediately and the physician should be notified. The oxytocin should be discontinued if uterine hyperstimulation occurs. Uterine contractions that occur every 8 to 10 minutes do not qualify as hyperstimulation. The woman needing to void is not an indication to discontinue the oxytocin induction immediately or to call the physician. Unless a change occurs in the FHR pattern that is nonreassuring or the woman experiences uterine hyperstimulation, the oxytocin does not need to be discontinued. The physician should be notified that the woman's membranes have ruptured.

A nurse is caring for a client whose labor is being augmented with oxytocin. The nurse recognizes that the oxytocin should be discontinued immediately if there is evidence of: Uterine contractions occurring every 8 to 10 minutes. A fetal heart rate (FHR) of 180 with absence of variability. The client needing to void. Rupture of the client's amniotic membranes.

A fetal heart rate (FHR) of 180 with absence of variability. A fetal heart rate (FHR) of 180 with absence of variability is nonreassuring; the oxytocin should be immediately discontinued and the physician should be notified. The oxytocin should also be discontinued if uterine hyperstimulation occurs. Uterine contractions that occur every 8 to 10 minutes do not qualify as hyperstimulation. The client needing to void is not an indication to discontinue the oxytocin induction immediately or to call the physician. The oxytocin does not need to be discontinued when the membranes rupture, but the physician should be notified.

What is a hydatidiform mole?

A form of gestational trophoblastic disease that occurs when the trophoblasts, which are the peripheral cells that attach the fertilized ovum to the uterine wall, develop abnormally. The mole manifests as an edematous grape-like cluster that may be nonmalignant or may develop into choriocarcinoma

Hydatidiform mole (molar pregnancy)

A group of pregnancy-related trophoblastic proliferative disorders without a viable fetus; they are caused by abnormal fertilization.

The nurse is caring for a patient whose delivery was assisted by forceps. Which assessment finding should the nurse report to the health care provider immediately? Pain with defecation Fundus is firm on palpation A hard, turgid area on the labia minora Episiotomy incision flush with the surrounding skin

A hard, turgid area on the labia minora A hard, turgid area in the perineal area is a sign of a hematoma and should be reported to the health care provider immediately because it can result in significant blood loss.

The nurse is performing an initial assessment on a woman receiving anticoagulant therapy. Which assessment finding prompts the nurse to alert the health care provider immediately? Dark red lochia A heart rate of 115 bpm Blood pressure of 120/72 mmHg Pain and tenderness in the left leg that has not stopped since starting anticoagulant therapy

A heart rate of 115 bpm Heart rate of 115 bpm is characteristic of hypovolemia and could indicate internal bleeding. The health care provider should be notified immediately.

Precipitous Labor

A labor that lasts less than 3 hours from the onset of contractions to the time of birth Causes: -hypertonic uterine contractions that are tetanic in intensity -placental abruption -uterine tachysystole -recent cocaine use Maternal complications: -uterine rupture -lacerations of the birth canal -amniotic fluid empolism -postpartum hemorrhage Fetal complications: -fetal dystocia -hypoxia -intracranial trauma due to rapid birth

HELLP syndrome

A laboratory diagnosis for a variant of severe preeclampsia that involves hepatic dysfunction; it is characterized byhemolysis, elevated liver enzymes, and low platelets.

A pregnant client with severe preeclampsia who is being transported to a tertiary care center needs to be administered magnesium sulfate injection for seizure activity. What actions does the nurse take when administering the drug? A 10-g dose is administered in the buttock. A local anesthetic is added to the solution. The Z-track technique is used to inject the drug. The injection site is massaged after the injection. The subcutaneous route is used to inject the drug.

A local anesthetic is added to the solution. The Z-track technique is used to inject the drug. The injection site is massaged after the injection. Rationale The nurse adds a local anesthetic to the solution to reduce pain that is caused by the injection. The Z-track technique is used to inject the drug so that the drug is injected in the intramuscular (IM) tissue safely. The nurse gently massages the site after administering the injection to reduce pain. The nurse administers two separate injections of 5 g in each buttock. Magnesium sulfate injections are administered in the IM layer and not the subcutaneous layer.

hematoma

A localized collection of blood in the tissues and can occur internally, involving the vaginal sulcus or other organs; vulvar hematomas are the most common. Predisposing conditions include operative delivery with forceps and injury to a blood vessel. •Can be a life-threatening condition.

Dysfunctional Labor (dystocia)

A long, difficult or abnormal labor Causes: -ineffective uterine contractions or maternal bearing-down efforts (powers) -alterations in the pelvic structure (the passage) -fetal causes- including abnormal presentation or position, anomalies, excessive size, and number of fetuses (passenger) -maternal position during labor and birth -psychologic responses to past experiencs, culture, and support systems

Amniotic Membrane Stripping

A method of inducin labor through the release of prostaglandins and oxytocin -involves separation of the membrane from the wall of the cervix and lower uterine segment by inserting a finger into internal cervical os and rotating 360 degrees -works best in a primigravida at term with an unripe cervix -Uncomfortable procedure -increases risk of infection, ROM, bleeding, and precipitous labor and birth

Which client situation presents the greatest risk for the occurrence of hypotonic dysfunction during labor? A primigravida who is 17 years old A 22-year-old multiparous woman with ruptured membranes A primigravida who has requested no analgesia during her labor A multiparous woman at 39 weeks of gestation who is expecting twins

A multiparous woman at 39 weeks of gestation who is expecting twinsA multiparous woman at 39 weeks of gestation who is expecting twins Rationale Overdistention of the uterus in a multiple pregnancy is associated with hypotonic dysfunction, because the stretched uterine muscle contracts poorly. A young primigravida usually will have good muscle tone in the uterus. This prevents hypotonic dysfunction. There is no indication that this woman's uterus is overdistended, which is the main cause of hypotonic dysfunction. A primigravida usually will have good uterine muscle tone, and there is no indication of an overdistended uterus.

The charge nurse is making patient assignments for the next shift of nurses. The nurse anticipates that induction of labor likely will be indicated for which patient? A patient at >42 weeks' gestation A patient with lupus and who is in remission A patient with adequately controlled diabetes A patient experiencing Braxton-Hicks contractions

A patient at >42 weeks' gestation Postterm pregnancy (>42 weeks) is an indication for induction of labor. Postterm pregnancy is linked with both fetal and maternal health complications; as a result, health care providers usually do everything they can to ensure that an infant is delivered as close to the due date as possible.

Which patients have the highest risk for experiencing fetal demise? Select all that apply. A patient with a BMI of 30 A patient carrying twins A patient with a history of anorexia A patient with a history of a gastric bypass A patient who uses heroin during pregnancy

A patient carrying twins A multifetal gestation pregnancy increases the patient's risk of experiencing fetal demise, because of an increased incidence of preterm births, which carries a higher risk of neonatal mortality. A patient who uses heroin during pregnancy Illicit drug use increases the risk of fetal demise. Chronic untreated heroin use is associated with fetal death.

The nurse should anticipate preparing for an operative delivery when caring for which patient? A patient with a history of perineal laceration A patient who rates her pain at ten out of ten A patient whose bladder is distended and who is unable to void A patient who has been pushing for three hours with minimal fetal progress

A patient who has been pushing for three hours with minimal fetal progress Maternal exhaustion is an indication for operative delivery. If the patient is exhausted and making minimal progress, there may be a need to assist with forceps or vacuum.

The nurse is caring for several patients in labor. Which patient exhibits a risk that indicates the need for an operative vaginal delivery? A patient whose previous birth was assisted with the use of forceps A patient whose fetus has a heart rate between 125 and 140 beats/min A patient who stopped responding to cues to push, stating she is exhausted A patient whose most recent contractions suggest uterine hyperstimulation

A patient who stopped responding to cues to push, stating she is exhausted Maternal exhaustion may require operative delivery if the patient is unable to push sufficiently to facilitate vaginal birth without operative interventions.

The nurse is caring for two women in early labor. Which patient is at greatest risk for perineal laceration? A primipara A patient with prolonged labor A patient experiencing a vaginal delivery after cesarean delivery A patient whose delivery will be assisted with a vacuum extractor

A patient whose delivery will be assisted with a vacuum extractor Patients who experience operative delivery, which is delivery assisted by forceps or vacuum extractor, are at increased risk for perineal laceration because of the introduction of a mechanical instrument into the vagina.

The nurse is caring for a patient in the second stage of labor. Which patient condition is most likely to result in the need for an episiotomy? A patient with a history of perineal laceration A patient receiving oxytocin for induction of labor A patient whose fetus is experiencing shoulder dystocia A patient who had an episiotomy during a previous delivery

A patient whose fetus is experiencing shoulder dystocia Shoulder dystocia is an indication for episiotomy because it is necessary to allow as much room as possible for the delivery of the shoulder.

The nurse arrives in the obstetric unit and receives the shift report. Which patient should the nurse assess first? A patient whose fundus is firm and who is saturating one perineal pad every hour. A patient whose infant was delivered with use of a vacuum extractor and whose heart rate is 89 beats/min. A patient with a boggy fundus who has been saturating one perineal pad every two hours. A patient who had a cesarean delivery and reports tenderness around the incision site.

A patient whose fundus is firm and who is saturating one perineal pad every hour. Saturating one perineal pad per hour is a sign of potential postpartum hemorrhage, and the nurse should assess this patient first.

Which patient is most at risk for abruptio placentae? A patient who is primipara A 35-year-old Asian woman A patient whose first child was delivered by c-section A patient with a blood pressure reading of 160/90 mmHg

A patient with a blood pressure reading of 160/90 mmHg Hypertension places pregnant women at higher risk for abruptio placentae.

Which pregnant patient is a likely candidate for expectant management? A molar pregnancy that has a risk for vaginal bleeding A patient who is 32 6⁄ 7 weeks of gestation, who is likely to deliver soon A patient diagnosed with preeclampsia who is on an antihypertensive medication A patient with a threatened miscarriage that has no bleeding or infection

A patient with a threatened miscarriage that has no bleeding or infection Rationale If bleeding and infection does not occur after a threatened miscarriage, then the patient is managed expectantly to continue the pregnancy successfully. Molar pregnancy is terminated as soon as it is identified and hence there is no need for expectant management. A patient with 32 6⁄ 7 weeks of gestation who is likely to deliver soon does not need to be managed expectantly. A patient with preeclampsia who is on an antihypertensive medication does not need expectant management as the condition can be managed at home.

Which patient is at greatest risk for mastitis? A patient who is breastfeeding twins A patient with dry, cracked skin on one nipple A patient who pumps the breasts between feedings A patient who uses a supplemental nursing system (SNS)

A patient with dry, cracked skin on one nipple Injury to the nipple area, such as cracked skin, increases the risk for the patient developing mastitis.

Missed Miscarriage- What is it? Clinical manifestations and management

A pregnancy in which the fetus has died but the products of conception are retained in utero for up to several weeks -may be diagnosed by ultrasound after the uterus stops increasing in size or starts decreasing in size -there may be no bleeding or cramping -cervical os remains closed -no passage of tissue -no cervical dilation Management: -If spontaneous evacuation of the uterus does not occur in 1 month, pregnancy is terminated by method appropriate to duration of pregnancy -blood clotting factors are monitored until uterus i empty -DIC and incoagulability of blood with uncontrolled hemorrhage may develop with fetal death after the twelfth week if products of conception or retained for longer than 5 weeks -may be treated with dilation and curettage or misoprostol given orally or vaginally

Miscarriage (spontaneous abortion)

A pregnancy that ends as a result of natural causes before 20 weeks of gestation A fetal weight <500 g also may be used to define abortion

indomethacin (Indocin)

A prostaglandin synthesis inhibitor that relaxes uterine smooth muscles; it is administered orally."

Proteinuria

A protein concentration at or greater than 300 mg/dL in a 24-hour urine collection.

Bishop Score

A rating system that is used to evaluate inducibility -13 point -point scale -A score of 8 or more indicates that the cervix is soft, anterior, 50% or more effaced, and dilated 2 cm or more and the presenting part is engaged (menas induction of labor is successful) Score of 0-3 on the following categories: -Dilation (cm) -Effacement (%) -Station (cm) -Cervical consistency -Cervical position

A woman with severe preeclampsia is receiving a magnesium sulfate infusion. The nurse becomes concerned after assessment when the woman exhibits what? A sleepy, sedated affect. A respiratory rate of 10 breaths/min. Deep tendon reflexes of 2+. Absent ankle clonus.

A respiratory rate of 10 breaths/min. Rationale A respiratory rate of 10 breaths per minute indicates that the patient is experiencing respiratory depression (bradypnea) from magnesium toxicity. Because magnesium sulfate is a central nervous system (CNS) depressant, the woman will most likely become sedated when the infusion is initiated. Deep tendon reflexes of 2+ are a normal finding. Absent ankle clonus is a normal finding.

A woman with severe preeclampsia is receiving a magnesium sulfate infusion. The nurse becomes concerned after assessment when the woman exhibits: A sleepy, sedated affect. A respiratory rate of 10 breaths/min. Deep tendon reflexes of 2+. Absence of ankle clonus.

A respiratory rate of 10 breaths/min. A respiratory rate of 10 breaths/min indicates that the client is experiencing respiratory depression (bradypnea) from magnesium toxicity. Because magnesium sulfate is a central nervous system (CNS) depressant, the client will most likely become sedated when the infusion is initiated. Deep tendon reflexes of 2+ are a normal finding, as is absence of ankle clonus.

A woman with severe preeclampsia is receiving a magnesium sulfate infusion. The nurse becomes concerned after assessment when the woman exhibits what? A sleepy, sedated affect A respiratory rate of 10 breaths/minute Deep tendon reflexes of 2+ Absent ankle clonus

A respiratory rate of 10 breaths/minute Rationale A respiratory rate of 10 breaths/minute indicates that the client is experiencing respiratory depression (bradypnea) from magnesium toxicity. Because magnesium sulfate is a central nervous system (CNS) depressant, the client will most likely become sedated when the infusion is initiated. Deep tendon reflexes of 2+ are a normal finding. Absent ankle clonus is a normal finding.

Dilation and Curettage (D&C)

A surgical procedure in which the cervix is dilated if necessary and a curette is inserted to scrape the uterine walls and remove uterine contents May also use a suction curettage (using a catheter attached to an electric-powered vacuum source) -Painful- pain-relief is usually achieved by administering analgesics or sedative IV or orally -A paracervical block may also be administered -a woman who is Rh negative may be given Rhogam

Hypertension;

A systolic blood pressure greater than 140 mm Hg or a diastolic blood pressure greater than 90 mm Hg. The elevated values must be present on 2 separate occasions at least 4-6 hours apart but within a maximum of a 1-week period.

Which woman is at greatest risk for wound infection? A woman who delivered twins with an episiotomy A 41-year-old woman with a vertical cesarean incision A woman with diabetes and a retroperitoneal hematoma A woman experiencing postpartum hemorrhage due to a cervical laceration

A woman experiencing postpartum hemorrhage due to a cervical laceration Hemorrhage increases the risk for wound infection, therefore this patient would be at greatest risk for wound infection.

Which pregnant women are at risk for PPROM? Select all that apply. A woman who is bearing twins A woman who sits eight hours a day at her job A woman who eats spicy foods at each meal A woman who is depressed over losing her job A woman with a Gardnerella vaginalis infection

A woman who is bearing twins Bearing twins can result in overdistention of the uterus; this is a possible cause of PROM. A woman who is depressed over losing her job A pregnant woman who is depressed over losing her job is experiencing maternal stress, which is recognized as a possible cause of PROM. A woman with a Gardnerella vaginalis infection Gardnerella vaginalis infections predispose pregnant women to PROM.

At 32 weeks of gestation, Maria—with hypertension since 28 weeks, hyperactive deep tendon reflexes (DTRs) with clonus, and proteinuria of 4+—has a convulsion.

A. Eclampsia

When is a prophylactic cerclage for an incompetent cervix usually placed (in weeks of gestation)? a. 12 to 14 b. 6 to 8 c. 23 to 24 d. After 24

ANS: A A prophylactic cerclage is usually placed at 12 to 14 weeks of gestation. The cerclage is electively removed when the woman reaches 37 weeks of gestation or when her labor begins. Six to 8 weeks of gestation is too early to place the cerclage. Cerclage placement is offered if the cervical length falls to less than 20 to 25 mm before 23 to 24 weeks. Although no consensus has been reached, 24 weeks is used as the upper gestational age limit for cerclage placement.

A client at 39 weeks of gestation has been admitted for an external version. Which intervention would the nurse anticipate the provider to order? a. Tocolytic drug b. Contraction stress test (CST) c. Local anesthetic d. Foley catheter

ANS: A A tocolytic drug will relax the uterus before and during the version, thus making manipulation easier. CST is used to determine the fetal response to stress. A local anesthetic is not used with external version. Although the bladder should be emptied, catheterization is not necessary.

A woman is undergoing a nipple-stimulated CST. She is having contractions that occur every 3 minutes. The fetal heart rate (FHR) has a baseline heart rate of approximately 120 beats per minute without any decelerations. What is the correct interpretation of this test? a. Negative b. Positive c. Satisfactory d. Unsatisfactory

ANS: A Adequate uterine activity necessary for a CST consists of three contractions in a 10-minute time frame. If no decelerations are observed in the FHR pattern with the contractions, then the findings are considered to be negative. A positive CST indicates the presence of repetitive late FHR decelerations. The terms satisfactory or unsatisfactory are not applicable.

Which information is the highest priority for the nurse to comprehend regarding the BPP? a. BPP is an accurate indicator of impending fetal well-being. b. BPP is a compilation of health risk factors of the mother during the later stages of pregnancy. c. BPP consists of a Doppler blood flow analysis and an amniotic fluid index (AFI). d. BPP involves an invasive form of an ultrasonic examination.

ANS: A An abnormal BPP score is one indication that labor should be induced. The BPP evaluates the health of the fetus, requires many different measures, and is a noninvasive procedure.

What is the correct definition of a spontaneous termination of a pregnancy (abortion)? a. Pregnancy is less than 20 weeks. b. Fetus weighs less than 1000 g. c. Products of conception are passed intact. d. No evidence exists of intrauterine infection

ANS: A An abortion is the termination of pregnancy before the age of viability (20 weeks). The weight of the fetus is not considered because some older fetuses may have a low birth weight. A spontaneous abortion may be complete or incomplete and may be caused by many problems, one being intrauterine infection.

A client asks her nurse, "My doctor told me that he is concerned with the grade of my placenta because I am overdue. What does that mean?" What is the nurse's best response? a. "Your placenta changes as your pregnancy progresses, and it is given a score that indicates how well it is functioning." b. "Your placenta isn't working properly, and your baby is in danger." c. "We need to perform an amniocentesis to detect if you have any placental damage." d. "Don't worry about it. Everything is fine."

ANS: A An explanation of what is meant by the "grade of my placenta" is the most appropriate response. If the client desires further information, the nurse can explain that calcium deposits are significant in postterm pregnancies, and ultrasonography can also be used to determine placental aging. Although stating that the client's placenta is not working properly and that the baby is in danger may be a valid response, it does not reflect therapeutic communication techniques and is likely to alarm the client. An ultrasound, not amniocentesis, is the method of assessment used to determine placental maturation. Telling the client not to worry is not appropriate and discredits her concerns.

A 39-year-old primigravida woman believes that she is approximately 8 weeks pregnant, although she has had irregular menstrual periods all her life. She has a history of smoking approximately one pack of cigarettes a day; however, she tells the nurse that she is trying to cut down. Her laboratory data are within normal limits. What diagnostic technique would be useful at this time? a. Ultrasound examination b. Maternal serum alpha-fetoprotein (MSAFP) screening c. Amniocentesis d. Nonstress test (NST)

ANS: A An ultrasound examination could be performed to confirm the pregnancy and to determine the gestational age of the fetus. An MSAFP screening is performed at 16 to 18 weeks of gestation; therefore, it is too early in the woman's pregnancy to perform this diagnostic test. An amniocentesis is performed if the MSAFP levels are abnormal or if fetal or maternal anomalies are detected. An NST is performed to assess fetal well-being in the third trimester.

A woman in preterm labor at 30 weeks of gestation receives two 12-mg intramuscular (IM) doses of betamethasone. What is the purpose of this pharmacologic intervention? a. To stimulate fetal surfactant production b. To reduce maternal and fetal tachycardia associated with ritodrine administration c. To suppress uterine contractions d. To maintain adequate maternal respiratory effort and ventilation during magnesium sulfate therapy

ANS: A Antenatal glucocorticoids administered as IM injections to the mother accelerate fetal lung maturity. Propranolol (Inderal) is given to reduce the effects of ritodrine administration. Betamethasone has no effect on uterine contractions. Calcium gluconate is given to reverse the respiratory depressive effects of magnesium sulfate therapy.

What is the highest priority nursing intervention when admitting a pregnant woman who has experienced a bleeding episode in late pregnancy? a. Assessing FHR and maternal vital signs b. Performing a venipuncture for hemoglobin and hematocrit levels c. Placing clean disposable pads to collect any drainage d. Monitoring uterine contractions

ANS: A Assessment of the FHR and maternal vital signs will assist the nurse in determining the degree of the blood loss and its effect on the mother and fetus. The most important assessment is to check the well-being of both the mother and the fetus. The blood levels can be obtained later. Assessing future bleeding is important; however, the top priority remains mother/fetal well-being. Monitoring uterine contractions is important but not a top priority.

Which description most accurately describes the augmentation of labor? a. Is part of the active management of labor that is instituted when the labor process is unsatisfactory b. Relies on more invasive methods when oxytocin and amniotomy have failed c. Is a modern management term to cover up the negative connotations of forceps-assisted birth d. Uses vacuum cups

ANS: A Augmentation is part of the active management of labor that stimulates uterine contractions after labor has started but is not progressing satisfactorily. Augmentation uses amniotomy and oxytocin infusion, as well as some more gentle, noninvasive methods. Forceps-assisted births are less common than in the past and not considered a method of augmentation. A vacuum-assisted delivery occurs during childbirth if the mother is too exhausted to push. Vacuum extraction is not considered an augmentation methodology.

The client is being induced in response to worsening preeclampsia. She is also receiving magnesium sulfate. It appears that her labor has not become active, despite several hours of oxytocin administration. She asks the nurse, "Why is this taking so long?" What is the nurse's most appropriate response? a. "The magnesium is relaxing your uterus and competing with the oxytocin. It may increase the duration of your labor." b. "I don't know why it is taking so long." c. "The length of labor varies for different women." d. "Your baby is just being stubborn."

ANS: A Because magnesium sulfate is a tocolytic agent, its use may increase the duration of labor. The amount of oxytocin needed to stimulate labor may be more than that needed for the woman who is not receiving magnesium sulfate. The nurse should explain to the client the effects of magnesium sulfate on the duration of labor. Although the length of labor varies for different women, the most likely reason this woman's labor is protracted is the tocolytic effects of magnesium sulfate. The behavior of the fetus has no bearing on the length of labor.

Of these psychosocial factors, which has the least negative effect on the health of the mother and/or fetus? a. Moderate coffee consumption b. Moderate alcohol consumption c. Cigarette smoke d. Emotional distress

ANS: A Birth defects in humans have not been related to caffeine consumption. Pregnant women who consume more than 300 mg of caffeine daily may be at increased risk for miscarriage or IUGR. Although the exact effects of alcohol in pregnancy have not been quantified, it exerts adverse effects on the fetus including fetal alcohol syndrome, fetal alcohol effects, learning disabilities, and hyperactivity. A strong, consistent, causal relation has been established between maternal smoking and reduced birth weight. Childbearing triggers profound and complex physiologic and psychologic changes on the mother. Evidence suggests a relationship between emotional distress and birth complications.

Which intervention is most important when planning care for a client with severe gestational hypertension? a. Induction of labor is likely, as near term as possible. b. If at home, the woman should be confined to her bed, even with mild gestational hypertension. c. Special diet low in protein and salt should be initiated. d. Vaginal birth is still an option, even in severe cases.

ANS: A By 34 weeks of gestation, the risk of continuing the pregnancy may be considered greater than the risks of a preterm birth. Strict bed rest is controversial for mild cases; some women in the hospital are even allowed to move around. Diet and fluid recommendations are essentially the same as for healthy pregnant women, although some authorities have suggested a diet high in protein. Women with severe gestational hypertension should expect a cesarean delivery.

Which order should the nurse expect for a client admitted with a threatened abortion? a. Bed rest b. Administration of ritodrine IV c. Nothing by mouth (nil per os [NPO]) d. Narcotic analgesia every 3 hours, as needed

ANS: A Decreasing the woman's activity level may alleviate the bleeding and allow the pregnancy to continue. Ritodrine is not the first drug of choice for tocolytic medications. Having the woman placed on NPO is unnecessary. At times, dehydration may produce contractions; therefore, hydration is important. Narcotic analgesia will not decrease the contractions and may mask the severity of the contractions.

The nurse sees a woman for the first time when she is 30 weeks pregnant. The client has smoked throughout the pregnancy, and fundal height measurements now are suggestive of intrauterine growth restriction (IUGR) in the fetus. In addition to ultrasound to measure fetal size, what is another tool useful in confirming the diagnosis? a. Doppler blood flow analysis b. Contraction stress test (CST) c. Amniocentesis d. Daily fetal movement counts

ANS: A Doppler blood flow analysis allows the examiner to study the blood flow noninvasively in the fetus and the placenta. It is a helpful tool in the management of high-risk pregnancies because of IUGR, diabetes mellitus, multiple fetuses, or preterm labor. Because of the potential risk of inducing labor and causing fetal distress, a CST is not performed on a woman whose fetus is preterm. Indications for an amniocentesis include diagnosis of genetic disorders or congenital anomalies, assessment of pulmonary maturity, and the diagnosis of fetal hemolytic disease, not IUGR. Fetal kick count monitoring is performed to monitor the fetus in pregnancies complicated by conditions that may affect fetal oxygenation. Although this may be a useful tool at some point later in this woman's pregnancy, it is not used to diagnose IUGR.

A woman with severe preeclampsia has been receiving magnesium sulfate by intravenous infusion for 8 hours. The nurse assesses the client and documents the following findings: temperature of 37.1° C, pulse rate of 96 beats per minute, respiratory rate of 24 breaths per minute, BP of 155/112 mm Hg, 3+ DTRs, and no ankle clonus. The nurse calls the provider with an update. The nurse should anticipate an order for which medication? a. Hydralazine b. Magnesium sulfate bolus c. Diazepam d. Calcium gluconate

ANS: A Hydralazine is an antihypertensive medication commonly used to treat hypertension in severe preeclampsia. Typically, it is administered for a systolic BP higher than 160 mm Hg or a diastolic BP higher than 110 mm Hg. An additional bolus of magnesium sulfate may be ordered for increasing signs of CNS irritability related to severe preeclampsia (e.g., clonus) or if eclampsia develops. Diazepam is sometimes used to stop or shorten eclamptic seizures. Calcium gluconate is used as the antidote for magnesium sulfate toxicity. The client is not currently displaying any signs or symptoms of magnesium toxicity.

The exact cause of preterm labor is unknown but believed to be multifactorial. Infection is thought to be a major factor in many preterm labors. Which type of infection has not been linked to preterm birth? a. Viral b. Periodontal c. Cervical d. Urinary tract

ANS: A Infections that increase the risk of preterm labor and birth are bacterial and include cervical, urinary tract, periodontal, and other bacterial infections. Therefore, early, continual, and comprehensive participation by the client in her prenatal care is important. Recent evidence has shown a link between periodontal infections and preterm labor. Researchers recommend regular dental care before and during pregnancy, oral assessment as a routine part of prenatal care, and scrupulous oral hygiene to prevent periodontal infections.

Which condition is considered a medical emergency that requires immediate treatment? a. Inversion of the uterus b. Hypotonic uterus c. ITP d. Uterine atony

ANS: A Inversion of the uterus is likely to lead to hypovolemic shock and therefore is considered a medical emergency. Although hypotonic uterus, ITP, and uterine atony are serious conditions, they are not necessarily medical emergencies that require immediate treatment.

Which analysis of maternal serum may predict chromosomal abnormalities in the fetus? a. Multiple-marker screening b. L/S ratio c. BPP d. Blood type and crossmatch of maternal and fetal serum

ANS: A Maternal serum can be analyzed for abnormal levels of alpha-fetoprotein, human chorionic gonadotropin, and estriol. The multiple-marker screening may predict chromosomal defects in the fetus. The L/S ratio is used to determine fetal lung maturity. A BPP is used for evaluating fetal status during the antepartum period. Five variables are used, but none is concerned with chromosomal problems. The blood type and crossmatch would not predict chromosomal defects in the fetus.

Which neonatal complications are associated with hypertension in the mother? a. Intrauterine growth restriction (IUGR) and prematurity b. Seizures and cerebral hemorrhage c. Hepatic or renal dysfunction d. Placental abruption and DIC

ANS: A Neonatal complications are related to placental insufficiency and include IUGR, prematurity, and necrotizing enterocolitis. Seizures and cerebral hemorrhage are maternal complications. Hepatic and renal dysfunction are maternal complications of hypertensive disorders in pregnancy. Placental abruption and DIC are conditions related to maternal morbidity and mortality.

Screening at 24 weeks of gestation reveals that a pregnant woman has gestational diabetes mellitus (GDM). In planning her care, the nurse and the client mutually agree that an expected outcome is to prevent injury to the fetus as a result of GDM. This fetus is at the greatest risk for which condition? a. Macrosomia b. Congenital anomalies of the central nervous system c. Preterm birth d. Low birth weight

ANS: A Poor glycemic control later in pregnancy increases the rate of fetal macrosomia. Poor glycemic control during the preconception time frame and into the early weeks of the pregnancy is associated with congenital anomalies. Preterm labor or birth is more likely to occur with severe diabetes and is the greatest risk in women with pregestational diabetes. Increased weight, or macrosomia, is the greatest risk factor for this fetus.

In caring for the woman with DIC, which order should the nurse anticipate? a. Administration of blood b. Preparation of the client for invasive hemodynamic monitoring c. Restriction of intravascular fluids d. Administration of steroids

ANS: A Primary medical management in all cases of DIC involves a correction of the underlying cause, volume replacement, blood component therapy, optimization of oxygenation and perfusion status, and continued reassessment of laboratory parameters. Central monitoring would not be initially ordered in a client with DIC because it could contribute to more areas of bleeding. Management of DIC would include volume replacement, not volume restriction. Steroids are not indicated for the management of DIC.

With regard to hemorrhagic complications that may occur during pregnancy, what information is most accurate? a. An incompetent cervix is usually not diagnosed until the woman has lost one or two pregnancies. b. Incidences of ectopic pregnancy are declining as a result of improved diagnostic techniques. c. One ectopic pregnancy does not affect a woman's fertility or her likelihood of having a normal pregnancy the next time. d. Gestational trophoblastic neoplasia (GTN) is one of the persistently incurable gynecologic malignancies.

ANS: A Short labors and recurring losses of pregnancy at progressively earlier gestational ages are characteristics of reduced cervical competence. Because diagnostic technology is improving, more ectopic pregnancies are being diagnosed. One ectopic pregnancy places the woman at increased risk for another one. Ectopic pregnancy is a leading cause of infertility. Once invariably fatal, GTN now is the most curable gynecologic malignancy.

Which information is an important consideration when comparing the CST with the NST? a. The NST has no known contraindications. b. The CST has fewer false-positive results when compared with the NST. c. The CST is more sensitive in detecting fetal compromise, as opposed to the NST. d. The CST is slightly more expensive than the NST.

ANS: A The CST has several contraindications. The NST has a high rate of false-positive results and is less sensitive than the CST but relatively inexpensive.

Which major neonatal complication is carefully monitored after the birth of the infant of a diabetic mother? a. Hypoglycemia b. Hypercalcemia c. Hypobilirubinemia d. Hypoinsulinemia

ANS: A The neonate is at highest risk for hypoglycemia because fetal insulin production is accelerated during pregnancy to metabolize excessive glucose from the mother. At birth, the maternal glucose supply stops and the neonatal insulin exceeds the available glucose, thus leading to hypoglycemia. Hypocalcemia is associated with preterm birth, birth trauma, and asphyxia, all common problems of the infant of a diabetic mother. Excess erythrocytes are broken down after birth, and large amounts of bilirubin are released into the neonate's circulation, with resulting hyperbilirubinemia. Because fetal insulin production is accelerated during pregnancy, hyperinsulinemia develops in the neonate.

A pregnant woman's BPP score is 8. She asks the nurse to explain the results. How should the nurse respond at this time? a. "The test results are within normal limits." b. "Immediate delivery by cesarean birth is being considered." c. "Further testing will be performed to determine the meaning of this score." d. "An obstetric specialist will evaluate the results of this profile and, within the next week, will inform you of your options regarding delivery."

ANS: A The normal biophysical score ranges from 8 to 10 points if the amniotic fluid volume is adequate. A normal score allows conservative treatment of high-risk clients. Delivery can be delayed if fetal well-being is indicated. Scores less than 4 should be investigated, and delivery could be initiated sooner than planned. The results of the BPP are usually available immediately after the procedure is performed. Since this score is within normal range, no further testing is required at this time.

A pregnant woman's amniotic membranes have ruptured. A prolapsed umbilical cord is suspected. What intervention would be the nurse's highest priority? a. Placing the woman in the knee-chest position b. Covering the cord in sterile gauze soaked in saline c. Preparing the woman for a cesarean birth d. Starting oxygen by face mask

ANS: A The woman is assisted into a modified Sims position, Trendelenburg position, or the knee-chest position in which gravity keeps the pressure of the presenting part off the cord. Although covering the cord in sterile gauze soaked saline, preparing the woman for a cesarean, and starting oxygen by face mark are appropriate nursing interventions in the event of a prolapsed cord, the intervention of top priority would be positioning the mother to relieve cord compression.

A perinatal nurse is caring for a woman in the immediate postbirth period. Assessment reveals that the client is experiencing profuse bleeding. What is the most likely cause for this bleeding? a. Uterine atony b. Uterine inversion c. Vaginal hematoma d. Vaginal laceration

ANS: A Uterine atony is significant hypotonia of the uterus and is the leading cause of postpartum hemorrhage. Uterine inversion may lead to hemorrhage; however, it is not the most likely source of this client's bleeding. Further, if the woman were experiencing a uterine inversion, it would be evidenced by the presence of a large, red, rounded mass protruding from the introitus. A vaginal hematoma may be associated with hemorrhage. However, the most likely clinical finding for vaginal hematoma is pain, not the presence of profuse bleeding. A vaginal laceration should be suspected if vaginal bleeding continues in the presence of a firm, contracted uterine fundus.

Which assessments are included in the fetal BPP? (Select all that apply.) a. Fetal movement b. Fetal tone c. Fetal heart rate d. AFI e. Placental grade

ANS: A, B, C, D Fetal movement, tone, heart rate, and AFI are all assessed in a BPP. The placental grade is determined by ultrasound and is not included in the criteria of assessment factors for a BPP.

What are the complications and risks associated with cesarean births? (Select all that apply.) a. Pulmonary edema b. Wound dehiscence c. Hemorrhage d. Urinary tract infections e. Fetal injuries

ANS: A, B, C, D, E Pulmonary edema, wound dehiscence, hemorrhage, urinary tract infections, and fetal injuries are possible complications and risks associated with cesarean births.

Women who are obese are at risk for several complications during pregnancy and birth. Which of these would the nurse anticipate with an obese client? (Select all that apply.) a. Thromboembolism b. Cesarean birth c. Wound infection d. Breech presentation e. Hypertension

ANS: A, B, C, E A breech presentation is not a complication of pregnancy or birth for the client who is obese. Venous thromboembolism is a known risk for obese women. Therefore, the use of thromboembolism-deterrent (TED) hose and sequential compression devices may help decrease the chance for clot formation. Women should also be encouraged to ambulate as soon as possible. In addition to having an increased risk for complications with a cesarean birth, in general, obese women are also more likely to require an emergency cesarean birth. Many obese women have a pannus (i.e., large roll of abdominal fat) that overlies a lower transverse incision made just above the pubic area. The pannus causes the area to remain moist, which encourages the development of infection. Obese women are more likely to begin pregnancy with comorbidities such as hypertension and type 2 diabetes.

IUGR is associated with which pregnancy-related risk factors? (Select all that apply.) a. Poor nutrition b. Maternal collagen disease c. Gestational hypertension d. Premature rupture of membranes e. Smoking

ANS: A, B, C, E Poor nutrition, maternal collagen disease, gestational hypertension, and smoking are risk factors associated with the occurrence of IUGR. Premature rupture of membranes is associated with preterm labor, not IUGR.

Transvaginal ultrasonography is often performed during the first trimester. While preparing a 6-week gestational client for this procedure, she expresses concerns over the necessity for this test. The nurse should explain that this diagnostic test may be indicated for which situations? (Select all that apply.) a. Multifetal gestation b. Obesity c. Fetal abnormalities d. Amniotic fluid volume e. Ectopic pregnancy

ANS: A, B, C, E Transvaginal ultrasound is useful in women who are obese whose thick abdominal layers cannot be penetrated with traditional abdominal ultrasound. This procedure is also used to identify multifetal gestation, ectopic pregnancy, estimating gestational age, confirming fetal viability, and identifying fetal abnormalities. Amniotic fluid volume is assessed during the second and third trimester; conventional ultrasound would be used.

Which medications are used to manage PPH? (Select all that apply.) a. Oxytocin b. Methergine c. Terbutaline d. Hemabate e. Magnesium sulfate

ANS: A, B, D Oxytocin, Methergine, and Hemabate are medications used to manage PPH. Terbutaline and magnesium sulfate are tocolytic medications that are used to relax the uterus, which would cause or worsen PPH.

The reported incidence of ectopic pregnancy has steadily risen over the past 2 decades. Causes include the increase in sexually transmitted infections (STIs) accompanied by tubal infection and damage. The popularity of contraceptive devices such as the IUD has also increased the risk for ectopic pregnancy. The nurse suspects that a client has early signs of ectopic pregnancy. The nurse should be observing the client for which signs or symptoms? (Select all that apply.) a. Pelvic pain b. Abdominal pain c. Unanticipated heavy bleeding d. Vaginal spotting or light bleeding e. Missed period

ANS: A, B, D, E A missed period or spotting can be easily mistaken by the client as an early sign of pregnancy. More subtle signs depend on exactly where the implantation occurs. The nurse must be thorough in her assessment because pain is not a normal symptom of early pregnancy. As the fallopian tube tears open and the embryo is expelled, the client often exhibits severe pain accompanied by intraabdominal hemorrhage, which may progress to hypovolemic shock with minimal or even no external bleeding. In approximately one half of women, shoulder and neck pain results from irritation of the diaphragm from the hemorrhage.

Diabetes refers to a group of metabolic diseases characterized by hyperglycemia resulting from defects in insulin action, insulin secretion, or both. Over time, diabetes causes significant changes in the microvascular and macrovascular circulations. What do these complications include? (Select all that apply.) a. Atherosclerosis b. Retinopathy c. Intrauterine fetal death (IUFD) d. Nephropathy e. Neuropathy f. Autonomic neuropathy

ANS: A, B, D, E These structural changes will most likely affect a variety of systems, including the heart, eyes, kidneys, and nerves. IUFD (stillbirth) remains a major complication of diabetes in pregnancy; however, this is a fetal complication.

One of the most important components of the physical assessment of the pregnant client is the determination of BP. Consistency in measurement techniques must be maintained to ensure that the nuances in the variations of the BP readings are not the result of provider error. Which techniques are important in obtaining accurate BP readings? (Select all that apply.) a. The client should be seated. b. The client's arm should be placed at the level of the heart. c. An electronic BP device should be used. d. The cuff should cover a minimum of 60% of the upper arm. e. The same arm should be used for every reading.

ANS: A, B, E BP readings are easily affected by maternal position. Ideally, the client should be seated. An alternative position is left lateral recumbent with the arm at the level of the heart. The arm should always be held in a horizontal position at approximately the level of the heart. The same arm should be used at every visit. The manual sphygmomanometer is the most accurate device. If manual and electronic devices are used in the care setting, then the nurse must use caution when interpreting the readings. A proper size cuff should cover at least 80% of the upper arm or be approximately 1.5 times the length of the upper arm.

A serious but uncommon complication of undiagnosed or partially treated hyperthyroidism is a thyroid storm, which may occur in response to stress such as infection, birth, or surgery. What are the signs and symptoms of this emergency disorder? (Select all that apply.) a. Fever b. Hypothermia c. Restlessness d. Bradycardia e. Hypertension

ANS: A, C Fever, restlessness, tachycardia, vomiting, hypotension, and stupor are symptoms of a thyroid storm. Fever, not hypothermia; tachycardia, not bradycardia; and hypotension, not hypertension, are symptoms of thyroid storm.

Approximately 10% to 15% of all clinically recognized pregnancies end in miscarriage. What are possible causes of early miscarriage? (Select all that apply.) a. Chromosomal abnormalities b. Infections c. Endocrine imbalance d. Systemic disorders e. Varicella

ANS: A, C, D, E Infections are not a common cause of early miscarriage. At least 50% of pregnancy losses result from chromosomal abnormalities. Endocrine imbalances such as hypothyroidism or diabetes are also possible causes for early pregnancy loss. Other systemic disorders that may contribute to pregnancy loss include lupus and genetic conditions. Although infections are not a common cause of early miscarriage, varicella infection in the first trimester has been associated with pregnancy loss.

The induction of labor is considered an acceptable obstetric procedure if it is in the best interest to deliver the fetus. The charge nurse on the labor and delivery unit is often asked to schedule clients for this procedure and therefore must be cognizant of the specific conditions appropriate for labor induction. What are appropriate indications for induction? (Select all that apply?) a. Rupture of membranes at or near term b. Convenience of the woman or her physician c. Chorioamnionitis (inflammation of the amniotic sac) d. Postterm pregnancy e. Fetal death

ANS: A, C, D, E The conditions listed are all acceptable indications for induction. Other conditions include intrauterine growth restriction (IUGR), maternal-fetal blood incompatibility, hypertension, and placental abruption. Elective inductions for the convenience of the woman or her provider are not recommended; however, they have become commonplace. Factors such as rapid labors and living a long distance from a health care facility may be valid reasons in such a circumstance. Elective delivery should not occur before 39 weeks of completed gestation.

Which adverse prenatal outcomes are associated with the HELLP syndrome? (Select all that apply.) a. Placental abruption b. Placenta previa c. Renal failure d. Cirrhosis e. Maternal and fetal death

ANS: A, C, E The HELLP syndrome is associated with an increased risk for adverse perinatal outcomes, including placental abruption, acute renal failure, subcapsular hepatic hematoma, hepatic rupture, recurrent preeclampsia, preterm birth, and fetal and maternal death. The HELLP syndrome is associated with an increased risk for placental abruption, not placenta previa. It is also associated with an increased risk for hepatic hematoma, not cirrhosis.

What is a maternal indication for the use of vacuum-assisted birth? a. Wide pelvic outlet b. Maternal exhaustion c. History of rapid deliveries d. Failure to progress past station 0

ANS: B A mother who is exhausted may be unable to assist with the expulsion of the fetus. The client with a wide pelvic outlet will likely not require vacuum extraction. With a rapid delivery, vacuum extraction is not necessary. A station of 0 is too high for a vacuum-assisted birth.

The client being cared for has severe preeclampsia and is receiving a magnesium sulfate infusion. Which new finding would give the nurse cause for concern? a. Sleepy, sedated affect b. Respiratory rate of 10 breaths per minute c. DTRs of 2 d. Absent ankle clonus

ANS: B A respiratory rate of 10 breaths per minute indicates the client is experiencing respiratory depression from magnesium toxicity. Because magnesium sulfate is a CNS depressant, the client will most likely become sedated when the infusion is initiated. DTRs of 2 and absent ankle clonus are normal findings.

A number of methods can be used for inducing labor. Which cervical ripening method falls under the category of mechanical or physical? a. Prostaglandins are used to soften and thin the cervix. b. Labor can sometimes be induced with balloon catheters or laminaria tents. c. Oxytocin is less expensive and more effective than prostaglandins but creates greater health risks. d. Amniotomy can be used to make the cervix more favorable for labor.

ANS: B Balloon catheters or laminaria tents are mechanical means of ripening the cervix. Ripening the cervix, making it softer and thinner, increases the success rate of induced labor. Prostaglandin E1 is less expensive and more effective than oxytocin but carries a greater risk. Amniotomy is the artificial rupture of membranes, which is used to induce labor only when the cervix is already ripe.

The nurse who elects to work in the specialty of obstetric care must have the ability to distinguish between preterm birth, preterm labor, and low birth weight. Which statement regarding this terminology is correct? a. Terms preterm birth and low birth weight can be used interchangeably. b. Preterm labor is defined as cervical changes and uterine contractions occurring between 20 and 37 weeks of gestation. c. Low birth weight is a newborn who weighs below 3.7 pounds. d. Preterm birth rate in the United States continues to increase.

ANS: B Before 20 weeks of gestation, the fetus is not viable (miscarriage); after 37 weeks, the fetus can be considered term. Although these terms are used interchangeably, they have different meanings: preterm birth describes the length of gestation (before 37 weeks), regardless of the newborn's weight; low birth weight describes only the infant's weight at the time of birth (2500 g or less), whenever it occurs. Low birth weight is anything below 2500 g or approximately pounds. In 2011, the preterm birth rate in the United States was 11.7 %; it has dropped every year since 2008.

An 18-year-old client who has reached 16 weeks of gestation was recently diagnosed with pregestational diabetes. She attends her centering appointment accompanied by one of her girlfriends. This young woman appears more concerned about how her pregnancy will affect her social life than her recent diagnosis of diabetes. A number of nursing diagnoses are applicable to assist in planning adequate care. What is the most appropriate diagnosis at this time? a. Risk for injury, to the fetus related to birth trauma b. Deficient knowledge, related to diabetic pregnancy management c. Deficient knowledge, related to insulin administration d. Risk for injury, to the mother related to hypoglycemia or hyperglycemia

ANS: B Before a treatment plan is developed or goals for the outcome of care are outlined, this client must come to an understanding of diabetes and the potential effects on her pregnancy. She appears more concerned about changes to her social life than adopting a new self-care regimen. Risk for injury to the fetus related to either placental insufficiency or birth trauma may come later in the pregnancy. At this time, the client is having difficulty acknowledging the adjustments that she needs to make to her lifestyle to care for herself during pregnancy. The client may not yet be on insulin. Insulin requirements increase with gestation. The importance of glycemic control must be part of health teaching for this client. However, she has not yet acknowledged that changes to her lifestyle need to be made and may not participate in the plan of care until understanding takes place.

A perinatal nurse is giving discharge instructions to a woman, status postsuction, and curettage secondary to a hydatidiform mole. The woman asks why she must take oral contraceptives for the next 12 months. What is the bestresponse by the nurse? a. "If you get pregnant within 1 year, the chance of a successful pregnancy is very small. Therefore, if you desire a future pregnancy, it would be better for you to use the most reliable method of contraception available." b. "The major risk to you after a molar pregnancy is a type of cancer that can be diagnosed only by measuring the same hormone that your body produces during pregnancy. If you were to get pregnant, then it would make the diagnosis of this cancer more difficult." c. "If you can avoid a pregnancy for the next year, the chance of developing a second molar pregnancy is rare. Therefore, to improve your chance of a successful pregnancy, not getting pregnant at this time is best." d. "Oral contraceptives are the only form of birth control that will prevent a recurrence of a molar pregnancy."

ANS: B Beta-human chorionic gonadotropin (beta-hCG) hormone levels are drawn for 1 year to ensure that the mole is completely gone. The chance of developing choriocarcinoma after the development of a hydatidiform mole is increased. Therefore, the goal is to achieve a zero human chorionic gonadotropin (hCG) level. If the woman were to become pregnant, then it may obscure the presence of the potentially carcinogenic cells. Women should be instructed to use birth control for 1 year after treatment for a hydatidiform mole. The rationale for avoiding pregnancy for 1 year is to ensure that carcinogenic cells are not present. Any contraceptive method except an intrauterine device (IUD) is acceptable.

A client with maternal phenylketonuria (PKU) has come to the obstetrical clinic to begin prenatal care. Why would this preexisting condition result in the need for closer monitoring during pregnancy? a. PKU is a recognized cause of preterm labor. b. The fetus may develop neurologic problems. c. A pregnant woman is more likely to die without strict dietary control. d. Women with PKU are usually mentally handicapped and should not reproduce.

ANS: B Children born to women with untreated PKU are more likely to be born with mental retardation, microcephaly, congenital heart disease, and low birth weight. Maternal PKU has no effect on labor. Women without dietary control of PKU are more likely to miscarry or bear a child with congenital anomalies. Screening for undiagnosed maternal PKU at the first prenatal visit may be warranted, especially in individuals with a family history of the disorder, with low intelligence of an uncertain cause, or who have given birth to microcephalic infants.

Which laboratory marker is indicative of DIC? a. Bleeding time of 10 minutes b. Presence of fibrin split products c. Thrombocytopenia d. Hypofibrinogenemia

ANS: B Degradation of fibrin leads to the accumulation of multiple fibrin clots throughout the body's vasculature. Bleeding time in DIC is normal. Low platelets may occur but are not indicative of DIC because they may be the result from other coagulopathies. Hypofibrinogenemia occurs with DIC.

An MSAFP screening indicates an elevated level of alpha-fetoprotein. The test is repeated, and again the level is reported as higher than normal. What is the next step in the assessment sequence to determine the well-being of the fetus? a. PUBS b. Ultrasound for fetal anomalies c. BPP for fetal well-being d. Amniocentesis for genetic anomalies

ANS: B If MSAFP findings are abnormal, then follow-up procedures include genetic counseling for families with a history of NTD, repeated MSAFP screenings, an ultrasound examination, and possibly amniocentesis. Indications for the use of PUBS include prenatal diagnosis of inherited blood disorders, karyotyping of malformed fetuses, detection of fetal infection, determination of the acid-base status of fetuses with IUGR, and assessment and treatment of isoimmunization and thrombocytopenia in the fetus. A BPP is a method of assessing fetal well-being in the third trimester. Before an amniocentesis, the client would have an ultrasound for direct visualization of the fetus.

Which maternal condition always necessitates delivery by cesarean birth? a. Marginal placenta previa b. Complete placenta previa c. Ectopic pregnancy d. Eclampsia

ANS: B In complete placenta previa, the placenta completely covers the cervical os. A cesarean birth is the acceptable method of delivery. The risk of fetal death occurring is due to preterm birth. If the previa is marginal (i.e., 2 cm or greater away from the cervical os), then labor can be attempted. A cesarean birth is not indicated for an ectopic pregnancy. Labor can be safely induced if the eclampsia is under control.

The nurse who is caring for a woman hospitalized for hyperemesis gravidarum would expect the initial treatment to involve what? a. Corticosteroids to reduce inflammation b. Intravenous (IV) therapy to correct fluid and electrolyte imbalances c. Antiemetic medication, such as pyridoxine, to control nausea and vomiting d. Enteral nutrition to correct nutritional deficits

ANS: B Initially, the woman who is unable to down clear liquids by mouth requires IV therapy to correct fluid and electrolyte imbalances. Corticosteroids have been successfully used to treat refractory hyperemesis gravidarum, but they are not the expected initial treatment for this disorder. Pyridoxine is vitamin B6, not an antiemetic medication. Promethazine, a common antiemetic, may be prescribed. In severe cases of hyperemesis gravidarum, enteral nutrition via a feeding tube may be necessary to correct maternal nutritional deprivation but is not the initial treatment for this client.

Which client is at greatest risk for early PPH? a. Primiparous woman (G 2, P 1-0-0-1) being prepared for an emergency cesarean birth for fetal distress b. Woman with severe preeclampsia on magnesium sulfate whose labor is being induced c. Multiparous woman (G 3, P 2-0-0-2) with an 8-hour labor d. Primigravida in spontaneous labor with preterm twins

ANS: B Magnesium sulfate administration during labor poses a risk for PPH. Magnesium acts as a smooth muscle relaxant, thereby contributing to uterine relaxation and atony. A primiparous woman being prepared for an emergency cesarean birth for fetal distress, a multiparous woman with an 8-hour labor, and a primigravida in spontaneous labor with preterm twins do not indicate risk factors or causes of early PPH.

What is the correct terminology for an abortion in which the fetus dies but is retained within the uterus? a. Inevitable abortion b. Missed abortion c. Incomplete abortion d. Threatened abortion

ANS: B Missed abortion refers to the retention of a dead fetus in the uterus. An inevitable abortion means that the cervix is dilating with the contractions. An incomplete abortion means that not all of the products of conception were expelled. With a threatened abortion, the woman has cramping and bleeding but no cervical dilation.

A pregnant woman is being discharged from the hospital after the placement of a cervical cerclage because of a history of recurrent pregnancy loss, secondary to an incompetent cervix. Which information regarding postprocedural care should the nurse emphasize in the discharge teaching? a. Any vaginal discharge should be immediately reported to her health care provider. b. The presence of any contractions, rupture of membranes (ROM), or severe perineal pressure should be reported. c. The client will need to make arrangements for care at home, because her activity level will be restricted. d. The client will be scheduled for a cesarean birth.

ANS: B Nursing care should stress the importance of monitoring for the signs and symptoms of preterm labor. Vaginal bleeding needs to be reported to her primary health care provider. Bed rest is an element of care. However, the woman may stand for periods of up to 90 minutes, which allows her the freedom to see her physician. Home uterine activity monitoring may be used to limit the woman's need for visits and to monitor her status safely at home. The cerclage can be removed at 37 weeks of gestation (to prepare for a vaginal birth), or a cesarean birth can be planned.

A new mother with a thyroid disorder has come for a lactation follow-up appointment. Which thyroid disorder is a contraindication for breastfeeding? a. Hyperthyroidism b. PKU c. Hypothyroidism d. Thyroid storm

ANS: B PKU is a cause of mental retardation in infants; mothers with PKU pass on phenylalanine and therefore should elect not to breastfeed. A woman with either hyperthyroidism or hypothyroidism would have no particular reason not to breastfeed. A thyroid storm is a complication of hyperthyroidism and is not a contraindication to breastfeeding.

In contrast to placenta previa, what is the most prevalent clinical manifestation of abruptio placentae? a. Bleeding b. Intense abdominal pain c. Uterine activity d. Cramping

ANS: B Pain is absent with placenta previa and may be agonizing with abruptio placentae. Bleeding may be present in varying degrees for both placental conditions. Uterine activity and cramping may be present with both placental conditions.

The nurse is planning the care for a laboring client with diabetes mellitus. This client is at greater risk for which clinical finding? a. Oligohydramnios b. Polyhydramnios c. Postterm pregnancy d. Chromosomal abnormalities

ANS: B Polyhydramnios or amniotic fluid in excess of 2000 ml is 10 times more likely to occur in the client with diabetes mellitus rather than in nondiabetic pregnancies. This complication places the mother at risk for premature rupture of membranes, premature labor, and postpartum hemorrhage. Prolonged rupture of membranes, IUGR, intrauterine fetal death, and renal agenesis (Potter syndrome) place the client at risk for developing oligohydramnios. Anencephaly, placental insufficiency, and perinatal hypoxia contribute to the risk for postterm pregnancy. Maternal age older than 35 years and balanced translocation (maternal and paternal) are risk factors for chromosomal abnormalities.

Preconception counseling is critical in the safe management of diabetic pregnancies. Which complication is commonly associated with poor glycemic control before and during early pregnancy? a. Frequent episodes of maternal hypoglycemia b. Congenital anomalies in the fetus c. Hydramnios d. Hyperemesis gravidarum

ANS: B Preconception counseling is particularly important since strict metabolic control before conception and in the early weeks of gestation is instrumental in decreasing the risk of congenital anomalies. Frequent episodes of maternal hypoglycemia may occur during the first trimester (not before conception) as a result of hormonal changes and the effects on insulin production and use. Hydramnios occurs approximately 10 times more often in diabetic pregnancies than in nondiabetic pregnancies. Typically, it is observed in the third trimester of pregnancy. Hyperemesis gravidarum may exacerbate hypoglycemic events because the decreased food intake by the mother and glucose transfer to the fetus contribute to hypoglycemia

A 41-week pregnant multigravida arrives at the labor and delivery unit after a NST indicated that her fetus could be experiencing some difficulties in utero. Which diagnostic tool yields more detailed information about the condition of the fetus? a. Ultrasound for fetal anomalies b. Biophysical profile (BPP) c. MSAFP screening d. Percutaneous umbilical blood sampling (PUBS)

ANS: B Real-time ultrasound permits a detailed assessment of the physical and physiologic characteristics of the developing fetus and a cataloging of normal and abnormal biophysical responses to stimuli. The BPP is a noninvasive, dynamic assessment of a fetus that is based on acute and chronic markers of fetal disease. An ultrasound for fetal anomalies would most likely have occurred earlier in the pregnancy. It is too late in the pregnancy to perform an MSAFP. Furthermore, it does not provide information related to fetal well-being. Indications for PUBS include prenatal diagnosis or inherited blood disorders, karyotyping of malformed fetuses, detection of fetal infection, determination of the acid-base status of the fetus with IUGR, and assessment and treatment of isoimmunization and thrombocytopenia in the fetus.

Which nursing intervention is paramount when providing care to a client with preterm labor who has received terbutaline? a. Assess deep tendon reflexes (DTRs). b. Assess for dyspnea and crackles. c. Assess for bradycardia. d. Assess for hypoglycemia.

ANS: B Terbutaline is a beta2-adrenergic agonist that affects the mother's cardiopulmonary and metabolic systems. Signs of cardiopulmonary decompensation include adventitious breath sounds and dyspnea. An assessment for dyspnea and crackles is important for the nurse to perform if the woman is taking magnesium sulfate. Assessing DTRs does not address the possible respiratory side effects of using terbutaline. Since terbutaline is a beta2-adrenergic agonist, it can lead to hyperglycemia, not hypoglycemia. Beta2-adrenergic agonist drugs cause tachycardia, not bradycardia.

What is the primary nursing responsibility when caring for a client who is experiencing an obstetric hemorrhage associated with uterine atony? a. Establishing venous access b. Performing fundal massage c. Preparing the woman for surgical intervention d. Catheterizing the bladder

ANS: B The initial management of excessive postpartum bleeding is a firm massage of the uterine fundus. Although establishing venous access may be a necessary intervention, fundal massage is the initial intervention. The woman may need surgical intervention to treat her postpartum hemorrhage, but the initial nursing intervention is to assess the uterus. After uterine massage, the nurse may want to catheterize the client to eliminate any bladder distention that may be preventing the uterus from properly contracting.

A 26-year-old pregnant woman, gravida 2, para 1-0-0-1, is 28 weeks pregnant when she experiences bright red, painless vaginal bleeding. On her arrival at the hospital, which diagnostic procedure will the client most likely have performed? a. Amniocentesis for fetal lung maturity b. Transvaginal ultrasound for placental location c. Contraction stress test (CST) d. Internal fetal monitoring

ANS: B The presence of painless bleeding should always alert the health care team to the possibility of placenta previa, which can be confirmed through ultrasonography. Amniocentesis is not performed on a woman who is experiencing bleeding. In the event of an imminent delivery, the fetus is presumed to have immature lungs at this gestational age, and the mother is given corticosteroids to aid in fetal lung maturity. A CST is not performed at a preterm gestational age. Furthermore, bleeding is a contraindication to a CST. Internal fetal monitoring is also contraindicated in the presence of bleeding.

A 26-year-old primigravida has come to the clinic for her regular prenatal visit at 12 weeks. She appears thin and somewhat nervous. She reports that she eats a well-balanced diet, although her weight is 5 pounds less than it was at her last visit. The results of laboratory studies confirm that she has a hyperthyroid condition. Based on the available data, the nurse formulates a plan of care. Which nursing diagnosis is most appropriate for the client at this time? a. Deficient fluid volume b. Imbalanced nutrition: less than body requirements c. Imbalanced nutrition: more than body requirements d. Disturbed sleep pattern

ANS: B This client's clinical cues include weight loss, which supports a nursing diagnosis of "Imbalanced nutrition: less than body requirements." No clinical signs or symptoms support a nursing diagnosis of deficient fluid volume. This client reports weight loss, not weight gain. Although the client reports nervousness, the most appropriate nursing diagnosis, based on the client's other clinical symptoms, is "Imbalanced nutrition: less than body requirements."

A pregnant woman at 29 weeks of gestation has been diagnosed with preterm labor. Her labor is being controlled with tocolytic medications. She asks when she might be able to go home. Which response by the nurse is mostaccurate? a. "After the baby is born." b. "When we can stabilize your preterm labor and arrange home health visits." c. "Whenever your physician says that it is okay." d. "It depends on what kind of insurance coverage you have."

ANS: B This client's preterm labor is being controlled with tocolytics. Once she is stable, home care may be a viable option for this type of client. Care of a client with preterm labor is multidisciplinary and multifactorial; the goal is to prevent delivery. In many cases, this goal may be achieved at home. Managed care may dictate an earlier hospital discharge or a shift from hospital to home care. Insurance coverage may be one factor in client care, but ultimately, client safety remains the most important factor.

In terms of the incidence and classification of diabetes, which information should the nurse keep in mind when evaluating clients during their ongoing prenatal appointments? a. Type 1 diabetes is most common. b. Type 2 diabetes often goes undiagnosed. c. GDM means that the woman will receive insulin treatment until 6 weeks after birth. d. Type 1 diabetes may become type 2 during pregnancy.

ANS: B Type 2 diabetes often goes undiagnosed because hyperglycemia gradually develops and is often not severe. Type 2, sometimes called adult-onset diabetes, is the most common type of diabetes. GDM refers to any degree of glucose intolerance first recognized during pregnancy; insulin may or may not be needed. People do not go back and forth between type 1 and type 2 diabetes.

A laboring woman with no known risk factors suddenly experiences spontaneous ROM. The fluid consists of bright red blood. Her contractions are consistent with her current stage of labor. No change in uterine resting tone has occurred. The fetal heart rate (FHR) begins to decline rapidly after the ROM. The nurse should suspect the possibility of what condition? a. Placenta previa b. Vasa previa c. Severe abruptio placentae d. Disseminated intravascular coagulation (DIC)

ANS: B Vasa previa is the result of a velamentous insertion of the umbilical cord. The umbilical vessels are not surrounded by Wharton jelly and have no supportive tissue. The umbilical blood vessels thus are at risk for laceration at any time, but laceration occurs most frequently during ROM. The sudden appearance of bright red blood at the time of ROM and a sudden change in the FHR without other known risk factors should immediately alert the nurse to the possibility of vasa previa. The presence of placenta previa most likely would be ascertained before labor and is considered a risk factor for this pregnancy. In addition, if the woman had a placenta previa, it is unlikely that she would be allowed to pursue labor and a vaginal birth. With the presence of severe abruptio placentae, the uterine tonicity typically is tetanus (i.e., a boardlike uterus). DIC is a pathologic form of diffuse clotting that consumes large amounts of clotting factors, causing widespread external bleeding, internal bleeding, or both. DIC is always a secondary diagnosis, often associated with obstetric risk factors such as the hemolysis, elevated liver enzyme levels, and low platelet levels (HELLP) syndrome. This woman did not have any prior risk factors.

Which nursing intervention is necessary before a first-trimester transabdominal ultrasound? a. Place the woman on nothing by mouth (nil per os [NPO]) for 12 hours. b. Instruct the woman to drink 1 to 2 quarts of water. c. Administer an enema. d. Perform an abdominal preparation.

ANS: B When the uterus is still in the pelvis, visualization may be difficult. Performing a first-trimester transabdominal ultrasound requires the woman to have a full bladder, which will elevate the uterus upward and provide a better visualization of the fetus; therefore, being NPO is not appropriate. Neither an enema nor an abdominal preparation is necessary for this procedure.

Indications for a primary cesarean birth are often nonrecurring. Therefore, a woman who has had a cesarean birth with a low transverse scar may be a candidate for vaginal birth after cesarean (VBAC). Which clients would beless likely to have a successful VBAC? (Select all that apply.) a. Lengthy interpregnancy interval b. African-American race c. Delivery at a rural hospital d. Estimated fetal weight <4000 g e. Maternal obesity (BMI >30)

ANS: B, C, E Indications for a low success rate for a VBAC delivery include a short interpregnancy interval, non-Caucasian race, gestational age longer than 40 weeks, maternal obesity, preeclampsia, fetal weight greater than 4000 g, and delivery at a rural or private hospital.

The nurse recognizes that uterine hyperstimulation with oxytocin requires emergency interventions. What clinical cues alert the nurse that the woman is experiencing uterine hyperstimulation? (Select all that apply.) a. Uterine contractions lasting <90 seconds and occurring >2 minutes in frequency b. Uterine contractions lasting >90 seconds and occurring <2 minutes in frequency c. Uterine tone <20 mm Hg d. Uterine tone >20 mm Hg e. Increased uterine activity accompanied by a nonreassuring FHR and pattern

ANS: B, D, E Uterine contractions that occur less frequently than 2 minutes apart and last longer than 90 seconds, a uterine tone over 20 mm Hg, and a nonreassuring FHR and pattern are indications of uterine hyperstimulation with oxytocin administration. Uterine contractions that occur more frequently than 2 minutes apart and last less than 90 seconds are the expected goal of oxytocin induction. A uterine tone less than 20 mm Hg is normal.

In the past, factors to determine whether a woman was likely to develop a high-risk pregnancy were primarily evaluated from a medical point of view. A broader, more comprehensive approach to high-risk pregnancy has been adopted today. Four categories have now been established, based on the threats to the health of the woman and the outcome of pregnancy. Which category should not be included in this group? a. Biophysical b. Psychosocial c. Geographic d. Environmental

ANS: C A geographic category is correctly referred to as sociodemographic risk. These factors stem from the mother and her family. Ethnicity may be one of the risks to pregnancy; however, it is not the only factor in this category. Low income, lack of prenatal care, age, parity, and marital status also are included. Biophysical is one of the broad categories used for determining risk. These include genetic considerations, nutritional status, and medical and obstetric disorders. Psychosocial risks include smoking, caffeine, drugs, alcohol, and psychologic status. All of these adverse lifestyles can have a negative effect on the health of the mother or fetus. Environmental risks are risks that can affect both fertility and fetal development. These include infections, chemicals, radiation, pesticides, illicit drugs, and industrial pollutants.

Which preexisting factor is known to increase the risk of GDM? a. Underweight before pregnancy b. Maternal age younger than 25 years c. Previous birth of large infant d. Previous diagnosis of type 2 diabetes mellitus

ANS: C A previous birth of a large infant suggests GDM. Obesity (body mass index [BMI] of 30 or greater) creates a higher risk for gestational diabetes. A woman younger than 25 years is not generally at risk for GDM. The person with type 2 diabetes mellitus already has diabetes and thus will continue to have it after pregnancy. Insulin may be required during pregnancy because oral hypoglycemia drugs are contraindicated during pregnancy.

A woman arrives at the emergency department with complaints of bleeding and cramping. The initial nursing history is significant for a last menstrual period 6 weeks ago. On sterile speculum examination, the primary care provider finds that the cervix is closed. The anticipated plan of care for this woman would be based on a probable diagnosis of which type of spontaneous abortion? a. Incomplete b. Inevitable c. Threatened d. Septic

ANS: C A woman with a threatened abortion has spotting, mild cramps, and no cervical dilation. A woman with an incomplete abortion would have heavy bleeding, mild-to-severe cramping, and cervical dilation. An inevitable abortion demonstrates the same symptoms as an incomplete abortion: heavy bleeding, mild-to-severe cramping, and cervical dilation. A woman with a septic abortion has malodorous bleeding and typically a dilated cervix.

At 35 weeks of pregnancy, a woman experiences preterm labor. Although tocolytic medications are administered and she is placed on bed rest, she continues to experience regular uterine contractions and her cervix is beginning to dilate and efface. What is an important test for fetal well-being at this time? a. PUBS b. Ultrasound for fetal size c. Amniocentesis for fetal lung maturity d. NST

ANS: C Amniocentesis is performed to assess fetal lung maturity in the event of a preterm birth. The fluid is examined to determine the lecithin to sphingomyelin (L/S) ratio. Indications for PUBS include prenatal diagnosis or inherited blood disorders, karyotyping of malformed fetuses, detection of fetal infection, determination of the acid-base status of the fetus with IUGR, and assessment and treatment of isoimmunization and thrombocytopenia in the fetus. Determination of fetal size by ultrasound is typically performed during the second trimester and is not indicated in this scenario. An NST measures the fetal response to fetal movement in a noncontracting mother.

A woman with worsening preeclampsia is admitted to the hospital's labor and birth unit. The physician explains the plan of care for severe preeclampsia, including the induction of labor, to the woman and her husband. Which statement by the husband leads the nurse to believe that the couple needs further information? a. "I will help my wife use the breathing techniques that we learned in our childbirth classes." b. "I will give my wife ice chips to eat during labor." c. "Since we will be here for a while, I will call my mother so she can bring the two boys—2 years and 4 years of age—to visit their mother." d. "I will stay with my wife during her labor, just as we planned."

ANS: C Arranging a visit with their two children indicates that the husband does not understand the importance of the quiet, subdued environment that is needed to prevent his wife's condition from worsening. Implementing breathing techniques is indicative of adequate knowledge related to pain management during labor. Administering ice chips indicates an understanding of nutritional needs during labor. Staying with his wife during labor demonstrates the husband's support for his wife and is appropriate.

A client in the third trimester has just undergone an amniocentesis to determine fetal lung maturity. Which statement regarding this testing is important for the nurse in formulating a care plan? a. Because of new imaging techniques, an amniocentesis should have been performed in the first trimester. b. Despite the use of ultrasonography, complications still occur in the mother or infant in 5% to 10% of cases. c. Administration of Rho(D) immunoglobulin may be necessary. d. The presence of meconium in the amniotic fluid is always a cause for concern.

ANS: C As a result of the possibility of fetomaternal hemorrhage, administration of Rho(D) immunoglobulin is the standard of practice after amniocentesis for women who are Rh negative. Amniocentesis is possible after the 14th week of pregnancy when the uterus becomes an abdominal organ. Complications occur in less than 1% of cases; many have been minimized or eliminated through the use of ultrasonography. Meconium in the amniotic fluid before the beginning of labor is not usually a problem.

What is the primary purpose for the use of tocolytic therapy to suppress uterine activity? a. Drugs can be efficaciously administered up to the designated beginning of term at 37 weeks gestation. b. Tocolytic therapy has no important maternal (as opposed to fetal) contraindications. c. The most important function of tocolytic therapy is to provide the opportunity to administer antenatal glucocorticoids. d. If the client develops pulmonary edema while receiving tocolytic therapy, then intravenous (IV) fluids should be given.

ANS: C Buying time for antenatal glucocorticoids to accelerate fetal lung development may be the best reason to use tocolytic therapy. Once the pregnancy has reached 34 weeks, however, the risks of tocolytic therapy outweigh the benefits. Important maternal contraindications to tocolytic therapy exist. Tocolytic-induced edema can be caused by IV fluids.

While working with the pregnant client in her first trimester, what information does the nurse provide regarding when CVS can be performed (in weeks of gestation)? a. 4 b. 8 c. 10 d. 14

ANS: C CVS can be performed in the first or second trimester, ideally between 10 and 13 weeks of gestation. During this procedure, a small piece of tissue is removed from the fetal portion of the placenta. If performed after 9 completed weeks of gestation, then the risk of limb reduction is no greater than in the general population.

What condition indicates concealed hemorrhage when the client experiences abruptio placentae? a. Decrease in abdominal pain b. Bradycardia c. Hard, boardlike abdomen d. Decrease in fundal height

ANS: C Concealed hemorrhage occurs when the edges of the placenta do not separate. The formation of a hematoma behind the placenta and subsequent infiltration of the blood into the uterine muscle results in a very firm, boardlike abdomen. Abdominal pain may increase. The client will have shock symptoms that include tachycardia. As bleeding occurs, the fundal height increases.

A woman arrives for evaluation of signs and symptoms that include a missed period, adnexal fullness, tenderness, and dark red vaginal bleeding. On examination, the nurse notices an ecchymotic blueness around the woman's umbilicus. What does this finding indicate? a. Normal integumentary changes associated with pregnancy b. Turner sign associated with appendicitis c. Cullen sign associated with a ruptured ectopic pregnancy d. Chadwick sign associated with early pregnancy

ANS: C Cullen sign, the blue ecchymosis observed in the umbilical area, indicates hematoperitoneum associated with an undiagnosed ruptured intraabdominal ectopic pregnancy. Linea nigra on the abdomen is the normal integumentary change associated with pregnancy and exhibits a brown pigmented, vertical line on the lower abdomen. Turner sign is ecchymosis in the flank area, often associated with pancreatitis. A Chadwick sign is a blue-purple cervix that may be seen during or around the eighth week of pregnancy.

Which is the initial treatment for the client with vWD who experiences a PPH? a. Cryoprecipitate b. Factor VIII and von Willebrand factor (vWf) c. Desmopressin d. Hemabate

ANS: C Desmopressin is the primary treatment of choice for vWD and can be administered orally, nasally, and intravenously. This medication promotes the release of factor VIII and vWf from storage. Cryoprecipitate may be used; however, because of the risk of possible donor viruses, other modalities are considered safer. Treatment with plasma products such as factor VIII and vWf is an acceptable option for this client. Because of the repeated exposure to donor blood products and possible viruses, this modality is not the initial treatment of choice. Although the administration of the prostaglandin, Hemabate, is known to promote contraction of the uterus during PPH, it is not effective for the client who has a bleeding disorder.

During a prenatal visit, the nurse is explaining dietary management to a woman with pregestational diabetes. Which statement by the client reassures the nurse that teaching has been effective? a. "I will need to eat 600 more calories per day because I am pregnant." b. "I can continue with the same diet as before pregnancy as long as it is well balanced." c. "Diet and insulin needs change during pregnancy." d. "I will plan my diet based on the results of urine glucose testing."

ANS: C Diet and insulin needs change during the pregnancy in direct correlation to hormonal changes and energy needs. In the third trimester, insulin needs may double or even quadruple. The diet is individualized to allow for increased fetal and metabolic requirements, with consideration of such factors as prepregnancy weight and dietary habits, overall health, ethnic background, lifestyle, stage of pregnancy, knowledge of nutrition, and insulin therapy. Energy needs are usually calculated on the basis of 30 to 35 calories per kilogram of ideal body weight. Dietary management during a diabetic pregnancy must be based on blood, not urine, glucose changes.

The labor of a pregnant woman with preeclampsia is going to be induced. Before initiating the oxytocin (Pitocin) infusion, the nurse reviews the woman's latest laboratory test findings, which reveal a platelet count of 90,000 mm3, an elevated aspartate aminotransaminase (AST) level, and a falling hematocrit. The laboratory results are indicative of which condition? a. Eclampsia b. Disseminated intravascular coagulation (DIC) syndrome c. Hemolysis, elevated liver enzyme levels, and low platelet levels (HELLP) syndrome d. Idiopathic thrombocytopenia

ANS: C HELLP syndrome is a laboratory diagnosis for a variant of severe preeclampsia that involves hepatic dysfunction characterized by hemolysis (H), elevated liver (EL) enzymes, and low platelets (LP). Eclampsia is determined by the presence of seizures. DIC is a potential complication associated with HELLP syndrome. Idiopathic thrombocytopenia is the presence of low platelets of unknown cause and is not associated with preeclampsia.

To manage her diabetes appropriately and to ensure a good fetal outcome, how would the pregnant woman with diabetes alter her diet? a. Eat six small equal meals per day. b. Reduce the carbohydrates in her diet. c. Eat her meals and snacks on a fixed schedule. d. Increase her consumption of protein.

ANS: C Having a fixed meal schedule will provide the woman and the fetus with a steady blood sugar level, provide a good balance with insulin administration, and help prevent complications. Having a fixed meal schedule is more important than the equal division of food intake. Approximately 45% of the food eaten should be in the form of carbohydrates.

Which statement regarding the laboratory test for glycosylated hemoglobin Alc is correct? a. The laboratory test for glycosylated hemoglobin Alc is performed for all pregnant women, not only those with or likely to have diabetes. b. This laboratory test is a snapshot of glucose control at the moment. c. This laboratory test measures the levels of hemoglobin Alc, which should remain at less than 7%. d. This laboratory test is performed on the woman's urine, not her blood.

ANS: C Hemoglobin Alc levels greater than 7% indicate an elevated glucose level during the previous 4 to 6 weeks. This extra laboratory test is for diabetic women and defines glycemic control over the previous 4 to 6 weeks. Glycosylated hemoglobin level tests are performed on the blood.

What is the primary purpose for magnesium sulfate administration for clients with preeclampsia and eclampsia? a. To improve patellar reflexes and increase respiratory efficiency b. To shorten the duration of labor c. To prevent convulsions d. To prevent a boggy uterus and lessen lochial flow

ANS: C Magnesium sulfate is the drug of choice used to prevent convulsions, although it can generate other problems. Loss of patellar reflexes and respiratory depression are signs of magnesium toxicity. Magnesium sulfate can also increase the duration of labor. Women are at risk for a boggy uterus and heavy lochial flow as a result of magnesium sulfate therapy.

The nurse is preparing to administer methotrexate to the client. This hazardous drug is most often used for which obstetric complication? a. Complete hydatidiform mole b. Missed abortion c. Unruptured ectopic pregnancy d. Abruptio placentae

ANS: C Methotrexate is an effective nonsurgical treatment option for a hemodynamically stable woman whose ectopic pregnancy is unruptured and measures less than 4 cm in diameter. Methotrexate is not indicated or recommended as a treatment option for a complete hydatidiform mole, for a missed abortion, or for abruptio placentae.

Women with mild gestational hypertension and mild preeclampsia can be safely managed at home with frequent maternal and fetal evaluation. Complete or partial bed rest is still frequently ordered by some providers. Which complication is rarely the result of prolonged bed rest? a. Thrombophlebitis b. Psychologic stress c. Fluid retention d. Cardiovascular deconditioning

ANS: C No evidence has been found that supports the practice of bed rest to improve pregnancy outcome. Fluid retention is not an adverse outcome of prolonged bed rest. The woman is more likely to experience diuresis with accompanying fluid and electrolyte imbalance and weight loss. Prolonged bed rest is known to increase the risk for thrombophlebitis. Psychologic stress is known to begin on the first day of bed rest and continue for the duration of the therapy. Therefore, restricted activity, rather than complete bed rest, is recommended. Cardiovascular deconditioning is a known complication of bed rest.

Which information should nurses provide to expectant mothers when teaching them how to evaluate daily fetal movement counts (DFMCs)? a. Alcohol or cigarette smoke can irritate the fetus into greater activity. b. Kick counts should be taken every hour and averaged every 6 hours, with every other 6-hour stretch off. c. The fetal alarm signal should go off when fetal movements stop entirely for 12 hours. d. A count of less than four fetal movements in 1 hour warrants future evaluation.

ANS: C No movement in a 12-hour period is cause for investigation and possibly intervention. Alcohol and cigarette smoke temporarily reduce fetal movement. The mother should count fetal activity (kick counts) two or three times daily for 60 minutes each time. A count of less than 3 in 1 hour warrants further evaluation by a NST.

The management of the pregnant client who has experienced a pregnancy loss depends on the type of miscarriage and the signs and symptoms. While planning care for a client who desires outpatient management after a first-trimester loss, what would the nurse expect the plan to include? a. Dilation and curettage (D&C) b. Dilation and evacuation (D&E) c. Misoprostol d. Ergot products

ANS: C Outpatient management of a first-trimester loss is safely accomplished by the intravaginal use of misoprostol for up to 2 days. If the bleeding is uncontrollable, vital signs are unstable, or signs of infection are present, then a surgical evacuation should be performed. D&C is a surgical procedure that requires dilation of the cervix and scraping of the uterine walls to remove the contents of pregnancy. This procedure is commonly performed to treat inevitable or incomplete abortion and should be performed in a hospital. D&E is usually performed after 16 weeks of pregnancy. The cervix is widely dilated, followed by removal of the contents of the uterus. Ergot products such as Methergine or Hemabate may be administered for excessive bleeding after miscarriage.

A woman who has recently given birth complains of pain and tenderness in her leg. On physical examination, the nurse notices warmth and redness over an enlarged, hardened area. Which condition should the nurse suspect, and how will it be confirmed? a. Disseminated intravascular coagulation (DIC); asking for laboratory tests b. von Willebrand disease (vWD); noting whether bleeding times have been extended c. Thrombophlebitis; using real-time and color Doppler ultrasound d. Idiopathic or immune thrombocytopenic purpura (ITP); drawing blood for laboratory analysis

ANS: C Pain and tenderness in the extremities, which show warmth, redness, and hardness, is likely thrombophlebitis. A Doppler ultrasound examination is a common noninvasive way to confirm the diagnosis. A diagnosis of DIC is made according to clinical findings and laboratory markers. With DIC, a physical examination will reveal symptoms that may include unusual bleeding, petechiae around a blood pressure cuff on the woman's arm, and/or excessive bleeding from the site of a slight trauma such as a venipuncture site. Symptoms of vWD, a type of hemophilia, include recurrent bleeding episodes, prolonged bleeding time, and factor VIII deficiency. A risk for PPH exists with vWD but does not exhibit a warm or reddened area in an extremity. ITP is an autoimmune disorder in which the life span of antiplatelet antibodies is decreased. Increased bleeding time is a diagnostic finding, and the risk of postpartum uterine bleeding is increased.

A number of metabolic changes occur throughout pregnancy. Which physiologic adaptation of pregnancy will influence the nurse's plan of care? a. Insulin crosses the placenta to the fetus only in the first trimester, after which the fetus secretes its own. b. Women with insulin-dependent diabetes are prone to hyperglycemia during the first trimester because they are consuming more sugar. c. During the second and third trimesters, pregnancy exerts a diabetogenic effect that ensures an abundant supply of glucose for the fetus. d. Maternal insulin requirements steadily decline during pregnancy.

ANS: C Pregnant women develop increased insulin resistance during the second and third trimesters. Insulin never crosses the placenta; the fetus starts making its own around the 10th week. As a result of normal metabolic changes during pregnancy, insulin-dependent women are prone to hypoglycemia (low levels). Maternal insulin requirements may double or quadruple by the end of pregnancy.

Prostaglandin gel has been ordered for a pregnant woman at 43 weeks of gestation. What is the primary purpose of prostaglandin administration? a. To enhance uteroplacental perfusion in an aging placenta b. To increase amniotic fluid volume c. To ripen the cervix in preparation for labor induction d. To stimulate the amniotic membranes to rupture

ANS: C Preparations of prostaglandin E1 and E2 are effective when used before labor induction to ripen (i.e., soften and thin) the cervix. Uteroplacental perfusion is not altered by the use of prostaglandins. The insertion of prostaglandin gel has no effect on the level of amniotic fluid. In some cases, women will spontaneously begin laboring after the administration of prostaglandins, thereby eliminating the need for oxytocin. It is not common for a woman's membranes to rupture as a result of prostaglandin use.

In planning for home care of a woman with preterm labor, which concern should the nurse need to address? a. Nursing assessments are different from those performed in the hospital setting. b. Restricted activity and medications are necessary to prevent a recurrence of preterm labor. c. Prolonged bed rest may cause negative physiologic effects. d. Home health care providers are necessary.

ANS: C Prolonged bed rest may cause adverse effects such as weight loss, loss of appetite, muscle wasting, weakness, bone demineralization, decreased cardiac output, risk for thrombophlebitis, alteration in bowel functions, sleep disturbance, and prolonged postpartum recovery. Nursing assessments differ somewhat from those performed in the acute care setting, but this concern does not need to be addressed. Restricted activity and medications may prevent preterm labor but not in all women. In addition, the plan of care is individualized to meet the needs of each client. Many women receive home health nurse visits, but care is individualized for each woman.

A primigravida is being monitored at the prenatal clinic for preeclampsia. Which finding is of greatest concern to the nurse? a. Blood pressure (BP) increase to 138/86 mm Hg b. Weight gain of 0.5 kg during the past 2 weeks c. Dipstick value of 3+ for protein in her urine d. Pitting pedal edema at the end of the day

ANS: C Proteinuria is defined as a concentration of 1+ or greater via dipstick measurement. A dipstick value of 3+ alerts the nurse that additional testing or assessment should be performed. A 24-hour urine collection is preferred over dipstick testing attributable to accuracy. Generally, hypertension is defined as a BP of 140/90 mm Hg or an increase in systolic pressure of 30 mm Hg or diastolic pressure of 15 mm Hg. Preeclampsia may be demonstrated as a rapid weight gain of more than 2 kg in 1 week. Edema occurs in many normal pregnancies, as well as in women with preeclampsia. Therefore, the presence of edema is no longer considered diagnostic of preeclampsia.

A pregnant woman has been receiving a magnesium sulfate infusion for treatment of severe preeclampsia for 24 hours. On assessment, the nurse finds the following vital signs: temperature 37.3° C, pulse rate 88 beats per minute, respiratory rate 10 breaths per minute, BP 148/90 mm Hg, absent deep tendon reflexes (DTRs), and no ankle clonus. The client complains, "I'm so thirsty and warm." What is the nurse's immediate action? a. To call for an immediate magnesium sulfate level b. To administer oxygen c. To discontinue the magnesium sulfate infusion d. To prepare to administer hydralazine

ANS: C Regardless of the magnesium level, the client is displaying the clinical signs and symptoms of magnesium toxicity. The first action by the nurse should be to discontinue the infusion of magnesium sulfate. In addition, calcium gluconate, the antidote for magnesium, may be administered. Hydralazine is an antihypertensive drug commonly used to treat hypertension in severe preeclampsia. Typically, hydralazine is administered for a systolic BP higher than 160 mm Hg or a diastolic BP higher than 110 mm Hg.

The nurse is teaching a client with preterm premature rupture of membranes (PPROM) regarding self-care activities. Which activities should the nurse include in her teaching? a. Report a temperature higher than 40° C. b. Tampons are safe to use to absorb the leaking amniotic fluid. c. Do not engage in sexual activity. d. Taking frequent tub baths is safe.

ANS: C Sexual activity should be avoided because it may induce preterm labor. A temperature higher than 38° C should be reported. To prevent the risk of infection, tub baths should be avoided and nothing should be inserted into the vagina. Further, foul-smelling vaginal fluid, which may be a sign of infection, should be reported.

A woman who is 30 weeks of gestation arrives at the hospital with bleeding. Which differential diagnosis would not be applicable for this client? a. Placenta previa b. Abruptio placentae c. Spontaneous abortion d. Cord insertion

ANS: C Spontaneous abortion is another name for miscarriage; it occurs, by definition, early in pregnancy. Placenta previa is a well-known reason for bleeding late in pregnancy. The premature separation of the placenta (abruptio placentae) is a bleeding disorder that can occur late in pregnancy. Cord insertion may cause a bleeding disorder that can also occur late in pregnancy.

The most effective and least expensive treatment of puerperal infection is prevention. What is the most important strategy for the nurse to adopt? a. Large doses of vitamin C during pregnancy b. Prophylactic antibiotics c. Strict aseptic technique, including hand washing, by all health care personnel d. Limited protein and fat intake

ANS: C Strict adherence by all health care personnel to aseptic techniques during childbirth and the postpartum period is extremely important and the least expensive measure to prevent infection. Good nutrition to control anemia is a preventive measure. Increased iron intake assists in preventing anemia. Antibiotics may be administered to manage infections; they are not a cost-effective measure to prevent postpartum infection. Limiting protein and fat intake does not help prevent anemia or prevent infection.

Which statement most accurately describes the HELLP syndrome? a. Mild form of preeclampsia b. Diagnosed by a nurse alert to its symptoms c. Characterized by hemolysis, elevated liver enzymes, and low platelets d. Associated with preterm labor but not perinatal mortality

ANS: C The acronym HELLP stands for hemolysis (H), elevated liver (EL) enzymes, and low platelets (LP). The HELLP syndrome is a variant of severe preeclampsia and is difficult to identify because the symptoms are not often obvious. The HELLP syndrome must be diagnosed in the laboratory. Preterm labor is greatly increased; therefore, so is perinatal mortality.

A 30-year-old gravida 3, para 2-0-0-2 is at 18 weeks of gestation. Which screening test should the nurse recommend be ordered for this client? a. BPP b. Chorionic villi sampling c. MSAFP screening d. Screening for diabetes mellitus

ANS: C The biochemical assessment MSAFP test is performed from week 15 to week 20 of gestation (weeks 16 to 18 are ideal). A BPP is a method of biophysical assessment of fetal well-being in the third trimester. Chorionic villi sampling is a biochemical assessment of the fetus that should be performed from the 10th to 12th weeks of gestation. Screening for diabetes mellitus begins with the first prenatal visit.

A primigravida at 40 weeks of gestation is having uterine contractions every to 2 minutes and states that they are very painful. Her cervix is dilated 2 cm and has not changed in 3 hours. The woman is crying and wants an epidural. What is the likely status of this woman's labor? a. She is exhibiting hypotonic uterine dysfunction. b. She is experiencing a normal latent stage. c. She is exhibiting hypertonic uterine dysfunction. d. She is experiencing precipitous labor.

ANS: C The contraction pattern observed in this woman signifies hypertonic uterine activity. Typically, uterine activity in this phase occurs at 4- to 5-minute intervals lasting 30 to 45 seconds. Women who experience hypertonic uterine dysfunction, or primary dysfunctional labor, are often anxious first-time mothers who are having painful and frequent contractions that are ineffective at causing cervical dilation or effacement to progress. With hypotonic uterine dysfunction, the woman initially makes normal progress into the active stage of labor; then the contractions become weak and inefficient or stop altogether. Precipitous labor is one that lasts less than 3 hours from the onset of contractions until time of birth.

Which finding on a prenatal visit at 10 weeks of gestation might suggest a hydatidiform mole? a. Complaint of frequent mild nausea b. Blood pressure of 120/80 mm Hg c. Fundal height measurement of 18 cm d. History of bright red spotting for 1 day, weeks ago

ANS: C The uterus in a hydatidiform molar pregnancy is often larger than would be expected on the basis of the duration of the pregnancy. Nausea increases in a molar pregnancy because of the increased production of hCG. A woman with a molar pregnancy may have early-onset pregnancy-induced hypertension. In the client's history, bleeding is normally described as brownish.

The American College of Obstetricians and Gynecologists (ACOG) has developed a comprehensive list of risk factors associated with the development of preeclampsia. Which client exhibits the greatest number of these risk factors? a. 30-year-old obese Caucasian with her third pregnancy b. 41-year-old Caucasian primigravida c. 19-year-old African American who is pregnant with twins d. 25-year-old Asian American whose pregnancy is the result of donor insemination

ANS: C Three risk factors are present in the 19-year-old African-American client. She has African-American ethnicity, is at the young end of the age distribution, and has a multiple pregnancy. In planning care for this client, the nurse must frequently monitor her BP and teach her to recognize the early warning signs of preeclampsia. The 30-year-old obese Caucasian client has only has one known risk factor: obesity. Age distribution appears to be U-shaped, with women younger than 20 years of age and women older than 40 years of age being at greatest risk. Preeclampsia continues to be more frequently observed in primigravidas; this client is a multigravida woman. Two risk factors are present for the 41-year-old Caucasian primigravida client. Her age and status as a primigravida place her at increased risk for preeclampsia. Caucasian women are at a lower risk than are African-American women. The 25-year-old Asian-American client exhibits only one risk factor. Pregnancies that result from donor insemination, oocyte donation, and embryo donation are at an increased risk of developing preeclampsia.

A woman is having her first child. She has been in labor for 15 hours. A vaginal examination performed 2 hours earlier revealed the cervix to be dilated to 5 cm and 100% effaced, and the presenting part of the fetus was at station 0; however, another vaginal examination performed 5 minutes ago indicated no changes. What abnormal labor pattern is associated with this description? a. Prolonged latent phase b. Protracted active phase c. Secondary arrest d. Protracted descent

ANS: C With a secondary arrest of the active phase, the progress of labor has stopped. This client has not had any anticipated cervical change, indicating an arrest of labor. In the nulliparous woman, a prolonged latent phase typically lasts longer than 20 hours. A protracted active phase, the first or second stage of labor, is prolonged (slow dilation). With a protracted descent, the fetus fails to descend at an anticipated rate during the deceleration phase and second stage of labor.

Hypothyroidism occurs in 2 to 3 pregnancies per 1000. Because severe hypothyroidism is associated with infertility and miscarriage, it is not often seen in pregnancy. Regardless of this fact, the nurse should be aware of the characteristic symptoms of hypothyroidism. Which do they include? (Select all that apply.) a. Hot flashes b. Weight loss c. Lethargy d. Decrease in exercise capacity e. Cold intolerance

ANS: C, D, E Symptoms include weight gain, lethargy, decrease in exercise capacity, and intolerance to cold. Other presentations might include constipation, hoarseness, hair loss, and dry skin. Thyroid supplements are used to treat hyperthyroidism in pregnancy.

A woman with gestational diabetes has had little or no experience reading and interpreting glucose levels. The client shows the nurse her readings for the past few days. Which reading signals the nurse that the client may require an adjustment of insulin or carbohydrates? a. 75 mg/dl before lunch. This is low; better eat now. b. 115 mg/dl 1 hour after lunch. This is a little high; maybe eat a little less next time. c. 115 mg/dl 2 hours after lunch. This is too high; it is time for insulin. d. 50 mg/dl just after waking up from a nap. This is too low; maybe eat a snack before going to sleep.

ANS: D 50 mg/dl after waking from a nap is too low. During hours of sleep, glucose levels should not be less than 60 mg/dl. Snacks before sleeping can be helpful. The premeal acceptable range is 60 to 99 mg/dl. The readings 1 hour after a meal should be less than 129 mg/dl. Two hours after eating, the readings should be less than 120 mg/dl.

The nurse who elects to practice in the area of women's health must have a thorough understanding of miscarriage. Which statement regarding this condition is most accurate? a. A miscarriage is a natural pregnancy loss before labor begins. b. It occurs in fewer than 5% of all clinically recognized pregnancies. c. Careless maternal behavior, such as poor nutrition or excessive exercise, can be a factor in causing a miscarriage. d. If a miscarriage occurs before the 12th week of pregnancy, then it may be observed only as moderate discomfort and blood loss.

ANS: D Before the sixth week, the only evidence might be a heavy menstrual flow. After the 12th week, more severe pain, similar to that of labor, is likely. Miscarriage is a natural pregnancy loss, but it occurs, by definition, before 20 weeks of gestation, before the fetus is viable. Miscarriages occur in approximately 10% to 15% of all clinically recognized pregnancies. Miscarriages can be caused by a number of disorders or illnesses outside the mother's control or knowledge.

A woman at 26 weeks of gestation is being assessed to determine whether she is experiencing preterm labor. Which finding indicates that preterm labor is occurring? a. Estriol is not found in maternal saliva. b. Irregular, mild uterine contractions are occurring every 12 to 15 minutes. c. Fetal fibronectin is present in vaginal secretions. d. The cervix is effacing and dilated to 2 cm.

ANS: D Cervical changes such as shortened endocervical length, effacement, and dilation are predictors of imminent preterm labor. Changes in the cervix accompanied by regular contractions indicate labor at any gestation. Estriol is a form of estrogen produced by the fetus that is present in plasma at 9 weeks of gestation. Levels of salivary estriol have been shown to increase before preterm birth. Irregular, mild contractions that do not cause cervical change are not considered a threat. The presence of fetal fibronectin in vaginal secretions between 24 and 36 weeks of gestation could predict preterm labor, but it has only a 20% to 40% positive predictive value. Of more importance are other physiologic clues of preterm labor such as cervical changes.

If nonsurgical treatment for late PPH is ineffective, which surgical procedure would be appropriate to correct the cause of this condition? a. Hysterectomy b. Laparoscopy c. Laparotomy d. Dilation and curettage (D&C)

ANS: D D&C allows the examination of the uterine contents and the removal of any retained placental fragments or blood clots. Hysterectomy is the removal of the uterus and is not the appropriate treatment for late PPH. A laparoscopy is the insertion of an endoscope through the abdominal wall to examine the peritoneal cavity, but it, too, is not the appropriate treatment for this condition. A laparotomy is the surgical incision into the peritoneal cavity to explore it but is also not the appropriate treatment for late PPH.

The nurse has evaluated a client with preeclampsia by assessing DTRs. The result is a grade of 3+. Which DTR response most accurately describes this score? a. Sluggish or diminished b. Brisk, hyperactive, with intermittent or transient clonus c. Active or expected response d. More brisk than expected, slightly hyperactive

ANS: D DTRs reflect the balance between the cerebral cortex and the spinal cord. They are evaluated at baseline and to detect changes. A slightly hyperactive and brisk response indicates a grade 3+ response.

Nurses need to understand the basic definitions and incidence data regarding PPH. Which statement regarding this condition is most accurate? a. PPH is easy to recognize early; after all, the woman is bleeding. b. Traditionally, it takes more than 1000 ml of blood after vaginal birth and 2500 ml after cesarean birth to define the condition as PPH. c. If anything, nurses and physicians tend to overestimate the amount of blood loss. d. Traditionally, PPH has been classified as early PPH or late PPH with respect to birth.

ANS: D Early PPH is also known as primary, or acute, PPH; late PPH is known as secondary PPH. Unfortunately, PPH can occur with little warning and is often recognized only after the mother has profound symptoms. Traditionally, a 500-ml blood loss after a vaginal birth and a 1000-ml blood loss after a cesarean birth constitute PPH. Medical personnel tend to underestimate blood loss by as much as 50% in their subjective observations.

The nurse is performing an assessment on a client who thinks she may be experiencing preterm labor. Which information is the most important for the nurse to understand and share with the client? a. Because all women must be considered at risk for preterm labor and prediction is so variable, teaching pregnant women the symptoms of preterm labor probably causes more harm through false alarms. b. Braxton Hicks contractions often signal the onset of preterm labor. c. Because preterm labor is likely to be the start of an extended labor, a woman with symptoms can wait several hours before contacting the primary caregiver. d. Diagnosis of preterm labor is based on gestational age, uterine activity, and progressive cervical change.

ANS: D Gestational age of 20 to 37 weeks, uterine contractions, and a cervix that is 80% effaced or dilated 2 cm indicates preterm labor. It is essential that nurses teach women how to detect the early symptoms of preterm labor. Braxton Hicks contractions resemble preterm labor contractions, but they are not true labor. Waiting too long to see a health care provider could result in essential medications failing to be administered. Preterm labor is not necessarily long-term labor.

The nurse suspects that her postpartum client is experiencing hemorrhagic shock. Which observation indicates or would confirm this diagnosis? a. Absence of cyanosis in the buccal mucosa b. Cool, dry skin c. Calm mental status d. Urinary output of at least 30 ml/hr

ANS: D Hemorrhage may result in hemorrhagic shock. Shock is an emergency situation during which the perfusion of body organs may become severely compromised, and death may occur. The presence of adequate urinary output indicates adequate tissue perfusion. The assessment of the buccal mucosa for cyanosis can be subjective. The presence of cool, pale, clammy skin is associated with hemorrhagic shock. Hemorrhagic shock is associated with lethargy, not restlessness.

The client has been on magnesium sulfate for 20 hours for the treatment of preeclampsia. She just delivered a viable infant girl 30 minutes ago. What uterine findings does the nurse expect to observe or assess in this client? a. Absence of uterine bleeding in the postpartum period b. Fundus firm below the level of the umbilicus c. Scant lochia flow d. Boggy uterus with heavy lochia flow

ANS: D High serum levels of magnesium can cause a relaxation of smooth muscle such as the uterus. Because of this tocolytic effect, the client will most likely have a boggy uterus with increased amounts of bleeding. All women experience uterine bleeding in the postpartum period, especially those who have received magnesium therapy. Rather than scant lochial flow, however, this client will most likely have a heavy flow attributable to the relaxation of the uterine wall caused by magnesium administration.

A woman with preeclampsia has a seizure. What is the nurse's highest priority during a seizure? a. To insert an oral airway b. To suction the mouth to prevent aspiration c. To administer oxygen by mask d. To stay with the client and call for help

ANS: D If a client becomes eclamptic, then the nurse should stay with the client and call for help. Nursing actions during a convulsion are directed toward ensuring a patent airway and client safety. Insertion of an oral airway during seizure activity is no longer the standard of care. The nurse should attempt to keep the airway patent by turning the client's head to the side to prevent aspiration. Once the seizure has ended, it may be necessary to suction the client's mouth. Oxygen is administered after the convulsion has ended.

Which statement related to the induction of labor is most accurate? a. Can be achieved by external and internal version techniques b. Is also known as a trial of labor (TOL) c. Is almost always performed for medical reasons d. Is rated for viability by a Bishop score

ANS: D Induction of labor is likely to be more successful with a Bishop score of 9 or higher for first-time mothers or 5 or higher for veterans. Version is the turning of the fetus to a better position by a physician for an easier or safer birth. A TOL is the observance of a woman and her fetus for several hours of active labor to assess the safety of vaginal birth. Two thirds of cases of induced labor are elective and not done for medical reasons.

When would an internal version be indicated to manipulate the fetus into a vertex position? a. Fetus from a breech to a cephalic presentation before labor begins b. Fetus from a transverse lie to a longitudinal lie before a cesarean birth c. Second twin from an oblique lie to a transverse lie before labor begins d. Second twin from a transverse lie to a breech presentation during a vaginal birth

ANS: D Internal version is used only during a vaginal birth to manipulate the second twin into a presentation that allows it to be vaginally born. For internal version to occur, the cervix needs to be completely dilated.

Nurses should be aware of the strengths and limitations of various biochemical assessments during pregnancy. Which statement regarding monitoring techniques is the most accurate? a. Chorionic villus sampling (CVS) is becoming more popular because it provides early diagnosis. b. MSAFP screening is recommended only for women at risk for NTDs. c. PUBS is one of the triple-marker tests for Down syndrome. d. MSAFP is a screening tool only; it identifies candidates for more definitive diagnostic procedures.

ANS: D MSAFP is a screening tool, not a diagnostic tool. CVS provides a rapid result, but it is declining in popularity because of advances in noninvasive screening techniques. An MSAFP screening is recommended for all pregnant women. MSAFP screening, not PUBS, is part of the triple-marker tests for Down syndrome.

Which classification of placental separation is not recognized as an abnormal adherence pattern? a. Placenta accreta b. Placenta increta c. Placenta percreta d. Placenta abruptio

ANS: D Placenta abruptio is premature separation of the placenta as opposed to partial or complete adherence. This classification occurs between the 20th week of gestation and delivery in the area of the decidua basalis. Symptoms include localized pain and bleeding. Placenta accreta is a recognized degree of attachment. With placenta accreta, the trophoblast slightly penetrates into the myometrium. Placenta increta is a recognized degree of attachment that results in deep penetration of the myometrium. Placenta percreta is the most severe degree of placental penetration that results in deep penetration of the myometrium. Bleeding with complete placental attachment occurs only when separation of the placenta is attempted after delivery. Treatment includes blood component therapy and, in extreme cases, hysterectomy may be necessary.

Which physiologic alteration of pregnancy most significantly affects glucose metabolism? a. Pancreatic function in the islets of Langerhans is affected by pregnancy. b. Pregnant women use glucose at a more rapid rate than nonpregnant women. c. Pregnant women significantly increase their dietary intake. d. Placental hormones are antagonistic to insulin, thus resulting in insulin resistance.

ANS: D Placental hormones, estrogen, progesterone, and human placental lactogen (HPL) create insulin resistance. Insulin is also broken down more quickly by the enzyme placental insulinase. Pancreatic functioning is not affected by pregnancy. The glucose requirements differ because of the growing fetus. The pregnant woman should increase her intake by 200 calories a day.

What is one of the initial signs and symptoms of puerperal infection in the postpartum client? a. Fatigue continuing for longer than 1 week b. Pain with voiding c. Profuse vaginal lochia with ambulation d. Temperature of 38° C (100.4° F) or higher on 2 successive days

ANS: D Postpartum or puerperal infection is any clinical infection of the genital canal that occurs within 28 days after miscarriage, induced abortion, or childbirth. The definition used in the United States continues to be the presence of a fever of 38° C (100.4° F) or higher on 2 successive days of the first 10 postpartum days, starting 24 hours after birth. Fatigue is a late finding associated with infection. Pain with voiding may indicate a urinary tract infection (UTI), but it is not typically one of the earlier symptoms of infection. Profuse lochia may be associated with endometritis, but it is not the first symptom associated with infection.

Which assessment is least likely to be associated with a breech presentation? a. Meconium-stained amniotic fluid b. Fetal heart tones heard at or above the maternal umbilicus c. Preterm labor and birth d. Postterm gestation

ANS: D Postterm gestation is not likely to occur with a breech presentation. The presence of meconium in a breech presentation may be a result of pressure on the fetal wall as it traverses the birth canal. Fetal heart tones heard at the level of the umbilical level of the mother are a typical finding in a breech presentation because the fetal back would be located in the upper abdominal area. Breech presentations often occur in preterm births.

A 32-year-old primigravida is admitted with a diagnosis of ectopic pregnancy. Which information assists the nurse in developing the plan of care? a. Bed rest and analgesics are the recommended treatment. b. She will be unable to conceive in the future. c. A D&C will be performed to remove the products of conception. d. Hemorrhage is the primary concern.

ANS: D Severe bleeding occurs if the fallopian tube ruptures. The recommended treatment is to remove the pregnancy before rupture to prevent hemorrhaging. If the tube must be removed, then the woman's fertility will decrease; however, she will not be infertile. A D&C is performed on the inside of the uterine cavity. The ectopic pregnancy is located within the tubes.

The obstetric provider has informed the nurse that she will be performing an amniotomy on the client to induce labor. What is the nurse's highest priority intervention after the amniotomy is performed? a. Applying clean linens under the woman b. Taking the client's vital signs c. Performing a vaginal examination d. Assessing the fetal heart rate (FHR)

ANS: D The FHR is assessed before and immediately after the amniotomy to detect any changes that might indicate cord compression or prolapse. Providing comfort measures, such as clean linens, for the client is important but not the priority immediately after an amniotomy. The woman's temperature should be checked every 2 hours after the rupture of membranes but not the priority immediately after an amniotomy. The woman would have had a vaginal examination during the procedure. Unless cord prolapse is suspected, another vaginal examination is not warranted. Additionally, FHR assessment provides clinical cues to a prolapsed cord.

How does the nurse document a NST during which two or more FHR accelerations of 15 beats per minute or more occur with fetal movement in a 20-minute period? a. Nonreactive b. Positive c. Negative d. Reactive

ANS: D The NST is reactive (normal) when two or more FHR accelerations of at least 15 beats per minute (each with a duration of at least 15 seconds) occur in a 20-minute period. A nonreactive result means that the heart rate did not accelerate during fetal movement. A positive result is not used with NST. CST uses positive as a result term. A negative result is not used with NST. CST uses negative as a result term.

In evaluating the effectiveness of magnesium sulfate for the treatment of preterm labor, which finding alerts the nurse to possible side effects? a. Urine output of 160 ml in 4 hours b. DTRs 2+ and no clonus c. Respiratory rate (RR) of 16 breaths per minute d. Serum magnesium level of 10 mg/dl

ANS: D The therapeutic range for magnesium sulfate management is 4 to 7.5 mg/dl. A serum magnesium level of 10 mg/dl could lead to signs and symptoms of magnesium toxicity, including oliguria and respiratory distress. Urine output of 160 ml in 4 hours, DTRs of 2+, and a RR of 16 breaths per minute are all normal findings.

In comparing the abdominal and transvaginal methods of ultrasound examination, which information should the nurse provide to the client? a. Both require the woman to have a full bladder. b. The abdominal examination is more useful in the first trimester. c. Initially, the transvaginal examination can be painful. d. The transvaginal examination allows pelvic anatomy to be evaluated in greater detail.

ANS: D The transvaginal examination allows pelvic anatomy to be evaluated in greater detail than the abdominal method and also allows intrauterine pregnancies to be diagnosed earlier. The abdominal examination requires a full bladder; the transvaginal examination requires an empty one. The transvaginal examination is more useful in the first trimester; the abdominal examination works better after the first trimester. Neither the abdominal nor the transvaginal method of ultrasound examination should be painful, although the woman will feel pressure as the probe is moved during the transvaginal examination.

A woman arrives at the clinic seeking confirmation that she is pregnant. The following information is obtained: She is 24 years old with a body mass index (BMI) of 17.5. She admits to having used cocaine "several times" during the past year and occasionally drinks alcohol. Her blood pressure is 108/70 mm Hg. The family history is positive for diabetes mellitus and cancer. Her sister recently gave birth to an infant with a neural tube defect (NTD). Which characteristics places this client in a high-risk category? a. Blood pressure, age, BMI b. Drug and alcohol use, age, family history c. Family history, blood pressure (BP), BMI d. Family history, BMI, drug and alcohol abuse

ANS: D The woman's family history of an NTD, her low BMI, and her drug and alcohol use abuse are high risk factors of pregnancy. The woman's BP is normal, and her age does not put her at risk. Her BMI is low and may indicate poor nutritional status, which is a high risk.

A woman at 39 weeks of gestation with a history of preeclampsia is admitted to the labor and birth unit. She suddenly experiences increased contraction frequency of every 1 to 2 minutes, dark red vaginal bleeding, and a tense, painful abdomen. Which clinical change does the nurse anticipate? a. Eclamptic seizure b. Rupture of the uterus c. Placenta previa d. Abruptio placentae

ANS: D Uterine tenderness in the presence of increasing tone may be the earliest sign of abruptio placentae. Women with preeclampsia are at increased risk for an abruption attributable to decreased placental perfusion. Eclamptic seizures are evidenced by the presence of generalized tonic-clonic convulsions. Uterine rupture exhibits hypotonic uterine activity, signs of hypovolemia, and, in many cases, the absence of pain. Placenta previa exhibits bright red, painless vaginal bleeding.

Which statement best describes chronic hypertension? a. Chronic hypertension is defined as hypertension that begins during pregnancy and lasts for the duration of the pregnancy. b. Chronic hypertension is considered severe when the systolic BP is higher than 140 mm Hg or the diastolic BP is higher than 90 mm Hg. c. Chronic hypertension is general hypertension plus proteinuria. d. Chronic hypertension can occur independently of or simultaneously with preeclampsia.

ANS: D Women with chronic hypertension may develop superimposed preeclampsia, which increases the morbidity for both the mother and the fetus. Chronic hypertension is present before pregnancy or diagnosed before the 20 weeks of gestation and persists longer than 6 weeks postpartum. Chronic hypertension becomes severe with a diastolic BP of 110 mm Hg or higher. Proteinuria is an excessive concentration of protein in the urine and is a complication of hypertension, not a defining characteristic.

If cerclage is not a good option, what can be done to prevent miscarriage in women with extremely short cervixes?

Abdominal cerclage -usually done at 11-13 weeks of gestation laparascopically -suture is placed at the junction of the lower uterine segment and the cervix -C-section birth is necessary -suture is left in place for future pregnancies

A client is diagnosed with ectopic pregnancy. Which signs associated with ectopic pregnancy can be found in the client? Select all that apply. Bradycardia Hypertension Abdominal pain Delayed menses Abnormal vaginal bleeding

Abdominal pain Delayed menses Abnormal vaginal bleeding Rationale Ectopic pregnancy is a condition in which the fertilized ovum is implanted outside the uterine cavity. A client with ectopic pregnancy would have colicky pain due to the stretching of the fallopian tube because of the growth of the embryo. Ectopic pregnancy delays menses and can produce abnormal vaginal bleeding. Mild to moderate dark red or brown intermittent vaginal bleeding is observed in ectopic pregnancy. Ectopic pregnancy may cause tubal rupture and result in internal bleeding. This type of pregnancy affects the heart rate and causes tachycardia. Bradycardia is not observed in ectopic pregnancy. The internal bleeding causes hypotension. Hypertension is not observed in ectopic pregnancy.

A pregnant client with chronic hypertension is at risk for placental abruption. Which symptoms of abruption does the nurse instruct the client to be alert for? Select all that apply. Weight loss Abdominal pain Vaginal bleeding Shortness of breath Uterine tenderness

Abdominal pain Vaginal bleeding Uterine tenderness Rationale The nurse instructs the pregnant client to be alert for abdominal pain, vaginal bleeding, and uterine tenderness, because they indicate placental abruption. Weight loss indicates fluid and electrolyte loss, not placental abruption. Shortness of breath indicates inadequate oxygen, which is usually seen in a clientwho is having cardiac arrest.

Soft tissue dystocia

Abnormal labor caused by obstruction of the birth passage by an anatomic abnormality other than that involving the bony pelvis; the obstruction may result from placenta previa, leiomyomas (uterine fibroid tumors), ovarian tumors, or a full bladder or rectum.

Hypertonic uterine dysfunction; primary dysfunctional labor

Abnormal uterine activity often experienced by an anxious first-time mother who is having painful, uncoordinated, and frequent contractions that are ineffective in causing cervical dilation and effacement to progress; they usually occur in the latent phase of the first stage of labor.

Hypotonic uterine dysfunction; secondary uterine inertia

Abnormal uterine activity that usually occurs when a woman initially makes normal progress into the active phase of the first stage of labor but then uterine contractions become weak and inefficient or stop altogether.

A woman at 39 weeks of gestation with a history of preeclampsia is admitted to the labor and birth unit. She suddenly experiences increased contraction frequency of every 1 to 2 minutes; dark red vaginal bleeding; and a tense, painful abdomen. The nurse suspects the onset of what? Eclamptic seizure Rupture of the uterus Placenta previa Abruptio placentae

Abruptio placentae Rationale Women with hypertension are at increased risk for an abruption. Eclamptic seizures are evidenced by the presence of generalized tonic-clonic convulsions. Uterine rupture presents as hypotonic uterine activity, signs of hypovolemia, and in many cases the absence of pain. Placenta previa presents with bright red, painless vaginal bleeding. Uterine tenderness in the presence of increasing tone may be the earliest finding of premature separation of the placenta (abruptio placentae or placental abruption).

Which of the following is the most common kind of placental adherence seen in pregnant woman? Accreta Placenta previa Percreta Increta

Accreta Rationale Placenta accreta is the most common kind of placental adherence seen in pregnant woman and is characterized by slight penetration of myometrium.A placenta previa is when the placenta does not embed correctly and results in what is known as a low lying placenta. It can be marginal, partial or complete in that it covers the cervical os and places the patient at increased risk for painless vaginal bleeding during the pregnancy and/or delivery process. Placenta percreta leads to perforation of the uterus and is the most serious and invasive of all types of accrete. Placenta increta leads to deep penetration of the myometrium.

Dependent edema;

Accumulation of fluid in the tissues of the lowest or most dependent parts of the body, where hydrostatic pressure is the greatest. For ambulating women it is first evident in ankles; if the pregnant woman is confined to bed, it occurs in the sacrum.

Diabetic ketoacidosis (DKA

Accumulation of ketones in the blood resulting from hyperglycemia and leading to metabolic acidosis; it occurs most often during the second and third trimesters when the diabetogenic effect of pregnancy is the greatest."

Which parameter is measured to detect neural tube defects in a fetus? Acetylcholinesterase Phosphatidylglycerol (PG) Lecithin-to-sphingomyelin (L/S) ratio Surfactant-to-albumin (S/A) ratio

Acetylcholinesterase Rationale Acetylcholinesterase and amniotic fluid alpha-fetoprotein (AFP) measurement are used to diagnose neural tube defects. The presence of Phosphatidylglycerol (PG) is used in amniocentesis to determine fetal lung maturity. The lecithin-to-sphingomyelin (L/S) ratio is used in amniocentesis to determine fetal lung maturity. The surfactant-to-albumin (S/A) ratio helps determine fetal lung maturity in the TDx FLM assay.

The nurse is performing a fetal acoustic stimulation test (FAST) in conjunction with a nonstress test. The nurse observes a nonreactive baseline fetal heart rate (FHR) after 5 minutes. What is the best nursing intervention in this situation? Performing the test after an interval of 2 hours Performing a contraction stress test immediately Administering 0.5 milliunits/min oxytocin for 20 minutes Activating a stimulator for 3 seconds on the patient's abdomen

Activating a stimulator for 3 seconds on the patient's abdomen Rationale If the nurse observes a nonreactive baseline FHR, then the sound source, a laryngeal stimulator, should be activated for 3 seconds on the maternal abdomen over the fetal head. Performing the test after 2 hours would not help acquire the required test results. If the FAST test result is nonreactive, the same test is performed again a total of three times. If the test is nonreactive for all three attempts, then the contraction stress test is performed; 0.5 milliunits/minute oxytocin for 20 minutes is administered by the nurse while performing the oxytocin-stimulated contraction test.

A woman who has experienced recent fetal demise continues to call out for her infant. According to the Miles and Demi Model of Parental Grief, which phase of grief is this patient likely experiencing? Intense grief Acute distress Reorganization Anger and denial

Acute distress A woman who calls out for her infant after a fetal demise is in the acute distress phase. She has not accepted the loss and has a sense of unreality.

A nurse, caring for a patient who has experienced a fetal demise, is presenting the infant to the parents. Which action is appropriate for the nurse to make? Address the infant by name. Tell the parents you sympathize with their loss. Wait for the parents to initiate the conversation. Offer to explain how the infant died to the patient's other children.

Address the infant by name. Addressing the infant by name acknowledges right of being named which has been identified as an important intervention for parents.

A postpartum client shows clinical manifestations of hemorrhage and pain. Upon vaginal examination, the nurse finds a smooth mass through the dilated cervix. Which intervention should be performed to prevent blood loss and pain? Select all that apply. Administer an oxytocic agent to the client. Administer light nitrous oxide to the client. Administer halogenated anesthetics to the client. Administer intravenous nitroglycerin to the client. Administer broad-spectrum antibiotics to the client.

Administer an oxytocic agent to the client. Administer intravenous nitroglycerin to the client. Rationale Uterine inversion manifests as hemorrhage, shock, and pain. The presence of a palpable, smooth mass through the dilated cervix indicates incomplete uterine inversion. This is a life-threatening situation that should be treated immediately. In the event of uterine inversion, the nurse should immediately perform maternal fluid resuscitation followed by the administration of tocolytic agents or oxytocic agents. Tocolytic agents, such as intravenous nitroglycerin, and oxytocic agents cause uterine relaxation, thus facilitating the repositioning of the uterus. Oxytocic agents are administered to a patient with uterine inversion following the replacement of the uterus in the pelvic cavity. Light nitrous oxide is used to anesthetize a client who retains the placenta. Halogenated anesthetics cause uterine atony and thus should not be administered. Broad-spectrum antibiotics are used to prevent infection in clients with uterine inversion. Antibiotics are initiated after repositioning the uterus into the pelvic cavity.

A pregnant patient is administered terbutaline (Brethine). The nurse reports to the primary health care provider that the patient has a heart rate of 134 beats per minute and blood pressure of 80/60 mm Hg. Which intervention would be helpful in preventing complications related to terbutaline (Brethine)? Administer propranolol (Inderal). Monitor serum potassium levels. Administer 1gcalcium gluconate. Assess for the presence of oligohydramnios.

Administer propranolol (Inderal). Rationale Terbutaline (Brethine) is a tocolytic agent that is used in the treatment of preterm labor. A heart rate of 134 beats per minute (tachycardia) combined withblood pressure that is less than 80/60 mm Hg indicates intolerable adverse effects of the drug on the cardiovascular system. Propranolol (Inderal) is administered to reverse the cardiovascular adverse effects of terbutaline (Brethine). Serum potassium levels should be monitored in the patient receiving terbutaline (Brethine). However, it is not a priority intervention. Calcium gluconate is administered to reverse the effects of magnesium sulfate. Oligohydramnios (low amniotic fluid volume) is the adverse effect of indomethacin (Indocin) and may not be associated with terbutaline (Brethine).

Which is a priority nursing action when a pregnant client with severe gestational hypertension is admitted to the health care facility? Prepare the client for cesarean delivery. Administer intravenous (IV) and oral fluids. Provide diversionary activities during bed rest. Administer the prescribed magnesium sulfate.

Administer the prescribed magnesium sulfate. Rationale The nurse administers the prescribed magnesium sulfate to the client to prevent eclamptic seizures. IV oral fluids are indicated when there is severe dehydration in the client. It is important to provide diversionary activities during bed rest, but it is secondary in this case. A client who has experienced a multisystem trauma is prepared for caesarean delivery, which increases the chance of maternal survival, if there is no evidence of a maternal pulse.

The nurse examines a client at 30 weeks of gestation for cervical dilation. The nurse understands that the infant may be at risk of cerebral palsy if it is born preterm. Which intervention would help to prevent cerebral palsy? Shifting the client to an obstetric facility Administering antibiotic medications to the client Administering antenatal glucocorticoids to the client Administering magnesium sulfate (Epsom salts) to the client

Administering magnesium sulfate (Epsom salts) to the client Rationale When preterm birth appears inevitable, magnesium sulfate (Epsom salts) is administered to the client at 24 to 32 weeks of gestation to prevent the risk of cerebral palsy. Clients in preterm labor should be shifted to a healthcare facility that is well-equipped to handle emergencies and take care of preterm infants. Antibiotics are administered to prevent infections. Antenatal glucocorticoids are administered to pregnant clients to prevent the risk of respiratory depression in the fetus, caused by structurally and functionally immature lungs.

The nurse finds that a client in labor has developed spontaneous bruises over the skin. The fetal heart activity monitor indicates late decelerations in the fetal heart rate (FHR). What is the best intervention to prevent fetal complications, if the nurse suspects disseminated intravascular coagulation (DIC) in the client? Assisting the client to supine position Increasing the rate of oxytocin intravenous (IV) infusion Administering oxygen through a face mask Administering intravenous (IV) fluids to the client

Administering oxygen through a face mask Rationale DIC may be caused secondary to placental abruption, which may result in late deceleration of the FHR. Late deceleration is due to decreased blood flow and oxygen to the fetus during the uterine contractions (UCs). Therefore, an oxygen mask is applied to the client. A side-lying tilt would provide proper blood flow by decreasing the pressure on the uterus. Hence, the client should be assisted to a side-lying position and not to a supine position. An IV oxytocin infusion is given to increase the UCs during labor, but it is discontinued when a late deceleration is noted. IV fluids are administered to the client to promote fluid balance, but it may not directly help in improving the fetal heart rate.

In caring for the woman with disseminated intravascular coagulation (DIC), what order should the nurse anticipate? Administration of blood Preparation of the woman for invasive hemodynamic monitoring Restriction of intravascular fluids Administration of steroids

Administration of blood Rationale Primary medical management in all cases of DIC involves correction of the underlying cause, volume replacement, blood component therapy, optimization of oxygenation and perfusion status, and continued reassessment of laboratory parameters. Central monitoring would not be ordered initially in a woman with DIC because this can contribute to more areas of bleeding. Management of DIC includes volume replacement, not volume restriction. Steroids are not indicated for the management of DIC.

In caring for the woman with disseminated intravascular coagulation (DIC), what order should the nurse anticipate? Administration of blood Preparation of the woman for invasive hemodynamic monitoring Restriction of intravascular fluids Administration of steroids

Administration of blood Primary medical management in all cases of DIC involves correction of the underlying cause, volume replacement (not volume restriction), blood component therapy, optimization of oxygenation and perfusion status, and continued reassessment of laboratory parameters. Central monitoring would not be ordered initially in a woman with DIC because it could contribute to more areas of bleeding. Steroids are not indicated for the management of DIC.

A postpartum woman with hypovolemic shock is given an intravenous infusion of crystalloids. Upon assessing the reports of the blood tests done after the infusion, the nurse finds that the client has decreased clotting factors and platelet count. Which intervention would help in restoring the normal levels of platelets and clotting factors in the client? Administration of fresh frozen plasma Administration of packed red blood cells Administration of normal saline solution Administration of lactated Ringer's solution

Administration of fresh frozen plasma Rationale Hypovolemic shock is initially treated by restoring blood volume and eliminating the underlying cause of the shock. The client is initially treated by administering crystalloids in order to perform fluid resuscitation. Because the fresh frozen plasma is rich in clotting factors and platelets, administration of fresh frozen plasma restores the normal levels of platelets and clotting factors in the client. Packed red blood cells are administered if the client is actively bleeding. Normal saline solution and lactated Ringer's solution are both crystalline solutions that are administered to restore the circulating blood volume in the condition of hypovolemic shock. However, these solutions are not helpful in restoring normal levels of platelets and clotting factors.

A pregnant client experiences thyroid storm following delivery of the infant. What interventions would the nurse anticipate to be ordered by the physician? Restriction of intravenous fluids to prevent fluid overload Administration of oxygen Antipyretics Synthroid PTU

Administration of oxygen Antipyretics PTU Rationale Oxygen would be provided. Antipyretics would be given to reduce fever. PTU would be administered. IV fluids would be administered to the client in order to reverse the hypotension that the client would be experiencing. Synthroid would not be given, because it is used to treat hypothyroidism, and with thyroid storm, the client is suffering from hyperthyroidism

A pregnant patient experiences thyroid storm following delivery of her infant. What interventions would the nurse anticipate to be ordered by the physician? Select all that apply. Restriction of intravenous fluids to prevent fluid overload Administration of oxygen Antipyretics Synthroid PTU

Administration of oxygen Oxygen would be provided, antipyretics would be given to reduce fever, and PTU would be administered. IV fluids would be administered to the patient in order to reverse the hypotension that the patient would be experiencing. Synthroid would not be given because it is used to treat hypothyroidism, and with thyroid storm, the patient is suffering from hyperthyroidism. Antipyretics Oxygen would be provided, antipyretics would be given to reduce fever, and PTU would be administered. IV fluids would be administered to the patient in order to reverse the hypotension that the patient would be experiencing. Synthroid would not be given because it is used to treat hypothyroidism, and with thyroid storm, the patient is suffering from hyperthyroidism. PTU Oxygen would be provided, antipyretics would be given to reduce fever, and PTU would be administered. IV fluids would be administered to the patient in order to reverse the hypotension that the patient would be experiencing. Synthroid would not be given because it is used to treat hypothyroidism, and with thyroid storm, the patient is suffering from hyperthyroidism.

How should the nurse modify the plan of care for a woman with mastitis? Instruct the patient to pump the breasts or feed every 3 hours. Advise the patient to breastfeed or pump every 1.5 to 2 hours. Instruct the patient to start the feeding on the affected breast. Discourage the mother from pumping if one breast is too sore to feed.

Advise the patient to breastfeed or pump every 1.5 to 2 hours. Feeding or pumping the breasts consistently every 1.5 to 2 hours helps to prevent milk stasis and to clear the infection. The patient should be instructed to feed or pump every 1.5 to 2 hours.

A postpartum woman with asthma who had manual repositioning of her uterus into the pelvic cavity after uterine inversion, experiences prolonged lochial discharge and hemorrhage. Upon examination, the nurse finds a large, boggy uterus. Which nursing intervention may pose the most risk to the client? Removal of retained placental fragments Administration of carboprost (Hemabate) Aggressive massage of the uterine fundus Administration of intravenous oxytocin (Pitocin)

Aggressive massage of the uterine fundus Rationale Aggressive fundal massage should be avoided in a client who underwent a manual repositioning of the uterus, because this may increase the risk of bleeding. Subinvolution of the uterus is the delayed return of the uterus to its normal size and function. Subinvolution of the uterus is characterized by prolonged lochial discharge, excessive bleeding, hemorrhage, and a large, boggy uterus. Retained placental fragments should be removed by performing dilation and curettage. This may reduce excessive bleeding. Carboprost (Hemabate), a uterotonic medication, is used if the uterus is nonresponsive to the continuous oxytocin (Pitocin) infusion. Carboprost (Hemabate) is contraindicated in clients with asthma. Because the client has asthma, carboprost (Hemabate) should not be administered to the client. Administration of intravenous oxytocin (Pitocin) along with normal saline promotes restoration of normal uterine tone and size.

A neonate born at 27 weeks' gestation is receiving gavage feedings every 3 hours. The neonate's abdominal girth has increased by 3 cm since the previous shift. Which is the most important action for the nurse to take? Check for residuals before the next feeding. Test the next stool for reducing substances. Complete an abdominal assessment prior to feeding. Alert the health care provider to the increased abdominal circumference.

Alert the health care provider to the increased abdominal circumference. An increase in abdominal girth by 3 cm indicates the presence of intestinal complications. The health care provider must be alerted to this finding immediately.

The nurse is teaching a client insulin self-administration techniques. What does the nurse include in the lesson? Allow the alcohol to dry before injecting the insulin. Puncture the skin at a 45-degree angle. Ensure that the insulin is injected rapidly. Apply gentle pressure after the injection. Record the dose and time of the injection.

Allow the alcohol to dry before injecting the insulin. Apply gentle pressure after the injection. Record the dose and time of the injection. Rationale The nurse should instruct the client to ensure that the injection site is clean. The site does not need to be cleaned with alcohol; if alcohol is applied, insulin should be injected only after the alcohol dries. Gentle pressure should be applied with sterile gauze while withdrawing the needle after injection to prevent bleeding. The client should maintain a record of the dose and time of injection. Insulin should be administered by puncturing the skin at a 90-degree angle. Insulin should be injected slowly into the skin.

Which diagnostic test would be used by the health care team to detect anencephaly in the fetus? Amniocentesis Doppler blood flow analysis Nonstress test Contraction stress test

Amniocentesis Rationale Anencephaly is a neural tube defect, indicated by elevated levels of alpha-fetoprotein, which results in leakage of cerebrospinal fluid. This abnormality is detected by performing amniocentesis, which detects any abnormal finding in the amniotic fluid. Doppler blood flow analysis is used to study blood flow and muscle motion. It detects trisomy 13 or 18 in the fetus. Nonstress test and contraction stress tests are used to determine fetal well-being in relation to the fetal activity. These tests may not be helpful in detecting anencephaly.

Which are prenatal tests used for diagnosing fetal defects in pregnancy? Select all that apply. Amniocentesis Polyhydramnios Amniotic fluid index (AFI) Chorionic villus sampling (CVS) Daily fetal movement count (DFMC) Percutaneous umbilical blood sampling (PUBS)

Amniocentesis Chorionic villus sampling (CVS) Percutaneous umbilical blood sampling (PUBS) Rationale Amniocentesis, PUBS, and CVS are prenatal tests used for diagnosing fetal defects in pregnancy. Polyhydramnios is increased amniotic fluid. AFI is a method in which the vertical depths (in centimeters) of the largest pocket of amniotic fluid in all four quadrants surrounding the maternal umbilicus are totaled. DFMC, the kick count, is frequently used to monitor the fetus in pregnancies complicated by conditions that may affect fetal oxygenation.

Amniotic Fluid Embolus (AFE)

Anaphylactoid Syndrome of Pregnancy -rare but involves sudden, acute onset of hypoxia, hypotension, cardiovascular collapse, and coagulopathy -occurs during labor, during birth, or within 30 minutes of birth -Amniotic fluid embolism is introduced into maternal circulation Risk factors: -preeclampsia/eclampsia -placenta previa -placental abrutption -advanced maternal age -hypertensive disorders -diabetes -labor induction -forceps-assisted or cesarean birth -cervical laceration -uterine rupture

The nurse is caring for a pregnant client who had an onset of labor during 34 weeks' gestation. What does the nurse expect the primary health care provider (PHP) to prescribe? Select all that apply. Antibiotics Glucocorticoids Synthetic oxytocin Magnesium sulfate Progesterone supplementations

Antibiotics Glucocorticoids Rationale The onset of labor during 34 weeks' gestation indicates that the client has preterm labor. In such a condition, antibiotics and glucocorticoids should be prescribed and administered to the patient. Antibiotics are prescribed to prevent neonatal group B streptococcal infection. Glucocorticoids are prescribed to reduce the neonatal morbidity and mortality. Synthetic oxytocin is administered in clients to induce labor. Therefore synthetic oxytocin will not be prescribed to the client because of the onset of labor. Magnesium sulfate is administered when the labor is induced before 32 weeks' gestation. Progesterone supplementation is administered before the onset of labor to prevent preterm birth.

The nurse is caring for a patient with an episiotomy 4 hours after delivery. While examining the wound, the nurse notices a hard, turgid area alongside the incision. Which is the next step in nursing management? Massage the hardened area Take the patient's temperature Apply an ice pack to the perineum Apply a warm, damp towel to the perineum

Apply an ice pack to the perineum Ice should first be applied to the site to promote vasoconstriction. With less blood flowing to the area of the incision, inflammation may decrease. The nurse should follow up by notifying the health care provider.

A patient's fundus is boggy and rests to the left above the umbilicus. Which action should the nurse take first? Begin fundal massage. Ask the patient to void. Notify the health care provider. Assess the patient's vital signs.

Ask the patient to void. A boggy, displaced fundus above the umbilicus can be caused by a full bladder. Having the patient void should be the first action the nurse takes.

Which medication is contraindicated in a client who is on anticoagulant therapy? Aspirin (Ecotrin) Clindamycin (Cleocin) Misoprostol (Cytotec) Ergonovine (Ergotrate)

Aspirin (Ecotrin) Rationale Aspirin (Ecotrin) is contraindicated in clients undergoing anticoagulant therapy, because it inhibits the synthesis of clotting factors and can lead to prolonged clotting time. Clindamycin (Cleocin) does not inhibit the synthesis of clotting factors and can be administered in clients undergoing anticoagulant therapy. Misoprostol (Cytotec) and ergonovine (Ergotrate) do not affect the clotting factors and do not interact with anticoagulants. Thus, it is safe to administer misoprostol (Cytotec) and ergonovine (Ergotrate) to clients undergoing anticoagulant therapy.

A client with severe gestational hypertension is prescribed hydralazine (Apresoline). What is a priority nursing intervention in this case? Assess for visual disturbances. Assess airway, breathing, and pulse. Assess blood pressure frequently. Prepare the patient for nonstress testing.

Assess blood pressure frequently. Rationale Hydralazine (Apresoline) is an antihypertensive medication. The nurse assesses the client's BP frequently, because a precipitous drop in BP can lead to shock and placental abruption. Visual disturbances are symptoms of severe preeclampsia, not a side effect of hydralazine (Apresoline). The nurse needs to assess airway, breathing, and pulse to stabilize a pregnant client after a convulsion. Nonstress testing is performed once or twice weekly to assess fetal well being.

The nurse is assessing a postpartum client 4 hours after delivery. The nurse observes that the client has cool, pale, and clammy skin with severe restlessness and thirst. What should the immediate nursing intervention be? Begin fundal massage and start oxygen therapy. Begin an hourly pad count and reassure the client. Elevate the head of the bed and assess vital signs. Assess for hypovolemia and notify the primary health care provider.

Assess for hypovolemia and notify the primary health care provider. Rationale The presence of cool, pale, and clammy skin with severe restlessness and thirst are the diagnostic signs of hypovolemia. If the nurse observes these signs it should be immediately reported to the primary health care provider. Fundal massage will only be effective for clients with postpartum hemorrhage due to uterine bleeding. Pad count is associated with postpartum hemorrhage and is unrelated to cold and clammy skin. Elevating the head of the bed may affect the client's vital signs. However, it is not associated with hypovolemia.

Which nursing interventions are implemented while caring for a pregnant client with disseminated intravascular coagulation? Assess for signs of bleeding. Monitor urinary output. Initiate electronic fetal monitoring. Administer prescribed oxygen. Provide suctioning once a day.

Assess for signs of bleeding. Monitor urinary output. Initiate electronic fetal monitoring. Administer prescribed oxygen. Rationale DIC causes extensive external bleeding due to the destruction of clotting factors. Therefore, the nurse needs to assess the client for signs of bleeding, rapid replacement of blood products, and clotting factors. DIC also causes renal failure, so the nurse needs to monitor urinary output in the client as well. Urinary output needs to be more than 30 ml/ hr. Continuous electronic fetal monitoring is necessary to assess if the fetus has sufficient oxygenation. The nurse needs to administer the prescribed oxygen through a nonrebreather facemask to maintain adequate fetal oxygenation. Suctioning is necessary in patients when the airway is obstructed, for instance, after a convulsion.

The nurse assesses a neonate born to a mother with a history of heroin use during pregnancy. Which is a priority assessment for the nurse to complete? Observe for flexed upper extremities when startled. Assess for tremors when unwrapping the neonate. Observe for a dark black, viscous stool in the diaper. Observe for decreased weight loss from the previous day.

Assess for tremors when unwrapping the neonate. Tremors are associated with neonatal abstinence syndrome (NAS) and are expected in a neonate born to a mother with a history of heroin use.

A woman presents to the emergency department with left lower quadrant pain, and the nurse suspects a spontaneous abortion. Which of these nursing care interventions are priorities? Select all that apply. Assess pulse and blood pressure Prepare patient for a chest x-ray Initiate intravenous fluids as ordered Monitor for lightheadedness and dizziness Prepare patient for a laparoscopic surgery Administer intravenous antibiotics as ordered

Assess pulse and blood pressure An increase in pulse rate and a drop in blood pressure can indicate hypovolemic shock. The nurse should closely monitor for hypovolemic shock. Initiate intravenous fluids as ordered Maintaining fluid and electrolyte balance is important in a patient who is at risk for hypovolemic shock. Correct Monitor for lightheadedness and dizziness Symptoms of lightheadedness and dizziness can indicate bleeding, which may lead to hypovolemic shock. The nurse should closely monitor for hypovolemic shock.

A woman in preterm labor is receiving intravenous magnesium sulfate. Which are the nurse's priority assessments? Select all that apply. Assess respirations Monitor urine output Check the capillary refill Assess for deep tendon reflexes Check the level of consciousness

Assess respirations Assessing for at least 12 respirations per minute is a priority for this woman because an adverse effect of magnesium sulfate is respiratory paralysis. Monitor urine output Assessing that the woman's urine output is at least 30 mL/hr is a priority in caring for this woman, because urinary output will serve as determinant factor whether treatment will be continued or terminated by the provider. Assess for deep tendon reflexes Assessing for the presence of deep tendon reflexes is a priority in caring for this woman because depressed or absent reflexes is indicative of early magnesium intoxication.

What is a priority nursing action after administering magnesium sulfate to a pregnant client? Assess the client's weight. Assess serum magnesium level. Restrict fluid intake to 250 ml/hr. Evaluate fetal movement counts hourly.

Assess serum magnesium level. Rationale Magnesium sulfate can cause toxicity in the client if the renal system does not function properly. Therefore the nurse needs to assess the serum magnesium levels so that prompt action can be taken. The nurse needs to assess the client's weight if there is a risk for edema. Fluids are restricted if the client is at risk for edema. Magnesium sulfate does not affect the fetal heart rate (FHR), so assessing fetal movements is not a priority.

A premature neonate who receives gavage feedings every three hours is regurgitating formula and has a slightly distended abdomen. Which assessments are critical for the nurse to perform? Select all that apply. Test every stool for excessive potassium. Assess the axillary temperature every 4 hours. Assess the color and consistency of the vomitus. Assess the respiratory rate before each feeding. Measure the abdominal circumference once a shift. Aspirate the stomach contents to measure the residual.

Assess the color and consistency of the vomitus. Assessing the color and consistency of the vomitus will assist the nurse in determining if regurgitations are present because of overfeeding or intestinal complications. Measure the abdominal circumference once a shift. The nurse should measure the abdominal circumference every 4-8 hours. An increasing abdominal circumference is caused by abdominal distension, which signals intolerance of feedings or intestinal complications. Aspirate the stomach contents to measure the residual. With the presence of abdominal distension and regurgitation with feedings, the nurse must aspirate the stomach contents to measure residuals. Distention, regurgitation, and increasing residuals are all signs of intestinal complications.

While performing fundal massage, the nurse observes that vaginal bleeding has decreased. The fundus was boggy before massage and currently feels firm. Which is the next step in nursing management? Notify the health care provider. Assess the patient's vital signs. Weigh the patient's perineal pads and linens. Inform the patient that she should remain in bed.

Assess the patient's vital signs. Assessing the patient's vitals should be the next step after bleeding has decreased in order to monitor the patient for signs of hypovolemia.

What is the first priority when a pregnant woman says she is experiencing nausea and vomiting?

Assess to determine the severity of the problem -tell her to come in to her HCP office and/or emergency room Assessment should include -frequency, severity, and duration of episodes of nausea and vomiting -precipitating factors -any pharmacologic or nonpharmacologic treatment measures used -prepregnancy weight and documented weight gain or loss during pregnancy vital signs -urinalysis -CBC -electrolytes -liver enzymes -bilirubin -assess mother's anxiety

Daily Fetal Movement Count

Assessment of fetal activity by the mother; it is a simple yet valuable method for monitoring the condition of the fetus. The fetal alarm signal refers to the cesssation of fetal movements entirely for 12 hours.

The primary health care provider suspects a pregnant client to be at risk for an ectopic pregnancy and instructs the nurse to prepare the client for ultrasonography. How does the nurse prepare the client for the test? Instruct the client not to void prior to the test. Assist the client into a lithotomy position during the test. Inform the client that moderate pain will be felt during the test. Use pillows to elevate the client's head and knees during the test.

Assist the client into a lithotomy position during the test. Rationale An ectopic pregnancy is detected using transvaginal ultrasonography. The client should be placed in a lithotomy position, because this position enables the primary health care provider to conduct the test with ease and provides comfort to the client. If abdominal and not transvaginal ultrasonography is being performed, then the client would be instructed to avoid voiding. Transvaginal ultrasonography does not cause pain; the client would only feel a slight pressure due to insertion of the probe. The client 's head and knees are elevated using pillows while performing abdominal ultrasonography. While performing transvaginal ultrasonography, the nurse should place a pillow to elevate the client 's pelvis.

A primary health care provider prescribed 15-methylprostaglandin F2α (Carboprost) to a client. What history does the nurse check before administering this drug to the patient to prevent complications? Asthma Diabetes Joint pain Migraine pain

Asthma Rationale Methylprostaglandin F2α (Carboprost) causes uterine contractions (UCs) and is used to prevent postpartum hemorrhage (PPH). The medication also causes bronchoconstriction. If it is administered to an asthmatic client, it may aggravate bronchoconstriction and cause status asthmaticus. Therefore it is contraindicated in the asthmatic client. Thus, the nurse should check for the history of asthma in the client before administering 15-methylprostaglandin F2α (Carboprost). The medication 15-methylprostaglandin F2α (Carboprost) does not interfere with blood sugar levels, joint structure, or cerebral blood flow. Therefore it can be safely administered to clients with diabetes, joint pain, and migraine pain.

A neonate born at 33 weeks' gestation is admitted to the NICU. The neonate is grunting and retracting with each respiration, and the neonate's skin is mottled and cool to the touch. Which priority assessments should the nurse complete? Select all that apply. Assess the neonate's muscle tone. Auscultate the neonate's lung sounds. Assess the neonate's axillary temperature. Assess the neonate's oxygen saturation levels. Auscultate the neonate's heart sounds to detect a murmur.

Auscultate the neonate's lung sounds. Assessing the lung sounds of the neonate provides information regarding aeration and oxygenation. The airway of the neonate is a priority to ensure proper oxygenation. Correct Assess the neonate's axillary temperature. Cool, mottled skin, grunting, and retracting with each respiration may be associated with inadequate thermoregulation. Measuring the axillary temperature will provide data for the thermoregulatory status of the neonate. Correct Assess the neonate's oxygen saturation levels. The oxygenation levels of the neonate provide information regarding the ventilator status of the neonate.

When do nausea and vomiting associated with pregnancy begin? When do they normally go away?

Begin at 4-10 weeks of gestation Usually go away by 20 weeks of gestation Possible causes of nausea and vomiting during pregnancy -Relaxation of the smooth muscle of the stomach -increasing levels of estrogen and hCG -gastroesophageal reflux -reduced gastric mobility -gastric dysrhythmias

A pregnant client at 30 weeks of gestation has preterm labor. The nurse understands that the preterm baby may be born with immature lungs. What medication would be administered to help in lung maturation? Hydralazine (Apresoline) Calcium gluconate (Kalcinate) Low-dose aspirin (Anacin) Betamethasone (Celestone)

Betamethasone (Celestone) Rationale The primary health care provider prescribes corticosteroids such as betamethasone (Celestone) to enhance fetal lung maturation for gestations less than 34 weeks. Hydralazine (Apresoline) is an antihypertensive, which is not used to develop fetal lungs. Calcium gluconate (Kalcinate) is used as an antidote for magnesium sulfate (Sulfamag) toxicity. Low-dose aspirin (Anacin) prevents preeclampsia but does not help develop the fetus' lungs.

When would the best timeframe be to establish gestational age based on ultrasound? At term 8 weeks Between 14 and 22 weeks 36 weeks

Between 14 and 22 weeks Ultrasound determination of gestational age dating is best done between 14 and 22 weeks. It is less reliable after that period because of variability in fetal size. Standard sets of measurements relative to gestational age are noted around 10 to after 12 weeks and include crown-rump length (after 10), biparietal diameter (after 12), femur length, and head and abdominal circumferences.

When would the best timeframe to establish gestational age based on ultrasonography? At term 8 weeks Between 14 and 22 weeks of gestation 36 weeks

Between 14 and 22 weeks of gestation Rationale Between 14 and 22 weeks of gestation is the period of time in which US is the most reliable in determining gestational age. It is less reliable after that period because of variability in fetal size. A standard set of measurements relative to gestational age is noted from around 10 to after 12 weeks, and includes crown-rump length (after 10 weeks), biparietal diameter (after 12 weeks), femur length, head and abdominal circumference.

In the past, factors to determine whether a woman was likely to develop a high risk pregnancy were evaluated primarily from a medical point of view. A broader, more comprehensive approach to high risk pregnancy has been adopted. There are now four categories based on threats to the health of the woman and the outcome of pregnancy. Which of the options listed below is considered included as a category? Select all that apply. Biophysical Psychosocial Geographic Environmental Sociodemographic

Biophysical Psychosocial Environmental Sociodemographic Rationale Ethnicity may be one of the risks to pregnancy; however, it is not the only factor in this category. Low income, lack of prenatal care, age, parity, and marital status are included. Biophysical is one of the broad categories used for determining risk. These include genetic considerations, nutritional status, and medical and obstetric disorders. Psychosocial risks include smoking; using caffeine, drugs, or alcohol; and psychologic status. All of these adverse indicators can have a negative effect on the health of the mother or fetus. Environmental risks are those that can affect fertility and fetal development. These include infections, chemicals, radiation, pesticides, illicit drugs, and industrial pollutants. Geographic should be correctly referred to as sociodemographic risk. These factors stem from the mother and her family.

Very preterm birth

Birth that occurs before 32 weeks of gestation; the great majority of deaths and most serious morbidity occur to infants born at this time.

Grade 1 Placental Abruption

Bleeding is minimal (<500 mL lost) -dark red blood -shock, coagulopathy rare -normal uterine tonicity -pain absent -normal location of placenta -normal fetal heart rate and pattern -10-20% separation

A newborn of a mother with diabetes is jittery and hypotonic. The nurse should complete which assessment first in the neonate? Respiratory rate Axillary temperature Blood glucose level Readiness to nipple feed

Blood glucose level A blood glucose level confirms the presence of hypoglycemia in the newborn of a mother with diabetes. Jitteriness and hypotonia are signs of hypoglycemia.

A neonate with a history of asphyxia from birth is hypotonic, has audible grunting, and has severe subcostal retractions. Which findings should the nurse report to the health care provider? Select all that apply. Blood pH level of 7.1 Substernal retractions Respiratory rate of 56 bpm Pulse oximeter reading of 88% on room air Cyanosis in the hands and feet of the neonate

Blood pH level of 7.1 A blood pH level of 7.1 indicates acidemia, which is possible in a neonate with a history of asphyxia at birth and signs of respiratory distress. Substernal retractions Substernal retractions are a sign of respiratory distress syndrome, and the nurse observes for signs of developing and worsening symptoms and reports them to the health care provider. Pulse oximeter reading of 88% on room air Asphyxia at birth can lead to the development of persistent pulmonary hypertension (PPH) which is associated with low arterial oxygen blood levels. Saturation levels of 88% along with hypotonia, audible grunting and severe retractions are all signs of PPH

The nurse is caring for a client with preeclampsia who gave birth by cesarean section. The primary health care provider prescribes a nonsteroidal antiinflammatory pain medication to the client. What parameter does the nurse closely monitor in this client? White blood cell (WBC) count Respiratory rate Blood pressure Prothrombin time

Blood pressure Rationale Nonsteroidal antiinflammatory drugs should be used with caution in clients with preeclampsia. Because preeclampsia is associated with hypertension, these medications may have the potential to further increase such clients' blood pressure. Therefore, the nurse should closely monitor the client's blood pressure. White blood cells are unaffected by nonsteroidal antiinflammatory drugs, and monitoring the WBC count is not required. Monitoring the respiratory rate is essential in preeclampsia associated with seizures and is unrelated to the use of nonsteroidal antiinflammatory drugs. The prothrombin time is usually unchanged in preeclampsia and is not a complication of the administration of nonsteroidal antiinflammatory drugs.

The nurse is caring for a client with preeclampsia who gave birth by cesarean section. The primary health care provider prescribes a nonsteroidal antiinflammatory pain medication to the client. What parameter does the nurse closely monitor in this client? White blood cell (WBC) count Respiratory rate Blood pressure Prothrombin time

Blood pressure Rationale Nonsteroidal antiinflammatory drugs should be used with caution in clients with preeclampsia. Because preeclampsia is associated with hypertension, these medications may have the potential to further increase such clients' blood pressure. Therefore, the nurse should closely monitor the client's blood pressure. White blood cells are unaffected by nonsteroidal antiinflammatory drugs, and monitoring the WBC count is not required. Monitoring the respiratory rate is essential in preeclampsia associated with seizures and is unrelated to the use of nonsteroidal antiinflammatory drugs. The prothrombin time is usually unchanged in preeclampsia and is not a complication of the administration of nonsteroidal antiinflammatory drugs.

The nurse is caring for a pregnant client who has been prescribed terbutaline (Brethine) to relax the uterus. Following the assessment, the nurse informs the primary health care provider (PHP) that it is not safe to administer terbutaline (Brethine) to the client. Which client condition leads the nurse to such a conclusion? Blood pressure of 80/60 mmHg. Short episode of hyperglycemia. Irregular episodes of dysrhythmias. Heart rate of less than 120 beats/min.

Blood pressure of 80/60 mmHg. Rationale Terbutaline (Brethine) relaxes the smooth muscles and inhibits uterine activity (UA). However, the drug can adversely affect the cardiovascular system. Presence of a blood pressure lower than 90/60 mm Hg indicates an adverse effect on the cardiovascular system, and the nurse should stop the treatment to prevent further damage. Short and irregular episodes of hyperglycemia and dysrhythmias are mild and tolerable adverse effects of terbutaline (Brethine), so those conditions would not warrant the discontinuation of the medication. If the client develops tachycardia greater than 130 beats/min, then the treatment should be stopped.

The nurse is caring for a pregnant client who has been prescribed terbutaline (Brethine) to relax the uterus. Following the assessment, the nurse informs the primary health care provider (PHP) that it is not safe to administer terbutaline (Brethine) to the client. Which client condition leads the nurse to such a conclusion? Blood pressure of 80/60 mmHg. Short episode of hyperglycemia. Irregular episodes of dysrhythmias. Heart rate of less than 120 beats/min.

Blood pressure of 80/60 mmHg. Rationale Terbutaline (Brethine) relaxes the smooth muscles and inhibits uterine activity (UA). However, the drug can adversely affect the cardiovascular system. Presence of a blood pressure lower than 90/60 mm Hg indicates an adverse effect on the cardiovascular system, and the nurse should stop the treatment to prevent further damage. Short and irregular episodes of hyperglycemia and dysrhythmias are mild and tolerable adverse effects of terbutaline (Brethine), so those conditions would not warrant the discontinuation of the medication. If the client develops tachycardia greater than 130 beats/min, then the treatment should be stopped.

After assessing a client, the nurse concludes that the client has threatened miscarriage. What signs in the client would be consistent with the diagnosis? Select all that apply. Blood spotting Heavy bleeding Closed cervical os Mild uterine cramping Decreased urine output

Blood spotting Closed cervical os Mild uterine cramping Rationale A pregnancy that ends from natural causes before 20 weeks of gestation results in miscarriage, or spontaneous abortion. Blood spotting, a closed cervical os, and mild uterine cramping are signs of a threatened miscarriage. Heavy bleeding is observed in inevitable and incomplete miscarriages. A threatened miscarriage does not affect urine output. Decreased urine output is a sign of placenta previa.

Which risk factor for DVT is most likely to occur during delivery? Venous stasis Blood vessel injury Prolonged inactivity Increased coagulation

Blood vessel injury Blood vessel injury is likely to occur during delivery due to operative or cesarean delivery or due to injury to the vaginal or perineal tissue.

In which way does pregnancy affect the risk of thrombus formation? The fibrinolytic system is activated. Potential for vascular damage is decreased. Factors that promote clot formation are decreased. Blood vessels in the legs and pelvis are compressed.

Blood vessels in the legs and pelvis are compressed. Blood vessels in the legs and pelvis are compressed by the expanding uterus during pregnancy.

mastitis

Breast infection -found in breastfeeding mothers Symptoms: -malaise -fever -flulike symptoms -sore area in breast

The nurse is assessing a client with postpartum hemorrhage (PPH). During the physical assessment, the nurse finds that there are deep lacerations in the cervix. Which observation allows the nurse to conclude that the PPH is due to cervical lacerations? Dark red blood Bright red blood Clots in the blood Foul-smelling blood

Bright red blood Rationale If the color of blood is bright red, it indicates that hemorrhage has occurred due to deep lacerations of the cervix. Foul-smelling blood during the postpartum period indicates infection. Bleeding due to varices or superficial lacerations of the birth canal is dark red in color. Clots in the blood characterize the PPH caused by disseminated intravascular coagulation (DIC).

A breastfeeding woman's cesarean birth occurred 2 days ago. Investigation of the pain, tenderness, and swelling in her left leg led to a medical diagnosis of DVT. Care management for this woman during the acute stage of DVT select all that apply A Explaining that she will need to stop breastfeeding until anticoagulation therapy is completed B Administer warfarin (Coumadin orally) C Placing the woman on bed rest with her left leg elevated D Fitting the woman with elastic stockings so that she can exercise her legs E Telling her to avoid changing her position for the first 24 hours F Adminsitering heparin intravenously for 3-5 days

C, F C Placing the woman on bed rest with her left leg elevated F Adminsitering heparin intravenously for 3-5 days

At 37 weeks of gestation, Mary's BP rose from a prepreg- nant baseline of 118/66 mm Hg to 142/88 mm Hg. No other problematic signs and symptoms, including proteinuria, were noted.

C. Gestational hypertension

Magnesium sulfate

CNS depressant -relaxes smooth muscles including uterus -given as piggyback IV, loading dose, maintenance dose Maternal Adverse side effects -hot flushes -sweating -burning at insertion site -nausea and vomiting -drowsiness -blurred vision -headache -lethargy -hypocalcemia -dyspnea -transient hypotension Fetal (uncommon) -decreased FHR variability -nonreactive NST -decreased breathing movement

What is magnesium sulfate?

CNS depressant, is drug of choice for prevention & management of seizures with preeclamptic & eclamptic women. Drops BP for 30-45mins

The nurse observes that intravenous (IV) administration of magnesium sulfate has resulted in magnesium toxicity in a pregnant client with preeclampsia. The nurse immediately discontinues the infusion and reports to the primary health care provider (PHP). For which drug does the nurse obtain a prescription from the PHP? Calcium gluconate Nifedipine (Adalat) Hydralazine (Apresoline) Labetalol hydrochloride (Normodyne)

Calcium gluconate Rationale The nurse needs to obtain a prescription for calcium gluconate, because it acts as an antidote to magnesium toxicity. Nifedipine (Adalat) and labetalol hydrochloride (Normodyne) are antihypertensive medications, which are prescribed for gestational hypertension or severe preeclampsia. Hydralazine (Apresoline) is also an antihypertensive medication used for treating hypertension intrapartum.

A nurse completes an assessment of a 28 weeks' neonate. The findings include an axillary temperature of 89.6 degrees Fahrenheit, listlessness, and a moderate emesis of digested breastmilk. Which is a priority action for the nurse to take? Assess for gastric residuals. Assess the abdominal girth for distension. Call the health care provider with the assessment findings. Observe the neonate for tachypnea, grunting, and costal retractions.

Call the health care provider with the assessment findings. Hypothermia, hypotonia, and regurgitation of feedings are all signs of sepsis. Early identification is important to the overall welfare of the neonate.

What is the pathologic change associated with diabetic ketoacidosis? > Loss of electrolytes Metabolic alkalosis Cellular dehydration Increase in the blood volume

Cellular dehydratio Rationale Diabetic ketoacidosis is characterized by an increase in blood sugar and ketone levels. This causes osmotic diuresis, which ultimately results in cellular dehydration. There is loss of electrolytes in ketoacidosis. Ketone bodies in the blood would result in decreased blood pH, which is referred to as metabolic acidosis. In this condition, the kidneys excrete large amounts of fluid, which results in a decrease in blood volume.

The nurse is caring for a pregnant client who has a history of second trimester loss and preterm birth. After reviewing the ultrasound reports, the nurse finds that the client has a cervix less than 30 mm. What treatment strategy should be included in the treatment plan? Cerclage insertion Cesarean delivery Corticosteroid therapy Methotrexate (MTX) therapy

Cerclage insertion Rationale A client who has a history of second trimester loss and preterm delivery has a risk of premature cervical opening called cervical insufficiency. Cerclage insertion decreases preterm birth through a cervical suture placed surgically to tie the cervix and keep it closed. It is not necessary for a client with cervical insufficiency to have a cesarean delivery. If the client has proper treatment with cerclage or progesterone, it is possible for the client to have a vaginal delivery. Corticosteroid therapy is given to the client at risk of having a preterm delivery to accelerate fetal lung maturity. MTX therapy is used to treat ectopic pregnancy.

Upon assessment of a pregnant client, the nurse concludes that the client is less likely to have a preterm delivery. Which client clinical finding led the nurse to conclude this? Previous cesarean birth. Preexisting diabetes mellitus. Cervical length is more than 30 mm. Symptoms of chronic hypertension.

Cervical length is more than 30 mm. Rationale The cervical length is a good predictor of preterm birth. For childbirth, the cervix needs to prepare itself, in terms of effacement and dilatation. Clients having a cervical length of more than 30 mm would not have preterm labor, even if they have symptoms of preterm labor. A previous cesarean birth may not rule out the risk of preterm delivery. Chronic hypertension and preexisting diabetes mellitus might not increase the risk of preterm labor.

Which intrapartum factors contribute to an increased risk of endometritis? Select all that apply. Episiotomy Premature labor Low birth weight Cesarean delivery Vaginal laceration Use of a vacuum extractor

Cesarean delivery Cesarean delivery may increase the risk of endometritis, as the incisions in the skin and uterine lining provide a site for pathogens to enter the bloodstream. Correct Vaginal laceration Vaginal laceration may increase the risk of endometritis, as the wound provides a site for pathogens to enter the bloodstream. Correct Use of a vacuum extractor Use of a vacuum extractor may increase the risk of endometritis, as use of an extractor increases the risk of vaginal and perineal trauma. The wounds caused by the use of an extractor provide a site for bacteria to enter the bloodstream. Episiotomy Episiotomies may increase the risk of endometritis, as the incision provides a site for pathogens to enter the bloodstream.

The nurse is caring for a postpartum client who is diagnosed with superficial venous thrombosis. Which interventions will the nurse perform for a client with this condition? Select all that apply. Change the client's position frequently. Apply moist heat to relieve the client's pain. Inspect the affected site by gently palpating it. Massage the affected area suitably for comfort. Allow the client to rest with legs in a low position.

Change the client's position frequently. Apply moist heat to relieve the client's pain. Inspect the affected site by gently palpating it. Rationale The client with superficial venous thrombosis is instructed to change positions frequently to avoid the pooling of blood. Application of moist heat eases circulation and provides pain relief. Routine inspection of the affected site prevents further complications of the condition. Massaging the affected area is contraindicated, because this may cause the clot to break, resulting in embolism. The client should not rest by placing the legs in a low position, because this causes the pooling of blood in the lower extremities.

A 28-year-old multipara delivered a 9 pound, 3 ounce baby girl an hour ago after a 22-hour labor with a forceps-assisted birth. As the patient is holding her daughter, she keeps shifting position and is becoming increasingly irritable and annoyed with everyone in the room. What action should the nurse initially take? Massage the fundus Check her perineum. Assess her vital signs. Check the tone of her fundus.

Check her perineum. Rationale The client is exhibiting increasing anxiety, which can signal the presence of postpartum hemorrhage. Risk factors for postpartum hemorrhage include a large fetus, prolonged labor, and a forceps-assisted birth. Because vital signs change late, the fastest way to see the amount of current hemorrhage is to check the perineum. The fundus would be massaged and additional nursing and medical interventions would be instituted.

After removal of the retained placental fragment, the client is recovering on the mother-baby (postpartum) unit. What should the nurse identify as the priority of care? Monitoring the infant's ability to breastfeed. Checking the perineum frequently. Assessing vital signs frequently. Encouraging the client to ambulate.

Checking the perineum frequently. Rationale Although the retained placental fragment has been removed, the client is still at risk for postpartum hemorrhage. Because vital signs change late, checking the perineum is the best way to assess for postpartum hemorrhage and is the priority. Monitoring the infant's ability to breastfeed is unrelated to removal of retained placental fragments. Assessment of vital signs is a required part of the nursing care plan for this client. Increasing ambulation is not a priority at this time in the postpartum period.

A pregnant woman is scheduled for a transvaginal ultrasound test to establish gestational age. In preparing this woman for the test the nurse should: A. Place the woman in a supine position with her hips elevated on a folded pillow B. Instruct her the woman to come for the test with a full bladder C. Administer an analgesic 30 minutes before the test D. Lubricate the vaginal probe with transmission gel"

Choice A is correct; a supine position with hips ele- vated enhances the view of the uterus; a lithotomy position may also be used; a full bladder is not re- quired for the vaginal ultrasound but is needed for most abdominal ultrasounds; during the test the woman may experience some pressure but medica- tion for pain relief before the test is not required; contact gel is used with the abdominal ultrasound; water-soluble lubricant may be used to ease insertion of the vaginal probe." A. Place the woman in a supine position with her hips elevated on a folded pillow

A pregnant woman at 42 weeks of gestation is undergoing a nonstress test. During the test an evaluation of the monitor tracing indicated that two accelerations of the fetal heart rate occurred within a 20-minute period. The first acceleration of 18 beats/minute lasted 20 seconds and the second acceleration of 20 beats/minute lasted 16 seconds. The nurse conducting the test records the test result as: A. Reactive B. Positive C. Nonreactive D. Equivocal hyperstimulatory"

Choice A is correct; choices B and D are types of results for the contraction stress test; see Box 26-8 for a full explanation of results for the nonstress test and Table 26-5 for the contraction stress test." A. Reactive

A pregnant woman at 38 weeks of gestation and diagnosed with marginal placenta previa has just given birth to a healthy newborn male. The nurse recognizes that the immediate focus for the care of this woman is: A. Preventing hemorrhage B. Relieving acute pain C. Preventing infection D. Fostering attachment of the woman with her new son

Choice A is correct; hemorrhage is a major potential postpartum complication because the implantation site of the placenta is in the lower uterine segment, which has a limited capacity to contract after birth; infection is another major complication but it is not the immediate focus of care; choices B and D are also important but not to the same degree as hemorrhage, which is life threatening. A. Preventing hemorrhage

A woman's labor is being suppressed using IV magnesium sulfate. Which measure should be implemented during the infusion? A. Limit intravenous fluid intake to 125 mL/hour. B. Discontinue infusion if maternal respirations are less than 14 breaths/minute. C. Ensure that indomethacin is available should toxicity occur. D. Assist woman to maintain a comfortable semirecumbent position.

Choice A is correct; magnesium sulfate is a CNS depressant; woman should alternate lateral positions to decrease pressure on cervix, which could stimulate uterine contractions; calcium gluconate would be used if toxicity occurs; infusion should be discontinued if respiratory rate is less than 12 breaths/ minute. A. Limit intravenous fluid intake to 125 mL/hour.

A nurse caring for a pregnant woman suspected of being in preterm labor recognizes which sign as diagnostic of preterm labor? A. Cervical dilation of at least 2 cm B. Uterine contractions occurring every 15 minutes C. Spontaneous rupture of the membranes D. Presence of fetal fibronectin in cervical secretions

Choice A is correct; the definitive sign of preterm labor is significant change in the cervix; while uterine contractions do occur, they must occur at a frequency of more than six contractions per hour and cause significant changes in the cervix; fetal fibronectin indicates that the risk for preterm labor exists but it does not mean that preterm labor is occurring. A. Cervical dilation of at least 2 cm

A primigravida at 10 weeks of gestation reports mild uterine cramping and slight vaginal spotting without passage of tissue. When she is examined, no cervical dilation is noted. The nurse caring for this woman should: A. Anticipate that the woman will be sent home with instructions to limit her activity and to avoid stress or orgasm B. Prepare the woman for a dilation and curettage C. Notify a grief counselor to assist the woman with the imminent loss of her fetus D. Tell the woman that the doctor will most likely perform a cerclage to help her maintain her pregnancy

Choice A is correct; the woman is experiencing a threatened miscarriage; therefore expectant management is attempted first, although there are no research-proven therapies; choices B and C reflect management of an inevitable and complete or in- complete abortion; cerclage or suturing of the cervix is done for recurrent spontaneous miscarriage associated with premature dilation of the cervix (reduced cervical competence). A. Anticipate that the woman will be sent home with instructions to limit her activity and to avoid stress or orgasm

When measuring the blood pressure to ensure consistency and to facilitate early detection of blood pressure changes consistent with preeclampsia, the nurse should: A. Place the woman in a sitting position with feet flat on the floor B. Allow the woman to rest for 5 minutes after positioning her before measuring her blood pressure C. Record Korotkoff phase IV (muffled sound) as the diastolic pressure D. Use a proper-sized cuff that covers at least 50% of her upper arm

Choice A is correct; the woman should rest for at least 10 minutes after assuming her position and the cuff should cover 80% of the upper arm or be 1.5 times the length of the upper arm; either Korotkoff phase V alone or with phase IV should be used when recording the diastolic pressure. A. Place the woman in a sitting position with feet flat on the floor

A woman's labor is being induced. The nurse assesses the woman's status and that of her fetus and the labor process just before an infusion increment of 2 milliunits/minute. The nurse discontinues the infusion and notifies the woman's primary health care provider if during this assessment she notes: A. Frequency of uterine contractions: every 11⁄2 minutes B. Variability of fetal heart rate (FHR): present C. Deceleration patterns: early decelerations noted with several contractions D. Intensity of uterine contractions at their peaks: 80 to 85 mm Hg

Choice A is correct; uterine contractions should not occur more frequently than five contractions in 10 minutes to allow for an adequate rest period between contractions; choices B, C, and D are all expected findings within the normal range. A. Frequency of uterine contractions: every 11⁄2 minutes

A woman, who is at 35 weeks of gestation with preeclampsia, has a seizure. Immediately after the seizure the nurse's priority action is to: A. Evaluate fetal heart rate and pattern for signs of decreasing variability, late decelerations, or bradycardia B. Assess status of the maternal airway, respiratory effort, and pulse C. Determine if membranes have ruptured and if the amniotic fluid contains meconium D. Prepare to increase the amount of magnesium sulfate being infused from 1 g/hour to 2 g/hour"

Choice B is correct; although choices A, C, and D are all appropriate actions, they are not the first priority; remember the ABCs—airway, breathing, circulation—when considering the priority action. B. Assess status of the maternal airway, respiratory effort, and pulse

A woman with preeclampsia gave birth by cesarean section 1 hour ago. She is still receiving a magnesium sulfate infusion at 1 g/hour. A major concern regarding the administration of magnesium sulfate at this time is: A. Increased risk for seizures B. Central nervous system depression C. Hypotension D. Diuresis

Choice B is correct; magnesium sulfate is a CNS depressant that potentiates the action of other CNS depressants such as opioid analgesics; it reduces the risk for seizures but is unlikely to cause hypotension; diuresis is a common expected finding in the post- partum period. B. Central nervous system depression

A woman has just been admitted with a diagnosis of hyperemesis gravidarum. She has been unable to retain any oral intake and as a result has lost weight and is exhibiting signs of dehydration with electrolyte imbalance and aceton- uria. The nurse anticipates that the care management of this woman will include: A. Administering vitamin K to control nausea and vomiting B. Separating liquids from solids, alternating them every 2 to 3 hours once she is able to tolerate oral intake C. Avoiding oral hygiene until the woman is able to tolerate oral fluids D. Providing three daily meals of bland foods with warm fluids once the woman is able to tolerate oral intake"

Choice B is correct; oral hygiene is important when nothing by mouth (NPO) and after vomiting epi- sodes to maintain the integrity of oral mucosa; taking fluids between (not with) meals reduces nausea, thereby increasing tolerance for oral nutrition; vita- min B6 (pyridoxine) and/or doxylamine (Unisom) are often the first-line antiemetic drugs used. B. Separating liquids from solids, alternating them every 2 to 3 hours once she is able to tolerate oral intake

A 40-year-old woman at 18 weeks of gestation is having a quad marker test performed. She is obese, and her health history reveals that she is Rh negative. The primary purpose of this test is to screen for: A. Spina bifida B. Down syndrome C. Gestational diabetes D. Rh antibodies"

Choice B is correct; the quad marker test is used to screen the older pregnant woman for the possibility that her fetus has Down syndrome; serum levels of AFP, unconjugated estriol, human chorionic gonado- tropin (hCG), and inhibin A are measured; maternal serum AFP alone is the screening test for open neural tube defects such as spina bifida; a 1-hour, 50-g glu- cose test is used to screen for gestational diabetes; amniocentesis and Coombs testing are used to check for Rh antibodies and sensitization." B. Down syndrome

During her pregnancy a woman with pregestational diabetes has been monitoring her blood glucose level several times a day. Which level requires further assessment? A. 85 mg/dL—15 minutes prior to breakfast B. 98 mg/dL—prior to lunch C. 140 mg/dL—2 hours after lunch D. 126 mg/dL—1 hour after supper"

Choice C is correct; a 2-hour postprandial blood glucose should be 130 mg/dL or less; choices A, B, and D all fall within the expected normal ranges for premeal and postmeal levels (see Table 29-3). C. 140 mg/dL—2 hours after lunch

A woman is in active labor. On spontaneous rupture of her membranes, the nurse caring for this woman notices variable deceleration patterns during evaluation of the monitor tracing. When preparing to perform a vaginal examination, the nurse observes a small section of the umbilical cord protruding from the vagina. What should the nurse do next? A. Increase the IV drip rate. B. Administer oxygen to the woman via mask at 8 to 10 L/minute. C. Place a sterile gloved hand into the vagina and hold the presenting part off the cord while calling for assistance. D. Wrap the cord loosely with a sterile towel saturated with warm normal saline.

Choice C is correct; although choices A, B, and D are appropriate actions along with changes in her position, removing pressure from the cord to preserve perfusion is the priority; see Emergency—Prolapsed Umbilical Cord box." C. Place a sterile gloved hand into the vagina and hold the presenting part off the cord while calling for assistance.

During a contraction stress test, four contractions lasting 45 to 55 seconds each were recorded in a 10-minute period. A late deceleration was noted during the third contraction. The nurse conducting the test documents the result as: A. Negative B. Positive C. Equivocal or suspicious D. Unsatisfactory"

Choice C is correct; an equivocal suspicious result is recorded when late decelerations occur with fewer than 50% of the contractions; see Table 26-5 for a full explanation of the results." C. Equivocal or suspicious

A woman is admitted through the emergency department with a medical diagnosis of ruptured ectopic pregnancy. The primary nursing diagnosis at this time is: A. Acute pain related to irritation of the peritoneum with blood B. Risk for infection related to tissue trauma C. Deficient fluid volume related to blood loss associated with rupture of the uterine tube D. Anticipatory grieving related to unexpected pregnancy outcome

Choice C is correct; choices A, B, and D are appropriate nursing diagnoses but deficient fluid volume is the most immediate concern since it places the woman's well-being at greatest risk. C. Deficient fluid volume related to blood loss associated with rupture of the uterine tube

A physician has ordered that dinoprostone (Cervidil) be administered to ripen a pregnant woman's cervix in prepa- ration for an induced labor. In fulfilling this order, the nurse should: A. Insert the dinoprostone in the cervical canal just below the internal os B. Tell the woman to remain in bed for at least 15 minutes C. Observe the woman for signs of uterine tachysystole D. Remove the dinoprostone as soon as the woman begins to experience uterine contractions

Choice C is correct; it is inserted in the posterior vaginal fornix; the woman should remain in bed for 2 hours; caution should be used if the woman has asthma; therefore ensure that physician is aware of asthma; the insert is removed for severe side effects such as uterine tachysystole; dinoprostone (Cervidil) often stimulates contractions and may even induce the onset of labor, eliminating or reducing the need for oxytocin (Pitocin); it should be removed after 12 hours or with onset of active labor. C. Observe the woman for signs of uterine tachysystole

A woman with preeclampsia with severe features is receiving nifedipine (Procardia). She asks the nurse what this medication is for. The nurse should tell her that nifedipine is used to: A. Prevent seizures B. Relieve the headaches she is starting to have C. Decrease her blood pressure D. Reduce the edema in her hands and legs

Choice C is correct; nifedipine (Procardia) is a calcium channel blocker used to decrease blood pressure. C. Decrease her blood pressure

Following vaginal birth 2 hours ago a woman with preeclampsia is experiencing a heavy flow as a result of a boggy uterus. It is determined that she will require medication to reduce the amount of blood loss. Which medication would the nurse anticipate administering? A. Methergine (methylergonovine) B. Calcium gluconate C. Pitocin (oxytocin) D. Normodyne (labetalol)

Choice C is correct; oxytocin (Pitocin) as an oxytocic medication is safe and effective to use to contract the uterus and reduce blood loss because it will not increase blood pressure as can methylergonovine (Methergine), another oxytocic medication; calcium gluconate is the antidote used for magnesium sulfate toxicity; labetalol (Normodyne) is an antihyperten- sive medication. C. Pitocin (oxytocin)

A pregnant woman at 32 weeks of gestation comes to the emergency department because she has begun to experi- ence bright red vaginal bleeding. She reports that she has no pain. The admission nurse suspects that the woman is experiencing: A. Abruptio placentae B. Disseminated intravascular coagulation C. Placenta previa D. Preterm labor

Choice C is correct; the clinical manifestations of placenta previa are described; bleeding and clots with abdominal pain and uterine tenderness are characteristic of abruptio placentae; massive bleeding from many sites is associated with DIC; bleeding is not an expected sign of preterm labor. C. Placenta previa

A laboring woman's vaginal examination reveals the following: 3 cm, 50%, LSA, 0. The nurse caring for this woman should: A. Place the ultrasound transducer in the left lower quadrant of the woman's abdomen B. Recognize that passage of meconium would be a definitive sign of fetal distress C. Expect the progress of fetal descent to be slower than usual D. Assist the woman into a knee-chest position for each contraction

Choice C is correct; the presentation of this fetus is breech; the soft buttocks are a less efficient dilating wedge than the fetal head; therefore labor may be slower; the ultrasound transducer should be placed to the left of the umbilicus at a level at or above it; pas- sage of meconium is an expected finding as a result of pressure on the abdomen during descent; knee- chest position would be used for occipito posterior positions. C. Expect the progress of fetal descent to be slower than usual

When caring for a woman with preeclampsia without severe features, it is critical that during assessment the nurse be alert for signs of progress to preeclampsia with severe features. Progress to preeclampsia with severe features is indicated by which of the following assessment findings? A. Serum creatinine 0.9 mg/dL B. Platelet count of 180,000/mm3 C. Positive ankle clonus response with DTRs 4+ bilaterally D. Blood pressure of 150/88 and 154/96 mm Hg, 6 hours apart

Choice C is correct; with severe preeclampsia the DTRs should be 31 with positive ankle clonus indicating increased cerebral involvement, the BP should be .160/110 mm Hg, with a platelet count of ,100,000/mm3, which reflects thrombocytopenia; serum creatinine would be greater than 1.1 mg/dL. C. Positive ankle clonus response with DTRs 41 bilaterally

A nulliparous woman experiencing a postterm pregnancy is admitted for labor induction. Assessment reveals a Bishop score of 9. The nurse should: A. Call the woman's primary health care provider to order a cervical ripening agent B. Mix 20 units of oxytocin (Pitocin) in 500 mL of 5% glucose in water C. Piggyback the oxytocin solution into the port nearest the drip chamber of the primary IV tubing D. Begin the infusion at a rate of 1 milliunit/minute as determined by the induction protocol

Choice D is correct; a Bishop score of 9 indicates that the cervix is already sufficiently ripe for successful induction; it is currently recommended that 30 units of oxytocin (Pitocin) be mixed in 500 mL of an electrolyte solution such as Ringers lactate; the oxytocin solution is piggybacked at the proximal port (port nearest the insertion site). D. Begin the infusion at a rate of 1 milliunit/minute as determined by the induction protocol

A 34-year-old woman at 36 weeks of gestation has been scheduled for a biophysical profile. She asks the nurse why the test needs to be performed. The nurse tells her that the test is performed because it: A. Determines how well her baby will breathe after it is born B. Evaluates the response of her baby's heart to uterine contractions C. Measures her baby's head and length D. Observes her baby's activities in utero to ensure that her baby is getting enough oxygen"

Choice D is correct; an amniocentesis with analysis of amniotic fluid for the lecithin/sphingomyelin (L/S) ratio and presence of phosphatidylglycerol (Pg) is used to determine pulmonary maturity; choice B re- fers to a contraction stress test; choice C refers to serial measurements of fetal growth using ultrasound." D. Observes her baby's activities in utero to ensure that her baby is getting enough oxygen"

A nurse is caring for a pregnant woman at 30 weeks of gestation in preterm labor. The woman's physician orders betamethasone 12 mg IM for two doses, with the first dose to begin at 11 am. In implementing this order the nurse should: A. Consult the physician, because the dose is too high B. Explain to the woman that this medication will reduce her heart rate and help her to breathe easier C. Prepare to administer the medication intravenously between contractions D. Schedule the second dose for 11 am on the next day

Choice D is correct; the dosage is correct at 12 mg 3 2 doses; this medication will stimulate her baby's lungs to produce surfactant and help the baby to breathe more easily should birth occur; dosages should be spaced 24 hours apart; therefore the sec- ond dose should be given at 11 am on the next day; this medication is administered intramuscularly. D. Schedule the second dose for 11 am on the next day

A pregestational diabetic woman at 20 weeks of gestation exhibits the following: thirst, nausea and vomiting, abdominal pain, drowsiness, and increased urination. Her skin is flushed and dry and her breathing is rapid, with a fruity odor. A priority nursing action when caring for this woman is to: A. Provide the woman with a simple carbohydrate immediately B. Request an order for an antiemetic C. Assist the woman into a lateral position to rest D. Administer insulin according to the woman's blood glucose level"

Choice D is correct; the woman is exhibiting signs of DKA; insulin is the required treatment, with the dos- age dependent on blood glucose level; IV fluids may also be required; choice A is the treatment for hypo- glycemia; although they may increase the woman's comfort, choices B and C are not the priority. D. Administer insulin according to the woman's blood glucose level

Specific guidelines should be followed when planning a diet with a pregestational diabetic woman (BMI 24) to ensure a euglycemic state. Which dietary practices does the woman need to modify? (Circle all that apply.) A. Follows a diet that reflects 45 kcal per kg daily based on her preconception BMI B. Eats three meals a day along with a midmorning, a mid afternoon, and a bedtime snack C. Drinks a cup of tea and a piece of dry toast as her bedtime snack D. Divides her daily carbohydrate intake as 50% simple carbohydrates and 50% complex carbohydrates E. Maintains a fat intake of approximately 25% of the total daily kcal recommendation F. Monitors the appropriateness of her nutritional intake by checking her blood glucose levels before and after meals"

Choices A, C, and D are correct; based on a normal BMI (19.8 to 26), calories should be 30 to 35 kcal/kg each day; the bedtime snack needs to be substantial to prevent starvation ketoacidosis; simple carbohydrates should be avoided. A. Follows a diet that reflects 45 kcal per kg daily based on her preconception BMI C. Drinks a cup of tea and a piece of dry toast as her bedtime snack D. Divides her daily carbohydrate intake as 50% simple carbohydrates and 50% complex carbohydrates

A woman at 27 weeks of gestation experiences some mild uterine cramping. Which actions should she take? (Circle all that apply.) A. Empty her bladder. B. Call her nurse-midwife immediately. C. Relax in a chair. D. Drink two to three glasses of water or juice. E. Palpate her uterus for 1 hour. F. Resume the activity she was doing if the cramping subsides.

Choices A, D, and E are correct; see Teaching for Self-Management—What to Do If Symptoms of Preterm Labor Occur box. A. Empty her bladder. D. Drink two to three glasses of water or juice. E. Palpate her uterus for 1 hour.

A woman has been diagnosed with preeclampsia without severe features and will be treated at home. In teaching this woman about her treatment regimen for preeclampsia, the nurse should tell her to: (Circle all that apply.) A. Follow a low-salt diet B. Use a dipstick to check a clean catch specimen of her urine for protein C. Maintain a fluid intake of six to eight 8-ounce glasses of water each day D. Increase the roughage in her diet E. Perform gentle range-of-motion exercises of her upper and lower extremities F. Ask her friends to avoid visiting or calling her because she needs to rest

Choices B, C, D, and E are correct; a clean catch midstream urine specimen should be used to assess urine for protein using a dipstick; fluid intake should be six to eight 8-ounce glasses a day along with roughage to prevent constipation; gentle exercise improves circulation and helps to preserve muscle tone and a sense of well-being; no sodium restriction is required except for limiting excessively salty foods; diversional activities, including contact with friends, will decrease boredom and stress. B. Use a dipstick to check a clean catch specimen of her urine for protein C. Maintain a fluid intake of six to eight 8-ounce glasses of water each day D. Increase the roughage in her diet E. Perform gentle range-of-motion exercises of her upper and lower extremities

When assessing a pregnant woman, the nurse is alert for factors associated with preterm labor. Which factor, if exhibited by this woman, increases her risk for spontaneous preterm labor and birth? (Circle all that apply.) A. Caucasian race B. Obstetric history of 3-0-2-0-1 C. History of bleeding at 20 weeks D. Currently being treated for second bladder infection in 2 months E. Employed as a nurse in a trauma intensive care unit (ICU) F. Body mass index (BMI) of 22 and height of 158 cm

Choices B, C, D, and E are correct; women who are underweight or overweight/obese; have high-stress jobs; are members of the non-Hispanic African- American race; or have a history of preterm birth, multiple miscarriages, infections of the genitouri- nary tract (including UTIs) and reproductive tract (such as bacterial vaginosis), and bleeding in the second trimester are at increased risk for preterm labor and birth. B. Obstetric history of 3-0-2-0-1 C. History of bleeding at 20 weeks D. Currently being treated for second bladder infection in 2 months E. Employed as a nurse in a trauma intensive care unit (ICU)

A woman diagnosed with an ectopic pregnancy is to receive methotrexate. The nurse should explain to the woman that: (Circle all that apply.) A. Methotrexate is an analgesic that will relieve the dull abdominal pain she is experiencing. B. She should double-flush the toilet with the lid down for 72 hours after receiving methotrexate. C. She will receive the medication intramuscularly. D. She must stop taking folic acid supplements as long as she is on methotrexate. E. Her partner should use a condom during intercourse. F. She must return weekly for a measurement of her progesterone level to determine if the methotrexate therapy has been effective.

Choices B, C, and D are correct; methotrexate destroys rapidly dividing cells, in this case the fetus and placenta, to avoid rupture of tube and need for surgery; folic acid increases the risk for side effects with this medication; the woman should not put any- thing into her vagina; she needs to return to check her hCG level. B. She should double-flush the toilet with the lid down for 72 hours after receiving methotrexate. C. She will receive the medication intramuscularly. D. She must stop taking folic acid supplements as long as she is on methotrexate.

An obese pregnant woman with gestational diabetes is learning self-injection of insulin. While evaluating the woman's technique for self-injection of rapid-acting and NPH insulin, the nurse evaluates that the woman under- stands the instructions when she: (Circle all that apply.) A. Washes her hands and puts on a pair of clean gloves B. Gently rotates the NPH insulin vial to fully mix the insulin C. Draws the NPH insulin into her syringe first when mixing it with regular insulin D. Spreads her skin taut and punctures the skin at a 90-degree angle E. Cleanses the top of each insulin vial thoroughly with warm water and soap F. Wipes the injection site gently with alcohol, waiting until it dries to administer the insulin"

Choices B, D, and F are correct; washing hands is important but gloves are not necessary for self-injec- tion; vial should be rotated gently, not shaken; regu- lar insulin should be drawn into the syringe first; because she is obese, a 90-degree angle with skin taut is recommended; vial tops should be cleansed with alcohol prior to each use. B. Gently rotates the NPH insulin vial to fully mix the insulin D. Spreads her skin taut and punctures the skin at a 90-degree angle F. Wipes the injection site gently with alcohol, waiting until it dries to administer the insulin

A woman with preeclampsia is admitted to the hospital and her primary health care provider has ordered that an infusion of magnesium sulfate be started. In implementing this order, the nurse should: (Circle all that apply.) A. Prepare a solution of 20 g of magnesium sulfate in 100 mL of 5% glucose in water B. Monitor maternal vital signs, fetal heart rate (FHR) patterns, and uterine contractions every hour C. Expect the maintenance dose to be approximately 1 to 3 g/hour D. Administer a loading dose of 4 to 6 g over 15 to 30 minutes E. Prepare to administer hydralazine (Apresoline) if signs of magnesium toxicity occur F. Report a respiratory rate of less than 12 breaths/minute to the primary health care provider immediately

Choices C, D, and F are correct; a respiratory rate of less than 12 breaths/minute indicates central nervous system (CNS) depression caused by the magnesium sulfate; the solution should be 40 g in 1000 mL of Ringer lactate; assessment should occur every 15 to 30 minutes and the maintenance dose should be 1 to 3 g/hour; calcium gluconate is the antidote for mag- nesium sulfate toxicity. C. Expect the maintenance dose to be approximately 1 to 3 g/hour D. Administer a loading dose of 4 to 6 g over 15 to 30 minutes F. Report a respiratory rate of less than 12 breaths/minute to the primary health care provider immediately

Which of the following presentations is associated with early pregnancy loss occurring in less than 12 weeks gestation? Select all that apply. Chromosomal abnormalities Infection Cystitis Antiphospholipid syndrome Hypothyroidism Caffeine use

Chromosomal abnormalities Antiphospholipid syndrome Hypothyroidism Rationale Fifty percent of early pregnancy losses result from genetic abnormalities. Antiphospholipid syndrome is associated with early pregnancy loss. Hypothyroidism is associated with early pregnancy loss. Infection is not a likely source of early pregnancy loss. Cystitis in not associated with early pregnancy loss. Caffeine use is associated with second-trimester losses because of maternal behavior.

What are the possible causes of miscarriage during early pregnancy? Select all that apply. > Premature dilation of cervix Chromosomal abnormalities Endocrine imbalance Hypothyroidism Antiphospholipid antibodies

Chromosomal abnormalities Endocrine imbalance Hypothyroidism Antiphospholipid antibodies Rationale Chromosomal abnormalities account for 50% of all early pregnancy losses. Endocrine imbalance is caused by luteal phase defects, hypothyroidism, and diabetes mellitus in pregnant patients and results in miscarriage. Antiphospholipid antibodies also increase the chances of miscarriage in pregnant patients. Premature dilation of the cervix may cause a second-trimester loss, and is usually seen in patients between 12 and 20 weeks of gestation.

Which of the following presentations is associated with early pregnancy loss, occurring in less than 12 weeks gestation? Select all that apply. Chromosomal abnormalities Infection Cystitis Antiphospholipid syndrome Hypothyroidism Caffeine use

Chromosomal abnormalities 50% of early pregnancy loss results from genetic abnormalities. Hypothyroidism and antiphospholipid syndrome are associated with early pregnancy loss. Caffeine use is associated with second-trimester losses as a result of maternal behavior. Infection is not a likely source of early pregnancy loss. Cystitis in not associated with early pregnancy loss. Antiphospholipid syndrome 50% of early pregnancy loss results from genetic abnormalities. Hypothyroidism and antiphospholipid syndrome are associated with early pregnancy loss. Caffeine use is associated with second-trimester losses as a result of maternal behavior. Infection is not a likely source of early pregnancy loss. Cystitis in not associated with early pregnancy loss. Hypothyroidism 50% of early pregnancy loss results from genetic abnormalities. Hypothyroidism and antiphospholipid syndrome are associated with early pregnancy loss. Caffeine use is associated with second-trimester losses as a result of maternal behavior. Infection is not a likely source of early pregnancy loss. Cystitis in not associated with early pregnancy loss

Which hypertensive disorders can occur during pregnancy? Select all that apply. Chronic hypertension Preeclampsia-eclampsia Hyperemesis gravidarum Gestational hypertension Gestational trophoblastic disease

Chronic hypertension Preeclampsia-eclampsia Gestational hypertension Rationale Chronic hypertension refers to hypertension that developed in the pregnant patient before 20 weeks of gestation. Preeclampsia refers to hypertension and proteinuria that develops 20 weeks after gestation. Eclampsia is the onset of seizure activity in a pregnant patient with preeclampsia. Gestational hypertension is the onset of hypertension after 20 weeks of gestation. Gestational trophoblastic disease and hyperemesis gravidarum are not hypertensive disorders. Gestational trophoblastic disease refers to a disorder without a viable fetus that is caused by abnormal fertilization. Hyperemesis gravidarum is excessive vomiting during pregnancy that may result in weight loss and electrolyte imbalance.

What are the risk factors associated with preterm premature rupture of membranes (PROM)? Select all that apply. Preeclampsia Long cervical length Cigarette smoking Urinary tract infection Uterine overdistention

Cigarette smoking Urinary tract infection Uterine overdistention Rationale Conditions such as smoking, urinary tract infection, and uterine overdistention may cause early rupturing of membranes in a pregnant patient. Therefore, these factors are considered risk factors associated with preterm PROM. Preeclampsia is the common cause of indicated preterm birth and is not associated with preterm PROM. Short cervical length would increase the risk of PROM and may not be observed in clients with long cervical length. Non-Caucasian women are at a higher risk for spontaneous preterm birth than Caucasian women.

White's Classification of Diabetes in Pregnancy- Gestational Diabetes

Class A1- woman has 2+ abnormal values on OGTT with normal fasting blood sugar. Blood glucose levels are diet controlled Class A2- woman was not known to have diabetes before pregnancy but requires medication for blood glucose control

White's Classification of Pregestational Diabetes

Class B- onset of disease occurs after age 20 and duration of illness <10 years Class C- onset of disease occurs between 10-19 years of age or duration of illness for 10 to 19 years or both Class D- onset of disease occurs at <10 years of age or duration of illness >20 years or both Class F- client has developed diabetic nephropathy Class R- client has developed retinitis proliferans Class T- client has had a renal transplant

Which indicator would lead the nurse to suspect that the postpartum client experiencing hemorrhagic shock is getting worse? Restoration of blood pressure levels to normal range Capillary refill brisk Client complains of headache and reaction time increases to asking questions Client states she sees "stars"

Client complains of headache and reaction time increases to asking questions Rationale A client complaining of a headache with an increased reaction (response) time indicates that cerebral hypoxia is getting worse. Blood pressure returning to normal range would indicate resolving symptoms. Brisk capillary refill is a normal finding. A client seeing "stars" may occur early on in decreased blood flow states.

In which situations would the use of methergine or prostaglandin be contraindicated even if the client were experiencing a postpartum significant bleed? Delivered twin pregnancies. Client's blood pressure postpartum is 180/90 Client has a history of asthma Client has a mitral valve prolapse Client is a grand multipara

Client's blood pressure postpartum is 180/90 Client has a history of asthma Client has a mitral valve prolapse Rationale If a client is hypertensive, then these medications would not be used. If a client has a history of asthma, then prostaglandin would not be used. If the client had cardiovascular disease, then these medications would not be used. Twin pregnancies successfully delivered would not be a contraindication to the use of these medications. The fact that a client is a grand multipara would not be a contraindication to the use of these medications.

The postpartum client who delivered a day ago reports, "I feel tired very often and experience pain in my lower abdomen." Upon further observation, the nurse finds that the client also has profuse foul-smelling vaginal discharge and an increased pulse. Which medication would be added to the client's prescription? Warfarin (Coumadin) Clindamycin (Cleocin) Misoprostol (Cytotec) Ergonovine (Ergotrate)

Clindamycin (Cleocin) Rationale Endometrial infection is characterized by tiredness and lower abdominal pain, profuse foul-smelling discharge, and increased pulse. Clindamycin (Cleocin) is an antibiotic used to treat endometrial infections. Warfarin (Coumadin) is prescribed to postpartum clients with thromboembolic disorders. Misoprostol (Cytotec) is prescribed to clients with excessive bleeding caused by uterine atony. Ergonovine (Ergotrate) is prescribed to treat subinvolution of the uterus.

During the assessment of a postpartum client, the nurse finds that she has endometritis. Which medication should be involved in the treatment plan of this client? Clindamycin (Cleocin) Misoprostol (Cytotec) Ergonovine (Ergotrate) Methylergonovine (Methergine)

Clindamycin (Cleocin) Rationale Endometritis is a common postpartum infection. It usually begins as a localized infection at the placental site and spreads to the entire endometrium. Endometritis is usually managed by giving the patient a broad-spectrum antibiotic drug, like Clindamycin (Cleocin). Therefore clindamycin (Cleocin) should be involved in the treatment plan of the client for management of endometritis. Misoprostol (Cytotec), ergonovine (Ergotrate), and methylergonovine (Methergine) are uterotonic drugs used to manage postpartum hemorrhage (PPH) caused by uterine atony.

A client had a previous cesarean birth. What are the criteria in order to try having a vaginal birth during the second pregnancy? Select all that apply. A history of postpartum hemorrhage A previous classical vertical incision Clinically adequate pelvis Previous low transverse incision No history of uterine rupture

Clinically adequate pelvis Previous low transverse incision No history of uterine rupture Rationale A vaginal birth is possible after a previous caesarean delivery if the pelvis is found to be adequate to provide room for childbirth. A previous low transverse incision poses less risk of rupture and a vaginal delivery may be possible. A client with no history of uterine rupture would have less risk of uterine rupture during the vaginal delivery. A history of postpartum hemorrhage may not affect the risk associated with a second vaginal delivery in women with a history of first caesarean delivery. A previous vertical incision on the uterus increases the risk of uterine rupture

The nurse is preparing to perform a fetal fibronectin test for a pregnant client. Which intervention should the nurse perform to collect the sample for the test? Take a blood sample from the forearm. Take a sample of patient's amniotic fluid. Ask the patient to provide a urine sample. Collect the vaginal secretions using a swab.

Collect the vaginal secretions using a swab. Rationale The fetal fibronectin test is conducted to assess whether a client is at risk for preterm labor. Fetal fibronectin is a glycoprotein found in the vaginal secretions during early and late pregnancy. In order to conduct the test the nurse should collect the vaginal secretions using a swab and send it for analysis. Urine, blood, and amniotic fluid are not collected for a fetal fibronectin test, because they may not contain adequate glycoprotein levels.

Uterine Rupture

Complete nonsurgical disruption of all of the uterine layers -rare but life-threatening -major risk factor- scarred uterus as a result of previous cesarean birth or other uterine surgery - occurs most often with a previous classic incision

The nurse is assessing a pregnant client who takes nifedipine (Adalat). What instruction does the nurse provide to ensure the client's safety? Consume adequate fluids. Take medication on an empty stomach. Avoid eating foods high in carbohydrates. Administer medication under medical supervision.

Consume adequate fluids. Rationale The potent vasodilator effect of nifedipine (Adalat) causes variations in the blood pressure of a pregnant client. So, the nurse advises the client to consume adequate fluids to maintain blood pressure. Nifedipine (Adalat) is best tolerated when taken with food. Hence, the nurse does not ask the client to take the medication on empty stomach. Clients on glucocorticoids are advised to avoid carbohydrate-rich foods, because glucocorticoids increase glucose levels in the body, and are unrelated to nifedipine (Adalat). Nifedipine (Adalat) is taken orally and does not require medical supervision to administer it.

The nurse is caring for a patient in labor and has identified a severe deceleration in fetal heart rate. The health care provider has subsequently diagnosed a cord prolapse. Which actions should the nurse perform? Select all that apply. Continue assessing fetal heart rate. Assess the patient's level of anxiety. Ensure that the patient's hips are elevated above the head. Palpate the prolapsed umbilical cord to verify presence of pulse. Evaluate the family's understanding of the need for immediate delivery.

Continue assessing fetal heart rate. Assessing fetal heart rate allows the nurse to ensure that pressure on the cord is minimized. Correct Assess the patient's level of anxiety. The nurse should assess the patient's anxiety and provide education to address the patient's concerns. Correct Ensure that the patient's hips are elevated above the head. Ensuring that the patient's hips are elevated above the head helps maintain relief of cord compression and ensure fetal oxygenation. Evaluate the family's understanding of the need for immediate delivery. The nurse should include the family in education as much as possible and evaluate the family's understanding of the patient's condition.

The nurse observes that eclampsia has developed in a pregnant client after starting magnesium sulfate therapy. What action does the nurse take? Continue to administer magnesium sulphate per protocol Administer regional anesthesia to the client Administer calcium gluconate simultaneously Prepare the clientfor immediate caesarean birth

Continue to administer magnesium sulphate per protocol Rationale The nurse needs to administer additional magnesium sulfate, because it will help in treating eclamptic seizures and preventing repeated seizures. Regional anesthesia is not recommended for eclamptic clients due to the risk of maternal complications. Calcium gluconate is administered as an antidote for magnesium toxicity. Immediate cesarean birth is a priority when the client is in shock after a trauma.

Prolapsed Umbilical cord

Cord is displaced b/w the presenting part and amnion or protruding thru the cervix, causing compression of the cord and compromising fetal circulation. S/S client has feeling something is coming thru vagina opening, cord visible or palpable, FHR irregular and slow., FHR monitor shows variable decelerations or bradycardia, If FHR severe, violent fetal activity may occur then cease.

A 2-day-old neonate has a serum bilirubin level of 18.9 mg/dL. Double bank phototherapy is ordered. Which is the most important intervention for the nurse when caring for the neonate? Placing the neonate naked under the phototherapy Supplementing with formula after each breastfeeding Weighing the newborn before and after every breastfeeding Covering the eyes with an eye mask when under phototherapy lights

Covering the eyes with an eye mask when under phototherapy lights Covering the eyes with an eye mask is the most important intervention when initiating phototherapy. Phototherapy lights can injure the neonate's eyes if left uncovered.

The nurse is caring for a woman who is at 24 weeks of gestation with suspected severe preeclampsia. Which signs and symptoms should the nurse expect to observe? Select all that apply. Decreased urinary output and irritability Transient headache and +1 proteinuria Ankle clonus and epigastric pain Platelet count of less than 100,000/mm 3 and visual problems Seizure activity and hypotension

Decreased urinary output and irritability Ankle clonus and epigastric pain Platelet count of less than 100,000/mm 3 and visual problems Rationale Decreased urinary output and irritability are signs of severe eclampsia. Ankle clonus and epigastric pain are signs of severe eclampsia. Platelet count of less than 100,000/mm 3 and visual problems are signs of severe preeclampsia. A transient headache and +1 proteinuria are signs of preeclampsia and should be monitored. Seizure activity and hyperreflexia are signs of eclampsia.

In planning for the care of a 30-year-old woman with pregestational diabetes, the nurse recognizes that which is the most important factor affecting pregnancy outcome? Mother's age Number of years since diabetes was diagnosed Amount of insulin required prenatally Degree of glycemic control during pregnancy

Degree of glycemic control during pregnancy Rationale Women with excellent glucose control and no blood vessel disease should have good pregnancy outcomes. Although advanced maternal age may pose some health risks, for the woman with pregestational diabetes the most important factor remains the degree of glycemic control during pregnancy. The number of years since diagnosis is not as relevant to outcomes as the degree of glycemic control. The key to reducing risk in the pregestational diabetic woman is not the amount of insulin required but rather the level of glycemic control.

In planning for the care of a 30-year-old woman with pregestational diabetes, the nurse recognizes that the most important factor affecting pregnancy outcome is the: Mother's age. Number of years since diabetes was diagnosed. Amount of insulin required prenatally. Degree of glycemic control during pregnancy

Degree of glycemic control during pregnancy. Women with excellent glucose control and no blood vessel disease should have good pregnancy outcomes. Although advanced maternal age may pose some health risks, the most important factor for the woman with pregestational diabetes remains the degree of glycemic control during pregnancy. The number of years since diagnosis and the amount of insulin required are not as relevant to outcomes as the degree of glycemic control.

Match the maternal effect of altered glucose metabolism with its description

Delayed or difficult birth of fetal shoulders after head is born Dystocia Excess volume of amniotic fluid Hydramnios Body uses fat for energy instead of glucose Ketoacidosis

What is the treatment that should be considered first for the woman with von Willebrand disease who experiences a postpartum hemorrhage? Cryoprecipitate Factor VIII and vWf Desmopressin Hemabate

Desmopressin Rationale Cryoprecipitate may be used; however, because of the risk of possible donor viruses, other modalities are considered safer. Treatment with plasma products, such as factor VIII and vWf, is an acceptable option for this client. Because of the repeated exposure to donor blood products and possible viruses, this is not the initial treatment of choice. Desmopressin is the primary treatment of choice. This hormone can be administered orally, nasally, and intravenously. This medication promotes the release of factor VIII and vWf from storage. Although the administration of this prostaglandin is known to promote contraction of the uterus during postpartum hemorrhage, it is not effective for the client who presents with a bleeding disorder.

What is the treatment that should be considered first for the client with von Willebrand disease who experiences a postpartum hemorrhage? Cryoprecipitate Factor VIII and vWf Desmopressin Hemabate

Desmopressin Rationale Desmopressin is the primary treatment of choice. This hormone can be administered orally, nasally, and intravenously. This medication promotes the release of factor VIII and vWf from storage. Cryoprecipitate may be used; however, because of the risk of possible donor viruses, other modalities are considered safer. Treatment with plasma products such as factor VIII and vWf is an acceptable option for this client. Because of the repeated exposure to donor blood products and possible viruses, this is not the initial treatment of choice. Although the administration of this prostaglandin is known to promote contraction of the uterus during postpartum hemorrhage, it is not effective for the client who presents with a bleeding disorder.

To major goals of antepartal assessment during the third trimester

Determine if the uterine environment continues to be supportive and determine the timing of childbirth for women at risk for uteroplacental insufficiency

A client reports mild vaginal bleeding, pain, and cramping in her lower abdomen at 6 weeks of gestation. Upon performing a pelvic examination, the nurse finds that the client's cervical os is closed. What is the priority intervention in this condition? Administer intravenous fluids to the client. Administer methylcarboprost tromethamine (Hemabate) to the client. Determine the client's human chorionic gonadotropin and progesterone levels. Prompt termination of pregnancy in the patient by dilation and curettage method.

Determine the client's human chorionic gonadotropin and progesterone levels. Rationale Mild vaginal bleeding, pain in the lower abdomen, and mild uterine cramps with a closed cervical os are the symptoms of threatened miscarriage. Therefore, the nurse should determine the human chorionic gonadotropin and progesterone levels in the client to find whether the fetus is alive in the uterus. The nurse should administer intravenous fluids if the client has severe bleeding. The nurse should administer methylcarboprost tromethamine (Hemabate) to prevent excessive bleeding after miscarriage. In this case, the client does not have a complete miscarriage or excessive bleeding; therefore, the nurse would not administer this medication to the client. Dilation and curettage is a surgical method to terminate a pregnancy. It is performed on a client with inevitable miscarriage, but not threatened miscarriage.

What is gestational diabetes? Diabetes that accompanies hypertension Diabetes that begins abruptly at a young age Diabetes caused by absolute insulin deficiency Diabetes that is diagnosed during pregnancy

Diabetes that is diagnosed during pregnancy Rationale Gestational diabetes is a type of diabetes that did not exist before it was diagnosed during pregnancy. Hypertension is a risk factor for type 2 diabetes. Type 1 diabetes usually begins abruptly at a young age. People with type 1 diabetes have absolute insulin deficiency resulting from the destruction of beta cells in the pancreas.

A pregnant woman at 28 weeks of gestation has been diagnosed with gestational diabetes. Of what should the nurse be aware regarding this? Oral hypoglycemic agents can be used if the woman is reluctant to give herself insulin Dietary modifications and insulin are both required for adequate treatment Glucose levels are monitored by testing urine four times a day and at bedtime Dietary management involves distributing nutrient requirements over three meals and two or three snacks

Dietary management involves distributing nutrient requirements over three meals and two or three snacks Rationale Small frequent meals over a 24-hour period help decrease the risk for hypoglycemia and ketoacidosis. Oral hypoglycemic agents can be harmful to the fetus and less effective than insulin in achieving tight glucose control. In some women gestational diabetes can be controlled with dietary modifications alone. Blood, not urine, glucose levels are monitored several times a day. Urine is tested for ketone content; results should be negative.

A pregnant woman at 28 weeks of gestation has been diagnosed with gestational diabetes. The nurse caring for this woman understands what? Oral hypoglycemic agents can be used if the woman is reluctant to give herself insulin. Dietary modifications and insulin are both required for adequate treatment. Glucose levels are monitored by testing urine 4 times a day and at bedtime. Dietary management involves distributing nutrient requirements over three meals and two or three snacks.

Dietary management involves distributing nutrient requirements over three meals and two or three snacks. Rationale Small frequent meals over a 24-hour period help decrease the risk for hypoglycemia and ketoacidosis. Oral hypoglycemic agents can be harmful to the fetus and less effective than insulin in achieving tight glucose control. In some women gestational diabetes can be controlled with dietary modifications alone. Blood, not urine, glucose levels are monitored several times a day. Urine is tested for ketone content; results should be negative.

A pregnant woman at 28 weeks of gestation has been diagnosed with gestational diabetes. The nurse caring for this client understands that: Oral hypoglycemic agents can be used if the woman is reluctant to give herself insulin. Dietary modifications and insulin are both required for adequate treatment. Glucose levels are monitored by testing urine four times a day and at bedtime. Dietary management involves distributing nutrient requirements over three meals and two or three snacks

Dietary management involves distributing nutrient requirements over three meals and two or three snacks. Small frequent meals over a 24-hour period help decrease the risk for hypoglycemia and ketoacidosis. In some women gestational diabetes can be controlled with dietary modifications alone. Blood, not urine, glucose levels are monitored several times a day. Urine is tested for ketone content; results should be negative.Oral hypoglycemic agents can be harmful to the fetus and less effective than insulin in achieving tight glucose control.

A nurse is working with a diabetic client who recently found out she is pregnant. In coordinating an interdisciplinary team to help manage the client throughout the pregnancy, the nurse would include whom? Family practice physician Dietician Perinatologist Occupational therapist Nephrologist Speech therapist

Dietician Perinatologist Nephrologist Rationale A dietician would be included in the interdisciplinary care team to help the client with dietary planning. A perinatologist would be included in the interdisciplinary care team to take care of both the mother and the fetus. A nephrologist would be included in the interdisciplinary care team to monitor renal function. A family practice physician would not be included but, rather, an internal medicine practitioner would be a member of the interdisciplinary care team. There is no need for an occupational therapist to be included unless there are other issues presented. There is no need for a speech therapist to be included unless there are other issues presented.

A nurse is working with a diabetic patient who recently found out she is pregnant. In coordinating an interdisciplinary team to help manage the patient throughout the pregnancy, the nurse would include: Select all that apply. Family practice physician Dietician Perinatologist Occupational therapist Nephrologist Speech therapist

Dietician An internal medicine practitioner rather than family practice physician would be included on the interdisciplinary care team. A dietician would be included to help the patient with dietary planning, a perinatologist to take care of the maternal-fetal unit, and a nephrologist to monitor renal function. There is no need for an occupational therapist or a speech therapist unless other issues arise. Perinatologist An internal medicine practitioner rather than family practice physician would be included on the interdisciplinary care team. A dietician would be included to help the patient with dietary planning, a perinatologist to take care of the maternal-fetal unit, and a nephrologist to monitor renal function. There is no need for an occupational therapist or a speech therapist unless other issues arise. Nephrologist An internal medicine practitioner rather than family practice physician would be included on the interdisciplinary care team. A dietician would be included to help the patient with dietary planning, a perinatologist to take care of the maternal-fetal unit, and a nephrologist to monitor renal function. There is no need for an occupational therapist or a speech therapist unless other issues arise.

Dystocia

Difficult labor that is prolonged or more painful Occurs because of problems caused by uterine contractions, fetus or bones and tissues of maternal pelvis. Fetus large, malpositioned or abnormal presentation. Hypertonic contractions- painful, frequently, uncoordinated Hypotonic contractions- short, irregular, weak, amniotomy and oxytocin tx. Can result in maternal dehydration, infection, fetal injury or death.

A client is administered magnesium sulfate (Epsom salts) as a part of tocolytic therapy. Which signs and symptoms should the nurse monitor in the client? Select all that apply. Diplopia Tremors Hot flushes Drowsiness Tachycardia

Diplopia Hot flushes Drowsiness Rationale Magnesium sulfate (Epsom salt) is a tocolytic agent that relaxes smooth muscles, including those of the uterus during preterm labor. Diplopia, hot flushes, and drowsiness are maternal adverse effects of magnesium sulfate (Epsom salt). Tremors and tachycardia are not associated with magnesium sulfate (Epsom salt).

The nurse is caring for a pregnant patient receiving intravenous (IV) oxytocin for induction of labor. The most recent fetal heart rates indicate fetal bradycardia unresponsive to repositioning. Which order would the nurse anticipate? Discontinue the oxytocin infusion Increase the rate of the oxytocin infusion Administer 5% dextrose with the oxytocin infusion Administer supplementary oxygen by simple face mask

Discontinue the oxytocin infusion Fetal bradycardia (fetal heart rate <110 beats/min lasting for more than 10 minutes) is a sign of potential uterine tachysystole and reduced placental exchange caused by oxytocin administration. Therefore, discontinuing the IV oxytocin infusion is the next step taken to relax the uterus, thus increasing blood flow to the fetus.

A pregnant female was diagnosed with type 1 diabetes mellitus prior to becoming pregnant. Which pieces of information should be included in patient teaching? Select all that apply. Discuss the increased risk for ketoacidosis Explain that insulin requirement may increase Demonstrate how to measure, mix, and inject insulin Explain that frequent prenatal visits may be necessary Explain that dietary requirements for glucose will increase

Discuss the increased risk for ketoacidosis Ketoacidosis is more likely in pregnant females if an appropriate diet is not maintained. Correct Explain that insulin requirement may increase Insulin requirements for pregnant women with diabetes may increase, depending on the trimester. Correct Demonstrate how to measure, mix, and inject insulin The nurse should ensure the woman is able to mix, measure, and administer insulin. Correct Explain that frequent prenatal visits may be necessary The health care provider may request more frequent visits to assess both mother and baby.

DIC

Disseminated Intravascular Coagulation -diffuse clotting that consumes large amounts of clotting factors and then causes widespread bleeding -never a primary diagnosis

What are the manifestations associated with hypoglycemia? Select all that apply. Dizziness Fruity breath Blurred vision Excessive hunger Presence of acetone in urine

Dizziness Blurred vision Excessive hunger Rationale Hypoglycemia refers to decreased blood sugar levels. Decreased availability of glucose impairs brain function, which results in dizziness and blurred vision. Decreased glucose levels stimulate the satiety center of the brain, which results in excessive hunger. Fruity breath and the presence of acetone in urine result from increased ketone levels in the blood. This complication may occur when the blood glucose increases over 300 mg/dl in the non-pregnant client and 200 mg/dl in the pregnant client.

The nurse sees a woman for the first time when she is 30 weeks pregnant. The woman has smoked throughout the pregnancy, and fundal height measurements now are suggestive of growth restriction in the fetus. In addition to ultrasound to measure fetal size, what tool is useful in confirming the diagnosis? Doppler blood flow analysis Contraction stress test (CST) Amniocentesis Daily fetal movement counts

Doppler blood flow analysis Rationale Doppler blood flow analysis allows the examiner to study the blood flow noninvasively in the fetus and placenta. It is a helpful tool in the management of high risk pregnancies because of intrauterine growth restriction (IUGR), diabetes mellitus, multiple fetuses, or preterm labor. Because of the potential risk of inducing labor and causing fetal distress, a CST is not performed on a woman whose fetus is preterm. Indications for an amniocentesis include diagnosis of genetic disorders or congenital anomalies, assessment of pulmonary maturity, and the diagnosis of fetal hemolytic disease, not IUGR. Fetal kick count monitoring is performed to monitor the fetus in pregnancies complicated by conditions that may affect fetal oxygenation. Although this may be a useful tool at some point later in this woman's pregnancy, it is not used to diagnose IUGR.

The nurse sees a woman for the first time when she is 30 weeks pregnant. The woman has smoked throughout the pregnancy, and fundal height measurements now are suggestive of growth restriction in the fetus. In addition to ultrasound to measure fetal size, what would be another tool useful in confirming the diagnosis? Doppler blood flow analysis Contraction stress test (CST) Amniocentesis Daily fetal movement counts

Doppler blood flow analysis Rationale Doppler blood flow analysis allows the examiner to study the blood flow noninvasively in the fetus and the placenta. It is a helpful tool in the management of high risk pregnancies due to intrauterine growth restriction (IUGR), diabetes mellitus, multiple fetuses, or preterm labor. Because of the potential risk of inducing labor and causing fetal distress, a CST is not performed on a woman whose fetus is preterm. Indications for an amniocentesis include diagnosis of genetic disorders or congenital anomalies, assessment of pulmonary maturity, and the diagnosis of fetal hemolytic disease, but not IUGR. Fetal kick count monitoring is performed to monitor the fetus in pregnancies complicated by conditions that may affect fetal oxygenation. Although this may be a useful tool at some point later in this woman's pregnancy, it is not used to diagnose IUGR

The nurse sees a woman for the first time when she is 30 weeks pregnant. The woman has smoked throughout the pregnancy, and fundal height measurements now are suggestive of growth restriction in the fetus. In addition to ultrasound to measure fetal size, what would be another tool useful in confirming the diagnosis? Doppler blood flow analysis Contraction stress test (CST) Amniocentesis Daily fetal movement counts

Doppler blood flow analysis Doppler blood flow analysis allows the examiner to study the blood flow noninvasively in the fetus and the placenta. It is a helpful tool in the management of high risk pregnancy due to intrauterine growth restriction (IUGR), diabetes mellitus, multiple fetuses, or preterm labor. Because of the potential risk of inducing labor and causing fetal distress, a CST is not performed in a woman whose fetus is preterm. Indications for an amniocentesis include diagnosis of genetic disorders or congenital anomalies, assessment of pulmonary maturity, and the diagnosis of fetal hemolytic disease, not IUGR. Fetal kick count monitoring is performed to monitor the fetus in pregnancies complicated by conditions that may affect fetal oxygenation. Although it may be a useful tool at some point later in this woman's pregnancy, it is not used to diagnose IUGR.

magnesium sulfate

Dosage is initial bolus of 4 - 6 gm by IVPB over 15-20 min. Maintenance dose of 2-3 g/hr diluted in LR solution is administered by infusion pump at 2-3 g/hr. ( 40 g in 1000 ml LR) HR increases, Respirations slow down, relaxes smooth muscles potentiate - narcotics, CNS depressants, calcium channel blocker any women with hypertension with headache, blurred vision or severe hypertension - put on magnesium sulfate for seizure prophylaxis

The nurse is caring for a client in the first trimester of pregnancy. The client's laboratory reports indicate a reduction in the levels of pregnancy-associated placental protein (PAPP-A) and an elevation in the levels of human chorionic gonadotropin (hCG) and nuchal translucency (NT). Based on these findings, which condition does the nurse suspect in the fetus? Spina bifida Down syndrome Potter syndrome Fetal cardiac disease

Down syndrome Rationale Down syndrome is a chromosomal abnormality. It is characterized by decreased pregnancy-associated placental protein (PAPP-A) and elevated human chorionic gonadotropin (hCG) and nuchal translucency. It can be detected in the first trimester with multiple markers in the serum. Spina bifida is a neural tube defect, which is identified by amniocentesis. Potter syndrome is the typical appearance of the fetus as a result of oligohydramnios, which is detected by amniocentesis. PAPP-A, hCG, and NT do not identify the presence of fetal cardiac disease.

While reviewing the laboratory reports of 38-year-old pregnant client, the nurse finds that the lecithin/sphingomyelin (L/S) ratio is 2:1, amniotic fluid index is 12 cm, and inhibin A level is elevated. Which condition may the fetus be at risk for based on these finding s? Lung immaturity Down syndrome Potter syndrome Neural tube defect

Down syndrome Rationale Inhibin A is the hormone that is secreted by the placenta. Elevated levels of inhibin A indicate that the fetus has a chromosomal abnormality, which indicates a risk for Down syndrome. A L/S ration of 2:1 is a normal finding and does not indicate fetal lung immaturity. The normal amniotic fluid index is 10 to 25 cm. If the amniotic fluid index is more than 25, the fetus has a risk for Potter syndrome. If the amniotic fluid index is less than 5 cm, the fetus has a chromosomal abnormality and a risk for neural tube defect. In this case, the amniotic fluid index falls in the normal range, so neither of these conditions are suspected.

The nurse is assessing a client for gestational diabetes mellitus (GDM) using the oral glucose tolerance test (OGTT). What intervention by the nurse is appropriate while caring for this client? Teach the client to eat an unrestricted diet the day before the test. Instruct the client to avoid caffeine for 6 hours before the test. Draw blood for a fasting blood glucose level just before the test. Obtain the plasma glucose level an hour after a 50 g oral glucose load.

Draw blood for a fasting blood glucose level just before the test. Rationale The nurse must draw blood for a fasting blood glucose level just before the test begins. This is the first sample, after which blood is drawn 1, 2, and 3 hours after providing the glucose load. The nurse must teach the client to eat an unrestricted diet that includes at least 150 g of carbohydrates for at least 3 days before the test. The client must be instructed to avoid caffeine for 12 hours before the test because it increases glucose levels. The client is given a 100 g oral glucose load, and then the client's blood glucose levels are determined every hour for up to 3 hours. The plasma glucose level is obtained after a 50 g oral glucose load in the first step of screening for GDM.

The nurse is caring for a diabetic client who is pregnant. What does the nurse instruct the client about self-care during illness? Avoid insulin if your appetite is less than normal. Drink as much fluid as possible. Obtain as much rest as possible. Check your blood glucose levels at regular intervals. Seek treatment if your glucose level exceeds 250 mg/dl.

Drink as much fluid as possible. Obtain as much rest as possible. Check your blood glucose levels at regular intervals. Rationale The nurse should teach the client to drink as much fluid as possible to prevent dehydration. The client should get plenty of rest to speed up recovery from illness. Blood glucose levels should be monitored at regular intervals as a precautionary measure to identify hypoglycemia. The client should be taught to take insulin as per her regular schedule even if her appetite is less than normal, because insulin requirements increase during illness. The client should seek emergency treatment as soon as her glucose level exceeds 200 mg/dl.

If glucose tabs are not available and woman is experiencing hypoglycemia (<70 mg/dl), what should she do?

Eat 15 g of carbohydrates: -1/2 cup unsweetened orange juice -1/2 cup of regular (not diet) soda -5-6 hard candies -1 cup of skim milk

When a pregnant woman with diabetes experiences hypoglycemia while hospitalized, what should the nurse have the woman do? Eat a candy bar. Eat 5 or 6 hard candies or drink 8 oz of milk. Drink 4 oz of orange juice followed by 8 oz of milk. Drink 8 oz of orange juice with 2 teaspoons of sugar added.

Eat 5 or 6 hard candies or drink 8 oz of milk. Rationale Crackers provide carbohydrates in the form of polysaccharides. A candy bar provides only monosaccharides. Milk is a disaccharide and orange juice is a monosaccharide. This will provide an increase in blood sugar but will not sustain the level. Orange juice and sugar will increase the blood sugar, but not provide a slow-burning carbohydrate to sustain the blood sugar.

The nurse is caring for a pregnant client with gestational diabetes. What does the nurse teach the client about diet during pregnancy? Eat three meals a day with two or three snacks. Avoid meals or snacks just before bedtime. Use artificial sweeteners instead of sugar. Avoid foods that are high in dietary fiber.

Eat three meals a day with two or three snacks. Rationale The nurse should teach the client to distribute her daily required calories into three meals with two or three snacks. In order to prevent hypoglycemia, the client should eat meals on time and never skip meals. The client should consume a bedtime snack of at least 25 g of complex carbohydrate with some protein or fat to prevent hypoglycemia and starvation ketosis during the night. The nurse should teach the client to avoid the use of sweeteners that are nonnutritive and foods high in refined sugar. The client should eat foods that are high in dietary fiber.

A nurse is examining a client who has been admitted for possible ectopic pregnancy who is approximately 8 weeks pregnant. Which finding would be a priority concern? No FHT heard via Doppler Scant vaginal bleeding noted on peri pad Ecchymosis noted around umbilicus Blood pressure 100/80

Ecchymosis noted around umbilicus Rationale Ecchymosis around the umbilicus indicates Cullen's sign, which could represent a ruptured intrabdominal ectopic pregnancy. Because the client is most likely in the early stages of pregnancy, FHT would not be able to be auscultated at this time. Scant vaginal bleeding would not be a priority concern but should still be monitored by the nurse.

A nurse is examining a client who has been admitted for possible ectopic pregnancy who is approximately 8 weeks pregnant. Which finding would be a priority concern? No FHT heard via Doppler Scant vaginal bleeding noted on peri pad Ecchymosis noted around umbilicus Blood pressure 100/80

Ecchymosis noted around umbilicus Rationale Ecchymosis around the umbilicus indicates Cullen's sign, which could represent a ruptured intrabdominal ectopic pregnancy. Because the client is most likely in the early stages of pregnancy, FHT would not be able to be auscultated at this time. Scant vaginal bleeding would not be a priority concern but should still be monitored by the nurse.

A nurse is examining a patient who has been admitted for possible ectopic pregnancy who is approximately 8 weeks pregnant. Which finding would be a priority concern? No FHT heard via Doppler Scant vaginal bleeding noted on peri pad Ecchymosis noted around umbilicus Blood pressure 100/80

Ecchymosis noted around umbilicus Because this patient is most likely in the early stages of pregnancy, FHT would not be able to be auscultated at this time. Scant vaginal bleeding would not be a priority concern but should still be monitored by the nurse. Ecchymosis around the umbilicus indicates Cullen sign, which indicates hematoperitoneum, and may also develop in an undiagnosed, ruptured intraabdominal ectopic pregnancy.

Which condition is characterized by implantation of fertilized ovum outside the uterine cavity? Placenta previa Molar pregnancy Ectopic pregnancy Cervical insufficiency

Ectopic pregnancy Rationale Ectopic pregnancy is a condition in which the fertilized ovum is implanted outside the uterine cavity. Placenta previa is a condition in which the placenta is implanted in the lower uterine segment. Molar pregnancy is a benign proliferative growth of the placental trophoblast. In this condition, the chorionic villi develop into cystic and avascular transparent vesicles that hang in a grapelike cluster. Cervical insufficiency is characterized by passive and painless dilation of the cervix. It may lead to recurrent preterm birth during the second trimester in the absence of other causes.

The nurse is caring for a client in the second month of pregnancy. Which condition can be diagnosed through ultrasonography? Polyhydramnios Ectopic pregnancy Congenital anomalies Intrauterine growth restriction

Ectopic pregnancy Rationale Ultrasonography is conducted during various stages of pregnancy to determine whether the client has any risks or complications. During the first trimester, the embryo implants in the uterus. By conducting ultrasonography, the nurse can find whether the client has a risk of ectopic pregnancy. Polyhydramnios is characterized by the presence of excessive amniotic fluid in the amniotic sac and can be detected during the second trimester, not in the first trimester. Congenital anomalies can be detected during either the second or third trimester but not in the first trimester. Intrauterine growth restriction is the condition in which the growth rate of the fetus is not normal. Intrauterine growth restriction is detected during the second or third trimester of pregnancy.

A woman with mastitis states that the infected breast is too sore to tolerate breastfeeding. Which action should the nurse take first? Notify the health care provider. Reassess the patient in one hour. Administer the patient's as-needed pain medication. Educate the patient on milk expression techniques for the affected breast

Educate the patient on milk expression techniques for the affected breast. If the affected breast is too painful to breast feed, the patient should be educated on expression techniques or pumping to prevent milk stasis, which can cause an abscess and additional discomfort.

Active Management of Labor

Efficient labor is defined as labor that occurs within 12 hours of admission to L&D unit -give pitocin to ensure that this happens -amniotomy performed within 1 hr of admission if a woman in labor has not had SROM

A pregnant client with eclampsia has developed convulsions. The nurse immediately elevates the head of the bed to prevent aspiration and turns the client onto her side. The nurse then assesses the status of the client's airway, breathing, and pulse. The nurse suctions the secretions from the glottis to clear the airway, inserts an oral airway tube, and administers oxygen at 10 L/min through a face mask. Finally, the nurse administers intravenous (IV) magnesium sulfate as ordered. Which intervention by the nurse needs correction? Administering oxygen at 10 L/min via face mask Suctioning secretions from the glottis to clear the airway Elevating the head of the bed to prevent aspiration Inserting an oral airway to ensure proper breathing

Elevating the head of the bed to prevent aspiration Rationale Immediately after a convulsion, the nurse should lower the head of the bed and turn the client onto her side to prevent aspiration of vomitus. The nurse should not elevate the head of the bed; hence this intervention needs correction. The nurse should administer oxygen at 10 L/min through a face mask to maintain the client 's respiratory rate. The nurse should suction the secretions from the glottis to clear the airway and should insert an oral airway to ensure unobstructed breathing.

A postpartum client who had undergone a cesarean reports to the nurse about fever, loss of appetite, pelvic pain, and foul-smelling lochia. Upon assessment, the nurse finds that the client has an increased pulse rate and uterine tenderness. The laboratory reports indicate significant leukocytosis. What clinical condition should the nurse suspect based on these findings? Cystocele. Rectocele. Hematoma. Endometritis.

Endometritis. Rationale Endometritis is a common postpartum infection. It usually begins as a localized infection at the placental site and spreads to the entire endometrium. Fever, loss of appetite, pelvic pain, and foul-smelling lochia are symptoms of endometritis. An increased pulse rate and uterine tenderness is also observed in this condition. Therefore the nurse can infer that the client has endometritis. Cystocele is the protrusion of the bladder downward into the vagina. Rectocele is the herniation of the anterior rectal wall through the relaxed or ruptured vaginal fascia and rectovaginal septum. The symptoms reported by the client are not indicative of these conditions. Because the nurse does not find any collection of blood in the client, the client does not have a hematoma.

A cesarean birth is planned for a diabetic client with fetal macrosomia. Which intervention by the nurse is appropriate when preparing the client for surgery? Instruct the client to avoid insulin the night before the surgery. Administer a full dose of insulin on the morning of the surgery. Ensure the client has nothing by mouth on the morning of the surgery. Infuse intravenous 5% dextrose if the client's glucose level is below 100 mg/dl

Ensure the client has nothing by mouth on the morning of the surgery. Rationale The nurse must ensure the client is not given anything by mouth on the morning of the surgery. The client must take a full dose of insulin at bedtime the night before surgery. The client is fasting; therefore insulin is not administered on the morning of the surgery. The client is given intravenous 5% dextrose if her glucose levels fall below 70 mg/dl during active labor.

What does the nurse administer to a client if there is excessive bleeding after suction curettage? Nifedipine (Procardia) Methyldopa (Aldomet) Hydralazine (Apresoline) Ergonovine (Methergine)

Ergonovine (Methergine) Rationale Ergonovine (Methergine) is an ergot product, which is administered to contract the uterus when there is excessive bleeding after suction curettage. Nifedipine (Procardia) is prescribed for gestational hypertension or severe preeclampsia. Methyldopa (Aldomet) is an antihypertensive medication indicated for pregnant clients with hypertension. Hydralazine (Apresoline) is also an antihypertensive medication used for treating hypertension intrapartum.

A nurse assesses a postpartum woman with deep vein thrombosis (DVT). Which findings would be expected in an assessment of the woman? Select all that apply. Inflammation in both legs Decreased pulse oximetry Erythema of the affected leg Tenderness of the affected leg Decreased pedal pulse in the affected leg

Erythema of the affected leg Redness of the affected extremity is a sign of a DVT due to inflammation of the leg. Tenderness of the affected leg Tenderness in the affected area is a sign of a DVT due to inflammation of the leg. Decreased pedal pulse in the affected leg Decreased pedal pulses can result from reflex arterial spasms and are a sign of a DVT.

The nurse is caring for a patient in active labor. The patient's cervix is 6 cm dilated and soft, unchanged from the previous assessment. The patient reports feeling exhausted and "stuck." Nursing management should include which intervention? Guide the patient in pursed-lip breathing Administer supplementary oxygen by nasal cannula Establish reliable peripheral intravenous (IV) access Assist the patient into a standing position for as long as she can tolerate it

Establish reliable peripheral intravenous (IV) access Augmentation of labor is indicated in cases of maternal exhaustion and ineffective contractions. Augmentation is performing by administering IV oxytocin. As a result, it is imperative that the patient have reliable IV access.

Dystocia S/S and Interventions

Excessive abdominal pain, abnormal contraction pattern, fetal distress, maternal or fetal tachycardia, lack of progress in labor 1. Assess excessive abdominal pain , monitor fetal distress 2. Monitor uterine contractions maternal and fetal tachycardia lack of progress in labor 3.Monitor maternal temp and HR 4. Assist w/ pelvic exam, us, measurements 5. Admin prophylactic antibiotic 6. Admin IV fluids 7. Monitor I&Os, Keep hydrated monitor amniotic fluids provide rest and comfort administer sedatives and pain medications assess for prolapse of the cord after membrane raptures instruct patient in breathing technique

Assessment of pregnant client with diabetes?

Excessive thirst, hunger, weight loss, frequent urination, blurred vision, recurrent urinary tract infections and vaginal yeast infections, glycosuria and ketonuria, signs of gestational hypertension, polyhydramnios, large fetus for gestational age

Hyperemesis gravidarum

Excessive vomiting during pregnancy resulting in a weight loss of at least 5% of prepregnancy weight and accompanied by dehydration, electrolyte imbalance, ketosis, and acetonuria."

oxytocin stimulated contraction test

Exogenous oxytocin can also be used to stimulate uterine contractions. An IV infusion is begun with a scalp needle. The oxytocin is diluted in an IV solution (usually 10 units in 1000 ml of fluid) and infused through a piggyback port into the tubing of the main IV device. An infusion pump is used to ensure accurate dosage. The oxytocin infusion usually is begun at 0.5 milliunits/min and increased by 0.5 milliunits/min at 15-to-30 minute intervals until three uterine contractions of good quality are observed within a 10-minute period. A rate of 10 milliunits/min is usually adequate to elicit uterine contractions.

Care Management of Woman with Abruptio Placentae

Expectant Management -if fetus is between 20-34 weeks and both the woman and fetus are stable -woman is monitored closely because abruption may reoccur at any time Active Management -immediate birth if the fetus is at term or bleeding is severe and mother or fetus are in jeopardy -large bore IV line should be started for blood transfusion -indwelling urinary catheter inserted for continuous monitoring of urine output -vaginal birth is preferable but C-section may be necessary

Uncontrolled diabetes may cause complications in which body areas? Select all that apply. Eyes Heart Kidneys Nerves of the brain Peripheral vasculature

Eyes The small vascular nature of the eyes, particularly the retina, makes the eyes susceptible to impaired glucose metabolism. Heart Sustained high blood glucose levels cause damage to the heart muscle. Kidneys The kidneys, which are responsible for glucose secretion, are sensitive to high blood glucose levels. Additionally, as muscle wasting, related to impaired glucose metabolism, occurs, protein passes through the kidneys and causes damage. Peripheral vasculature Increased blood glucose damages vessels and can lead to atherosclerosis.

A nursing instructor asks a nursing student to describe manifestations of hypoglycemia exhibited by the newborn. The student responds correctly by telling the instructor about which signs and symptoms? Select all that apply. Eyes remain open to painful stimuli Hands and feet appear bluish Extremities in the flexed position Respiratory rate of 16 breaths/minute Axillary temperature 97.2° F

Eyes remain open to painful stimuli Lethargy, exhibited by a newborn's eyes remaining open to painful stimuli, is a symptom of hypoglycemia in newborns. Respiratory rate of 16 breaths/minute A respiratory rate of 16 breaths per minute represents depressed respiration and is a symptom of hypoglycemia in newborns. Correct Axillary temperature 97.2° F An axillary temperature of 97.2° F is low and a symptom of hypoglycemia in newborns. Because this infant has low blood glucose, energy is consumed to maintain normal metabolic function, causing temperatures to decrease.

Shawna is a primigravida at 28 weeks of gestation. Her BP has risen to 160/110 mm Hg and higher. Proteinuria is at 3+. She has been complaining of a headache and states that she needs to wear sunglasses even indoors because light hurts her eyes. She reports that she has been using two pil- lows at night instead of one to breathe more easily when she sleeps.

F. Preeclampsia (severe) with severe features

Assessment of a hydatidiform mole?

FHR not detectable, vaginal bleeding which may occur by the 4th week or not until the 2nd trimester may be bright red or dark brown in color and may be slight, profuse, or intermittent, symptoms of gestational hypertension before the 20th week, fundal height greater than expected for gestational date, elevated human chorionic gonadotropin levels

FHR activity

Fetal death can be confirmed by lack of heart motion along with the presence of fetal scalp edema and maceration and overlap of the cranial bones beginning 6 weeks using transvaginal fetal heart anatomy begins visible 13 weeks

Poor glycemic control later in pregnancy increases the rate of what?

Fetal macrosomia A birth weight more than 4000-4500 g or greater than the 90th percentile -occurs in 40% of pregestational diabetic pregnancies -carries an increased risk of need for C-section -disporoprtionate increase in shoulder, trunk and chest size -greater risk for shoulder dystocia

The nurse is helping a pregnant client monitor the daily fetal movement. Which finding should be reported to the primary health care provider? Fetal movement was not detected for 12 hours. An episode of limb straightening was observed. One episode of fetal breathing was seen in 30 minutes. An amniotic fluid index value of more than 5 cm

Fetal movement was not detected for 12 hours. Rationale The presence of fetal movements is a reassuring sign of fetal health. If fetal movements are not felt by the mother for 12 hours, then it is reported as the fetal alarm signal. Limb straightening, breathing movement, and amniotic fluid index do not indicate the fetal alarm signal. One episode of limb straightening is also a normal finding. One episode of fetal breathing movement seen in 30 minutes is a normal finding. An amniotic fluid index greater than 5 cm is a normal finding.

The nurse is caring for a patient for whom induction is being considered. Which assessment data are necessary to calculate the patient's Bishop score? Select all that apply. Fetal station Fetal heart rate The patient's cervical dilatation The patient's week of gestation The position of the patient's cervix

Fetal station Bishop's score is calculated on the basis of five parameters. Fetal station would consequently need to be identified. The patient's cervical dilatation Bishop's score is calculated on the basis of five parameters. It would be necessary to determine the dilatation of the patient's cervix, measured in centimeters. The position of the patient's cervix Bishop's score is calculated on the basis of five parameters. The position of the patient's cervix would need to be determined.

The nurse is caring for a pregnant client who is scheduled for cordocentesis. What could be the consequences of the test on the fetus? Destruction of red blood cells Fetal hyperbilirubinemia Fetomaternal hemorrhage Deformity of extremities

Fetomaternal hemorrhage Rationale Cordocentesis is an invasive procedure also known as percutaneous umbilical blood sampling (PUBS). In this procedure, the fetal umbilical vessel is punctured. Therefore, there is a direct risk of fetomaternal hemorrhage. Fetal anemia is diagnosed using cordocentesis; it does not cause destruction of red blood cells. The test may not cause hyperbilirubinemia as there is no destruction of the red blood cells. Deformity of extremities is caused by certain medications like thalidomide (Thalomid). It is not caused by cordocentesis.

Which metabolic action is responsible for earlier gluconeogenesis in pregnant women? Destruction of erythrocytes increases bilirubin Placental hormones lead to impaired healing in the mother Fetus continually withdraws nutrients from maternal blood Protein cannot be excreted from the kidneys during pregnancy

Fetus continually withdraws nutrients from maternal blood During pregnancy, the fetus withdraws nutrients from maternal blood, causing the release of glucose from stores in the liver.

If a pregnant client suspects signs and symptoms of preterm labor, which conditions would lead the client to go to hospital immediately? Select all that apply. Nausea and vomiting Upper back pain Fluid leakage from vagina Presence of vaginal bleeding Contractions every 10 minutes

Fluid leakage from vagina Presence of vaginal bleeding Contractions every 10 minutes Rationale Fluid leakage from the vagina indicates rupture of the amniotic membranes. The client should seek immediate medical attention, because ruptured amniotic membranes can compromise fetal health. Presence of vaginal bleeding may indicate onset of labor or placental hemorrhage, which may compromise fetal perfusion. Therefore the client should go to the hospital immediately. Uterine contractions (UCs) after every 10 minutes indicate active labor and the client should go to the hospital immediately. Nausea and vomiting and upper back pain do not indicate labor. The client need not seek immediate medical attention for these conditions.

Preterm labor interventions

Focus on stopping labor Identify and treat infection Restrict activity Ensure hydration; admin fluids Bed rest; Lateral position Monitor fetal status administer fluids Admin meds (Tocolytics)

Which conditions during pregnancy can result in preeclampsia in the client? Genetic abnormalities Dietary deficiencies Abnormal trophoblast invasion Cardiovascular changes Maternal hypotension

Genetic abnormalities Dietary deficiencies Abnormal trophoblast invasion Cardiovascular changes Rationale Current theories consider that genetic abnormalities and dietary deficiencies can result in preeclampsia. Abnormal trophoblast invasion causes fetal hypoxia and results in maternal hypertension. Cardiovascular changes stimulate the inflammatory system and result in preeclampsia in the pregnant client. Maternal hypertension, and not hypotension, after 20 weeks of gestation is known as preeclampsia.

Which drug is administered to treat chorioamnionitis in a patient during labor? Clindamycin (Cleocin) Metronidazole (Flagyl) Gentamycin (Garamycin) Dexamethasone (Decadron)

Gentamycin (Garamycin) Rationale Chorioamnionitis is a bacterial infection of the amniotic cavity. The infection is treated with antibiotics like gentamycin (Garamycin) during labor. Clindamycin (Cleocin) and metronidazole (Flagyl) are antibiotics used to treat chorioamnionitis after cesarean birth. Dexamethasone (Decadron) is an antenatal glucocorticoid that is used to prevent the risk of respiratory distress syndrome in the fetus.

In the past, factors to determine whether a woman was likely to have a high risk pregnancy were evaluated primarily from a medical point of view. A broader, more comprehensive approach to high risk pregnancy has been adopted. There are now four categories based on threats to the health of the woman and the outcome of pregnancy. Which of the options listed here is not included as a category? Biophysical Psychosocial Geographic Environmental

Geographic The fourth category is correctly referred to as the sociodemographic risk category. The factors stem from the mother and her family. Ethnicity may be one of the risks to pregnancy; however, it is not the only factor in this category. Low income, lack of prenatal care, age, parity, and marital status are included. Biophysical is one of the broad categories used for determining risk. It includes genetic considerations, nutritional status, and medical and obstetric disorders. Psychosocial risks include smoking, caffeine, drugs, alcohol, and psychologic status. All of these adverse lifestyles can have a negative effect on the health of the mother or fetus. Environmental risks are those that can affect fertility and fetal development. They include infections, chemicals, radiation, pesticides, illicit drugs, and industrial pollutants.

Multifetal pregnancy

Gestation of twins, triplets, quadruplets, or more.

GTD

Gestational Trophoblastic Disease -a group of pregnancy-related trophoblastic proliferative disorders without a viable fetus that are caused by abnormal fertilization

Diabetes mellitus "

Group of metabolic diseases characterized by hyperglycemia resulting from defects in insulin secretion, insulin action, or both."

Which physical signs and symptoms are common in the postpartum period after a woman experiences a fetal demise? Select all that apply. Nausea Diarrhea Headache Poor appetite Elevated heart rate Low blood pressure

Headache A headache is a common physical symptom after a fetal demise. Poor appetite Poor appetite is a common physical symptom after a fetal demise. Elevated heart rate Elevated heart rate is a common physical sign after a fetal demise. Nausea Nausea is a common physical symptom after a fetal demise.

What are premonitory S&S of eclampsia?

Headache Blurred vision Severe epigastric pain Altered mental status Make sure to have an airway goal to take care of the patent airway and client safety

A woman who has given birth reports severe perineal pain. She had a vacuum extractor delivery. This woman is at risk for which early postpartum complication? Hematoma Laceration Subinvolution Retained placental fragments

Hematoma Delivery via vacuum extractor is a risk factor for hematomas.

Two hours after giving birth, a primiparous woman becomes anxious and complains of intense perineal pain with a strong urge to have a bowel movement. Her fundus is firm, at the umbilicus, and midline. Her lochia is moderate rubra with no clots. The nurse would suspect what? Bladder distention Uterine atony Constipation Hematoma formation

Hematoma formation Rationale Bladder distention would result in an elevation of the fundus above the umbilicus and deviation to the right or left of midline. Uterine atony would result in a boggy fundus. Constipation is unlikely at this time. Increasing perineal pressure along with a firm fundus and moderate lochial flow are characteristic of hematoma formation.

The nurse is caring for a client who is undergoing treatment for deep vein thrombosis (DVT). Which signs and symptoms will the nurse monitor in the client to evaluate the patient's response to treatment? Dysuria, petechiae, and vertigo Petechiae, hematuria, and dysuria Hematuria, increased lochia, and vertigo Hematuria, petechiae, and increased lochia

Hematuria, petechiae, and increased lochia Rationale DVT treatment includes anticoagulant therapy. The effectiveness of the treatment can be evaluated by the presence of bleeding. Hematuria is the presence of blood in the urine, which is a side effect of anticoagulant therapy. Generalized petechiae refer to the reddish patches on the skin as a result of the rupturing of the microcirculation beneath the skin. It, too, is an adverse effect associated with anticoagulant treatment. Increased lochial discharge refers to increased uterine bleeding following delivery, and is a result of excess anticoagulants. Dysuria is decreased urinary output, which is not a complication associated with anticoagulant therapy. Vertigo is not a side effect of anticoagulant therapy.

What are the manifestations of HELLP syndrome? Select all that apply. Hemolysis Tachycardia Hyperventilation Low platelet count Elevated liver enzymes

Hemolysis Low platelet count Elevated liver enzymes Rationale HELLP syndrome is a serious condition that may develop during pregnancy in a client with preeclampsia. It is characterized by hemolysis due to the breakdown of red blood cells. The client may have a low platelet count, increasing the risk of bleeding and elevated liver enzymes due to impaired functioning of the liver. HELLP is not associated with an increase in heart rate, and may not result in tachycardia. The pulmonary functioning is not impaired in the client with HELLP syndrome. Therefore, hyperventilation is not a manifestation of HELLP syndrome

Upon reviewing the ultrasonography reports of a pregnant client, the nurse finds that the placenta is at a distance of 2.5 cm from the internal cervical os. What complication is likely if the client has a vaginal delivery? Hemorrhage Hyperthyroidism Thrombocytopenia Hypofibrinogenemia

Hemorrhage Rationale A placenta implanted in the lower uterine segment 2.5 cm from the internal cervical os indicates that the client has marginal placenta previa. In placenta previa, disruption of placental blood vessels occurs with stretching and thinning of the lower uterine segment, which results in bleeding. Therefore, the major maternal complication associated with placenta previa is hemorrhage. Hyperthyroidism is one of the serious complications of hydatidiform mole. Thrombocytopenia and hypofibrinogenemia are complications of abruptio placentae.

A woman diagnosed with marginal placenta previa gave birth vaginally 15 minutes ago. At the present time she is at the greatest risk for what? Hemorrhage Infection Urinary retention Thrombophlebitis

Hemorrhage Rationale Hemorrhage is the most immediate risk because the lower uterine segment has limited ability to contract to reduce blood loss. Infection is a risk because of the location of the placental attachment site; however, it is not a priority concern at this time. Placenta previa poses no greater risk for urinary retention than with a normally implanted placenta. There is no greater risk for thrombophlebitis than with a normally implanted placenta.

A woman diagnosed with marginal placenta previa gave birth vaginally 15 minutes ago. At present she is at the greatest risk for: Hemorrhage. Infection. Urinary retention. Thrombophlebitis

Hemorrhage. Hemorrhage is the most immediate risk because the lower uterine segment has limited ability to contract to reduce blood loss. Infection is a risk because of the location of the placental attachment site; however, it is not a priority concern at this time. Placenta previa poses no greater risk for urinary retention or thrombophlebitis than does a normally implanted placenta.

An 8-month-pregnant client presents with preeclampsia. Which clinical findings in the client indicate that the disease has progressed to HELLP syndrome? Select all that apply. Hepatic dysfunction Elevated liver enzymes Vaginal bleeding Low platelet count Chronic hypertension

Hepatic dysfunction Elevated liver enzymes Low platelet count Rationale Hepatic dysfunction in a client with preeclampsia indicates that the disease has progressed to HELLP syndrome. It can result in both endothelial damage and fibrin deposits in the liver. Hepatic tissue damage results in elevated liver enzymes. Narrowed blood vessels damage the red blood cells (RBCs) and they become hemolyzed, resulting in a decreased RBC and platelet count. Vaginal bleeding is sometimes seen in clients with severe gestational hypertension or those who are at risk for miscarriage. Chronic hypertension is a condition in which clients develop hypertension before the pregnancy. It is not related to HELLP syndrome.

The nurse is caring for a client in the first trimester of pregnancy who is prescribed propylthiouracil (PTU) for hyperthyroidism. What are the side effects of this medication? Facial anomalies Hepatic toxicity Esophageal atresia Developmental delay

Hepatic toxicity Rationale Although PTU controls the symptoms of hyperthyroidism effectively, it can cause hepatic toxicity serious enough to require liver transplantation. PTU is generally used only in the first trimester of pregnancy, after which the client is prescribed methimazole (MM). If MM is taken in the first trimester of pregnancy, it can cause facial anomalies, esophageal atresia, and developmental delay in the fetus.

What are the causes of indicated preterm labor? Select all that apply. Herpes infection Multifetal gestation Gestational diabetes Chronic hypertension Second trimester bleeding

Herpes infection Gestational diabetes Chronic hypertension Rationale Preterm labor may be spontaneous or indicated. Indicated preterm labor is a means to resolve the maternal and fetal risk. The factors that can cause indicated preterm labor include gestational diabetes, chronic hypertension, and herpes infection. Spontaneous preterm labor is caused due to early initiation of the labor process. The factors responsible for spontaneous labor are multifetal gestation and bleeding during the second trimester.

What are the possible risk factors associated with indicated preterm birth? Select all that apply. Herpes infection Gestational diabetes Chronic hypertension History of preterm birth Second trimester bleeding

Herpes infection Gestational diabetes Chronic hypertension Rationale Preterm births can be either spontaneous or indicated. Conditions that pose a danger to fetal or maternal health may be resolved by indicated preterm birth. These conditions include herpes infection, gestational diabetes, and hypertension in the mother. The clients with these conditions undergo indicated preterm birth to ensure the neonate's safety. Clients with a history of preterm birth and second trimester bleeding are at risk of spontaneous preterm birth.

The nurse understands which factors increase a woman's risk for developing gestational diabetes? Select all that apply. History of migraines High blood pressure Advanced maternal age Family history of heart disease Family history of type 2 diabetes mellitus

High blood pressure Hypertension is associated with an increased incidence of gestational diabetes. Advanced maternal age Women over 35 years of age have a higher incidence of gestational diabetes. Family history of type 2 diabetes mellitus Women with a family history of type 2 diabetes mellitus are more likely to develop diabetes during pregnancy.

Which statement is most likely to be associated with a breech presentation? Least common malpresentation Descent rapid Diagnosis by ultrasound only High rate of neuromuscular disorders

High rate of neuromuscular disorders Fetuses with neuromuscular disorders have a higher rate of breech presentation, perhaps because they are less capable of movement within the uterus. Breech is the most common malpresentation, affecting 3% to 4% of all labors. Descent is often slow because the breech is not as good a dilating wedge as the fetal head. Diagnosis is made by abdominal palpation and vaginal examination, and is confirmed by ultrasound

A nurse is caring for a neonate whose mother received daily methadone treatment during pregnancy. The neonate is irritable, tremulous, and feeds poorly at the breast. The nurse should assess for which other symptoms? Select all that apply. Weight gain of 0.5% from yesterday. High-pitched cry during a diaper change. Aggressive sucking on the hands and fingers. Bottle feeding 15 mL formula at every feeding. Persistent flexion of the upper and lower extremities.

High-pitched cry during a diaper change. Characteristics of neonatal abstinence syndrome (NAS) include a high-pitched cry, which may be heard during a routine diaper change. Correct Aggressive sucking on the hands and fingers. Neonates experiencing neonatal abstinence syndrome (NAS) often have aggressive finger and hand sucking patterns. Persistent flexion of the upper and lower extremities. Hyperactive muscle tone is associated with neonatal abstinence syndrome (NAS). The neonate is experiencing withdrawals related to the mother's use of methadone during pregnancy.

A premature newborn is being discharged to home on continuous monitoring, nasal cannula oxygen, and tube feedings. Which discharge teaching is a priority for the nurse to provide? Information on community resources, such as WIC How to recognize illness in the newborn at home The date and time for the next follow-up appointment How to obtain more feeding tubes for when they need more

How to obtain more feeding tubes for when they need more The newborn is receiving continuous tube feedings. It is a priority that the nurse share information with the parents about how to purchase the supplies needed for continuous tube feedings.

A neonate born at 33 weeks' gestation is on CPAP and has increased secretions in the oropharynx. Which interventions should the nurse provide when caring for the neonate? Select all that apply. Humidify the oxygen. Apply cardiac monitoring leads. Suction the oropharynx with a suction catheter. Position the neonate prone under the radiant warmer. Assess for grunting, nasal flaring, and retractions every 4 hours.

Humidify the oxygen. Humidifying the oxygen assists in thinning the secretions in the oropharynx. Thinner secretions are easier to remove and manage. Suction the oropharynx with a suction catheter. Suctioning the oropharynx removes the increased secretions and clears the airway. Suctioning the oropharynx is an effective way to manage secretions in the neonate. Position the neonate prone under the radiant warmer. The prone position facilitates drainage of respiratory secretions and this will assist the nurse with removal of these secretions.

A woman with severe preeclampsia has been receiving magnesium sulfate by IV infusion for 8 hours. The nurse assesses the woman and documents the following findings: temperature 37.1° C, pulse rate 96 beats/minute, respiratory rate 24 breaths/minute, blood pressure 155/112 mm Hg, 3+ deep tendon reflexes, and no ankle clonus. The nurse calls the physician, anticipating an order for what medication? Hydralazine Magnesium sulfate bolus Diazepam Calcium gluconate

Hydralazine Rationale Hydralazine is an antihypertensive commonly used to treat hypertension in severe preeclampsia. An additional bolus of magnesium sulfate may be ordered for increasing signs of central nervous system irritability related to severe preeclampsia (e.g., clonus) or if eclampsia develops. Diazepam sometimes is used to stop or shorten eclamptic seizures. Calcium gluconate is used as the antidote for magnesium sulfate toxicity. The client is not currently displaying any signs or symptoms of magnesium toxicity.

A woman with severe preeclampsia has been receiving magnesium sulfate by intravenous infusion for 8 hours. The nurse assesses the woman and documents the following findings: temperature 37.1° C, pulse rate 96 beats/min, respiratory rate 24 breaths/min, blood pressure 155/112 mm Hg, 3+ deep tendon reflexes, and no ankle clonus. The nurse calls the physician, anticipating an order for what? Hydralazine Magnesium sulfate bolus Diazepam Calcium gluconate

Hydralazine Rationale Hydralazine is an antihypertensive commonly used to treat hypertension in severe preeclampsia. An additional bolus of magnesium sulfate may be ordered for increasing signs of central nervous system irritability related to severe preeclampsia (e.g., clonus) or if eclampsia develops. Diazepam sometimes is used to stop or shorten eclamptic seizures. Calcium gluconate is used as the antidote for magnesium sulfate toxicity. The patient is not currently displaying any signs or symptoms of magnesium toxicity.

A woman with severe preeclampsia has been receiving magnesium sulfate by IV infusion for 8 hours. The nurse assesses the woman and documents the following findings: temperature 37.1° C, pulse rate 96 beats/min, respiratory rate 24 breaths/min, blood pressure 155/112 mm Hg, 3+ deep tendon reflexes, and no ankle clonus. The nurse calls the physician, anticipating an order for: Hydralazine. Magnesium sulfate bolus. Diazepam. Calcium gluconate.

Hydralazine. Hydralazine is an antihypertensive commonly used to treat hypertension in severe preeclampsia. An additional bolus of magnesium sulfate may be ordered for increasing signs of central nervous system irritability related to severe preeclampsia (e.g., clonus) or if eclampsia develops. Diazepam sometimes is used to stop or shorten eclamptic seizures. Calcium gluconate is used as the antidote for magnesium sulfate toxicity. The client is not currently displaying any signs or symptoms of magnesium toxicity.

What is the primary cause of thromboembolic disease? Viral infection Hypercoagulation Corticosteroid therapy Deficient clotting factors

Hypercoagulation Rationale Hypercoagulation and venous state are the primary causes for thromboembolic disease. Thromboembolic disease is characterized by the formation of clots in the blood vessel mainly due to inflammation. Viral infection is not associated with the formation of clots. Administration of corticosteroids does not alter the clotting behavior in a patient. Deficiency of clotting factors results in bleeding; it is not associated with thromboembolic disease.

A nurse caring for pregnant women must be aware that what situation is the most common medical complication of pregnancy? Hypertension Hyperemesis gravidarum Hemorrhagic complications Infections

Hypertension Rationale Preeclampsia and eclampsia are two noted, deadly forms of hypertension. A large percentage of pregnant women have nausea and vomiting, but a relative few have the severe form called hyperemesis gravidarum. Hemorrhagic complications are the second most common medical complication of pregnancy; hypertension is the most common complication of pregnancy.

A nurse caring for pregnant women must be aware that the most common medical complication of pregnancy is: Hypertension. Hyperemesis gravidarum. Hemorrhagic complications. Infections.

Hypertension. Preeclampsia and eclampsia are two noted, deadly forms of hypertension, which is the most common medical complication of pregnancy. A large percentage of pregnant women have nausea and vomiting, but a relative few have the severe form called hyperemesis gravidarum. Hemorrhagic complications are the second most common medical complication of pregnancy.

A woman with severe preeclampsia is being treated with an intravenous infusion of magnesium sulfate. When is this treatment considered successful? If blood pressure is reduced to prepregnant baseline. If seizures do not occur. If deep tendon reflexes become hypotonic. If diuresis reduces fluid retention.

If seizures do not occur. Rationale Magnesium sulfate is a central nervous system (CNS) depressant given primarily to prevent seizures. A temporary decrease in blood pressure can occur; however, this is not the purpose of administering this medication. Hypotonia is a sign of an excessive serum level of magnesium. It is critical that calcium gluconate be on hand to counteract the depressant effects of magnesium toxicity. Diuresis is not an expected outcome of magnesium sulfate administration.

Nipple-stimulated contraction test

In one approach, the woman applies warm, moist washcloths to both breasts for several minutes. She is then asked to massage one nipple for 10 minutes. Massaging the nipples causes a release of oxytocin from the posterior pituitary. An alternative approach is for her to massage the nipple for 2 minutes, rest for 5 minutes, and repeat the cycles of massaging and rest as necessary to achieve adequate uterine activity. When adequate contraptions or hyper stimulation occurs, stimulation should be stopped.

A new mother expresses doubt in her mothering abilities and indicates she is nervous about taking the newborn home. Which nursing interventions should occur before this woman's discharge? Select all that apply. Include the mother's partner in woman education. Remind the mother that the newborn is her responsibility now. Explain to the woman that every new mother is nervous and that she will be fine. Explain to the woman the importance of taking care of herself, as well as the newborn. Explain to the woman that she does not have to take this on alone and ensure that adequate support systems are in place.

Include the mother's partner in woman education. Including the woman's partner in patient education helps engage the partner and encourages participation in newborn care. Explain to the woman the importance of taking care of herself, as well as the newborn. Explaining to the woman the importance of self-care reinforces woman care and emphasizes the mother's role as a person, separate from the role of a caregiver. Correct Explain to the woman that she does not have to take this on alone and ensure that adequate support systems are in place. Ensuring availability of adequate support systems helps establish resources for the woman after discharge, which can help mitigate feelings of stress and doubt.

Subinvolution

Incomplete involution or failure of the uterus to return to its normal size and condition

A patient who is receiving oxytocin demonstrates signs of tachycardia. The nurse places the patient in a lateral position and then notifies the health care provider. The nurse then discontinues the oxytocin infusion. Which is the next nursing action indicated for this patient? Elevate the head of the bed Discontinue the primary nonadditive infusion Increase the rate of intravenous fluids as prescribed Call an emergency code and prepare the patient for cesarean delivery

Increase the rate of intravenous fluids as prescribed Increased fluid volume has the potential to simultaneously reduce uterine tone and enhance oxygenation.

The nurse is caring for a pregnant client with preeclampsia. The client reports severe and persistent epigastric pain. The primary health care provider orders a blood test. What result may indicate a worsening liver function? Increased red blood cell levels Decreased serum creatinine levels Decreased platelet count Increased liver transaminase levels

Increased liver transaminase levels Rationale If a pregnant client with preeclampsia reports severe and persistent epigastric pain, it often indicates impaired liver function. An increase in liver transaminases to twice the normal levels in the blood confirms liver damage. Preeclampsia is characterized by a decrease in red blood cells. An increase in serum creatinine levels indicates renal insufficiency and is not related to liver damage. A decreased platelet count does occur in preeclampsia, but this does not cause severe persistent epigastric pain. Epigastric pain is pathognomonic of liver damage.

The nurse is teaching a group of pregnant clients about early identification of preterm labor. What signs and symptoms of preterm labor should the nurse include in the teaching? Select all that apply. Upper abdominal pain Increased vaginal discharge Presence of vaginal bleeding Decreased urinary frequency Painful uterine contractions (UCs)

Increased vaginal discharge Presence of vaginal bleeding Painful uterine contractions (UCs) Rationale Any pregnant client runs the risk of having preterm labor and should be educated to identify its signs and symptoms. Painful uterine contractions (UCs) are a sign of preterm labor, caused by the body's attempt to deliver the baby. The client may show signs of vaginal bleeding due to a rupture of the membranes. Preterm labor can also be identified by changes in the color or amount of vaginal discharge. During labor the vaginal discharge usually increases and becomes brown to red in color. Preterm labor is also characterized by an increase in urine frequency and pain in lower abdomen. Therefore a decrease in urine frequency and upper abdominal pain do not indicate preterm labor.

The nurse is reviewing lab values to determine Rh incompatibility between mother and fetus. Which specific lab result should the nurse assess? Indirect Coombs test Hemoglobin level hCG level Maternal serum alpha-fetoprotein (MSAFP)

Indirect Coombs test Rationale The indirect Coombs test is a screening tool for Rh incompatibility. If the maternal titer for Rh antibodies is greater than 1:8, amniocentesis for determination of bilirubin in amniotic fluid is indicated to establish the severity of fetal hemolytic anemia. Hemoglobin reveals the oxygen carrying capacity of the blood. hCG is the hormone of pregnancy. Maternal serum alpha-fetoprotein (MSAFP) levels are used as a screening tool for NTDs in pregnancy.

wound infection

Infected episiotomy, c-section incision, or lacerations -occurs commonly once woman has been discharged home Signs: -fever -erythema -edema -warmth -tenderness -pain -seropurulent drainage -wound separation Management: -antibiotic therapy -wound debridement -wounds may be opened and drained Comfort measures: -analgesics -sitz bath -warm compresses -perineal care Often develop after mothers are discharged home; Typically cesarean incision, repaired laceration, or episiotomy site. Signs and symptoms of wound infections fever, erythema, edema, warmth, tenderness, pain, seropurulent drainage, and wound separation. Nursing management for wound infections comfort measures, analgesics, sits baths, warm compress, antibiotic therapy and wound debridement, and perineal care.

endometritis

Infection of the lining of the uterus -most common postpartum infection -higher incidence of C-section Signs: -Fever > 100.4oF -increased pulse -chills -anorexia -nausea -fatigue and lethargy -pelvic pain -uterine tenderness -fowl-smelling, profuse lochia -leukocytosis -increased RBC sedimentation rate Treatment: -IV broad-sprectrum antibiotic therapy -hydration -rest -pain rleif Comfort measures -cool comprsses -warm blankets -perineal care -sitz baths Signs and symptoms of Endometritis fever, increased pulse, chills, anorexia, nausea, fatigue, and pelvic pain, uterine tenderness and foul smelling profuse lochia. Management of Endometritis IV broad spectrum antibiotic therapy, hydration, rest and pain relief; antibiotic therapy usually DC 24 hours after woman is asymptomatic. Nurse should do a culture and asses lochia and temperature regularly.

The nurse is preparing a patient for cesarean delivery. After the epidural anesthetic is initiated, the patient reports anxiety about being alone and awake in the operating room (OR) during delivery. Which nursing interventions are appropriate for this patient? Select all that apply. Reassure the patient that the procedure will not take very long. Inform the patient that a nurse will be present in the OR during the procedure. Describe the OR and the people who will be present during the delivery to the patient. Explain to the patient that because of the anesthesia, it is unlikely that the patient will remember very much about the delivery. Inform the patient that a family member is allowed in the OR for support, although he or she may have to wait a few minutes to come in.

Inform the patient that a nurse will be present in the OR during the procedure. The nurse preparing the patient is often the circulating nurse during cesarean delivery. Having a familiar face and voice present can reassure the patient. Describe the OR and the people who will be present during the delivery to the patient. Describing the OR and the people who will be present during delivery helps the patient know what to expect and may help alleviate anxiety. Inform the patient that a family member is allowed in the OR for support, although he or she may have to wait a few minutes to come in. The patient may have a support person in the OR during the delivery. Preparatory procedures such as catheter insertion may occur before the support person enters, which can take 30 minutes to 45 minutes. Knowing that a family member will be present may help alleviate the patient's anxiety.

A patient in prolonged labor expresses frustration when changing positions and states, "I didn't have to move around so much last time I had a baby." Which nursing intervention is most appropriate for this patient? Reduce the frequency of position changes to promote patient comfort Inform the patient that repositioning is necessary to promote progress of labor Educate the patient about how repositioning provides distraction, reducing pain perception Explain to the patient that repositioning is necessary to ensure adequate fetal oxygenation during prolonged labor

Inform the patient that repositioning is necessary to promote progress of labor Repositioning during prolonged labor is indicated to promote natural progress of labor.

A pregnant client has been administered terbutaline (Brethine) as prescribed. The nurse finds that the client has a heart rate of 140 beats/min and complains of chest pain. What is the best nursing action in this situation? Administer propanolol (Inderal). Administer intravenous fluids. Administer 1 g calcium gluconate. Inform the primary health care provider (PHP).

Inform the primary health care provider (PHP). Rationale A heart rate of 140 beats per minute and chest pain indicate that the client is having tachycardia, which is an adverse effect of terbutaline (Brethine). Therefore the nurse should report this to the PHP to obtain further instructions on the treatment. Propanolol (Inderal) is administered to reverse the cardiovascular adverse effects of terbutaline (Brethine). However, it needs to be prescribed by the PHP. Calcium gluconate is administered to reverse the effect of magnesium sulfate. Serum potassium should be monitored in the client receiving terbutline therapy; however, it is not a priority intervention. The client has tachycardia and is not in a state of hypovolemic shock. Therefore intravenous fluids need not be administered to the client.

A nullipara reports concerns about being "overwhelmed" by the prospect of becoming a new parent. Which action should the nurse take? Inform the woman that parenting will get easier with time. Reassure the woman that parenthood is challenging for everyone. Explain to the woman that "maternal instinct" usually develops after discharge. Inform the woman that these feelings are normal and ask for further clarification.

Inform the woman that these feelings are normal and ask for further clarification. Reassuring the woman that these concerns are normal and inviting her to clarify her statement allows her to express her feelings and to feel understood. This response from the nurse is appropriate and facilitates woman care.

The primary health care provider prescribes magnesium sulfate (Epsom salts) for a client to prevent preterm labor. Following administration, the nurse observes that the client has a respiratory rate of 10 breaths/minute and deep tendon reflexes. Based on these findings, what interventions would help to prevent complications in the client? Give an oral dose of 10 mg nifedipine (Adalat). Administer propranolol (Inderal) intravenously. Infuse 500 mg of calcium chloride intravenously for 30 minutes. Administer 6 mg of dexamethasone (Decadron) intramuscularly.

Infuse 500 mg of calcium chloride intravenously for 30 minutes. Rationale Magnesium sulfate is a tocolytic that is administered to the patient at 24 to 32 weeks of gestation to prevent the risk of preterm birth. A respiratory rate of 10 breaths/minute (below 12 breaths/minute) and deep tendon reflexes are intolerable adverse effects of the drug. Therefore, 500 mg of calcium chloride is infused intravenously for 30 minutes to reverse the magnesium sulfate (Epsom salt) toxicity. Nifedipine (Adalat) is a calcium channel blocker that should not be administered concurrently with magnesium sulfate (Epsom salt), because it results in skeletal muscle blockade. Propranolol (Inderal) is used to reverse the intolerable cardiovascular effects of terbutaline (Brethine). Dexamethasone (Decadron) is an antenatal glucocorticoid that is used to prevent the risk of respiratory distress syndrome in the fetus.

The primary health care provider prescribes magnesium sulfate (Epsom salts) for a client to prevent preterm labor. Following administration, the nurse observes that the client has a respiratory rate of 10 breaths/minute and deep tendon reflexes. Based on these findings, what interventions would help to prevent complications in the client? Give an oral dose of 10 mg nifedipine (Adalat). Administer propranolol (Inderal) intravenously. Infuse 500 mg of calcium chloride intravenously for 30 minutes. Administer 6 mg of dexamethasone (Decadron) intramuscularly.

Infuse 500 mg of calcium chloride intravenously for 30 minutes. Rationale Magnesium sulfate is a tocolytic that is administered to the patient at 24 to 32 weeks of gestation to prevent the risk of preterm birth. A respiratory rate of 10 breaths/minute (below 12 breaths/minute) and deep tendon reflexes are intolerable adverse effects of the drug. Therefore, 500 mg of calcium chloride is infused intravenously for 30 minutes to reverse the magnesium sulfate (Epsom salt) toxicity. Nifedipine (Adalat) is a calcium channel blocker that should not be administered concurrently with magnesium sulfate (Epsom salt), because it results in skeletal muscle blockade. Propranolol (Inderal) is used to reverse the intolerable cardiovascular effects of terbutaline (Brethine). Dexamethasone (Decadron) is an antenatal glucocorticoid that is used to prevent the risk of respiratory distress syndrome in the fetus.

A postpartum client with hemorrhagic shock has been administered intravenous (IV) infusion of crystalloid solution. Upon reviewing the client's laboratory reports, the nurse finds that platelet count and clotting factor levels are not improved. What is the best treatment option in this situation? Infusion of fresh frozen plasma Provide supplemental oxygenation Administer packed red blood cells Increase the dose of crystalloid solution

Infusion of fresh frozen plasma Rationale When a postpartum client has excessive bleeding due to hemorrhagic shock, an IV infusion of crystalloid solution is administered. If the platelet count is not restored even after the crystalloid IV infusion process, then fresh frozen plasma has to be infused. Fresh frozen plasma contains all the coagulation factors and it helps to restore platelet counts. Packed RBCs are administered if the client has active bleeding, despite the initial crystalloid administration. Increasing the dose or volume of crystalloid solution will not increase the levels of clotting factors and platelets. Supplementary oxygenation is given to compensate for the reduced tissue perfusion when the client has hypovolemic shock. Supplementary oxygenation does not increase the levels of clotting factors and platelets.

A postpartum client who underwent a cesarean delivery complains of minor perineal and rectal pain. Further assessment shows the presence of peritoneal hematomas. Which primary nursing interventions should be performed after surgically removing the hematomas and minimizing the bleeding? Select all that apply. Prepare for hysterectomy. Administer pain relief medication. Initiate fluid replacement therapy. Monitor serum hemoglobin levels. Monitor the amount of blood loss.

Initiate fluid replacement therapy. Monitor serum hemoglobin levels. Monitor the amount of blood loss. Rationale Early manifestations of retroperitoneal hematoma include shock, which is caused by excessive bleeding. After the surgical removal of the hematomas, the priority nursing intervention is to initiate fluid replacement therapy to treat the shock. Along with fluid replacement, the nurse should monitor the client's hemoglobin levels until they are restored to normal values. Monitoring blood loss helps the nurse assess the client's condition. Hysterectomy is performed when there is excessive bleeding that is not responsive to conservative measures. Retroperitoneal hematoma is associated with minimal pain, so administering pain relief medication is not a priority nursing intervention.

The nurse is caring for a client with gestational diabetes. What does the nurse teach the client about using insulin? Store unused vials of insulin in the freezer. Shake the prepared syringes well before use. Administer long-acting insulin before meals. Inject insulin in the abdomen.

Inject insulin in the abdomen. Rationale The nurse must teach the client that the abdomen is the preferred site for injecting insulin, because insulin is best absorbed in the abdominal area. Other suitable sites include the upper outer arm, the thighs, and the buttocks. Insulin must not be frozen, so unused vials of insulin are stored in the refrigerator, but not in the freezer. Syringes containing mixed insulin may be stored for up to 2 weeks in the refrigerator. The syringe must not be shaken; it must be gently rotated 20 times before injection. Long-acting insulin is usually administered at bedtime, because it provides glucose control for a longer duration.

The nurse is teaching a woman with gestational diabetes the technique to inject insulin. What should the nurse include in the teaching session? Select all that apply. Aspirate before injecting. Clean injection site with alcohol. Insert the needle at a 90-degree angle. Inject insulin slowly. After injection, cover site with sterile gauze.

Insert the needle at a 90-degree angle. Inject insulin slowly. After injection, cover site with sterile gauze. Rationale Insulin should be injected with the short needle inserted at a 45- to 90-degree angle, depending on fatty tissue. Insulin is injected slowly to allow tissue expansion and minimize pressure, which can cause insulin leakage. After injection, the site should be covered with sterile gauze. Gentle pressure should be applied to prevent bleeding. Aspirating when injecting into subcutaneous tissue is not necessary. The injection site should be clean, but using alcohol is not necessary.

A client who has postpartum bleeding due to uterine subinvolution has not recovered from drug therapy. Which procedure would be helpful in managing the bleeding in this client? Massaging the abdomen Using a nonrebreather mask Indwelling a urinary catheter Inserting a fist into the vagina

Inserting a fist into the vagina Rationale The client is continuously bleeding despite conventional therapy. In this situation, the client is subjected to bimanual compression. In this procedure the fist is inserted into the vagina and knuckles are pressed against the anterior side of the uterus. This maneuver positions the uterus in the pelvic cavity. Uterine massage is also helpful in managing postpartum hemorrhage (PPH). However, uterine massage is not done by massaging the abdomen, but by placing one hand on the abdomen and massaging the posterior side of the uterus. A urinary catheter is indwelled to determine the urinary output. A nonrebreather face mask is given to the client when the client has hypoxemia.

What measures will the nurse implement in her postpartum client to prevent postpartum infection and other possible complications? Select all that apply. Instruct the client to consume foods that are rich in iron. Instruct the client to wash hands with soap after urination. Perform fundal massage on the client when necessary. Instruct the client to change the perineal pads from front to back. Inform the client to avoid consuming an excess amount of water.

Instruct the client to consume foods that are rich in iron. Instruct the client to wash hands with soap after urination. Perform fundal massage on the client when necessary. Instruct the client to change the perineal pads from front to back. Rationale While teaching a client about preventive measures for postpartum infection, the nurse should emphasize diet pattern and hygiene. An iron-rich diet is recommended to prevent anemia during hemorrhage. Washing one's hands with soap will prevent the spread of infection. Fundal massage is performed during the postpartum period to promote uterine contraction.The perineal pads should be changed from front to back, because this prevents postpartum urinary tract infections by preventing the deposit of the microorganisms present in the lochia near the urethra. The postpartum client is advised to drink plenty of water to maintain hydration and prevent infection in the uterus.

A pregnant client with pregestational insulin-dependent diabetes is going for a week's vacation to another state. What should the nurse ask the client to carry with her in order to prevent complications? Select all that apply. Antibiotics Insulin vials Glucose tablets Antihypertensives Blood glucose meter

Insulin vials Glucose tablets Blood glucose meter Rationale A client with pregestational diabetes should be very careful and should be prepared to tackle any complications associated with high or low blood sugar levels. The nurse should ask the client to carry the routine insulin medications. Hormonal effects of pregnancy can also cause hypoglycemia, which can be normalized by taking glucose tablets. The client should be able to find out her blood sugar levels any time. Therefore, the nurse should ask the client to take a glucometer for routine monitoring of blood sugar levels. Antibiotics should be taken only if the client has an infection. Antihypertensives would be required if the client has high blood pressure.

The nurse explains to the nursing student that the most prevalent clinical manifestation of abruptio placentae (as opposed to placenta previa) is what? Bleeding Intense abdominal pain Uterine activity Cramping

Intense abdominal pain Rationale Pain is absent with placenta previa and may be agonizing with abruptio placentae. Bleeding may be present in varying degrees for both placental conditions. Uterine activity may be present with both placental conditions. Cramping is a form of uterine activity that may be present in both placental conditions.

The most prevalent clinical manifestation of abruptio placentae (as opposed to placenta previa) is: Bleeding. Intense abdominal pain. Uterine activity. Cramping.

Intense abdominal pain. Pain is absent with placenta previa and may be agonizing with abruptio placentae. Bleeding, uterine activity, and cramping may be present in varying degrees for both placental conditions

The nurse is caring for a 32-year-old pregnant client who had an onset of labor at 40 weeks' gestation. Following the labor, the nurse finds that the newborn has a low birth weight (LBW). What explanation will the nurse give to the client as to the etiology of the newborn's LBW? Preterm labor. Maternal age. Diabetic condition of the patient. Intrauterine growth restriction (IUGR).

Intrauterine growth restriction (IUGR). Rationale The low birth weight of the newborn is due to IUGR, a condition of inadequate fetal growth. It may be caused due to various conditions, such as gestational hypertension that interferes with uteroplacental perfusion. Interference with uteroplacental perfusion limits the flow of nutrients into the fetus and causes the low birth weight. The onset of labor is at 40 weeks' gestation. Therefore, it is not a preterm labor. The client's age is normal for pregnancy. Therefore, the client's age is not a reason for the low birth weight of the child. Infants born to clients with diabetes would have a high birth weight, not a low one.

A postpartum client with hemorrhagic shock is administered intravascular colloids. The nurse monitors the client carefully throughout the colloid administration. What risk factor in the client should the nurse be aware of? Excessive hemorrhage Von Willebrand disease Deep venous thrombosis Intravascular fluid overload

Intravascular fluid overload Rationale Clients who are given intravascular colloids are at a higher risk for intravascular fluid overload. Therefore, the nurse should monitor the client for symptoms of intravascular fluid overload. Intravascular colloid therapy does not cause excessive hemorrhage. Von Willebrand disease, a hereditary disease condition, is a type of hemophilia. Von Willebrand disease is caused by the deficiency of a blood clotting protein called Von Willebrand factor (vWF). Deep venous thrombosis is a venous thromboembolic disorder that is most commonly seen in the lower extremities. It is unrelated to intravascular colloid therapy.

The nurse is caring for a hypertensive pregnant client who is on magnesium sulfate therapy. The nurse finds that the client has drowsiness, slurred speech, and depressed respiration. What medication would help in treating magnesium toxicity? Intravenous diazepam (Valium) Intravenous nifedipine (Adalat) Intravenous hydralazine (Apresoline) Intravenous calcium gluconate (Kalcinate)

Intravenous calcium gluconate (Kalcinate) Rationale When treating a hypertensive pregnant client with magnesium sulfate therapy, the nurse should be alert for possible magnesium toxicity. Manifestations of magnesium toxicity include drowsiness, lethargy, slurred speech, depressed respiration, loss of deep tendon reflexes, and in severe cases, cardiac arrest. The effects of magnesium toxicity can be reversed by administering calcium gluconate (Kalcinate) intravenously. Diazepam (Valium) is an anticonvulsant drug; it is not used to reverse the effects of magnesium toxicity. Nifedipine (Adalat) is an antihypertensive drug; if used along with magnesium sulfate, it leads to muscle blockade. Hydralazine (Apresoline) is an antihypertensive drug; it does not reverse the symptoms of magnesium toxicity.

Which postpartum conditions are considered medical emergencies that require immediate treatment? Inversion of the uterus and hypovolemic shock Hypotonic uterus and coagulopathies Subinvolution of the uterus and idiopathic thrombocytopenic purpura Uterine atony and coagulopathies

Inversion of the uterus and hypovolemic shock Rationale Inversion of the uterus and hypovolemic shock are considered medical emergencies. A hypotonic uterus can be managed with massage and oxytocin. Coagulopathies should be identified before birth and treated accordingly. Although subinvolution of the uterus and ITP are serious conditions, they do not always require immediate treatment. ITP can be safely managed with corticosteroids or IV immunoglobulin. DIC and uterine atony are very serious obstetric complications; however, uterine inversion is a medical emergency requiring immediate intervention.

Which PPH conditions are considered medical emergencies that require immediate treatment? Inversion of the uterus and hypovolemic shock Hypotonic uterus and coagulopathies Subinvolution of the uterus and idiopathic thrombocytopenic purpura Uterine atony

Inversion of the uterus and hypovolemic shock Rationale Inversion of the uterus and hypovolemic shock are considered medical emergencies. A hypotonic uterus can be managed with massage and oxytocin. Coagulopathies should be identified prior to delivery and treated accordingly. Although subinvolution of the uterus and ITP are serious conditions, they do not always require immediate treatment. ITP can be safely managed with corticosteroids or IV immunoglobulin. Uterine atony is a very serious obstetric complication; however, uterine inversion is a medical emergency requiring immediate intervention.

Which PPH conditions are considered medical emergencies that require immediate treatment? Inversion of the uterus and hypovolemic shock Hypotonic uterus and coagulopathies Subinvolution of the uterus and idiopathic thrombocytopenic purpura (ITP) Uterine atony and disseminated intravascular coagulation (DIC).

Inversion of the uterus and hypovolemic shock Inversion of the uterus and hypovolemic shock are considered medical emergencies. A hypotonic uterus can be managed with massage and oxytocin; coagulopathies should have been identified prior to delivery and treated accordingly. Although subinvolution of the uterus and ITP are serious conditions, they do not always require immediate treatment; ITP can be safely managed with corticosteroids or IV immunoglobulin. DIC and uterine atony are very serious obstetric complications but are not medical emergencies requiring immediate intervention.

Nurses should be aware that what is associated with HELLP syndrome? Is a mild form of preeclampsia Can be diagnosed by a nurse alert to its symptoms Is characterized by hemolysis, elevated liver enzymes, and low platelets Is associated with preterm labor but not perinatal mortality

Is characterized by hemolysis, elevated liver enzymes, and low platelets Rationale The acronym HELLP stands for hemolysis (H), elevated liver enzymes (EL), and low platelets (LP). HELLP syndrome is a variant of severe preeclampsia. HELLP syndrome is difficult to identify, because the symptoms often are not obvious. It must be diagnosed in the laboratory. Preterm labor is greatly increased and so is perinatal mortality

A nurse providing care for the antepartum woman should understand what about the contraction stress test (CST)? Sometimes uses vibroacoustic stimulation Is an invasive test; however, contractions are stimulated Is considered negative if no late decelerations are observed with the contractions Is more effective than nonstress test (NST) if the membranes have already been ruptured

Is considered negative if no late decelerations are observed with the contractions Rationale No late decelerations indicate a negative CST. Vibroacoustic stimulation is sometimes used with NST. CST is invasive if stimulation is by IV oxytocin but not if by nipple stimulation. CST is contraindicated if the membranes have ruptured.

A nurse providing care for the antepartum woman should understand that the contraction stress test (CST): Sometimes uses vibroacoustic stimulation. Is an invasive test; however, contractions are stimulated. Is considered to have a negative result if no late decelerations are observed with the contractions. Is more effective than nonstress test (NST) if the membranes have already been ruptured.

Is considered to have a negative result if no late decelerations are observed with the contractions. No late decelerations indicate a positive CST result. Vibroacoustic stimulation is sometimes used with NST. CST is invasive if stimulation is performed by IV oxytocin but not if by nipple stimulation. CST is contraindicated if the membranes have ruptured.

Of what should a nurse providing care to a woman in labor be aware regarding cesarean birth? Is declining in frequency in the United States Is more likely to be done for the poor in public hospitals who do not get the nurse counseling that wealthier clients do Is performed primarily for the benefit of the fetus Can be either elected or refused by women as their absolute legal right

Is performed primarily for the benefit of the fetus Rationale The most common indications for cesarean birth are dangers to the fetus related to labor and birth complications. Cesarean births are increasing in the United States. Wealthier women who have health insurance and who give birth in a private hospital are more likely to experience cesarean birth. A woman's right to elect cesarean surgery is in dispute, as is her right to refuse it if in doing so she endangers the fetus. Legal issues are not absolutely clear.

A nurse providing care to a woman in labor should be aware that cesarean birth: Is declining in frequency in the United States. Is more likely to be performed in the poor in public hospitals who do not receive the nurse counseling that wealthier clients do. Is performed primarily for the benefit of the fetus. Can be either elected or refused by women as their absolute legal right.

Is performed primarily for the benefit of the fetus. The most common indications for cesarean birth are danger to the fetus related to labor and birth complications. Cesarean births are increasing in the United States. Wealthier women who have health insurance and who give birth in a private hospital are more likely to experience cesarean birth. A woman's right to elect cesarean surgery is in dispute, as is her right to refuse it if in doing so she endangers the fetus. Legal issues are not absolutely clear.

The nurse is using White's classification of diabetes in pregnancy. What are the features of White's classification? It considers the duration of diabetes in the client. It is based on the age at which diabetes was diagnosed. It is based on the involvement of the eye and the kidneys. It classified as type 1, type 2, others, and gestational diabetes. It considers two groups with and without vascular complications.

It considers the duration of diabetes in the client. It is based on the age at which diabetes was diagnosed. It is based on the involvement of the eye and the kidneys. Rationale The White's classification system considers the duration of diabetes in the client. It is based on the age at which the illness was diagnosed. It also considers the involvement of the end-organs, which are the eye and the kidneys. The American Diabetes Association (ADA) classifies diabetes into four mutually exclusive categories. They are type 1, type 2, others, and gestational diabetes. In this classification method, type 1 and type 2 diabetes are further classified into two groups. One group includes those with vascular complications and the other group includes those without vascular complications.

A nurse providing care to a woman in labor should be aware of which fact about cesarean birth? It is declining in frequency in the United States. It is more likely to be done for the poor in public hospitals who do not get the nurse counseling that wealthier patients do. It is performed primarily for the health of the mother and fetus. It can be either elected or refused by women as their absolute legal right.

It is performed primarily for the health of the mother and fetus. Rationale The most common indications for cesarean birth are to preserve the health of the mother and fetus. Cesarean births are increasing in the United States. Women who have health insurance and who give birth in a private hospital are more likely to experience cesarean birth. A woman's right to elect cesarean birth is in dispute, as is her right to refuse it if in doing so she endangers the fetus. Legal issues are not absolutely clear.

The nurse is advising a pregnant client who has been prescribed lispro (Humalog). What information does the nurse provide about the insulin? It is rapid-acting insulin preferred for use during pregnancy. It is injected just before meals and causes less hyperglycemia. It has shorter duration of action as compared to regular insulin. It is released slowly in small amounts with no pronounced peak. Its action lasts for 12 hours maintaining optimal blood glucose levels.

It is rapid-acting insulin preferred for use during pregnancy. It is injected just before meals and causes less hyperglycemia. It has shorter duration of action as compared to regular insulin. Rationale Lispro (Humalog) is rapid-acting insulin preferred for use during pregnancy. It is convenient, because it is injected just before mealtime. It causes less hyperglycemia and fewer hypoglycemic episodes. It has a shorter duration of action as compared to regular insulin. Lispro (Humalog) is not released slowly in small amounts. Glargine (Lantus) is long-acting insulin that is released slowly in small amounts with no pronounced peak. The action of lispro (Humalog) lasts for 3 to 5 hours. Therefore clients often need additional longer-acting insulin to maintain optimal blood glucose levels.

With regard to the use of tocolytic therapy to suppress uterine activity, nurses should be aware of what? The drugs can be given efficaciously up to the designated beginning of term at 37 weeks. There are no important maternal (as opposed to fetal) contraindications. Its most important function is to afford the opportunity to administer antenatal glucocorticoids. If the client develops pulmonary edema while on tocolytics, IV fluids should be given.

Its most important function is to afford the opportunity to administer antenatal glucocorticoids. Rationale Buying time for antenatal glucocorticoids to accelerate fetal lung development might be the best reason to use tocolytics. Once the pregnancy has reached 34 weeks, the risks of tocolytic therapy outweigh the benefits. There are important maternal contraindications to tocolytic therapy. Tocolytic-induced edema can be caused by IV fluids.

What should nurses be aware of with regard to the use of tocolytic therapy to suppress uterine activity? The drugs can be given efficaciously up to the designated beginning of term at 37 weeks. There are no important maternal (as opposed to fetal) contraindications. Its most important function is to afford the opportunity to administer antenatal glucocorticoids. If the woman develops pulmonary edema while on tocolytics, IV fluids should be given.

Its most important function is to afford the opportunity to administer antenatal glucocorticoids. Rationale There are important maternal contraindications to tocolytic therapy. After the pregnancy has reached 34 weeks, the risks of tocolytic therapy outweigh the benefits. Buying time for antenatal glucocorticoids to accelerate fetal lung development might be the best reason to use tocolytics. Tocolytic-induced edema can be caused by IV fluids.

With regard to the use of tocolytic therapy to suppress uterine activity, nurses should be aware that: The drugs can be given efficaciously up to the designated beginning of term at 37 weeks. There are no important maternal (as opposed to fetal) contraindications. Its most important function is to afford the opportunity to administer antenatal glucocorticoids. If pulmonary edema develops while the client is receiving tocolytics, IV fluids should be given.

Its most important function is to afford the opportunity to administer antenatal glucocorticoids. Buying time for antenatal glucocorticoids to accelerate fetal lung development might be the best reason to use tocolytics. Once the pregnancy has reached 34 weeks, the risks of tocolytic therapy outweigh the benefits. There are important maternal contraindications to tocolytic therapy. Tocolytic-induced edema can be caused by IV fluids.

For which condition should the nurse be alert after administering terbutaline (Brethine) to a pregnant client with diabetes mellitus? Dyspnea Infection Ketoacidosis Hypoglycemia

Ketoacidosis Rationale Terbutaline (Brethine) is a beta-mimetic drug administered for tocolysis to stop preterm labor. It may lead to hyperglycemia and cause ketoacidosis in the pregnant client. Dyspnea on exertion may be seen in a pregnant patient with acquired cardiac disease. Infection in pregnant women happens due to an alteration in the normal resistance of the body to infection. Hypoglycemia occurs if there is an increase in the insulin levels.

The blood glucose level of a pregnant client is 325 mg/dl. Which test should be performed on the patient to assess the risk of maternal or intrauterine fetal death? Ketones in urine Glucose in urine Arterial blood gases Abdominal ultrasound

Ketones in urine Rationale Diabetic ketoacidosis is a potentially fatal complication of diabetes that can lead to fetal death. This complication may occur if the client's blood glucose levels rise above 200mg/dl. Diabetic ketoacidosis can be confirmed by assessing the presence of ketones in the urine. The client's blood glucose level is 325 mg/dl and, hence, the patient has poorly controlled diabetes. Therefore, the nurse need not assess the urine glucose levels. Arterial blood gases and abdominal ultrasound are not useful for diagnosis of intrauterine fetal death.

A pregnant woman has contacted the nurse about severe nausea and vomiting. What is the priority assessment in evaluating a pregnant woman with severe nausea and vomiting? Fasting blood glucose level Ketonuria Bilirubin White Blood cell count

Ketonuria Rationale Determination of ketonuria would be a critical assessment that would lead toward determination of hyperemesis. A pregnant client with severe nausea and vomiting may have hyperemesis gravidarum and, as such, requires critical monitoring to determine the nature of the problem. A FBS, although informative would not be the priority assessment at this time. Although a bilirubin level would be needed, it would not be the priority assessment. A WBC count would indicate the possibility of an infectious source but it would not be a priority assessment in terms of the client's presentation.

Which factors associated with gestational diabetes may predispose the pregnant female to spontaneous abortion in the first trimester? Select all that apply. Ketosis Glucosuria Hypoglycemia Hyperglycemia Uterine distention

Ketosis Untreated ketoacidosis may predispose the mother with diabetes to spontaneous abortion and can lead to maternal death. Hypoglycemia Abnormally low blood or hypoglycemic glucose levels may predispose the mother with diabetes to spontaneous abortion. Hyperglycemia Abnormally high or hyperglycemic blood glucose levels may predispose the mother with diabetes to spontaneous abortion.

Which test is used to determine the presence of fetal-to-maternal bleeding in a pregnant patient? D-dimer test Non-stress test (NST) Kleihauer-Betke (KB) test Biophysical profile (BPP) test

Kleihauer-Betke (KB) test Rationale The KB test is used to determine the presence of fetal-to-maternal bleeding or transplacental hemorrhage. The D-dimer test is used to diagnose blood disorders, such as disseminated intravascular coagulation (DIC). The NST and BPP tests are used to determine fetal surveillance.

Precipitous Labor and Delivery

Labor lasting less than 3 hours -Interventions Delivery tray (hemostats, scissors, cord clamp) Stay with client Support; keep client calm Pant b/w contractions Prepare for ruptured membranes / head crowns do not try to prevent the fetus from being delivered -Before arrival of MD or none: 1. Apply gentle pressure to fetal head upward toward vagina to prevent damage support perineal area 2. Support infants body during delivery 3. Deliver infant b/w contractions; check for cord around neck 4. Use restitution to deliver posterior shoulder 5. Use gentle downward pressure to move anterior shoulder under public symphysis 6. *Bulb suction infant's mouth first then suction nares. 7. Dry and cover infant to keep warm 8. Allow the placenta to separate naturally 9. Place infant on mothers abdomen or breast to induce uterine contractions.

Prolonged latent phase

Labor pattern defined as a latent phase that exceeds 20 hours in nulliparas and exceeds 14 hours in multiparas.

Failure of descent

Labor pattern defined as no change in fetal progress through the birth canal during the deceleration phase and second stage of labor.

Precipitous labor

Labor pattern that lasts less than 3 hours from the onset of contractions to the time of birth, often resulting from hypertonic uterine contractions that are tetanic in intensity. For nulliparas, dilation is greater than 5 cm in 1 hour and, for multiparas, dilation is 10 cm in 1 hour.

What is HELLP syndrome?

Laboratory diagnostic variant of severe preeclampsia involves hepatic dysfunction, characterized by: Hemolysis of RBC's(H) Elevated liver enzymes (EL) Low platelets (LP) low hgb may result from arteriol vasospasm, endothilial cell dysfunction CBC and may need platelet transfusion

A woman with a firm fundus who presents with excessive vaginal bleeding may be at risk for which complication? Laceration Hematoma Uterine atony Retained placental fragments

Laceration Excessive vaginal bleeding with a firm fundus may be a sign of a vaginal, cervical, or perineal laceration.

Match the type of diabetes with its characterization.

Lack of insulin secretion Type 1 diabetes mellitus Insulin resistance Type 2 diabetes mellitus Early gluconeogenesis Gestational diabetes mellitus

What is the primary test performed to determine fetal lung maturity? TDx FLM assay Amniocentesis Doppler velocimetry Lamellar body count (LBC)

Lamellar body count (LBC) Rationale Lamellar bodies are surfactant-containing particles secreted by type II pneumocytes. Therefore, lamellar body count (LBC) is used to determine fetal lung maturity. TDx FLM assay is no longer available in United Sates as a primary test for determining fetal lung maturity. Amniocentesis is used as a secondary test if simpler and less expensive processes indicate lung immaturity. Doppler velocimetry is used to assess fetal blood supply.

A woman with gestational diabetes who is at 34 weeks' gestation voices concern regarding fetal compromise related to her diagnosis of gestational diabetes. Which recommendation should the nurse make to the woman to monitor for potential fetal complications? Assess blood sugar once daily Purchase a handheld Doppler Increase the amount of juice and fruit in her diet Lie down and count the number of kicks felt in an hour

Lie down and count the number of kicks felt in an hour Fetal kick counts allow the woman to monitor fetal movement.

Which risk is a fetus subject to if chorionic villus sampling (CVS) is conducted in the 7th week of gestation? Reduced heart rate Limb reduction defects Decreased lung maturity Neural tube defect

Limb reduction defects Rationale Chorionic villus sampling (CVS) can be performed in the first or second trimester, ideally between 10 and 13 weeks of gestation. Studies indicate that the fetus may be at increased risk for limb reduction defects when the test is performed before the ninth completed week of gestation. Ultrasonography is used to monitor the fetal heart rate. CVS does not affect the fetal heart rate. Fetal lung maturity is detected by amniocentesis. CVS does not impair fetal lung maturity. Neural tube defects are caused by improper folic acid supply to the fetus. CVS does not cause neural tube defects.

Which assessments should be the nurse complete for a neonate who has abdominal distension, bilious emesis, and bloody stools? Select all that apply. Assess abdominal girth daily. Feed the neonate 15 mL of formula. Listen to bowel sounds in all four quadrants. Assess hemoccult results on the current stool. Palpate abdomen for distended loops of bowel. Assess for grunting, tachypnea, and substernal retractions.

Listen to bowel sounds in all four quadrants. Abdominal distension, bilious emesis and bloody stools are signs of necrotizing enteral colitis. The nurse would assess bowel sounds in all four quadrants. Correct Assess hemoccult results on the current stool. Abdominal distention, bilious emesis and bloody stools are signs of necrotizing enteral colitis. The nurse would assess a hemoccult on the current stool, which confirms the presence of blood in the stool. Correct Palpate abdomen for distended loops of bowel. Necrotizing enteral colitis may be present with distended loops of bowel, besides distension, bilious emesis and bloody stools. The nurse must assess for distended loops of bowel since abdominal distension is present. Correct Assess for grunting, tachypnea, and substernal retractions. Abdominal distension may increase pressure on the abdomen causing the appearance of tachypnea, grunting and retractions.

The nurse observes that a client has a decreased uterine size and has lost 5 lbs. at 42 weeks of gestation. Which signs should the nurse assess in the neonate after birth? Seizures Long nails Dry and peeling skin High body temperature Meconium-stained skin

Long nails Dry and peeling skin Meconium-stained skin Rationale Reduced uterine size and a maternal weight loss of 3 lb/week or more are clinical manifestations of postterm pregnancies. Postterm infants are at an increased risk of meconium aspiration. These infants are most likely to have postmaturity syndrome, which is characterized by long nails, dry and peeling skin, and meconium-stained skin. Seizures (indicative of cerebral palsy) and high body temperature (indicative ofsepsis) are the complications associated with infants who are born to women with chorioamnionitis during labor.

A client gains excessive weight during pregnancy, her laboratory reports reveal low levels of placental growth factors, and she is expected to deliver during the colder months of the year. Which medication would help in reducing risk of preeclampsia in the patient? Nifedipine (Adalat) Low-dose aspirin (Anacin) Magnesium sulfate (Sulfamag) Vitamin C supplement (Vita-C)

Low-dose aspirin (Anacin) Rationale A nonobese client who gains excessive weight during pregnancy is at an increased risk of developing preeclampsia. The low levels of placental growth factor and expected delivery during colder months of the year also increase the risk for preeclampsia. Low-dose aspirin (Anacin) helps lower the risk of preeclampsia and maintain general health. Nifedipine (Adalat) is an antihypertensive drug, not highly effective in preventing preeclampsia. Magnesium sulfate (Sulfamag) is the drug of choice for treating eclamptic seizures and preventing repeated seizures. A vitamin C supplement (Vita-C) does not help prevent preeclampsia.

On reviewing the results of the Doppler umbilical flow analysis of a pregnant client, the nurse finds that the client has an elevated S/D ratio. Which does this finding signify in the client? Trisomy 13 Potter syndrome Down syndrome Lupus erythematosus

Lupus erythematosus Rationale Lupus erythematosus is an autoimmune disorder that affects different body systems. An elevated S/D ratio indicates a poorly perfused placenta, which may be due lupus erythematosus. An elevated S/D ratio is not indicative of trisomy 13, which is a chromosomal abnormality. Potter syndrome is an atypical physical appearance of the fetus. Oligohydramnios may result in Potter syndrome, but an elevated S/D ratio is not indicative of Potter syndrome. Down syndrome is a chromosomal abnormality. An elevated S/D ratio does not indicate Down syndrome.

Nurses should be aware of the strengths and limitations of various biochemical assessments during pregnancy, including that: chorionic villus sampling (CVS) is becoming more popular because it provides early diagnosis. screening for maternal serum alpha-fetoprotein (MSAFP) levels is recommended only for women at risk for neural tube defects. percutaneous umbilical blood sampling (PUBS) is one of the quad-screen tests for Down syndrome. MSAFP is a screening tool only; it identifies candidates for more definitive procedures.

MSAFP is a screening tool only; it identifies candidates for more definitive procedures. Rationale MSAFP is a screening tool, not a diagnostic tool. Further diagnostic testing is indicated after an abnormal MSAFP. CVS does provide a rapid result, but it is declining in popularity because of advances in noninvasive screening techniques. MSAFP screening is recommended for all pregnant women. MSAFP, not PUBS, is part of the quad-screen tests for Down syndrome.

Nurses should be aware of the strengths and limitations of various biochemical assessments during pregnancy, including that: Chorionic villus sampling (CVS) is becoming more popular because it provides early diagnosis. Screening for maternal serum alpha-fetoprotein (MSAFP) levels is recommended only for women at risk for neural tube defects. Percutaneous umbilical blood sampling (PUBS) is one of the quad-screen tests for Down syndrome. MSAFP is a screening tool only; it identifies candidates for more definitive procedures.

MSAFP is a screening tool only; it identifies candidates for more definitive procedures. CVS does provide a rapid result, but it is declining in popularity because of advances in noninvasive screening techniques. MSAFP screening is recommended for all pregnant women. MSAFP, not PUBS, is part of the quad-screen tests for Down syndrome. MSAFP is a screening tool, not a diagnostic tool. Further diagnostic testing is indicated after an abnormal result.

The nurse is caring for a client with poorly controlled gestational diabetes. What risks to the fetus does the nurse anticipate during the later pregnancy stages and birthing process? Select all that apply. Ketoacidosis Macrosomia Shoulder dystocia Facial nerve injury Hyperglycemia

Macrosomia Shoulder dystocia Facial nerve injury Rationale Poor glycemic control during the later stages of pregnancy increases the rate of fetal macrosomia, defined as a birth weight of more than 4000 g. The macrosomic fetus of the diabetic mother tends to have a disproportionate increase in shoulder, trunk, and chest size, leading to risk of shoulder dystocia. Failure of fetal descent or labor progress leads to difficult vaginal birth. A vaginal birth may lead to birth injuries in the infant, such as facial nerve injury. Ketoacidosis is a result of uncontrolled glycemia, which affects the mother during pregnancy. Hypoglycemia, not hyperglycemia, is a risk for infants born to diabetic mothers. In the later stages of pregnancy, hypoglycemia may occur as insulin doses are adjusted to maintain a normal blood glucose level.

Which drug prevents the risk of cerebral palsy in the fetus? Nifedipine (Adalat) Propranolol (Inderal) Dexamethasone (Decadron) Magnesium sulfate (Epsom salts

Magnesium sulfate (Epsom salts) Rationale Magnesium sulfate (Epsom salts) is a tocolytic agent used for preventing or reducing the risk of cerebral palsy in the fetus if preterm birth appears inevitable. Nifedipine (Adalat) is a calcium channel blocker used in the tocolytic therapy for preterm labor. Propranolol (Inderal) is used to reverse the intolerable cardiovascular effects of terbutaline (Brethine). Dexamethasone (Decadron) is an antenatal glucocorticoid that is used to prevent the risk of respiratory distress syndrome in the fetus.

If woman on bedrest for placenta previa starts having contractions, what should be done?

Magnesium sulfate infusion

A pregnant client experienced preterm labor at 30 weeks gestation. Upon assessing the client the nurse finds that the newborn is at risk of having cerebral palsy. Which medication administration should the nurse perform to prevent cerebral palsy in the newborn? Calcium gluconate. Magnesium sulfate. Glucocorticoid drugs. Antibiotic medications.

Magnesium sulfate. Rationale Newborns who are born before 32 weeks' gestation may be at risk of cerebral palsy. Administering magnesium sulfate to the client can prevent this risk, because it would delay delivery. Calcium gluconate is administered when the preterm child has magnesium toxicity. This intervention would not help to prevent cerebral palsy. Also, the newborn would not have a fully developed respiratory system. Therefore, administering glucocorticoids to the pregnant client would help to prevent risk of respiratory depression in the baby. However, it does not help in preventing cerebral palsy. Administering antibiotics during labor would help prevent neonatal group B streptococci infection.

A nurse is making rounds on a client who recently delivered via the vaginal route, and suspects that the client is having excessive postpartum bleeding. Which intervention would be the priority action taken by the nurse at this time? Call the physician Massage the uterine fundus Increase the rate of intravenous fluids Monitor pad count and perform catherization

Massage the uterine fundus Rationale Massaging of the uterine fundus would be a priority action to help expel clots and stimulate uterine contractions to constrict blood flow. Although the physician may well have to be called, the initial priority action would be for the nurse to massage the uterine fundus. Although the rate of intravenous fluids may have to be increased, this is not the priority action at this time. Monitoring of pad count and lochia flow is needed but it is not the priority action at this time. Additionally, catherization may be needed if bladder distension is noted but, again, it is not the priority action at this time.

Which postpartum infection is most often contracted by first-time mothers who are breastfeeding? Endometritis Wound infections Mastitis Urinary tract infections (UTIs)

Mastitis Rationale Mastitis is an infection in a breast, usually confined to a milk duct. Most women who suffer this are first-time mothers who are breastfeeding. Endometritis is the most common postpartum infection. Incidence is higher after a cesarean birth and not limited to first-time mothers. Wound infections are also a common postpartum complication. Sites of infection include both a cesarean incision and the episiotomy or repaired laceration. The gravidity of the mother and her feeding choice are not factors in the development of a wound infection. UTIs occur in 2% to 4% of all postpartum women. Risk factors include catheterizations, frequent vaginal examinations, and epidural anesthesia.

Which postpartum infection is most often contracted by first-time mothers who are breastfeeding? Endometritis Wound infections Mastitis Urinary tract infections (UTIs)

Mastitis Mastitis is infection in a breast, usually confined to a milk duct. Most women who get it are first-timers who are breastfeeding. Endometritis is the most common postpartum infection. Its incidence is higher after a cesarean birth, not in first-time mothers. Wound infections are also a common postpartum complication. Sites of infection include both a cesarean incision and the episiotomy or repaired laceration. The gravidity of the mother and her feeding choice are not factors in the development of a wound infection. UTIs occur in 2% to 4% of all postpartum women. Risk factors include catheterizations, frequent vaginal exams, and epidural anesthesia.

What is an appropriate indicator for performing a contraction stress test? Increased fetal movement and small for gestational age Maternal diabetes mellitus and postmaturity Adolescent pregnancy and poor prenatal care History of preterm labor and intrauterine growth restriction

Maternal diabetes mellitus and postmaturity Rationale Maternal diabetes mellitus and postmaturity are two indications for performing a contraction stress test. Decreased fetal movement is an indicator for performing a contraction stress test; the size (small for gestational age) is not an indicator. Although adolescent pregnancy and poor prenatal care are risk factors for poor fetal outcomes, they are not indicators for performing a contraction stress test. Intrauterine growth restriction is an indicator; history of a previous stillbirth, not preterm labor, is another indicator.

Key to an optimal pregnancy outcome for diabetics

Maternal glucose control before conception and throghout the gestational period

The nurse is caring for a pregnant client with type 2 diabetes. What does the nurse teach the client about glucose metabolism in the first trimester? Maternal glucose levels are affected by nausea and cravings. The client's insulin dose may need to be increased to prevent hyperglycemia. The fetus will produce insulin in the fifth week of gestation. The client's fasting blood glucose level will increase.

Maternal glucose levels are affected by nausea and cravings. Rationale Dietary fluctuations in early pregnancy due to nausea, vomiting, and cravings affect maternal glycemic control. These fluctuations may result in reduced blood glucose levels. The nurse reminds the client that her insulin dose may need to be decreased to prevent hypoglycemia. The fetus begins to produce enough insulin to utilize maternal glucose in the tenth week of gestation. Increases in estrogen and progesterone levels stimulate the pancreas to produce insulin. An increase in insulin promotes increased peripheral use of glucose and decreased blood glucose. This leads to decreased fasting blood glucose levels.

What are maternal and neonatal risks associated with gestational diabetes mellitus? Maternal premature rupture of membranes and neonatal sepsis. Maternal hyperemesis and neonatal low birth weight. Maternal preeclampsia and fetal macrosomia. Maternal placenta previa and fetal prematurity

Maternal preeclampsia and fetal macrosomia. Rationale Women with gestational diabetes have twice the risk of developing hypertensive disorders such as preeclampsia, and the baby usually has macrosomia. Premature rupture of membranes and neonatal sepsis are not risks associated with gestational diabetes. Hyperemesis is not seen with gestational diabetes, nor is there an association with low birth weight of the infant. Placental previa and subsequent prematurity of the neonate are not risks associated with gestational diabetes.

What are the metabolic changes associated with pregnancy? Select all that apply. Maternal production of insulin increases during the first trimester. Fasting blood glucose levels will decrease during the first trimester. The patient's tolerance to glucose increases in the second trimester. There is enough glucose for the fetus during the second trimester. Maternal insulin requirements increase during the first trimester.

Maternal production of insulin increases during the first trimester. Fasting blood glucose levels will decrease during the first trimester. There is enough glucose for the fetus during the second trimester. Rationale In the first trimester, an increase in estrogen and progesterone production stimulates the beta cells in the pancreas to increase insulin production. The beta cells also increase peripheral use of glucose and, in turn, decrease the overall blood glucose levels. This reduces fasting glucose levels by approximately 10%. During the second and third trimesters hormonal changes increase insulin resistance and ensure an abundant supply of glucose for the fetus. The body develops insulin resistance as a glucose-sparing mechanism. In the second trimester hormonal changes decrease tolerance to glucose. Maternal insulin requirements increase from 18 to 24 weeks of gestation, not in the first trimester.

Nurses should be aware of the strengths and limitations of various biochemical assessments during pregnancy, including what? Chorionic villus sampling (CVS) is becoming more popular because it provides early diagnosis. Screening for maternal serum alpha-fetoprotein (MSAFP) levels is recommended only for women at risk for neural tube defects. Percutaneous umbilical blood sampling (PUBS) is one of the quad-screen tests for Down syndrome. Maternal serum alpha-fetoprotein (MSAFP) is a screening tool only; it identifies candidates for more definitive procedures.

Maternal serum alpha-fetoprotein (MSAFP) is a screening tool only; it identifies candidates for more definitive procedures. Rationale MSAFP is a screening tool, not a diagnostic tool. Further diagnostic testing is indicated after an abnormal MSAFP. CVS does provide a rapid result, and it is declining in popularity because of advances in noninvasive screening techniques. MSAFP screening is recommended for all pregnant women. MSAFP, not PUBS, is part of the quad-screen tests for Down syndrome.

A pregnant client who is in preterm labor has been prescribed dexamethasone (Decadron). What benefit of the drug would the nurse identify in the client? Maturation of fetal lungs Relaxation of smooth muscles Inhibition of uterine contractions (UCs) Central nervous system (CNS) depression

Maturation of fetal lungs Rationale Dexamethasone (Decadron) is a glucocorticoid and is administered to clients having preterm labor, because it promotes fetal lung maturation. The drug facilitates the release of enzymes that induce production or release of lung surfactant. Tocolytics are used to inhibit UCs. Magnesium sulfate is a CNS depressant. Tocolytics also causes the relaxation of smooth muscles.

miscarriage care

Medical management if bleeding and infection are not present -Prostoglandin medications (e.g. mixoprostol (cytotec) are given and are effective in completing the miscarriage wihin 7 days Side effects: -nausea -vomiting -diarrhea If the cervix dilates, can the pregnancy continue? No, miscarriage is inevitable suction curretage if anything needs to be removed

Tocolytics

Medications given to arrest labor after uterine contractions and cervical changes have occurred -magnesium sulfate= most commmonly used because it has the least maternal and fetal adverse reactions -beta-adrenergic agonists (e.g. ritrodine and terbutaline (brethine)) -Nifedipine (Procardia)- calcium channel blocker that suppresses contractions

Preterm Premature Rupture of Membranes (Preterm PROM)

Membrane rupture before 37 0/7 weeks of gestation -associated with 10% of all preterm births -most likely occurs due to weakening of the amniotic membranes caused by inflammation, stress from uterine contractions, or other factors that cause increased intrauterine pressure -infection of urogenital tract Complications for fetus are related to intrauterine infection, cord prolapse, umbilical cord compression associated with oligohydramnios, and placental abruption

A woman who is at 36 weeks' gestation thinks she is experiencing labor. Which signs or symptoms would support the woman's suspicion? Select all that apply. Headache Menstrual-like cramps Reports of constipation Reports of pelvic pressure Reports that the baby is "balling up"

Menstrual-like cramps Menstrual-like cramps are a sign of preterm labor, which may be related to premature uterine contractions Reports of pelvic pressure Pelvic pressure is a symptom of preterm labor, and may also be related to premature uterine contractions Reports that the baby is "balling up" A sensation that the baby is frequently "balling up" is a symptom of preterm labor

What medication is used for ectopic pregnancy and how does it work?

Methotrexate (Trexall), a folic acid antagonist, may be prescribed to inhibit cell division in the developing embryo

Which of the following antihypertensive medications would cause a pregnant woman to have a positive Coombs test result? Nifedipine (Procardia) Methyldopa (Aldomet) Labetalol Hydrochloride (Transdate) Hydrazaline (Apresoline)

Methyldopa (Aldomet) Rationale A positive Coombs test result can occur in about 20% of patients on Methyldopa (Aldomet). Nifedipine, labetalol hydrochloride and hydrazaline would not cause this effect.

Which of the following antihypertensive medications would cause a pregnant woman to have a positive Coombs test result? Nifedipine (Procardia) Methyldopa (Aldomet) Labetalol hydrochloride (Trandate) Hydralazine (Apresoline)

Methyldopa (Aldomet) A positive Coombs test result can occur in about 20% of patients taking methyldopa (Aldomet). None of the other drugs listed would have this effect.

The nurse is teaching a client diagnosed with phenylketonuria (PKU) about foods to be avoided in the daily diet. Which foods can have an adverse effect on the mother and fetus? Milk Eggs Nuts Fruits Vegetables

Milk Eggs Nuts Rationale A client diagnosed with PKU must avoid milk, eggs, and nuts in her diet before conception and throughout the pregnancy. The client must have a modified diet that excludes all high-protein foods. The client lacks the enzyme phenylalanine hydrolase; this lack impairs her body's ability to metabolize the amino acid phenylalanine, found in all protein foods. Most fruits and vegetables contain very little or no proteins; the client may include these in her diet.

Poor gycemic control around the time of conception and in early weeks of pregnancy is associated with what?

Miscarriage

For what condition is a client at risk in early pregnancy due to poorly controlled hyperglycemia? Miscarriage Hydramnios Preeclampsia Ketoacidosis

Miscarriage Rationale Poorly controlled hyperglycemia at the time of conception and early pregnancy often leads to miscarriage. The risk of having a miscarriage increases with the duration and severity of the client's diabetes. Hydramnios occurs in the third trimester of pregnancy in the diabetic client. It may be due to increased glucose concentration in the amniotic fluid from maternal and fetal hyperglycemia, which induces fetal polyuria. Preeclampsia occurs in later pregnancy and in the postpartum period. Pregnant clients with poorly controlled hyperglycemia at the beginning of pregnancy, especially if combined with nephropathy and hypertension, are at higher risk of developing preeclampsia. Ketoacidosis occurs in the second and third trimesters. This is the accumulation of ketones in the body due to hyperglycemia, and it may lead to metabolic acidosis.

Hypothyroidism occurs in 2 to 3 pregnancies per 1000. Pregnant women with untreated hypothyroidism are at risk for what? Miscarriage Macrosomia Gestational hypertension Placental abruption Stillbirth

Miscarriage Gestational hypertension Placental abruption Stillbirth Rationale Hypothyroidism is often associated with both infertility and an increased risk for miscarriage. These outcomes can be improved with early diagnosis and treatment. Pregnant women with hypothyroidism are more likely to experience both preeclampsia and gestational hypertension. Placental abruption and stillbirth are risks associated with hypothyroidism. Placental abruption and stillbirth are risks associated with hypothyroidism. Infants born to mothers with hypothyroidism are more likely to be of low birth weight or preterm.

During the assessment of a pregnant client, the nurse finds that the client has cramps, closed cervical os without any tissue perforation, and absence of uterine bleeding. The client's ultrasonic report reveals that the fetal growth is less than expected per the gestational period, and absence of fetal heartbeat. Which drug would be added to the client's prescription? Ampicillin (Amcill) Methotrexate (MTX) Misoprostol (Cytotec) Ergonovine (Methergine)

Misoprostol (Cytotec) Rationale Cramps, a closed cervical os without any tissue perforation, absence of uterine bleeding, and fetal growth inappropriate to the gestational age and no fetal heartbeat indicate miscarriage. A miscarriage with no bleeding and infection should be treated conservatively. Misoprostol (Cytotec) is a prostaglandin drug that is administered either orally or vaginally to complete the miscarriage within 7 days. Ampicillin (Amcill) is an antibiotic used to treat infection. MTX is used to treat ectopic pregnancy. Ergonovine (Methergine) is used for uterine contractions.

The primary health care provider orders magnesium sulfate (Sulfamag) for a pregnant client who is being transported to the tertiary care center. What actions does the nurse follow according to the protocol? Administer the drug intravenously. Mix the drug with a local anesthetic agent. Administer 5 g to each buttock as a loading dose. Administer 5 g as a maintenance dose every 8 hours. Administer 5 g as a maintenance dose alternately to each buttock.

Mix the drug with a local anesthetic agent. Administer 5 g to each buttock as a loading dose. Administer 5 g as a maintenance dose alternately to each buttock Rationale The nurse should administer the magnesium sulfate (Sulfamag) intramuscularly to a pregnant client who is being transported to the tertiary care center. The medication should be mixed with a local anesthetic to reduce the pain caused by injection. The nurse should administer 5 g of magnesium sulfate (Sulfamag) to each buttock as a loading dose of 10 g total. A maintenance dose of 5 g should be administered alternately to each buttock every 4 hours. The intravenous route is not preferred in clients who are being transported to the tertiary care center. The nurse should not administer the maintenance dose every 8 hours, because it may lead to insufficient action of the medication.

The ultrasound report of a 12-week pregnant woman shows snowstorm pattern. Upon further examination, the nurse finds elevated human chorionic gonadotropin (hCG) levels and dark brown vaginal discharge. What complication does the nurse expect in the client? Hemorrhage Hypertension Hyperglycemia Molar pregnancy

Molar pregnancy Rationale Snowstorm pattern in the ultrasound, elevated hCG, and dark brown vaginal discharge indicate that the client has a hydatidiform mole. The risk of hemorrhage is predominant in a client with placenta previa. The blood pressure of the client is not affected by the hydatidiform mole. Therefore, the client would not be at a risk of hypertension. a hydatidiform mole does not alter the blood glucose levels. Therefore, the client would not necessarily have hyperglycemia.

Interventions for Mild HTN

Monitor BP Monitor fetal activity and fetal growth -Encourage frequent rest periods; lie in lateral position -Admin HTN meds Monitor I&Os -Eval renal function thru BUN, Creatinine, 24hr levels for creatinine clearance and protein

The nurse is preparing a patient for cesarean delivery because of umbilical cord prolapse. Which is the nurse's most appropriate action? Provide support to the patient Administer tocolytics as prescribed Monitor fetal heart rate (FHR) continuously Document the status of the fetus and mother at least every five minutes

Monitor fetal heart rate (FHR) continuously Cord prolapse presents an acute risk for fetal hypoxia, a problem that would manifest with decelerations in FHR. Monitoring FHR is a nursing assessment that directly addresses the most acute risk to the fetus and consequently should be prioritized.

Rupture uterus interventions

Monitor for and treat signs of shock ( admin oxygen, IV fluids, Blood products) Prepare client for c-section & possible hysterectomy Emotional support

Nsg interventions for an HIV client during the postpartum period?

Monitor for signs of infection, place the mother in protective isolation if she is immunocompromised, restrict breast feeding, instruct the mother to monitor for signs of infection and to report any signs if they occur

Nsg interventions for chorioamnionitis?

Monitor maternal vital signs and FHR, uterine tenderness, contractions, and fetal activity, monitor results of blood cultures, prepare for amniocentesis to obtain amniotic fluid for Gram stain and leukocyte count, administer antibiotics as prescribed after cultures have been obtained, administer oxytocic meds as prescribed to increse uterine tone, prepare to obtain neonatal cultures after delivery

A neonate who required CPAP and IV fluid therapy is being discharged to home. Which intervention is most important for the nurse to execute? Weigh the infant before and after feedings Measure the hematocrit on the day of discharge Complete the metabolic screening prior to discharge Monitor oxygen saturation levels with the neonate in the car seat the parents will use

Monitor oxygen saturation levels with the neonate in the car seat the parents will use Neonates who required CPAP should be evaluated for apnea and bradycardia when in the car seat the parents will use. This will help with identifying desaturation and bradycardia in the neonate.

A client is undergoing percutaneous umbilical blood sampling (PUBS). What is the best intervention for the nurse to perform after conducting the test? Monitor the fetal heart rate (FHR). Give fluids to the client frequently. Elevate the client's bed to a 60 -degree angle. Check the patient's blood glucose levels.

Monitor the fetal heart rate (FHR). Rationale Percutaneous umbilical blood sampling (PUBS) is an invasive procedure in which the fetal umbilical vessel is punctured for blood transfusion or sample collection. After 1 to 2 hours of performing this procedure, the nurse should monitor the fetal heart rate (FHR) to ensure safety. PUBS does not cause loss of fluids or mouth dryness. Therefore, the nurse does not need to give fluids to the patient frequently. The nurse should not elevate the client's bed to a 60-degee angle, because it may cause discomfort to the client. PUBS does not alter blood glucose levels. So, the nurse does not need to check the client's blood glucose levels.

Nsg interventions for the pregnant client with cardiac disease in labor?

Monitor vitals frequently, place client on cardiac monitor and on external fetal monitor, maintain bed rest with client lying on her side with her head and shoulders elevated, administer O2 as prescribed manage pain in early labor use conytolled pushing efforts to decrease cardiac stress

Nsg interventions for cardiac disease in pregnancy?

Monitor vitals, FHR, and condition of fetus, limit physical activities, stress the need for sufficient rest, monitor for signs of cardiac stress and decompensation (cough, fatigue, dyspnea, chest pain, and tachycardia), encourage adequate nutrition to prevent anemia, avoid excessive weight gain, monitor for signs of HF and pulmonary edema low sodium diet be prescribed for the pregnant client with cardiac disease To prevent fluid retention and heart failure

Nsg interventions for a hematoma?

Monitor vitals, monitor client for abnormal pain, apply ice to hematoma site, administer analgesics as prescribed, monitor I&Os, encourage fluids and voiding, client may need to be catheterized, administer blood replacement as prescribed, monitor for signs of infection, administer antibiotics as prescribed, prepare for incision and evacuation of hematoma if necessary after hematoma formation infection is often

Uterine Tachysystole

More than 5 contraction sin 10 minutes, averaged over a 30-minute window -can apply to both spontaneous and stimulated labor

Uterine tachysystole

More than five uterine contractions in 10 minutes averaged over a 30-minute window; it can occur during both spontaneous and stimulated labor.

Chorioamionitis

Most common maternal complication of preterm PROM -A bacterial infection of the amniotic cavity -usually diagnosed by maternal fever, maternal and fetal tachycardia, uterine tenderness, and foul odor of amniotic fluid -usually occurs after membranes rupture or labor begins -infection in mom leads to fetal inflammatory response which results in pulmonary and CNS damage (e.g. increases risk for cerebral palsy)

What happens to women with gestational diabetes after birth?

Most women return to a euglycemic state after delivery, however these individuals have an increased risk of developing diabetes mellitus in their lifetime

The nurse is caring for a postpartum client. The primary health care provider prescribes intravenous magnesium sulfate (Sulfamag). Which other medications does the nurse check for in the treatment regimen to ensure the client's safety? Select all that apply. Narcotics Diuretics Analgesics Calcium channel blockers Central nervous system (CNS) depressants

Narcotics Calcium channel blockers Central nervous system (CNS) depressants Rationale To ensure the client's safety, the nurse should check the treatment regimen before administering intravenous magnesium sulfate (Sulfamag). This drug may synergize the action of drugs such as narcotics, calcium channel blockers, and CNS depressants. Therefore, narcotics, calcium channel blockers, and CNS depressants should be used with caution in a client undergoing intravenous magnesium sulfate (Sulfamag) treatment. Diuretics and analgesics are not affected by magnesium sulfate (Sulfamag). Therefore, checking for these medications before administering intravenous magnesium sulfate (Sulfamag) is not necessary.

A client who is 32 weeks pregnant visits a maternal clinic for a routine health checkup. The ultrasound and magnetic resonance imagining (MRI) reveal that the woman is at risk of placenta accreta. Which intervention should be performed during the delivery to ensure client's safety? Blood transfusion Hysterectomy after delivery Natural removal of the placenta Administration of uterine contractile drugs

Natural removal of the placenta Rationale Artificial removal of the placenta leads to excessive bleeding, which can result in hemorrhage and sometimes death. So, the primary health care provider teaches the surgical team about the risk of placenta accrete and instructs them to remove the placenta naturally to reduce bleeding. Blood transfusion is a common intervention during labor that is used to restore blood volume and components. A hysterectomy is done when the bleeding is uncontrollable. Uterine contractile drugs, such asoxytocin (Pitocin), are administered to induce labor, and are not specific to clients who are at risk of placenta accrete

A patient who is pregnant presents to the clinic with large amounts of bright red vaginal bleeding. What data are priority for the nurse to obtain? Select all that apply. Douching history What the patient last ate Nature and location of the pain How many children the patient has Fundal height and last menstrual period

Nature and location of the pain A priority evaluation for a patient with late pregnancy hemorrhaging is nature and location of the pain. Correct How many children the patient has Obstetric history is important to determine a patient's risk for certain late-stage hemorrhagic conditions and should be a priority assessment. Correct Fundal height and last menstrual period The gestation period is important information to determine if the fetus is viable. Fundal height and the patient's last menstrual period can provide this information

The newborn and HIV description

Neonates born to HIV-positive mothers may test positive because antibodies received from the mother may persist for 18 months after birth, all neonates acquire maternal antibody to HIV infection, but not all acquire infection

The nurse is reviewing the amniocentesis reports of a client who has completed 20 weeks of pregnancy. The reports reveal the presence of high alpha-fetoprotein (AFP) levels. What can the nurse infer from this information related to the clinical condition of the fetus? Cardiac disorder. Neurologic disorder. Circulatory disorder. Pulmonary disorder.

Neurologic disorder. Rationale High AFP levels after 15 weeks of gestation indicate an open neural tube or other disorders relating to the central nervous system. AFP levels in amniotic fluid cannot test cardiac disorders. Doppler blood flow analysis can be used to assess circulatory disorders in the fetus. The lecithin/sphingomyelin [L/S] ratio of the amniotic fluid is a useful predictor of pulmonary disorders in the fetus.

What are the potentially fatal complications associated with diabetes? Neuropathy Nephropathy Atherosclerosis Cardiomyopathy Restrictive lung disease

Neuropathy Nephropathy Atherosclerosis Rationale Diabetes is a condition characterized by high blood sugar. High blood sugar results in damage to the peripheral nerve, which is referred to as neuropathy. Increased workload on the renal system imposed by high blood sugar results in nephropathy. Diabetes also accelerates the process of atherosclerosis. High blood sugar levels do not impair the function of cardiac muscles and lung function. Therefore, diabetes is not associated with cardiomyopathy or restrictive lung disease.

Psychologic Response of Mom- Impact on birth

Neurotransmitters released during stress can cause dysfunctional labor -increased pain perception -reduced uterine contractility -inhibited cervical dilation

Client Education for Thrombophlebitis

Never massage the leg. •Avoid crossing the legs. •Avoid prolonged sitting. •Avoid constrictive clothing. •Avoid pressure behind the knees. •Know how to apply elastic stockings (support hose) if prescribed. •Understand the importance of anticoagulant therapy if prescribed. •Understand the importance of follow-up with the health care provider.

Doppler blood flow analysis

Noninvasive study of blood flow in the fetus and placenta

Magnetic resonance imaging

Nonivasive radiologic technique used for obstetic and gynecologic diagnosis by providing excellent pictures of soft tissue without the use of ionizing radiation.

Rupture of the uterus

Nonsurgical disruption of all uterine layers; it is a rare but life-threatening ob- stetric injury occurring during labor and birth. The major risk factor for its occurrence is scarred uterus; usually occurs during trial of labor for attempted vaginal birth after cesarean (VBAC)

The fetal fibronectin test of a pregnant client is positive, and her cervical length is found to be 32 mm. What will the nurse interpret from these observations regarding the client's pregnancy status? Normal gestation labor. Indicated preterm labor. Spontaneous preterm labor. Miscarriage in the next week.

Normal gestation labor. Rationale The cervical length and fibronectin test help to identify the risk of preterm delivery in the client. If the cervical length of the client is greater than 30 mm, the client would not have preterm labor, irrespective of having the symptoms of preterm labor. Because the cervical length of the patient is 32 mm, the client may have normal gestational labor. Cervical length and the fibronectin test do not indicate whether the client would have a miscarriage. If the cervical length is less than 30 mm, the client may have indicated or spontaneous preterm labor.

Oral contraceptives and diabetics

Not recommended (controversial) because of risks of thromboembolitic events (which is already increased for diabetics anyway)

A woman with deep vein thrombosis (DVT) begins coughing during auscultation of the lungs. Which action should the nurse take first? Notify the health care provider Assess the woman's pulse oximetry Auscultate abnormal breath sounds during the cough Pause and resume auscultation once the woman has stopped coughing

Notify the health care provider Coughing from a woman with a deep vein thrombosis could indicate development of a pulmonary embolism. The nurse should notify the health care provider immediately.

The antepartum nurse is caring for a woman who is 35 weeks' gestation. The woman reports swelling, tenderness, and redness of the left leg, followed one week later by respiratory symptoms, including coughing and difficulty breathing/panting. Which is the first action the nurse should take? Assess the woman's pulse oximetry. Auscultate the woman's breath sounds. Notify the health care provider immediately. Lift the woman's affected leg and flex the foot toward the head.

Notify the health care provider immediately. This woman's symptoms are suggestive of a pulmonary embolism that resulted from a deep vein thrombosis. The health care provider should be notified immediately.

The nurse is examining a woman who delivered via cesarean section. The area around the incision is bright red and tender to the touch. Which action should the nurse take first? Notify the wound care team. Notify the health care provider. Clean the wound with soap and water. Administer the patient's as-needed pain medication.

Notify the health care provider. A bright red incision that is tender to the touch is a sign of wound infection and should be reported to the health care provider immediately.

A woman in the immediate postnatal period complains of severe pelvic pain. The fundus is firm, and no excessive bleeding is visible. Which action should the nurse take first? Massage the fundus. Reposition the patient. Notify the health care provider. Administer the patient's as-needed pain medication.

Notify the health care provider. Severe pelvic pain with no signs of excessive bleeding indicates the possibility of a hematoma. The health care provider should be notified immediately.

A woman who showed signs of postpartum depression at a previous visit reports sleeping little for the past week. However, the woman also reports feeling "great" and expresses frustration at the nurse's questions. Which action should the nurse take first? Notify the health care provider. Move on to the next part of the assessment. Repeat the questions using simplified phrasing. Refer the woman to the social services department.

Notify the health care provider. A period of sleeplessness and euphoria after a period of depression is characteristic of bipolar II disorder. The health care provider should be notified of this change in the woman's reported symptoms.

After reviewing the blood glucose levels of a client, the nurse finds that the client is hypoglycemic and gives three glucose tablets to the client. After 15 minutes, the nurse finds that the client is still hypoglycemic, and again administers three glucose tablets. What should the nurse do next, if the client's blood glucose is 60 mg/dl after 15 minutes? Notify the primary health care provider. Administer 50% dextrose intravenous push. Obtain blood samples for blood gas analysis. Give three more glucose tablets to the client.

Notify the primary health care provider. Rationale The hypoglycemic client's blood sugar level has not risen after giving three glucose tablets twice over an interval of 15 minutes each. In this situation, the nurse should notify the primary health care provider so that emergency intervention can be provided. Administering three more glucose tablets would not be of use, because this intervention had not been useful to improve the client's blood sugar level previously. An intravenous push of 50% dextrose should be done if the hypoglycemic client has become unconscious. In this case, arterial blood gas analysis would not be helpful in finding out the cause of persistent low blood sugar.

Nursing Interventions for Vacuum-Assisted Birth

Nursing Interventions for Vacuum-Assisted Birth -Assess FHR frequently and document -Encourage woman to push during contractions/ when traction is applied to vacuum -do not exceed the "green zone" of pressure indicated by the pump -Document the number of pulls attempted and any pop-offs that occur

A client has undergone an amniocentesis for evaluation of fetal well-being. Which intervention would be included in the nurse's plan of care postprocedure? Perform ultrasound to determine fetal positioning. Observe the client for possible uterine contractions. Administer Rhogam to the client if she is Rh negative Perform a mini catherization to obtain urine specimen to assess for bleeding. Administer Vitamin K to prevent infection from occurring.

Observe the client for possible uterine contractions. Administer Rhogam to the client if she is Rh negative Rationale The client should be observed postprocedure for possible onset of uterine contractions. A client who is Rh negative will require a Rhogam injection postprocedure. Ultrasound is used prior to the procedure as a visualization aid to assist with insertion of the transabdominal needle. There is no need to perform a minicatherization to obtain a urine specimen to assess for bleeding, because this is not considered a typical presentation or complication. Vitamin K is not administered and is not associated with infection risk, but with clotting.

Nsg interventions for the client with diabetes during the postpartum period?

Observe the mother closely for a hypoglycemic reaction because a precipitous decline in insulin requirements normally occurs (the mother may not require insulin for the first 24 hours), reregulate insulin needs according to blood glucose reading, assess dietary needs based on blood glucose testing, monitor for signs of infection or postpartum hemorrhage

A patient has undergone an amniocentesis for evaluation of fetal well-being. Which intervention would be included in the nurse's plan of care after the procedure? Select all that apply. Perform ultrasound to determine fetal positioning. Observe the patient for possible uterine contractions. Administer RhoGAM to the patient if she is Rh negative. Perform a minicatheterization to obtain a urine specimen to assess for bleeding.

Observe the patient for possible uterine contractions. Ultrasound is used prior to the procedure as a visualization aid to assist with insertion of transabdominal needle. There is no need to assess the urine for bleeding as this is not considered to be a typical presentation or complication. Administer RhoGAM to the patient if she is Rh negative. Ultrasound is used prior to the procedure as a visualization aid to assist with insertion of transabdominal needle. There is no need to assess the urine for bleeding as this is not considered to be a typical presentation or complication.

Amniotic fluid embolism (AFE); anaphylactoid syndrome of pregnancy

Obstetric emergency in which amniotic fluid enters the maternal circulation triggering a rapid, complex series of pathophysiologic events leading to disseminated intravascular coagulation (DIC), hypotension, and hypoxia

A pregnant client has a systolic blood pressure that exceeds 160 mm Hg. Which action should the nurse take for this client? Monitor uterine contractions Obtain a prescription for antihypertensive medications Restrict intravenous and oral fluids to 125 ml per hour Monitor fetal heart rate

Obtain a prescription for antihypertensive medications Rationale Systolic BP exceeding 160 mm Hg indicates severe hypertension in the client. The nurse should alert the health care provider and obtain a prescription for antihypertensive medications, such as nifedipine (Adalat) and labetalol hydrochloride (Normodyne). Oral and IV fluids are restricted when the client is at risk for pulmonary edema. Monitoring FHR and UCs is a priority when the client experiences a trauma, so that any complications can be addressed immediately.

Which actions would the nurse take when a pregnant client has convulsions? Obtain a prescription for magnesium sulfate Assess the client's airway, breathing, and pulse Lower the bed and turn the client onto one side Not leave the client for more than 10 minutes Raise the side rails of the bed and pads with pillows

Obtain a prescription for magnesium sulfate Assess the client's airway, breathing, and pulse Lower the bed and turn the client onto one side Raise the side rails of the bed and pads with pillows Rationale The nurse obtains a prescription for magnesium sulfate to prevent further convulsions. The nurse assesses the client's airway, breathing, and pulse to understand the maternal condition. The nurse may need to suction secretions to clear the airway and administer oxygen to maintain sufficient oxygenation in the client. The nurse lowers the bed and turns the client onto one side to prevent aspiration of vomitus. The nurse raises the side rails of the bed and pads with pillows to prevent a fall. The nurse may call for help, but should not leave the client's bedside, because the patient is in a serious condition.

The nurse is providing care for a patient in labor, and the health care provider has just stated the patient's need for a forceps-assisted delivery. What should the nurse's preparation include? Obtaining a urinary catheter Establishing intravenous (IV) access Performing a head-to-toe assessment Educating the patient about the risk for lacerations

Obtaining a urinary catheter The patient's bladder must be empty during an operative delivery. Use of an intermittent catheter is expected.

The nurse observes that a pregnant client with gestational hypertension who is on magnesium sulfate therapy is prescribed nifedipine (Adalat). What action does the nurse take? Evaluates the client's renal function test Obtains a prescription for a change of drug Reduces the nifedipine (Adalat) dose by 50% Administers both medications simultaneously

Obtains a prescription for a change of drug Rationale Concurrent use of nifedipine (Adalat) and magnesium sulfate can result in skeletal muscle blockade in the client. Therefore the nurse needs to report immediately to the primary health care provider (PHP) and obtain a prescription for a change of drug. The nurse assesses the client's renal function to determine the risk for toxicity after administering any drug. However, it is not a priority in this case. Reducing the nifedipine (Adalat) dose is not likely to prevent the drug interaction in the client. The nurse does not administer both drugs simultaneously, because it may be harmful for the client.

Preterm labor

Occurs after the 20th week but before 37 week. -Risk factors: H/O medical conditions, present & past obstetric problems, infection, and social/environmental factors, substance abuse. Multiple pregnancy, anemia, age 18, or 40 years old S/S Uterine contractions (paiful or painless), abdominal cramping, low back pain, pelvic pressure or heaviness, change in character and amount of usual discharge, rupture of amniotic membranes presence of fetal fibronectin in cervical canal shotening of cervical length

What is a hematoma?

Occurs following the escape of blood into the maternal tissue after delivery, predisposing conditions include operative delivery with forceps or injury to a blood vessel

What is gestational diabetes?

Occurs in pregnancy (during the 2nd or 3rd trimester) in clients not previously diagnosed as diabetic and occurs when the pancreas cannot respond to the demand for more insulin

Prolapse of Umbilical Cord

Occurs when the cord lies below the presenting part of the fetus Risk factors: -long cord -malpresentation (breech or transverse lie) -unengaged presenting part

Supine Hypotension (Vena Cava Syndrome)

Occurs when venous return to heart is impaired by wt of uterus on vena cava. -Results in partial occlusion of vena cava and aorta and reduced cardiac return, cardiac output and BP S/S Pallor, faintness, dizziness, breathlessness, tachycardia, hypotension, sweating, cool and damp skin, fetal distress. Interventions- position client on her side to shift wt of fetus off vena cava. Monitor vs and FHR

A patient with endometritis asks for something to help relieve abdominal cramping and rates the pain as 3/10. The patient's next dose of pain medication is in 2 hours. Which action should the nurse take first? Offer the patient a heating pad. Lower the head of the bed to 15 degrees. Place the patient in a side-lying position. Contact the health care provider for a one-time dose of pain medication.

Offer the patient a heating pad. Heating pads are a nursing intervention that can act as a comfort measure for patients with endometritis. Application of heat helps to relax the muscles and reduce abdominal cramping.

What are the predisposing conditions to gestational diabetes?

Older than 35 years old, obesity, multiple gestation, family hx of diabetes, large for gestational age fetus

The nurse is caring for a diabetic client who is breastfeeding her infant. Within what time frame following childbirth do the client's insulin requirements return to prepregnancy levels? Immediately after childbirth Seven to 10 days after childbirth On completion of weaning During the lactation period

On completion of weaning Rationale The breastfeeding mother's insulin requirements return to prepregnancy levels after the infant has been completely weaned. At birth, there is a sudden drop in the levels of insulinase following expulsion of the placenta, but they do not return to prepregnancy levels. When the mother is not breastfeeding, the insulin carbohydrate balance returns in 7 to 10 days. Maternal glucose is used up during lactation; therefore the breastfeeding mother's insulin requirement remains low.

For a woman at 42 weeks of gestation, which finding requires more assessment by the nurse? Fetal heart rate of 120 beats/min Cervix dilated 2 cm and 50% effaced Score of 8 on the biophysical profile One fetal movement noted in 1 hour of assessment by the mother

One fetal movement noted in 1 hour of assessment by the mother Rationale Self-care in a postterm pregnancy should include performing daily fetal kick counts three times per day. The mother should feel four fetal movements per hour. If fewer than four movements have been felt by the mother, she should count for 1 more hour. Fewer than four movements in that hour warrants evaluation. A fetal heart rate of 120 beats/min is a normal finding at 42 weeks of gestation. Cervical dilation of 2 cm with 50% effacement is a normal finding in a woman at 42 weeks of gestation. A score of 8 on the BPP is a normal finding in a pregnancy at 42 weeks.

For a woman at 42 weeks of gestation, which finding requires more assessment by the nurse? Fetal heart rate of 116 beats/minute Cervix dilated 2 cm and 50% effaced Score of 8 on the biophysical profile One fetal movement noted in 1 hour of assessment by the mother

One fetal movement noted in 1 hour of assessment by the mother Rationale Self-care in a postterm pregnancy should include performing daily fetal kick counts three times per day. The mother should feel four fetal movements per hour. If the mother has felt fewer than four movements, she should count for 1 more hour. Fewer than four movements in that hour warrant evaluation. A fetal heart rate of 116 beats/minute is a normal finding at 42 weeks of gestation. Cervical dilation of 2 cm with 50% effacement is a normal finding in a 42-week gestation woman. A score of 8 on the BPP is a normal finding in a 42-week gestation pregnancy.

For a woman at 42 weeks of gestation, which finding requires more assessment by the nurse? Fetal heart rate of 116 beats/min Cervix dilated 2 cm and 50% effaced Score of 8 on the biophysical profile One fetal movement noted in 1 hour of assessment by the mother

One fetal movement noted in 1 hour of assessment by the mother Self-care in a postterm pregnancy should include performing daily fetal kick counts three times per day. The mother should feel four fetal movements per hour. If she feels fewer than four movements, she should count for 1 more hour. Fewer than four movements in that hour warrants evaluation. The findings described in the other choices are normal at 42 weeks of gestation.

The nurse is preparing to educate a woman with an abnormal 1-hour glucose tolerance test. Which additional diagnostic test does the nurse anticipate the health care provider to ordering? Fasting glucose Hemoglobin A1C Repeat finger stick glucose Oral glucose tolerance test

Oral glucose tolerance test The oral 3-hour glucose tolerance test follows a failed 1-hour glucose tolerance test.

Common side effects of calcium-channel blockers (e.g nefeipine [procardia])

Orthostatic hypotension, dizziness -need to be sure to maintain adequate fluid balance

Causes of Uterine Atony

Overdistended uterus -Traumatic birth -High parity -Polyhydramnios -Fetal macrosomia -Mulitple gestations -Magnesium sulfate -Rapid or prlonged labor -Chorioamnionitis -Use of oxytocin for labor induction -uterine atony in a previous pregnancy -anesthesia and analgesia -forceps-assisted birth -Vacuum-assited birth -retained placental fragments -inversion of the uterus -ruptured uterus -placenta accreta, increta, percreta -placental abruption -placenta previa -uterine subinvolution

A postpartum client is bleeding continuously and excessively due to uterine atony. Which medications administered to the client may cause postpartum hemorrhage? Select all that apply. Oxytocin (Picotin) Halothane (Fluothane) Nitrous oxide (Anesoxyn) Nitroglycerine (Nitrostat) Magnesium sulfate (Generic)

Oxytocin (Picotin) Halothane (Fluothane) Magnesium sulfate (Generic) Rationale Oxytocin (Picotin), halothane (Fluothane), and magnesium sulfate (Generic) have a high potential to cause uterine atony. Administration of oxytocin (Picotin) to induce labor may lead to prolonged dilation of the uterus. Halogenated anesthetics, like halothane (Fluothane), also lead to reduced uterine muscle tone (uterine atony). When magnesium sulfate is administered during labor, it will increase the chances of uterine atony. Nitrous oxide (Anesoxyn), an anesthetic agent, is given to clients during manual removal of the placenta. Nitroglycerin (Nitrostat), a tocolytic agent, is used to promote uterine relaxation in the case of a retained placenta and uterine inversion.

Which medication should be administered to manage excessive bleeding in a woman after a miscarriage? Oxytocin (Pictoin) Ampicillin (Amcill) Ergonovine (Methergine) Magnesium sulfate (Sulfamag) Misoprostol (Cytotec)

Oxytocin (Pictoin) Ergonovine (Methergine) Rationale Ergonovine (Methergine) is used to promote uterine contractions (UCs), which, in turn, prevent excessive bleeding following a miscarriage. Oxytocin (Pitocin) is used to prevent uterine hemorrhage. Ampicillin (Amcill) is an antibiotic used to treat infection, but it does not prevent bleeding. Magnesium sulfate (Sulfamag) is used to suppress UCs. Misoprostol (Cytotec) is a prostaglandin medication that can be used to manage the miscarriage, but only if there is no bleeding and infection.

The nurse is preparing a client for abortion. Which medicine is administered to the client after the evacuation of the uterus to prevent hemorrhage? Oxytocin (Pitocin) Misoprostol (Cytotec) Vitamin K (Aqua-Mephyton) Magnesium sulfate (Sulfamag)

Oxytocin (Pitocin) Rationale Oxytocin (Pitocin) is administered to the client to prevent hemorrhage after evacuation of the uterus. It prevents hemorrhage by causing contractions of the uterus. Misoprostol (Cytotec) is used to complete a missed miscarriage within 7 days. It helps by expelling the products of conception from the uterus. Vitamin K (Aqua-Mephyton) is used for the treatment of disseminated intravascular coagulation (DIC). Magnesium sulfate (Sulfamag) is used for tocolysis to suppress uterine contractions

A woman at 37 weeks of gestation is admitted with a placental abruption after a motor vehicle accident. Which assessment data are most indicative of her condition worsening? P 112, R 32, BP 108/60; FHR 166-178 P 98, R 22, BP 110/74; FHR 150-162 P 88, R 20, BP 114/70; FHR 140-158 P 80, R 18, BP 120/78; FHR 138-150

P 112, R 32, BP 108/60; FHR 166-178 Rationale Bleeding, which impacts the mother's well-being as well as that of her fetus, is the most dangerous problem. The decreasing blood volume would cause increases in pulse and respirations and a decrease in blood pressure. The fetus often responds to decreased oxygenation as a result of bleeding, causing a decrease in perfusion. This causes the fetus' heart rate to increase above the normal range of 120-160 beats per minute. The other options have measurements that are in the "normal" range and would not reflect a deterioration of the patient's physical status.

Match the woman to the relevant high-risk condition.

PTL A woman at 28 weeks who is having regular uterine contractions. PROM A woman at 38 weeks whose amniotic membranes have ruptured, but she has not experienced any contractions. PPROM A woman at 32 weeks whose amniotic membranes have ruptured. Chorioamnionitis A woman at 33 weeks who has a temperature of 101.5 degrees F and abdominal tenderness.

A nurse in the neonatal intensive care unit observes a newborn with a furrowed brow, open mouth, and tightly closed eyes. The nurse recognizes these features as being associated with which complication? Pain Hyperstimulation Respiratory distress Necrotizing entercolitis

Pain A furrowed brow, with the mouth open and eyes shut tightly, is typically known as a "cry face" and is associated with pain.

While performing a vaginal examination on a patient in active labor, which finding would prompt the nurse to call the health care provider immediately? Soft cervical consistency Palpable fetal head behind the dilated cervix Increased cervical dilation since previous examination Palpable pulse that beats synchronously with fetal heart monitor

Palpable pulse that beats synchronously with fetal heart monitor A palpable pulse that beats synchronously with the fetal heartbeat indicates a prolapsed umbilical cord; the nurse is feeling the umbilical cord during this examination, which indicates a prolapse.

What is the first and most important nursing intervention when a nurse observes profuse postpartum bleeding? Call the woman's primary health care provider Administer the standing order for an oxytocic Palpate the uterus and massage it if it is boggy Assess maternal blood pressure and pulse for signs of hypovolemic shock

Palpate the uterus and massage it if it is boggy Rationale The initial management of excessive postpartum bleeding is firm massage of the uterine fundus. The most important nursing intervention is to stop the bleeding. Once the nurse has applied firm massage of the uterine fundus the primary health care provider should be notified or the nurse can delegate this task to another staff member. Administering the standard order for an oxytocic is appropriate after assessment and immediate steps have been taken to control the bleeding. Vital signs will need to be ascertained after fundal massage has been applied.

When a nurse observes profuse postpartum bleeding, the first and most important nursing intervention is to what? Call the woman's primary health care provider. Administer the standing order for an oxytocic. Palpate the uterus and massage it if it is boggy. Assess maternal blood pressure and pulse for signs of hypovolemic shock.

Palpate the uterus and massage it if it is boggy. Rationale The most important nursing intervention is to stop the bleeding. Once the nurse has applied firm massage of the uterine fundus, the primary health care provider should be notified or the nurse can delegate this task to another staff member. This intervention is appropriate after assessment and immediate steps have been taken to control the bleeding. The initial management of excessive postpartum bleeding is firm massage of the uterine fundus. Vital signs will need to be ascertained after fundal massage has been applied.

The first and most important nursing intervention when a nurse observes profuse postpartum bleeding is to: Call the woman's primary health care provider. Administer the standing order for an oxytocic. Palpate the uterus and massage it if it is boggy. Assess maternal blood pressure and pulse for signs of hypovolemic shock.

Palpate the uterus and massage it if it is boggy. The initial management of excessive postpartum bleeding is firm massage of the uterine fundus to stop the bleeding. This is the most important nursing intervention. Then the primary health care provider should be notified or the nurse can delegate this task to another staff member. Administering an oxytocic and ascertaining vital signs are appropriate after assessment has been made and immediate steps have been taken to control the bleeding.

Upon assessing a client who is in the third stage of labor, the nurse notices that the client is experiencing vaginal bleeding with spurts of blood and clots. What does the nurse believe is the cause of the bleeding? Deep cervical laceration Intravascular coagulation Superficial vaginal laceration Partial placental separation

Partial placental separation Rationale Vaginal bleeding characterized by the spurts of blood and clots indicates that the placenta is not separated properly from the uterus. Deep cervical laceration is indicated by bright red blood originating from the arteries. If the blood fails to clot after bleeding, it indicates the presence of intravascular coagulation. Superficial vaginal laceration is characterized by dark red blood of venous origin.

Which indicator would lead the nurse to suspect that a postpartum patient experiencing hemorrhagic shock is getting worse? Restoration of blood pressure levels to normal range Capillary refill brisk Patient complaint of headache and increased reaction time to questioning Patient statement that she sees "stars"

Patient complaint of headache and increased reaction time to questioning Patient complaint of a headache accompanied by an increased reaction (response) time indicates that cerebral hypoxia is getting worse. Return of blood pressure to normal range would indicate resolving symptoms. Brisk capillary refill is a normal finding. The patient may see "stars" early on in decreased blood flow states.

In which situations would the use of Methergine or prostaglandin be contraindicated even if the patient was experiencing a postpartum significant bleed? Select all that apply. Patient has delivered twin pregnancies. Patient's blood pressure postpartum is 180/90. Patient has a history of asthma. Patient has a mitral valve prolapse. Patient is a grand multip.

Patient's blood pressure postpartum is 180/90. Twin pregnancies successfully delivered and grand multiparity are not contraindications to the use of these medications. If a patient is hypertensive or has cardiovascular disease, these medications would not be used. If a patient has a history of asthma, prostaglandin medication would not be used. Patient has a history of asthma. Patient has a mitral valve prolapse.

The nurse is assessing a pregnant client at 6 weeks of gestation. Upon reviewing the client's medical history, the nurse finds that the client had undergone a large cone biopsy. Which treatment strategy does the nurse expect to be most beneficial to prevent miscarriage in the client? Administering misoprostol (Cytotec) orally for 7 days Performing abdominal cerclage at 11 weeks of gestation Placing a rescue cerclage in the cervix at 23 weeks of gestation Administering ergonovine (Methergine) during the 3rd trimester

Performing abdominal cerclage at 11 weeks of gestation Rationale A client who has undergone a large cone biopsy may be at risk of miscarriage due to the presence of an extremely short cervix. Therefore, the primary health care provider would perform an abdominal cerclage at 11 or 13 weeks of gestation in order to prevent the opening of the cervical os. Misoprostol (Cytotec) is a prostaglandin analogue that induces miscarriage. Therefore, misoprostol (Cytotec) does not help to prevent miscarriage in the patient. Ergonovine (Methergine) causes uterine contractions and helps prevent excessive bleeding, but it does not prevent miscarriage. A rescue cerclage cannot be performed in a client who has undergone a large cone biopsy, because the client has a short cervix.

A neonate has an Apgar score of 2 at 1 minute of life and 6 at 5 minutes of life. The neonate requires oxygen and IV fluid therapy. Which is the priority intervention for this neonate? Obtain blood pressure and vital signs. Apply a continuous pulse oximeter to the left foot. Discuss the treatment plan with the parents when they visit the neonatal intensive care unit (NICU). Place the neonate under a radiant warmer with a temperature probe attached to the right upper quadrant of the abdomen.

Place the neonate under a radiant warmer with a temperature probe attached to the right upper quadrant of the abdomen. Maintaining thermoregulation is very important throughout care. Hypothermia will worsen the neonate's condition.

A pregnant woman's amniotic membranes rupture. Prolapsed cord is suspected. Which intervention is the nurse's top priority? Place the woman in the knee-chest position. Cover the cord in a sterile towel saturated with warm normal saline. Prepare the woman for a cesarean birth. Start oxygen by face mask.

Place the woman in the knee-chest position. Rationale The woman is assisted into a position (e.g., modified Sims position, Trendelenburg position, or the knee-chest position) in which gravity keeps the pressure of the presenting part off the cord. If the cord is protruding from the vagina it may be covered with a sterile towel soaked in saline. Although this is an appropriate intervention, relieving pressure on the cord is the nursing priority. If the cervix is fully dilated, the nurse should prepare for immediate vaginal delivery. Cesarean birth is indicated only if cervical dilation is not complete. The nurse should administer oxygen by facial mask at 8 to 10 L/min until delivery is complete. This intervention should be initiated after pressure is relieved on the cord. Not only should the woman be placed in knee-chest position, the nurse may also use her gloved hand or two fingers to lift the presenting part off the cord.

A pregnant woman's amniotic membranes rupture. Prolapsed cord is suspected. Which intervention is the nurse's top priority? Place the woman in the knee-chest position. Cover the cord in a sterile towel saturated with warm normal saline. Prepare the woman for a cesarean birth. Start oxygen by face mask

Place the woman in the knee-chest position. The woman is assisted into a position (e.g., modified Sims position, Trendelenburg position, or knee-chest position) in which gravity keeps the pressure of the presenting part off the cord. Relieving pressure on the cord is the nursing priority. The nurse may also use her gloved hand or two fingers to lift the presenting part off the cord. If the cord is protruding from the vagina it may be covered with a sterile towel soaked in saline. The nurse should administer O2 by facial mask at 8 to 10 L/min until delivery is complete. If the cervix is fully dilated, the nurse should prepare for immediate vaginal delivery. Cesarean birth is indicated only if cervical dilation is not complete.

After reviewing the medical reports of a client, the nurse finds that the client has submucosal uterine fibroids. Which postpartum complication of pregnancy is the client likely to have? Placenta accreta Impaired lactation Vaginal hematomas Postpartum hemorrhage

Placenta accreta Rationale Placenta accreta is a slight penetration of the placenta into the myometrium of the uterus. The clientwho has submucosal fibroids has a higher risk of developing placenta accreta. The symptom of placenta accreta can be diagnosed before birth using an ultrasound and magnetic resonance imaging (MRI). Submucosal fibroids do not have any effect on lactation. Therefore, they do not pose a risk of impaired lactation in the client. Vaginal hematomas are associated with forceps-assisted birth, episiotomy, or primigravidity. Submucosal uterine fibroids do not have any effect on the integrity of the vaginal walls. Therefore, submucosal uterine fibroids do not cause vaginal hematomas. The client's reports suggests that the placenta is adherent to the uterine wall. This does not indicate that the client is at risk of postpartum hemorrhage.

Which intrapartum factors contribute to an increased risk for uterine atony? Select all that apply. Primiparity Placenta previa Maternal infection Epidural anesthesia Delivery by c-section Use of forceps during delivery

Placenta previa Placenta previa contributes to an increased risk for uterine atony. Correct Maternal infection Maternal infection contributes to an increased risk for uterine atony. Use of forceps during delivery Use of forceps during delivery contributes to an increased risk for uterine atony.

A woman at 39 weeks of gestation with a history of preeclampsia is admitted to the labor and birth unit. She suddenly experiences increased contraction frequency of every 1 to 2 minutes; dark red vaginal bleeding; and a tense, painful abdomen. The nurse suspects the onset of what condition? Eclamptic seizure Rupture of the uterus Placenta previa Placental abruption

Placental abruption Rationale Uterine tenderness in the presence of increasing tone may be the earliest finding of premature separation of the placenta ( abruptio placentae or placental abruption). Women with hypertension are at increased risk for an abruption. Eclamptic seizures are evidenced by the presence of generalized tonic-clonic convulsions. Uterine rupture presents as hypotonic uterine activity, signs of hypovolemia, and in many cases the absence of pain. Placenta previa presents with bright red, painless vaginal bleeding.

What are the complications associated with polyhydramnios? Ketoacidosis Placental abruption Uterine dysfunction Gestational diabetes Postpartum hemorrhage

Placental abruption Uterine dysfunction Postpartum hemorrhage Rationale Polyhydramnios is a condition in which the amniotic fluid index becomes greater than 25 cm. An increase in the amniotic fluid may lead to placental abruption, which needs immediate intervention. This condition also impairs the uterine function. Polyhydramnios also results in excessive blood loss after delivery, which is referred to as postpartum hemorrhage. Ketoacidosis may occur when the blood sugar levels go beyond 200 mg/dl. Gestational diabetes can cause polyhydramnios. However, it is not a complication of polyhydramnios.

A woman at 39 weeks of gestation with a history of preeclampsia is admitted to the labor and birth unit. She suddenly experiences increased contraction frequency to every 1 to 2 minutes; dark red vaginal bleeding; and a tense, painful abdomen. The nurse suspects the onset of: Eclamptic seizure. Rupture of the uterus. Placenta previa. Placental abruption.

Placental abruption. Uterine tenderness in the presence of increasing tone may be the earliest finding of premature separation of the placenta (abruptio placentae or placental abruption). Women with hypertension are at increased risk for an abruption. Eclamptic seizures are evidenced by the presence of generalized tonic-clonic convulsions. Uterine rupture manifests with hypotonic uterine activity, signs of hypovolemia, and in many cases the absence of pain, and placenta previa with bright red, painless vaginal bleeding.

The nurse is assessing a pregnant client who is a chronic smoker and lives in a high altitude area. Which pregnancy-related risk may be high in the client? Placental previa Molar pregnancy Abruptio placenta Ectopic pregnancy

Placental previa Rationale Pregnant women who live at high altitudes or are smokers have lower blood oxygenation. A decrease in the uteroplacental oxygenation leads to an increase in the placental surface area. Therefore, the client is more likely to have placenta previa. Molar pregnancy is more common in pregnant women who have had a prior molar pregnancy and those who are in their early teens or older than 40 years of age. Pregnant clients with hypertension may have a high risk of having abruptio placenta. Clients with tubal infection or damage are at risk of having ectopic pregnancies.

The nurse is preparing to administer intravenous magnesium sulfate to a client with preeclampsia. Meanwhile, the student nurse positions the client in a supine position, monitors the fetal heart rate (FHR), checks for baseline variability, and monitors for the absence of late decelerations. Which action by the student nurse needs correction? Checking for baseline variability Monitoring of the fetal heart rate (FHR) Placing the client in a supine position Monitoring for the absence of late decelerations

Placing the client in a supine position Rationale While caring for a pregnant client with preeclampsia and ineffective tissue perfusion, the nurse should place the client on her side, not in a supine position. This is done to maximize the uteroplacental blood flow and ensure efficient uteroplacental oxygenation. This intervention also helps decrease the client's blood pressure, promote diuresis, and prevents supine hypotension. The student nurse should check for baseline variability, monitor the fetal heart rate (FHR), and check for the absence of late decelerations. These interventions promote the safety of the fetus.

The nurse is supervising a student nurse while performing a non-stress test. Which action of the student nurse indicates the need for further teaching? Placing the client in the left side-lying position Instructing the client to press the handheld marker Offering glucose water prior to the test Applying a tocodynamometer with Doppler transducer

Placing the client in the left side-lying position Rationale The client should be assisted to a semi-Fowler position with a slight tilt to prevent supine-hypotension. In order to assess the number of fetal movements, the nurse should instruct the client to press the handheld event marker whenever a fetal movement is experienced. Glucose water or orange juice may be is offered to increase fetal activity, if there is no fetal movements after 20 minutes. A tocodynamometer is applied with Doppler transducer to monitor fetal movement or detect uterine contraction.

A pregnant woman's amniotic membranes rupture. Prolapsed cord is suspected. Which intervention is the nurse's top priority? Placing the woman in the knee-chest position. Covering the cord in a sterile towel saturated with warm normal saline. Preparing the woman for a cesarean birth. Starting oxygen by face mask.

Placing the woman in the knee-chest position. Rationale The woman is assisted into a position (e.g., modified Sims position, Trendelenburg position, or the knee-chest position) in which gravity keeps the pressure of the presenting part off the cord. If the cord is protruding from the vagina, it may be covered with a sterile towel soaked in saline. Although preparing the woman for a cesarean birth is an appropriate intervention, relieving pressure on the cord is the nursing priority. If the cervix is fully dilated, the nurse should prepare for immediate vaginal birth. Cesarean birth is indicated only if cervical dilation is not complete. The nurse should administer O 2 by facial mask at 8 to 10 L/min until birth is complete. This intervention should be initiated after pressure is relieved on the cord.

Which laboratory values would present in a client diagnosed with preeclampsia? Hemoglobin 8g/dl Platelet count of 75,000 LDH 100 units/L Burr cells BUN 25 mg/dl

Platelet count of 75,000 LDH 100 units/L BUN 25 mg/dl Rationale Thrombocytopenia below 100,000 would be noted. An increase in LDH and an increase in BUN would be noted. Hemoglobin levels would be increased and this would be a decreased level. Burr cells would not be present in preeclampsia but would be present with HELLP syndrome.

Which laboratory values would be found in a patient diagnosed with preeclampsia? Select all that apply. Hemoglobin 8g/dL Platelet count of 75,000 LDH 100 units/L Burr cells BUN 25 mg/dL

Platelet count of 75,000 Thrombocytopenia below 100,000, an increase in LDH, and an increase in BUN would be noted. Hemoglobin levels would be increased, but 8 g/dL reflects a decreased level.Burr cells would not be present in preeclampsia but would in HELLP syndrome. LDH 100 units/L Thrombocytopenia below 100,000, an increase in LDH, and an increase in BUN would be noted. Hemoglobin levels would be increased, but 8 g/dL reflects a decreased level.Burr cells would not be present in preeclampsia but would in HELLP syndrome. BUN 25 mg/dL Thrombocytopenia below 100,000, an increase in LDH, and an increase in BUN would be noted. Hemoglobin levels would be increased, but 8 g/dL reflects a decreased level.Burr cells would not be present in preeclampsia but would in HELLP syndrome.

A new mother reports feeling detached from the infant and expresses doubt about being able to provide care for the infant. Which nursing interventions are appropriate? Select all that apply. Point out infant cues and explain their meaning. Observe the mother's response when the infant cries. Encourage skin-to-skin care to help increase attachment. Leave the woman alone with the infant to allow time for bonding. Suggest that the woman's partner take a more active role in caregiving.

Point out infant cues and explain their meaning. Pointing out infant cues and explaining their meaning gives the woman information she needs, which should relieve some of her doubt. Correct Observe the mother's response when the infant cries. Observing the mother's response when the infant cries helps the nurse assess the woman's level of attachment to the infant. Correct Encourage skin-to-skin care to help increase attachment. Skin-to-skin care may help increase attachment and may help the woman feel better about herself and her ability to care for her infant.

A nurse is caring for a premature neonate who is tachypneic, is grunting, and has subcostal retractions. Which interventions are appropriate for the nurse to implement? Select all that apply. Obtain an axillary temperature. Position neonate prone under the radiant warmer. Apply a pulse oximeter to the neonate's righthand. Assist ventilations using a bag-valve-mask device and 100% FiO2. Obtain an arterial blood gas from the umbilical artery catheter (UAC). Suction oropharynx with 10 fr and suction catheter for 30 seconds at a time.

Position neonate prone under the radiant warmer. The prone position increases oxygenation and respiratory control and improves lung volume and breathing. This position may decrease the respiratory rate of the neonate as well as the degree of grunting and retracting. Correct Apply a pulse oximeter to the neonate's righthand. A premature neonate with signs of respiratory distress requires continuous pulse oximeter monitoring so oxygen can be titrated depending on pulse oximetry levels. Obtain an arterial blood gas from the umbilical artery catheter (UAC). Arterial blood may be drawn for testing oxygen levels in premature neonates who display grunting, tachypnea, and subcostal retractions.

A premature neonate has flexed arms, clenched fists, and a high-pitched cry while lying in an isolette. Which interventions are appropriate for the nurse when providing care? Select all that apply. Completing a pain assessment twice per shift Positioning the neonate prone in the isolette Swaddling the neonate in a blanket in the isolette Dipping the pacifier in sucrose before a blood draw Obtaining vital signs, followed by the physical assessment an hour later

Positioning the neonate prone in the isolette Positioning the neonate prone in the isolette may decrease the discomfort of the neonate and decrease the signs of pain currently exhibited by the neonate. Swaddling the neonate in a blanket in the isolette Swaddling the neonate in a blanket is a comfort measure used to calm a premature neonate who is crying with flexed arms and fists. Dipping the pacifier in sucrose before a blood draw Sucrose placed on a pacifier prior to a painful event increases pain relief, especially when used in conjunction with sucking.

A nurse is monitoring a client's reflexes (DTRs) while receiving magnesium sulfate therapy for treatment of preeclampsia. Which assessment finding indicates a cause for concern? Bilateral DTRs noted at 2+ DTRs response has been noted at 1+ since onset of therapy Positive clonus response elicited unilaterally Client reports no pain upon examination of DTRs by nurse.

Positive clonus response elicited unilaterally Rationale Positive clonus response elicited unilaterally is a cause for concern, because it suggests a hyperactive response. Bilateral DTRs noted at 2+ would indicate a normal finding. Even though this finding indicates a sluggish or decreased response, this is unchanged since the initiation of therapy. The nurse would continue to monitor. Typically, there is no pain associated with determination of DTRs so this finding would be considered normal.

A nurse is monitoring a patient's reflexes (DTRs) while receiving magnesium sulfate therapy for treatment of preeclampsia. Which assessment finding indicates a cause for concern? Bilateral DTRs noted at 2+ DTRs response has been noted at 1+ since onset of therapy Positive clonus response elicited unilaterally Patient reports no pain upon examination of DTRs by nurse

Positive clonus response elicited unilaterally Positive clonus response elicited unilaterally is a cause for concern as it suggests a hyperactive response. Typically, there is no pain associated with determination of DTRs so this finding would be considered to be normal, as would bilateral DTRs noted at 2+.Even though DTRs at 1+ indicate a sluggish or decreased response, this finding is unchanged since the initiation of therapy. The nurse would continue to monitor.

Which patient laboratory value would the nurse validate as part of routine preparation for cesarean delivery, in order to determine adequate fetal maturity? Rh-positive antibody test performed 4 weeks prior to surgery. Negative rubella titer performed 6 weeks prior to surgery. Positive serum pregnancy test performed at least 36 weeks prior to surgery. High serum alpha-fetoprotein (AFP) level performed 26 weeks prior to surgery.

Positive serum pregnancy test performed at least 36 weeks prior to surgery. Positive serum pregnancy test, performed at least 36 weeks prior to the date of cesarean delivery, can be used to verify gestational age.

A 17-year-old nullipara who reports low self-esteem because of the pregnancy is at increased risk for developing which postpartum mood disorder? Bipolar II disorder Postpartum depression Postpartum posttraumatic stress disorder Postpartum obsessive-compulsive disorder

Postpartum depression Risk factors for postpartum depression include first pregnancy and personality characteristics such as low self-esteem. This woman's history increases the risk for postpartum depression.

After reviewing the medical reports of a client, the nurse finds that the client has multifetal gestation. What could be the most likely complication associated with this? Vaginal hematomas Von Willebrand disease (vWD) Postpartum hemorrhage (PPH) Abnormal development of limbs

Postpartum hemorrhage (PPH) Rationale The uterine walls are overstretched due to multifetal gestation, so the uterus contracts poorly after birth. This may cause uterine atony, leading to PPH. Multifetal gestation does not cause vaginal hematomas, vWD, or abnormal limb development of the fetus. Vaginal hematomas occur more commonly in association with a forceps-assisted birth. vWD is a type of hemophilia, which is a hereditary bleeding disorder. Abnormal development of fetal limbs is usually a complication associated with teratogenic drugs.

A new mother indicates that "I have brought my mother into the home to help care for the newborn because I am inexperienced and might accidently hurt him or not meet his needs." This statement is suggestive of which mood disorder? Postpartum psychosis Postpartum depression Postpartum panic disorder Postpartum obsessive-compulsive disorder

Postpartum obsessive-compulsive disorder Postpartum obsessive-compulsive disorder is characterized by preoccupation with thoughts of potentially harming the newborn and fear of being alone with the newborn. This woman's statement is suggestive of postpartum obsessive-compulsive disorder.

The nurse receives a report on a woman who had an emergency cesarean delivery and required a transfusion afterward because of hemorrhage. The nurse knows that this woman is at risk for which mood disorder? Postpartum psychosis Postpartum depression Postpartum panic disorder Postpartum posttraumatic stress disorder

Postpartum posttraumatic stress disorder Risk factors for postpartum posttraumatic stress disorder include having a traumatic birth, an emergency cesarean delivery, or hemorrhage. This woman's history puts her at increased risk for postpartum posttraumatic stress disorder.

Which condition is associated with oligohydramnios? Fetal hydrops Potter syndrome Neural tube defects Fetal gastrointestinal obstruction

Potter syndrome Rationale Oligohydramnios is characterized by the presence of amniotic fluid volume of less than 5 cm. It is associated with an absence of fluid pockets in the uterine cavity, and may cause Potter syndrome. Fetal hydrops, neural tube defects, and fetal gastrointestinal obstruction are associated with polyhydramnios due to the presence of large pockets of fluids.

Eclampsia interventions

Preeclampic with SEIZURES Remain with client Ensure open airway; turn client on side (prevent aspiration); protect from injury Admin oxygen Monitor FHR Admin med to control seizures (Mag Sulfate) After seizure-> insert oral airway and suction mouth prepare to deliver fetus after stabilization document occurene and outcone -Note time of seizure & duration

The nurse expects to note which findings associated with gestational diabetes mellitus (GDM)? Select all that apply. Preeclampsia Small fetal size Fundal height higher than expected Frequent urinary tract infections (UTIs) Premature rupture of membranes (PROM)

Preeclampsia Rates of preeclampsia are higher in women with GDM than in the general population. Fundal height higher than expected Large fundal size is a sign of polyhydramnios, which is associated with gestational diabetes. Frequent urinary tract infections (UTIs) Urinary tract infections are more common in woman with gestational diabetes. Premature rupture of membranes (PROM) PROM is associated with gestational diabetes

Which obstetric or medical complications should the nurse be alert for when providing care to a pregnant client with diabetes mellitus? Select all that apply. Preeclampsia Hypoglycemia Hydramnios Monilial vaginitis Brachial plexus palsy

Preeclampsia Hypoglycemia Hydramnios Monilial vaginitis Rationale A pregnant client with diabetes mellitus is at risk for hypertension, which may result in preeclampsia. Hypoglycemia may occur due to an increase in insulin levels in the first trimester of pregnancy. Hydramnios may occur in the third trimester of pregnancy due to hyperglycemia. Monilial vaginitis is a vaginal infection that is seen in women with diabetes during pregnancy. This results from an alteration in the normal resistance of the body to infection. Brachial plexus palsy may be seen in the child born to a woman with diabetes, due to a difficult vaginal birth.

What are some common maternal and fetal indications for antepartum testing? Select all that apply. Gallstones Preeclampsia Previous stillbirth Fetal growth restriction Increased fetal movement Premature rupture of membranes

Preeclampsia Previous stillbirth Fetal growth restriction Premature rupture of membranes Rationale Some common maternal and fetal indications for antepartum testing include: preeclampsia, previous stillbirth, fetal growth restriction, and premature rupture of membranes. Gallstones and increased fetal movement are not indications for antepartum testing.

With regard to preeclampsia and eclampsia, nurses should be aware of what information? Preeclampsia is a condition of the first trimester; eclampsia is a condition of the second and third trimesters Preeclampsia results in decreased function in such organs as the placenta, kidneys, liver, and brain The causes of preeclampsia and eclampsia are well documented Severe preeclampsia is defined as preeclampsia plus proteinuria

Preeclampsia results in decreased function in such organs as the placenta, kidneys, liver, and brain Rationale Vasospasms diminish the diameter of blood vessels; this impedes blood flow to all organs. Preeclampsia occurs after week 20 of gestation and can run for the duration of the pregnancy. The causes of preeclampsia and eclampsia are unknown, although several have been suggested. Preeclampsia includes proteinuria; severe cases are characterized by greater proteinuria or any of nine other conditions.

With regard to preeclampsia and eclampsia, nurses should be aware that: Preeclampsia is a condition of the first trimester; eclampsia is a condition of the second and third trimesters. Preeclampsia results in decreased function in such organs as the placenta, kidneys, liver, and brain. The causes of preeclampsia and eclampsia are well documented. Severe preeclampsia is defined as preeclampsia plus proteinuria

Preeclampsia results in decreased function in such organs as the placenta, kidneys, liver, and brain. Vasospasms diminish the diameter of blood vessels, which impedes blood flow to all organs. Preeclampsia occurs after week 20 of gestation and can run the duration of the pregnancy. The causes of preeclampsia and eclampsia are unknown, although several have been suggested. Preeclampsia includes proteinuria; severe cases are characterized by greater proteinuria or any of nine other conditions.

A client who is pregnant already has type 2 diabetes and a hemoglobin A1c of 7. What does the nurse would categorize this client as having? Gestational diabetes Insulin-dependent diabetes complicated by pregnancy. Pregestational diabetes mellitus Non-insulin-dependent diabeteswith complications

Pregestational diabetes mellitus Rationale Pregestational diabetes mellitus is a term used to describe type 1 or type 2 diabetic clients in whom the diabetes existed prior to pregnancy. Gestational diabetes occurs when during the pregnancy, a woman becomes diabetic. A type 2 diabetic is non-insulin dependent. This option indicates type 1, or insulin-dependent, diabetes. There is no information presented that indicates complications at this point, because the hemoglobin A1c is within normal range, indicating adequate glycemic control.

Ectopic (tubal) pregnancy

Pregnancy in which the fertilized ovum is implanted outside the uterine cavity, usually in the ampulla or largest part of the uterine tube.

diabetes mellitus in the pregnant woman?

Pregnancy places demands on carbohydrate metabolism and causes insulin requirements to change maternal glucose cross the placenta, insulin doesnt

miscarriage

Pregnancy that ends as a result of natural causes before 20 weeks of gestation The type and management of miscarriage depend on the duration of pregnancy, and the signs and symptoms.

Postterm; postdate; prolonged;

Pregnancy that extends beyond the end of week 42 of gestation.

Preeclampsia

Pregnancy-specific condition in which hypertension develops after 20 weeks of gestation or in the early postpartum period in a previously normotensive woman; the presence of thrombocytope- nia, impaired liver function, newly developed renal insufficiency, pulmonary edema, or new onset cerebral or visual disturbances confirms the diagnosis.

Incompetent Cervix

Premature dilation of cervix; 4th or 5th month of pregnancy -Associated with structural or functional defects of cervix. Treatment- surgical placement of cervical cerclage S/S Vaginal bleeding, fetal membranes visible thru cervix

The nurse is assigned four premature neonates for a shift. Which neonate should the nurse assess first? Premature neonate with nasal flaring with each respiration Premature neonate with fine crackles in the lower lung fields bilaterally Premature neonate grunting, flaring, and retracting with each respiration Premature neonate with cessation of breathing for 5 seconds, occurring twice an hour

Premature neonate grunting, flaring, and retracting with each respiration Grunting, flaring, and retractions are signs of increasing respiratory distress. The ability of the premature neonate to breathe is compromised; therefore, this neonate should be assessed first.

Abruptio placentae

Premature separation of placenta from uterine wall after the 20th week of gestation and before the fetus is delivered S/S Dark Red vaginal bleeding, Uterine PAIN, tenderness, uterine rigidity, severe ABDOMINAL pain, S/S fetal distress, Maternal shock Interventions- Monitor vs, FHR, assess bleeding, abdominal pain, increase in fundal ht, bed rest, admin oxygen, IV fluids, blood, place client in Trendelenburg position or lateral position with the head of the bed flat to decrease pressure from fetus or hypovolemic shock occurs, monitor uterine activity Prepare for delivery of fetus monitor for signs of disseminated intravascular coagulation

Nuchal translucency

Prenatal screening method that uses ultrasound a measurement of fluid in the nape of the fetal neck between 10 and 14 weeks of gestation to identify possible fetal abnormalities

Maternal serum alpha-fetoprotein"

Prenatal test performed between 15 and 20 weeks of gestation (ideally 16 and 18 weeks) to screen for neural tube defects, for which the fetus of a diabetic mother is at higher risk

Nsg interventions for hydatidform mole?

Prepare client for uterine evacuation done by vacuum aspiration, oxytocin is administered after evacuation to contract the uterus, monitor for postprocedure hemorrhage and infection, tissue is sent to the laboratory for evaluation, follow up is important to detect changes suggestive of malignancy, human chorionic gonadotropin levels are monitored every 1-2 weeks until normal prepregnancy levels are attained (levels are checked every 1-2 months for 1 year), instruct the client about birth control measures so that pregnancy can be prevented during the 1-year follow-up

While assisting a primary health care provider performing amniotomy, the nurse observes part of the umbilical cord protruding from the client's vagina. The nurse immediately positions the client in the Trendelenburg position and inserts a finger into her vagina. What additional care does the client need to prevent complications? Perform large-bore catheter suction. Prepare for an emergency C-section. Administer calcium gluconate intravenously. Administer terbutaline (Brethine) subcutaneously.

Prepare for an emergency C-section. Rationale Amniotomy may cause prolapse of the umbilical cord, in which the cord lies below the presenting part of the fetus. A prolapsed cord causes fetal hypoxia, because the supply of oxygen to the fetus is reduced. A cesarean birth should be performed to prevent further complications. Large-bore catheter suction is performed to remove the aspirated meconium from the newborn, and is unrelated to cord prolapse. Calcium gluconate is administered to a pregnant client who develops magnesium sulfate toxicity. Calcium gluconate is unrelated to cord prolapse. Terbutaline (Brethine) is administered to treat tachysystole in the pregnant client and is unrelated to cord prolapse.

The nurse is caring for a client who is 35 weeks pregnant, and reports moderate vaginal bleeding. The ultrasonographic reports indicate abruptio placentae. Which immediate interventions should the nurse provide? Determine hemoglobin value. Prepare the client for delivery. Determine blood and fluid volume. Provide oxygen therapy.

Prepare the client for delivery. Rationale The nurse should immediately prepare the client for delivery because the client is near the end of gestation and has moderate bleeding. The client's hemoglobin and hematocrit values should be determined to monitor the bleeding. However, this intervention can be performed after the client is admitted for delivery. The nurse should assess the blood and fluid volume by inserting a catheter during delivery to determine if blood transfusion is necessary. Oxygen therapy may be given if the client has difficulty in breathing.

Which is an important nursing intervention when a client has an incomplete miscarriage with heavy bleeding? Initiate expectant management at once. Prepare the client for dilation and curettage. Administer the prescribed oxytocin (Pitocin). Obtain a prescription for ergonovine (Methergine).

Prepare the client for dilation and curettage. Rationale In the case of an incomplete miscarriage, sometimes there is heavy bleeding and excessive cramping and some part of fetal tissue remains in the uterus. Therefore the nurse needs to prepare the client for dilation and curettage for the removal of the fetal tissue. Expectant management is initiated if the pregnancy continues after a threatened miscarriage. Oxytocin (Pitocin) is administered to prevent hemorrhage after evacuation of the uterus. Ergonovine (Methergine) is administered to contract the uterus.

A pregnant client with gestational hypertension has very high blood pressure. The nurse learns that the gestational age of the fetus is 37 weeks. What is the best intervention to prevent complications in the client? Instruct the client to stay on bed Provide the client with a nutritious dietary plan Prepare the client for induction of labor Instruct the client to come next week

Prepare the client for induction of labor Rationale A gestational age of 37 weeks in a client with gestational hypertension and dangerously high blood pressure indicates that labor should be induced as soon as possible. After 37 weeks of gestation, there may be detrimental effects of gestational hypertension on the fetus. Bed rest may not help relieve high blood pressure and therefore is not beneficial to the fetus. Nutritious food is essential for the client throughout pregnancy irrespective of the fetus' gestational age, but not as important as inducing labor in this client at 37 weeks of gestation. Instructing the client to come next week may worsen the condition and may be fatal to the fetus.

A client diagnosed with placenta accreta has uncontrolled bleeding, despite administering medications. What should be the best choice for treatment in this situation? Massage the uterus Prepare the client for surgery Replace blood components as needed Apply traction on the umbilical cord

Prepare the client for surgery Rationale Placenta accreta is an obstetric complication in which the placenta adheres to and penetrates the myometrium. The patient with placenta accreta is at risk of having hemorrhage during childbirth. If bleeding is not stopped after the administration of medication to the client, then a hysterectomy has to be performed to prevent further complications. Replacement of blood components is not useful, because the client has uncontrolled bleeding. Massaging the uterus and applying traction to the umbilical cord is helpful to expel the placenta, but is not useful when the placenta is adhered to the uterus.

Signs of a threatened abortion (miscarriage) are noted in a woman at 8 weeks of gestation. What is an appropriate management approach? Prepare the woman for a dilation and curettage (D&C). Place the woman on bed rest for at least 1 week and reevaluate. Prepare the woman for an ultrasound and blood work. Comfort the woman by telling her that if she loses this baby, she may attempt to get pregnant again in 1 month.

Prepare the woman for an ultrasound and blood work. Rationale Repetitive transvaginal ultrasounds and measurement of human chorionic gonadotropin (hCG) and progesterone levels may be performed to determine if the fetus is alive and within the uterus. If the pregnancy is lost, the woman should be guided through the grieving process. D&C is not considered until signs of the progress to an inevitable abortion are noted or the contents are expelled and incomplete. Bed rest is recommended for 48 hours initially. Telling the woman that she can get pregnant again soon is not a therapeutic response because it discounts the importance of this pregnancy.

Signs of a threatened abortion (miscarriage) are noted in a woman at 8 weeks of gestation. What is an appropriate management approach for this type of abortion? Prepare the woman for a dilation and curettage (D&C). Put the woman on bed rest for at least 1 week and reevaluate. Prepare the woman for an ultrasound and blood work. Comfort the woman by telling her that if she loses this baby, she may attempt to get pregnant again in 1 month.

Prepare the woman for an ultrasound and blood work. Repetitive transvaginal ultrasounds and measurement of human chorionic gonadotropin (hCG) and progesterone levels may be performed to determine whether the fetus is alive and within the uterus. Bed rest is recommended for 48 hours initially. D&C is not considered until signs of the progress to inevitable abortion are noted or the contents are expelled and incomplete. If the pregnancy is lost, the woman should be guided through the grieving process. Telling the client that she can get pregnant again soon is not a therapeutic response because it discounts the importance of this pregnancy.

Signs of a threatened abortion (miscarriage) are noted in a woman at 8 weeks of gestation. What is an appropriate management approach for this type of abortion? Prepare the woman for a dilation and curettage (D&C). Place the woman on bed rest for at least 1 week and reevaluate. Prepare the woman for an ultrasound and bloodwork. Comfort the woman by telling her that if she loses this baby, she may attempt to get pregnant again in 1 month.

Prepare the woman for an ultrasound and bloodwork. Rationale Repetitive transvaginal ultrasounds and measurement of human chorionic gonadotropin (hCG) and progesterone levels may be performed to determine if the fetus is alive and within the uterus. If the pregnancy is lost, the woman should be guided through the grieving process. D&C is not considered until signs of the progress to inevitable abortion are noted or the contents are expelled and incomplete. Bed rest is recommended for 48 hours initially. Telling the client that she can get pregnant again soon is not a therapeutic response because it discounts the importance of this pregnancy.

It has been determined after ultrasound that a small piece of the placenta remains in the uterus over an hour after birth, causing the fundus not to be firm and excessive bleeding to continue. Because the client delivered a large infant with a small dose of intravenous pain medication, what action should the nurse take? Preparing the client for the removal of the retained placental fragment, including the use of anesthesia. Encouraging the consumption of oral fluids to expand the fluid volume. Preventing the mother from nursing her infant until her vital signs are stable. Encouraging the mother to nurse as much as possible to clamp down the fundal vessels.

Preparing the client for the removal of the retained placental fragment, including the use of anesthesia Rationale The client will need to have the retained placental fragment removed under anesthesia because of the time period since delivery. The client should be nothing by mouth (NPO) at this time because of the expected anesthesia. The infant can nurse, but the retained placental fragment will not allow the uterus to contract. After removal of a retained placenta, the woman is at continued risk for primary pulmonary hypertension (PPH) and infection.

The primary health care provider requests that the nurse test for proteinuria in a pregnant client. What preliminary examination does the nurse perform before testing for proteinuria in order to get accurate results? Presence of blood in the urine Presence of bacteria in the urine Presence of sodium in the urine Presence of uric acid in the urine Presence of amniotic fluid in the urine

Presence of blood in the urine Presence of bacteria in the urine Presence of amniotic fluid in the urine Rationale Before testing for the presence of proteinuria in a pregnant client, the nurse collects the urine sample for laboratory assessment to detect the presence of blood, bacteria, or amniotic fluid. These substances may interfere with the diagnostic results, leading to a false report. The presence of sodium and uric acid in the urine may help identify the condition of preeclampsia but does not give a false positive result in the test for protein in the urine.

What is chronic hypertension?

Present before the pregnancy or diagnosed before week 20 of gestation may acquire preeclampsia. persists longer than 12 weeks postpartum

Which factor should alert the nurse to the potential for a prolapsed umbilical cord? Oligohydramnios Pregnancy at 38 weeks of gestation Presenting part at a station of -3 Meconium-stained amniotic fluid

Presenting part at a station of -3 Rationale Because the fetal presenting part is positioned high in the pelvis and is not well applied to the cervix, a prolapsed cord could occur if the membranes rupture. Hydramnios puts the woman at risk for a prolapsed umbilical cord. A very small fetus, normally preterm, puts the woman at risk for a prolapsed umbilical cord. Meconium-stained amniotic fluid shows that the fetus already has been compromised, but it does not increase the chance of a prolapsed cord.

A woman at 34 weeks' gestation reports having four contractions in one hour and menstrual-like abdominal cramps. Which condition is the woman experiencing? Stillbirth Miscarriage Chorioamnionitis Preterm labor (PTL)

Preterm labor (PTL) A woman with four uterine contractions in one hour and menstrual-like abdominal cramps indicates PTL. PTL occurs when regular contractions begin to open a pregnant woman's cervix before 37 weeks.

Nsg interventions for an HIV client during the prenatal period?

Prevent opportunistic infections, avoid procedures that increase the risk of perinatal trasmission, such as amniocentesis and fetal scalp sampling

Which factor is known to increase the risk of gestational diabetes mellitus? Previous birth of large infant Maternal age younger than 25 Underweight before pregnancy Previous diagnosis of type 2 diabetes mellitus

Previous birth of large infant Rationale Previous birth of a large infant suggests gestational diabetes mellitus. A woman younger than 25 is not at risk for gestational diabetes mellitus. Obesity (greater than 90 kg or 198 lb) creates a higher risk for gestational diabetes. The person with type 2 diabetes mellitus already has diabetes and will continue to have it after pregnancy. Insulin may be required during pregnancy because oral hypoglycemia drugs are contraindicated during pregnancy.

Which patient history places the woman at the highest risk for placenta previa? Domestic violence Abdominal trauma Previous abruptio placentae Previous incomplete abortion

Previous incomplete abortion Previous suction curettage, which may have been performed in the case of an incomplete abortion, increases the risk for placenta previa.

A nurse is evaluating several obstetric patients for their risk for cervical insufficiency. Which patient would be considered to be most at risk? Primipara Grandmultip who has previously had all vaginal deliveries without a problem Primip who undergoes a cervical cone biopsy for cervical dysplasia prior to the pregnancy Multip who had her previous delivery via C section due to cephalopelvic disproportion (CPD)

Primip who undergoes a cervical cone biopsy for cervical dysplasia prior to the pregnancy Any patient who has had previous surgical interventions (cone biopsy) is at greater risk for cervical insufficiency. There is no indication that a primip is at risk for cervical insufficiency. A grandmultip who has previously had vaginal deliveries without incidence is not necessarily at an increased risk for cervical insufficiency. A multip who has delivered via C section as a result of CPD would not necessarily be at an increased risk as the issue involves pelvic adequacy as determined by pelvic measurements in relationship to the fetus.

A nurse is evaluating several obstetrical clients for their risk for cervical insufficiency. Which client would be considered most at risk? Primipara Grand multipara who has previously had all vaginal deliveries without a problem Primipara who has a cervical cone biopsy performed for cervical dysplasia prior to the pregnancy. Multipara who had her previous delivery via C section due to cephalopelvic disproportion (CPD).

Primipara who has a cervical cone biopsy performed for cervical dysplasia prior to the pregnancy. Rationale A client who has had previous surgical interventions (such as a cone biopsy) is at greater risk for cervical insufficiency. There is no indication that a primipara is at risk for cervical insufficiency. A grand multipara who has previously had vaginal deliveries without incident is not necessarily at an increased risk for cervical insufficiency. A multipara who has delivered via C section as a result of CPD would not necessarily be at an increased risk, because the issue involves pelvic adequacy as determined by pelvic measurements in relationship to the fetus.

Chronic villus sampling

Procedure that involves the removal of a small tissue specimen from the fetal portion of the placenta. Because this tissue orginates from the zygote, it reflects the genetic makeup of the fetus; it is performed between 10 and 13 weeks of gestation, either trascervically or transabdominally

A pregnant client in the first trimester reports spotting of blood with the cervical os closed and mild uterine cramping. What else does the nurse need to assess? Progesterone levels Transvaginal ultrasounds Human chorionic gonadotropin (hCG) measurement Blood pressure Kleihauer-Betke (KB) test reports

Progesterone levels Transvaginal ultrasounds Human chorionic gonadotropin (hCG) measurement Rationale The spotting of blood with the cervical os closed and mild uterine cramping in the first trimester indicate a threatened miscarriage. Therefore the nurse needs to assess progesterone levels, transvaginal ultrasounds, and measurement of hCG to determine if the fetus is alive and within the uterus. BP measurements do not help determine the fetal status. KB assay is prescribed to identify fetal-to-maternal bleeding, usually after a trauma.

Which condition in the client may lead to uterine atony? Select all that apply. Primigravidity Prolonged labor Polyhydramnios Precipitous birth Fetal macrosomia

Prolonged labor Polyhydramnios Fetal macrosomia Rationale Prolonged labor causes the uterus to expand for a longer period of time, which may result in uterine atony. Polyhydramnios causes the uterus to overstretch, leading to uterine atony. Excessive stretching of the uterine muscles may occur due to the presence of a large fetus. It is called fetal macrosomia. Primigravidity does not cause uterine atony, because it does not affect the uterine muscles. Precipitous birth, or rapid birth, does not cause uterine atony, but may lead to vaginal and cervical lacerations.

What medication can be given to prevent preterm birth?

Prophylactic progesterone supplementation -daily suppositories or creams + weekly IM injections -begins at 16 weeks of gestation and continues up through 36 weeks of gestation

Which finding in a urine specimen of a pregnant patient indicates the client has proteinuria? Value of greater than or equal to 0.5+ protein in a dipstick testing Protein concentration that is greater than 300 mg/24 hours Concentration of greater than or equal to 1 g protein in a 24-hour urine collection Protein concentration at 10 mg/dl in random urine specimen

Protein concentration that is greater than 300 mg/24 hours Rationale Proteinuria is determined from dipstick testing on a clean-catch or catheterized urine specimen or evaluation of a 24-hour urine collection. Protein concentration that is greater than 300 mg/24 hours in a 24-hour urine specimen indicates proteinuria. A concentration of greater than or equal to 5 g protein in a 24-hour urine collection will indicate severe preeclampsia. Protein concentration greater than 30 mg/dl in at least two random urine specimens collected at least 6 hours apart will indicate proteinuria. Value of greater than or equal to 1+ on dipstick measurement indicates proteinuria.

What does the nurse assess to detect the presence of a hypertensive disorder in a pregnant client? Select all that apply. Proteinuria Epigastric pain Placenta previa Presence of edema Blood pressure (BP)

Proteinuria Epigastric pain Presence of edema Blood pressure (BP) Rationale Proteinuria indicates hypertension in a pregnant client. Proteinuria is concentration ≥300 mg/24 hours in a 24-hour urine collection. The nurse needs to assess the client for epigastric pain, because it indicates severe preeclampsia. Hypertension is likely to cause edema or swollen ankles due to greater hydrostatic pressure in the lower parts of the body. Therefore the nurse needs to assess the client for the presence of edema. Accurate measurement of BP will help detect the presence of any hypertensive disorder. A systolic BP >140 mm Hg or a diastolic BP >90 mm Hg will indicate hypertension. Placenta previa is a condition wherein the placenta is implanted in the lower uterine segment covering the cervix, which causes bleeding when the cervix dilates.

What does the nurse include in the plan of care of a pregnant client with mild preeclampsia? Ensure prolonged bed rest. Provide diversionary activities. Encourage the intake of adequate fluids. Restrict sodium and zinc in the diet. Refer to Internet-based support group.

Provide diversionary activities. Encourage the intake of adequate fluids. Refer to Internet-based support group. Rationale Activity is restricted in clients with preeclampsia, so it is necessary to provide diversionary activities to such clients to prevent boredom. The nurse encourages the client to increase fluid intake to an adequate level (six to eight 8-ounce glasses of water per day) to enhance renal perfusion and bowel function. The nurse can suggest Internet-based support groups to reduce boredom and stress in the client. Clients need to restrict activity, but complete bed rest is not advised, because it may cause cardiovascular deconditioning, muscle atrophy, and psychologic stress. The client needs to include adequate zinc and sodium in the diet for proper fetal development.

A newborn's heart rate is 80 beats per minute. The nurse learns that during labor, the amniotic fluid was meconium stained. What further assistance should the nurse provide to the newborn? Provide a large-bore suction catheter and bulb syringe. Place the baby in an incubator, providing frequent backrubs. Provide endotracheal tube suction assistance with ventilation. Administer 5 mg of sucrose solution within the first five hours of birth.

Provide endotracheal tube suction assistance with ventilation. Rationale Oxytocin may cause uterine tachysystole, which may lead to meconium-stained amniotic fluid. Meconium contains waste products of the fetus. Meconium-stained amniotic fluid increases the risk of fetal meconium aspiration. Therefore, the newborn should be provided endotracheal suction to help remove the meconium aspirated into the lungs. The newborn's heart rate of 80 beats per minute indicates reduced heart rate that should be managed by providing ventilation support to the newborn. A large-bore suction catheter and bulb syringe are used to remove meconium ingested by the baby if the heart rate of the newborn is more than 100 beats per minute. The nurse should remove the ingested meconium first. Incubating the newborn and providing backrubs would not help to remove the meconium. A sucrose solution of 5 mg is administered to newborns with hypoglycemia. Sucrose solution is unrelated to meconium aspiration.

The nurse is caring for a pregnant client with a body mass index (BMI) of 32 kg/m 2. The client gives birth at 40 weeks of gestation by cesarean. Which postpartum intervention does the nurse plan to implement after childbirth? Select all that apply. Avoid ambulation until the client's BMI improves. Provide sequential compression device (SCD) boots. Provide the sufficient nutrition to the preterm neonate. Administer glucose therapy to the client. Implement strategies to prevent infection in the client.

Provide sequential compression device (SCD) boots. Implement strategies to prevent infection in the client. Rationale A BMI of 32 kg/m 2 indicates that the client is obese. Obesity increases the risk of thromboembolism following the delivery. SCD boots are used postoperatively to decrease the risk of clot formation. A cesarean delivery may increase the risk of infection in clients who are obese due to concurrent morbidities. Therefore, infection control measures should be taken. The client should be encouraged to ambulate early on to prevent pulmonary and vascular complications. The neonate born after 40 weeks of gestation is considered postterm; a preterm neonate is born before 37 weeks of gestation. Aggressive glucose is appropriate for a pregnant woman who is in labor to prevent hypoglycemia, but a postoperative client may not need glucose therapy.

The nurse is caring for a client with insulin-dependent diabetes mellitus in the first trimester of pregnancy. The client feels dizzy and lethargic and her blood glucose level is 50 mg/dl. What should the nurse do first in this situation? Ask the dietician to recommend a sugar free diet to the client. Assess the client for symptoms of retinopathy and nephropathy. Assess the serum progesterone and estrogen levels in the client. Provide the client a dose of glucose gel or a few glucose tablets.

Provide the client a dose of glucose gel or a few glucose tablets. Rationale Symptoms such as dizziness and lethargy, as well as a blood glucose level of 50mg/dl, indicate that the client may have hypoglycemia or be in insulin shock. Pregnant client with insulin-dependent diabetes mellitus are extremely prone to hypoglycemia during the first trimester, because estrogen and progesterone stimulate the release of insulin. In this situation, the nurse should give the client fast glucose in the form of a gel or tablets to quickly stabilize the client . If the client is hypoglycemic, the nurse should not ask the dietician to prescribe a completely sugar-free diet. The signs and symptoms of the client do not indicate that the client may have retinopathy and nephropathy. The progesterone and estrogen levels are high in a pregnant client In this situation, it is not important to assess the levels of these hormones.

The nurse is caring for a patient who had a forceps delivery that caused a perineal hematoma. Which nursing intervention is most appropriate? Administer topical analgesic ointment as prescribed Provide the patient with an ice pack and educate her about its use Encourage the patient to lie on her side as much as possible until the injury heals Educate the patient about the fact that the hematoma was caused by the introduction of forceps

Provide the patient with an ice pack and educate her about its use Ice packs can help relieve the pain of hematomas or lacerations after operative delivery by causing vasoconstriction and decreasing blood flow to the area, as well as decreasing edema. Patients should be guided toward use for the first 12 hours, followed by intermittent use.

The nurse is assessing a client with hyperemesis gravidarum during the early stages of pregnancy. Which nonpharmacologic measure is appropriate to alleviate the discomforts associated with nausea and vomiting? Having the client cook her favorite foods Allowing frequent visits from friends Providing environment that is free from odors Having the client eat warm, low-fat, soupy foods

Providing environment that is free from odors Rationale The client must be allowed to rest in an environment that is free from odors. This helps to alleviate the discomforts associated with hyperemesis gravidarum. Most clients find exposure to cooking odors nauseating; it is better to have other family members cook for the client. It is important for the client to have limited periods of visitation and receive adequate rest, because sleep disturbances accompany hyperemesis. The client is able to tolerate dry, cold foods better than warm, soupy foods.

The nurse is caring for a preterm baby in the neonatal intensive care unit (NICU) who was born to a mother with preeclampsia. The family is anxious about the baby. What nursing interventions can help relieve anxiety in the family? Select all that apply. Providing the family with photographs of the baby Temporarily releasing the baby into the family's care Allowing the partner/spouse to visit the NICU Arranging a bed for the mother beside the baby Informing family members about the baby's status

Providing the family with photographs of the baby Allowing the partner/spouse to visit the NICU Informing family members about the baby's status Rationale A mother with preeclampsia has an increased risk of preterm birth. The infant born in such a case may need to be cared for in a neonatal intensive care unit. The nurse can reduce the anxiety of the family members by providing photographs of the baby. This intervention assures them of the baby's well-being and acts as a psychologic relief to the anxious family. The nurse should encourage the partner of the baby to visit the NICU, which facilitates bonding and attachment with the baby. Informing family members about the baby's status helps relieve the family's anxiety. The nurse should not hand the baby over to the family for some time, because it may expose the baby to infections and compromise his or her health. The nurse should not arrange the mother's bed beside the baby. Instead, the mother can be taken to the NICU with the help of a wheelchair once her condition is stabilized.

A client has elevated nuchal translucency and low maternal serum marker levels. Which abnormality would be likely in the client's newborn? Pyelectasis Oligohydramnios Neural tube defect Fetal cardiac disease

Pyelectasis Rationale Pyelectasis is the enlargement of the renal pelvis and is considered a "soft marker" for Down syndrome. Elevated nuchal translucency and low maternal serum marker levels are indicative of pyelectasis. Oligohydramnios is a decrease in amniotic fluid. Elevated nuchal translucency and low maternal serum marker levels do not indicate oligohydramnios. Neural tube defects may be caused by reduced folic acid availability; they cannot be identified by elevated nuchal translucency and low maternal serum marker levels. Elevated nuchal translucency alone indicates an increased risk of fetal cardiac disease and chromosomal abnormalities.

Bishop score;

Rating system used to evaluate the inducibility of the cervix. The five character- istics assessed are dilation (cm); effacement (%); station (cm); cervical consistency; cervical position

The sibling of an expired infant wants to know how long it will take until the baby comes home. In which way should the nurse recommend the parents respond? Recommend that they answer simply and truthfully. Recommend that they have their clergy speak with the sibling. Recommend that they explain the details of the circumstances of the death to the sibling. Recommend that they avoid answering the question until they have been discharged home.

Recommend that they answer simply and truthfully. The nurse should encourage the parents to speak simply and truthfully when the siblings ask about the baby to allow the rest of the family to grieve the loss.

A pregnant woman with type 1 diabetes is on rapid-acting, short-acting, and intermediate-acting insulin injections. Which are rapid and short-acting insulins? NPH (Novolin N) Regular (Humalin) Lispro (Humalog) Aspart (NovoLog) Glargine (Lantus)

Regular (Humalin) Lispro (Humalog) Aspart (NovoLog) Rationale Humalog and NovoLog are rapid-acting insulins and Humalin is a short-acting insulin. Novolin N is an intermediate-acting insulin and Lantus is a long-acting insulin.

Nsg interventions for DIC?

Remove underlying cause, monitor vitals, assess for signs of bleeding or shock, prepare for O2 therapy, volume replacement, blood component therapy and possibly heparin therapy, monitor for complications associated with fluid and blood replacement and heparin therapy, monitor urine output and maintain at 30 mL/hr

The nurse is caring for a client who gave birth vaginally two days ago. The nurse finds that the client has a blood pressure of 160/105 mm Hg. For what complications does the nurse monitor the client to ensure her safety? Select all that apply. Renal failure Hypovolemia Liver damage Brain damage Pulmonary edema

Renal failure Pulmonary edema Rationale A blood pressure reading of 160/105 mm Hg two days after delivery indicates chronic hypertension. The nurse should closely monitor the client for complications of chronic hypertension such as renal failure, pulmonary edema, and hypertensive encephalopathy. Hypovolemia does not occur in chronic hypertension. In fact, hypertension is associated with edema, resulting in hypervolemia. Liver damage may not be directly caused by chronic hypertension. Brain damage may also not be a direct complication of chronic hypertension after delivery.

The nurse is caring for a client who has undergone a contraction stress test. The test results were found to be equivocal. What intervention should the nurse perform based on the test results? Perform a non-stress test Repeat the contraction stress test next day Resume the weekly testing schedule Prepare to admit the client

Repeat the contraction stress test next day Rationale If the results of contraction stress test are equivocal, it indicates decelerations that occur in the presence of contractions are more frequent than every 2 minutes. Therefore, the primary health care provider would ask the nurse to repeat the test the next day. The contraction stress test is prescribed after a non-stress test if the test results are nonreactive. If the contraction stress test is negative, then the primary health care provider would ask the client to resume the weekly testing schedule. If the contraction stress test is positive, then the primary health care provider would instruct the nurse to admit the client to the hospital.

A nurse is caring for a laboring patient with hydramnios. After the patient's membranes rupture, the fetal heart rate is 100 beats/min for longer than 90 seconds. The nurse suspects cord compression. Which action should the nurse perform first? Perform a vaginal examination Assess the patient's blood pressure Reposition the patient on her hands and knees with the head lowered Coach the patient in deep breathing exercises to promote oxygenation

Reposition the patient on her hands and knees with the head lowered Repositioning the patient onto her hands and knees with the head lowered relieves the pressure on the umbilical cord and maintains fetal oxygenation. Fetal hypoxia can have rapid and severe effects.

The nurse is caring for a patient experiencing prolonged labor. The patient reports intense back pain that radiates down to the leg. Which is the appropriate nursing intervention for this patient? Elevate the head of the bed. Notify the health care provider. Administer the ordered pain medication. Reposition the patient onto her hands and knees.

Reposition the patient onto her hands and knees. Back pain that radiates to the leg is also called "back labor", and it is often a sign of a fetus in the occiput posterior (OP) position. Repositioning the patient onto her hands and knees helps facilitate fetal rotation.

Which nursing action should be initiated first when there is evidence of prolapsed cord? Notify the health care provider. Apply a scalp electrode. Prepare the woman for an emergency cesarean birth. Reposition the woman with her hips higher than her head.

Reposition the woman with her hips higher than her head. Rationale The priority is to relieve pressure on the cord. Changing the maternal position will shift the position of the fetus so that the cord is not compressed. Notifying the health care provider is a priority but not the first action. Applying a scalp electrode is not appropriate at this time. Preparing the woman for an emergency cesarean birth is not the first priority.

The nurse administers magnesium sulfate (Epsom salts) to stop labor in a pregnant client. Which symptoms should the nurse monitor to ensure the client's safety? Swollen legs Respiratory rate Eating patterns Maternal chills

Respiratory rate Rationale Magnesium sulfate (Epsom salts) is administered to a pregnant client to stop labor. Magnesium sulfate (Epsom salts) causes respiratory depression as a toxic effect. Therefore, the nurse should monitor the respiratory rate of the client. Swollen legs or edema is acommon observation during labor, which is caused by increased abdominal contents. Edema is unrelated to magnesium sulfate. Magnesium sulfate (Epsom salts) does not alter a client's eating habits. Maternal chills are observed in clients with membrane rupture and are unrelated to magnesium sulfate (Epsom salts).

A pregnant client is on tocolytic therapy with magnesium sulfate. Under which client circumstance would the nurse suggest discontinuing the therapy? Blood pressure is 120/80 mm Hg. Respiratory rate is 10 breaths per minute. Urine output is 40 ml per hour. Serum magnesium level is 5 mEq/L.

Respiratory rate is 10 breaths per minute. Rationale Magnesium sulfate is used as a tocolytic. However, it can cause severe adverse effects. Therefore the nurse should closely monitor the client. A respiratory rate of 10 breaths per minute indicates that the client has respiratory depression, which is an adverse effect of magnesium sulfate. Therefore the nurse should stop administration of the drug. A blood pressure of 120/80 mm Hg is normal and does not require discontinuation of magnesium sulfate. Urine output of 40 ml per hour indicates normal urine output; hence, the nurse need not discontinue the therapy. The therapeutic serum magnesium level should be 5 mEq/L to exert its action. Therefore if the serum magnesium level is 5 mEq/L, the nurse need not discontinue the therapy, because it would not cause toxic effects.

The nurse is caring for a pregnant client who is administered magnesium sulfate to prevent preterm labor. Which parameters should the nurse assess in the patient to determine drug toxicity? Select all that apply. Fluid intake Respiratory status Body temperature Level of consciousness Deep tendon reflexes

Respiratory status Level of consciousness Deep tendon reflexes Rationale Magnesium sulfate, when used as a tocolytic agent, depresses the central nervous system (CNS). The CNS depressive effect would be enhanced if the drug reaches toxic levels. CNS activity can be determined by assessing the respiratory status, level of consciousness, and deep tendon reflexes. A low respiratory rate, decreased level of consciousness, and slow reflexes indicate magnesium sulfate toxicity. Fluid intake and body temperature are not affected by CNS depression.

The nurse is caring for a pregnant client who is administered magnesium sulfate to prevent preterm labor. Which parameters should the nurse assess in the patient to determine drug toxicity? Select all that apply. Fluid intake Respiratory status Body temperature Level of consciousness Deep tendon reflexes

Respiratory status Level of consciousness Deep tendon reflexes Rationale Magnesium sulfate, when used as a tocolytic agent, depresses the central nervous system (CNS). The CNS depressive effect would be enhanced if the drug reaches toxic levels. CNS activity can be determined by assessing the respiratory status, level of consciousness, and deep tendon reflexes. A low respiratory rate, decreased level of consciousness, and slow reflexes indicate magnesium sulfate toxicity. Fluid intake and body temperature are not affected by CNS depression.

A pregnant woman is admitted to labor and delivery unit after reports of preterm contractions. Which action is a priority for the nurse to take? Restrict the woman's activity Administer tocolytics immediately Provide patient education on preterm labor Call the radiology department for ultrasound

Restrict the woman's activity Activity restriction reduces uterine contractions and it is an appropriate priority nursing action at this stage as the nurse focuses on initial measures to delay preterm labor, such as reducing the woman's activity.

Which intervention will help prevent the risk of pulmonary edema in a pregnant client with severe preeclampsia? Assess fetal heart rate (FHR) abnormalities regularly. Place the client on bed rest in a darkened environment. Restrict total intravenous (IV) and oral fluids to 125 ml/hr. Ensure that magnesium sulfate is administered as prescribed

Restrict total intravenous (IV) and oral fluids to 125 ml/hr. Rationale Pulmonary edema may be seen in clients with severe preeclampsia. Hence the nurse needs to restrict total intravenous (IV) and oral fluids to 125 ml/hr. FHR monitoring helps to assess any fetal complications. The client is placed on bed rest in a darkened environment to prevent stress. Magnesium sulfate is administered to prevent eclamptic seizures.

Which intervention will help prevent the risk of pulmonary edema in a pregnant client with severe preeclampsia? Assess fetal heart rate (FHR) abnormalities regularly. Place the client on bed rest in a darkened environment. Restrict total intravenous (IV) and oral fluids to 125 ml/hr. Ensure that magnesium sulfate is administered as prescribed.

Restrict total intravenous (IV) and oral fluids to 125 ml/hr. Rationale Pulmonary edema may be seen in clients with severe preeclampsia. Hence the nurse needs to restrict total intravenous (IV) and oral fluids to 125 ml/hr. FHR monitoring helps to assess any fetal complications. The client is placed on bed rest in a darkened environment to prevent stress. Magnesium sulfate is administered to prevent eclamptic seizures.

Venous Thromboembolic Disorders

Results from formation of blood clots inside a blood vessel and causes inflammation (thrombophlebitis) 3 Types: 1. Superficial venous thrombosis 2. Deep Venous Thrombosis (DVT)- occurs in lower extremities 3. Pulmonary embolism (PE)- complication of DVT when clot dislodges and occludes a vessel in the lung When do venous thromboemoblic disorders occur? DVT occurs during pregnancy and PE occurs in the postpartum period (most commonly) -important reason to initiate ambulation shortly after birth -due to hypercoagulability state of pregnancy and venous stasis Risk factors for VTE -history of venous thrombosis -operative vaginal birth -pulmonary embolism -varicosities -obesity -maternal age >35 -smoking -multiparity What is the most common form of VTE? Superficial venous thrombosis -pain and tenderness in lower extremity -warmth, redness, swelling, hardened vein over site of thrombosis Treatment: -rest with elevation of affected leg -elastic compression stockings -local heat -analgesia Symptoms and Treatment of DVT Symptoms: -unilateral leg pain, calf tenderness, and swelling -redness and warmth -positive Homan's sign may be present Treatment: -anticoagulant therapy (e.g. IV heparin) -bed rest with affected leg elevated -analgesia -elastic compression stockings (after symptoms resolve) -anticoagulant therapy will be continued for a few months after birth What are the primary causes of thromboembolic disease? venous stasis and hypercoagulation.

A client who has undergone cesarean surgery reports to the nurse about having persistent perineal pain and feels pressure in the vagina. The nurse finds that the client is in shock. What clinical condition should the nurse suspect based on this assessment? Rectocele. Endometritis. Impaired lactation. Retroperitoneal hematoma.

Retroperitoneal hematoma. Rationale Accumulation of blood in the retroperitoneal space is called retroperitoneal hematoma. It is caused by the rupture of the cesarean scar during labor. Retroperitoneal hematoma is characterized by such symptoms as persistent perineal pain, a feeling of pressure in the vagina, and shock. Therefore it is evident that the client has this condition. Persistent perineal pain, a feeling of pressure in the vagina, and shock are not associated with rectocele, endometritis, and impaired lactation. Rectocele is the herniation of the anterior rectal wall through the relaxed or ruptured vaginal fascia and rectovaginal septum. Endometritis is characterized by fever, increased pulse rate, chills, anorexia, nausea, fatigue, pelvic pain, uterine tenderness, and foul-smelling lochia. Because the client did not report these symptoms, the client does not have endometritis. Perineal pain, a feeling of pressure in the vagina, and shock do not affect lactation, so the client does not have impaired lactation.

A pregnant woman at 14 weeks of gestation is admitted to the hospital with a diagnosis of hyperemesis gravidarum. The primary goal of her treatment at this time is to: Rest the gastrointestinal (GI) tract by restricting all oral intake for 48 hours. Reduce emotional distress by encouraging the woman to discuss her feelings. Reverse fluid, electrolyte, and acid-base imbalances. Restore the woman's ability to take and retain oral fluid and foods.

Reverse fluid, electrolyte, and acid-base imbalances. Fluid, electrolyte, and acid-base imbalances present the greatest immediate danger to the well-being of the mother and fetus and should be corrected as soon as possible. Resting the GI tract and discussing her feelings are components of treatment but are not immediate goals for this client. The ability to retain oral fluid and foods is a longer-term goal of treatment for this condition.

A 30 weeks' gestation neonate is born to a mother who tested positive for group beta strep (GBS). The nurse would consider which interventions appropriate when caring for this neonate? Select all that apply. Obtaining vital signs once a shift Reviewing the complete blood cell count Weighing all weight diapers throughout the shift Obtaining a blood culture after starting antibiotics Instructing the parents to wash their hands before touching the newborn Administering ampicillin and gentamycin intravenously one after each other

Reviewing the complete blood cell count The complete blood cell count provides information regarding whether or not an infection is present. Correct Weighing all weight diapers throughout the shift Weighing the diapers provides the nurse with data regarding the hourly output of the neonate. Fluid balance maintenance is important for the neonate whose mother is GBS positive. Instructing the parents to wash their hands before touching the newborn Careful and frequent handwashing is an important way to prevent infection. Correct Administering ampicillin and gentamycin intravenously one after each other Broad-spectrum antibiotics are given for suspected infection. The timing of administration should be coordinated when more than one antibiotic is ordered.

Rupture of uterus

S/S Complete or incomplete separation of the uterine tissue as a result of a tear in wall from stress of labor Risk factors- labor after previous c-section, overdistened uterus, hydramnios, abdominal trauma. S/S abd pain or tenderness, chest pain, contractions stop or fail to progress, rigid abdomen, absent FHR, s/s shock, fetus palpated outside of uterus (Complete rupture)

Fetus distress

S/S FHR <110 or >160, meconium-stained amniotic fluid, fetal hyperactivity/hypoactivity, progressive decrease in baseline variability, severe variable decelerations, late decelerations Interventions Place client in lateral position Admin oxygen 8-10 l via face mask discontinue Oxytocin Monitor fetal and maternal status -If fetal in distress -> c-section

Intrauterine fetal demise

S/S Loss of fetal movement, absence of fetal heart tones, DIC- disseminated intravascular coagulation screen, low Hemoglobin & Hct, low platelett, prolonged bleeding Interventions- encourage client to verbalize feelings, etc administer fluids, medications, blod

The primary health care provider asks the nurse to check for edema in a pregnant client with preeclampsia on bed rest. Which region of the client's body does the nurse check for edema? Foot region Ankle region Head region Sacral region

Sacral region Rationale The nurse should check the sacral region for the presence of edema if the client is confined to the bed. If the pregnant client is ambulatory, then edema may first be evident in the feet and ankles. Edema in the head region may not be evident first in pregnant clients who are confined to the bed.

Surgical Management of Ectopic Pregnancy

Salpingetctomy- removal of the entire uterine tube Salpingostomy- if tube has not ruptured and woman may want to become pregnant in the future

The nurse is assessing a client at 30 weeks of pregnancy with poorly controlled pregestational diabetes. The client has come for the antenatal visit on a Monday. When should the nurse ask the client to come next? Next week on a Monday Next week on a Thursday Same week on a Thursday Week after next week on a Monday

Same week on a Thursday Rationale An antepartum client with pregestational diabetes requires more frequent antenatal visits. A client with poorly controlled diabetes who is in the third trimester of pregnancy should visit the clinic twice a week. If the client visited the clinic on a Monday, the nurse should instruct the client to come for the next visit on the same week on a Thursday. A client who is in the first and second trimester of pregnancy can come once in a week or two weeks. Because this client is in the third trimester and has poorly controlled diabetes, the nurse should not ask the client to come on next week on Monday, Thursday, or the week after next on Monday.

The nurse observes that a client has a high fever, maternal and fetal tachycardia, uterine tenderness, and vaginal discharge with a foul odor in early labor. Which signs should the nurse assess in the neonate? Seizures Breathing difficulties Laceration on the head High body temperature Heart rate of 110 beats per minute

Seizures Breathing difficulties High body temperature Rationale Fever, maternal and fetal tachycardia, uterine tenderness, and vaginal discharge with a foul odor are the signs of a bacterial infection called chorioamnionitis. Women with chorioamnionitis are more likely to have a cesarean birth due to dysfunctional labor. Neonatal risks include pneumonia, sepsis, and cerebral palsy. Therefore, the nurse should monitor signs like seizures (indicative of cerebral palsy), breathing difficulties (indicative of pneumonia), and a high body temperature (indicative ofsepsis) in the neonates of these clients. Neonates who are born through forceps- or vacuum-assisted birth may have a laceration on the head; however, this is not associated with chorioamnionitis. A heart rate of 110 beats per minute is a normal observation in newborns and is not a serious sign.

Which conditions does the nurse remain alert for in a pregnant client with preeclampsia? Select all that apply. Seizures Scotoma Renal disease Cerebral edema Chronic hypertension

Seizures Scotoma Cerebral edema Rationale Seizures may be seen due to the central nervous system irritability in the patient. Scotoma is a visual disturbance that is seen in a patient with preeclampsia due to arteriolar vasospasms and decreased blood flow to the retina. Cerebral edema is a neurologic complication associated with preeclampsia. Chronic hypertension is seen in pregnant clients before pregnancy and is not associated with preeclampsia. Renal disease is a risk factor that may cause preeclampsia in the client.

The nurse is caring for a client with severe preeclampsia who is on an intravenous (IV) infusion of magnesium sulfate (Sulfamag). What assessment parameter indicates that the treatment is a success? Edema is reduced Seizures do not occur Blood pressure is reduced Respiratory rate is reduced

Seizures do not occur Rationale Magnesium sulfate (Sulfamag) is administered to a client with preeclampsia to prevent seizures. The treatment is evaluated as successful if the seizures cease. A decrease in edema and blood pressure indicates reduced severity of preeclampsia in the client but is not caused by the magnesium sulfate (Sulfamag). A decreased respiratory rate in the client is considered an adverse effect of magnesium sulfate (Sulfamag) and requires the primary health care provider to attend to the client immediately.

A woman with severe preeclampsia is being treated with an IV infusion of magnesium sulfate. This treatment is considered successful if what happens? Blood pressure is reduced to prepregnant baseline Seizures do not occur Deep tendon reflexes become hypotonic Diuresis reduces fluid retention

Seizures do not occur Rationale Magnesium sulfate is a central nervous system (CNS) depressant given primarily to prevent seizures. A temporary decrease in blood pressure can occur; however, this is not the purpose of administering this medication. Hypotonia is a sign of an excessive serum level of magnesium. It is critical that calcium gluconate be on hand to counteract the depressant effects of magnesium toxicity. Diuresis is not an expected outcome of magnesium sulfate administration.

A woman with severe preeclampsia is being treated with an IV infusion of magnesium sulfate. This treatment is considered successful if: Blood pressure is reduced to prepregnant baseline. Seizures do not occur. Deep tendon reflexes become hypotonic. Diuresis reduces fluid retention

Seizures do not occur. Magnesium sulfate is a central nervous system (CNS) depressant given primarily to prevent seizures. A temporary decrease in blood pressure can occur but is not the purpose of administering this medication. Hypotonia is a sign of an excessive serum level of magnesium. It is critical that calcium gluconate be on hand to counteract the depressant effects of magnesium toxicity. Diuresis is not an expected outcome of magnesium sulfate administration.

The nurse is assessing a pregnant client at 16 weeks of gestation. Which diagnostic test should the nurse say is used to identify neural tube defects in the fetus? Serum alpha-fetoprotein Fetal echocardiography Glycosylated hemoglobin Nonstress test (NST)

Serum alpha-fetoprotein Rationale Measurement of maternal serum alpha-fetoprotein is performed between 16 and 18 weeks of gestation to determine the risk of neural tube defects. The fetus is at increased risk for neural tube defects such as spina bifida, anencephaly, and microcephaly. Fetal echocardiography is performed between 20 and 22 weeks of gestation to detect cardiac anomalies. Glycosylated hemoglobin is measured to assess glycemic control over the previous 4 to 6 weeks. The nonstress test (NST) is performed between 28 and 32 weeks of gestation in clients with vascular disease or poor glucose control. The NST is used to evaluate fetal well-being.

How is HIV transmitted?

Sexual exposure to genital secretions of an infected person, parenteral exposure to infected blood and tissue, preinteral exposure of an infant to infected maternal secretions through birth or breast feeding

Which factors would lead to an increased likelihood of uterine rupture? Select all that apply. Preterm singleton pregnancy G3P3 with all vaginal deliveries Short interval between pregnancies Labor patient receiving a trial labor (TOL) following a VBAC delivery Patient who had a primary caesarean section with a classical incision

Short interval between pregnancies Labor patient receiving a trial labor (TOL) following a VBAC delivery Patient who had a primary caesarean section with a classical incision Rationale The shorter the interval time between pregnancies/deliveries increases the risk for uterine rupture. A labor patient who is having a TOL following a VBAC is at increased risk for uterine rupture. A patient who has a C section with a classical incision into the uterus is at increased risk for uterine rupture. A pregnant woman with a singleton pregnancy (one fetus) even if preterm is not considered to be at increased risk for uterine rupture. A multipara who has had all deliveries via the vaginal route is not considered to be at an increased risk for uterine rupture.

Which factors would lead to an increased likelihood of uterine rupture? Select all that apply. Preterm singleton pregnancy G3P3 with all vaginal deliveries Short interval between pregnancies Patient receiving a trial of labor (TOL) following a VBAC delivery Patient who had a primary caesarean section with a classic incision

Short interval between pregnancies The shorter the interval between pregnancies/deliveries, the higher the risk of uterine rupture. A patient who is having a TOL following a VBAC and a patient who has had a C section with a classic incision into the uterus are at increased risk for uterine rupture. A pregnant woman with a singleton pregnancy (one fetus), even if preterm, is not considered to be at increased risk for uterine rupture; nor is a multipara who has delivered all her infants vaginally. CORRECT Patient receiving a trial of labor (TOL) following a VBAC delivery The shorter the interval between pregnancies/deliveries, the higher the risk of uterine rupture. A patient who is having a TOL following a VBAC and a patient who has had a C section with a classic incision into the uterus are at increased risk for uterine rupture. A pregnant woman with a singleton pregnancy (one fetus), even if preterm, is not considered to be at increased risk for uterine rupture; nor is a multipara who has delivered all her infants vaginally. Patient who had a primary caesarean section with a classic incision The shorter the interval between pregnancies/deliveries, the higher the risk of uterine rupture. A patient who is having a TOL following a VBAC and a patient who has had a C section with a classic incision into the uterus are at increased risk for uterine rupture. A pregnant woman with a singleton pregnancy (one fetus), even if preterm, is not considered to be at increased risk for uterine rupture; nor is a multipara who has delivered all her infants vaginally.

What clinical significance does a maternal blood Coombs test with a titer of 1:8 and increasing indicate? Fetal lung maturity Significant Rh compatibility Significant Rh incompatibility Fetus with trisomy 13, 18, or 21

Significant Rh incompatibility Rationale The clinical significance of a maternal blood Coombs test with a titer of 1:8 and increasing indicates significant Rh incompatibility. Fetal lung maturity and fetus with trisomy 13, 18, or 21 are not clinically significant of a Coombs test with a titer of 1:8.

Which personal health history factors increase the risk of postpartum thromboembolic disorder? Select all that apply. Smoking Hemophilia Maternal age <20 History of episiotomy History of cesarean birth

Smoking Smoking increases the risk of developing postpartum thromboembolic disorder. Exposure to smoke (even secondhand smoke) can cause damage to blood vessel lining and increase coagulation. History of episiotomy A previous episiotomy can result in vascular damage and increases the risk of developing postpartum thromboembolic disorder. History of cesarean birth A previous cesarean birth can result in blood vessel trauma and significantly increases the risk of developing postpartum thromboembolic disorder.

The nurse finds that the blood pH of a pregnant client who is diabetic is 6.5. What should the nurse administer to normalize the client's blood pH? Dextrose solution Normal saline solution Sodium citrate solution Sodium bicarbonate solution

Sodium bicarbonate solution Rationale A blood pH of 6.5 indicates that the client has acidosis. In order to revert this state, the nurse should administer an alkaline solution, such as sodium bicarbonate solution. Dextrose and normal saline solution are neither acidic nor alkaline. These solutions would not help to normalize the blood pH. Sodium citrate solution would increase acidity and would worsen the client's condition.

The nurse is caring for a pregnant client at 19 weeks of gestation. On reviewing the ultrasound reports, the nurse notes that the fetus has a ventricular septal defect (VSD). Which type of ultrasound helps detect VSD? Limited examination Nonmedical examination Standard or basic examination Specialized or targeted examination

Specialized or targeted examination Rationale Specialized or targeted examinations are performed if a client is suspected to be carrying an anatomically or physiologically abnormal fetus. AVSD is an anatomic defect in the ventricular septum, the wall dividing the left and right ventricles of the heart. Therefore, a specialized or targeted ultrasound examination of the heart is performed to detect a VSD. A limited ultrasound examination is performed to determine the fetal position during delivery. A nonmedical examination is used to obtain 3D pictures of the fetus. However, these examinations may have associated risk and should be used with discretion. A standard or basic examination helps determine the fetal heart rate and position and the amniotic fluid volume in the client.

A pregnant client reports painless vaginal bleeding at 22 weeks of gestation. The client's ultrasonographic reports reveal a normally implanted placenta. What examination does the nurse perform immediately in the client? Non-stress test (NST) Speculum examination Biophysical profile (BPP) Kleihauer-Betke (KB) test

Speculum examination Rationale If the client has a normally implanted placenta, then a speculum examination should be performed immediately to rule out local causes of bleeding. NST and BPP are performed every 2 to 3 weeks for fetal surveillance. A KB test is performed to determine the presence of transplacental hemorrhage.

On reviewing the medical history of a pregnant client, the nurse finds that the client is taking carbamazepine (Tegretol). What consequence of the drug on the fetus should the nurse be aware of? Pyelectasis Spina bifida Omphalocele Lupus erythematosus

Spina bifida Rationale Carbamazepine (Tegretol) is a teratogenic drug that may cause neural tube defects. Therefore, the fetus may be at risk for spina bifida. Pyelectasis is enlargement of the renal pelvis, which is observed in Down syndrome. Omphalocele is an abdominal wall defect but not a neural tube defect. Lupus erythematosus is a collection of autoimmune diseases, seen in the mother but not in the fetu

Which characteristics influence a couple's grieving process following a fetal demise? Select all that apply. Culture Occupation Spiritual beliefs Parents' gender Level of education

Spiritual beliefs Spiritual beliefs do influence the way a patient feels and displays grief after a fetal demise. Most people with strong spiritual beliefs resolved their grief progressively over a period of time. Correct Parents' gender Gender does influence the way a patient feels and displays grief after a fetal demise. A mother's grief response may be different than the father's grief response. Mothers feel a sense of failure but often find it easier to express feelings of sadness. Many fathers feel a need to appear strong so that they can support their partners. Culture Culture does influence the way a patient feels and displays grief after a fetal demise. Expression of grief may be loud and open or parents may appear stoic, depending on cultural expectations. The nurse must be accepting of the family's method of coping with their loss.

Premature Rupture of the Membranes

Spontaneous rupture of amniotic membranes before the onset of labor. If ruptured is before term and delivery will be delay, Infections becomes a risk. S/S presence of fluid pooling in vaginal vault; nitrazine test is positive. Elevated temp= infection tachycardia in fetus may be maternal infection Assessment - amount color and consistency, odor of fluid, monitor VS, feta monitoring, Interventions: assist with tests to assess gestational age, AVOID vaginal exams because of risk of infections. Monitor maternal and fetal for s/s complications or infection, administer antibiotics

Preterm premature rupture of membranes (PPROM)

Spontaneous rupture of the amniotic sac and leakage of fluid before the completion of 37 weeks of gestation most likely occurring as a result of pathogenic weakening of the amniotic membranes by inflammation, stress of uterine contractions, or other factors that cause increased intrauterine pressure.

Which patient education topics are priority areas to review with parents whose infant was stillborn? Select all that apply. Stages of grief Support group resources Resources for dealing with grief Birth control education and prescriptions Prenatal care resources for subsequent pregnancies

Stages of grief Stages of grief should be reviewed with parents who experience a perinatal loss including the birth of a stillborn. Explaining the stages of grief can help parents understand what they will experience. Correct Support group resources Support group resources should be reviewed with parents who experience a perinatal loss including the birth of a stillborn. Support groups can serve as a great resource for parents during this difficult time. Correct Resources for dealing with grief Resources for dealing with grief should be reviewed with parents who experience a perinatal loss including the birth of a stillborn. This may include names of local mental health providers.

Recommended Protocol for administering oxytocin

Start at 1 mu/min and increased by 1-2 mu/min no more frequently than every 30 minutes

Augmentation of labor;

Stimulation of uterine contractions after labor has started spontaneously but progress is unsatisfactory. Common methods include oxytocin infusion and amniotomy

While performing a physical examination of a postpartum client, the nurse finds that the uterus is firm and contracted. The client reports dark red lochial discharge. Which treatment strategy may help prevent further complications in the client? Tocolytics Stool softeners Anticoagulants Halogenated anesthetics

Stool softeners Rationale The client's uterus is firm and contracted, which is a normal finding. Slow discharge of dark red blood indicates superficial laceration. Therefore, stool softener would help to establish a normal bowel tone without straining the laceration. Tocolytics are not given to the client, because they dilate the uterine muscles, which may pressurize the laceration. Anticoagulants are given in the case of thromboembolytic defects in the client. Halogenated anesthetics are given to clients to relax the uterine muscles while correcting the uterine inversion.

A premature infant repeatedly chokes and gasps when being fed by a bottle. The nurse teaches the parents that which actions are most appropriate to take if this situation occurs again at home? Select all that apply. Stop feeding and call the doctor. Watch the infant to see how fast he/she is breathing. Change to a slow flow nipple and keep feeding the baby. Position the infant in a sitting position and keep feeding the baby. Position the infant on his/her side with the head of the bed elevated.

Stop feeding and call the doctor. The neonate is demonstrating signs of feeding intolerance. The healthcare provider must be notified. Correct Watch the infant to see how fast he/she is breathing. Assessing the neonate's respiratory rate will provide data as to whether nipple feedings are contraindicated. Position the infant on his/her side with the head of the bed elevated. Positioning the neonate on the side with the head of bed elevated assists in mobilization of secretions. This will reduce the risk of future choking and gasping.

The primary health care provider prescribes methotrexate (MTX) to treat ectopic pregnancy in a client. The nurse instructs the client to avoid taking any analgesic stronger than acetaminophen (Tylenol). What could be the reason for giving this instruction? MTX has an interaction with other analgesics. Stronger analgesics can mask symptoms of the tubal rupture. Other analgesics may cause liver function impairment in the client. Other analgesics may cause renal function impairment in the client.

Stronger analgesics can mask symptoms of the tubal rupture. Rationale Methotrexate (MTX) is an antimetabolite and folic acid antagonist. The medication can destroy rapidly dividing cells. The therapy helps avoid surgery and is a safe and effective option. Strong analgesics can mask symptoms of tubal rupture. Therefore, the nurse instructs the client to avoid taking any analgesic stronger than acetaminophen (Tylenol). Methotrexate (MTX) does not have an interaction with other analgesics. Analgesics do not interact with methotrexate (MTX) to cause liver or renal dysfunction.

The nurse is assessing a pregnant client with multifetal gestation. Upon reviewing the medical history, the nurse finds that the client had preterm delivery during the first pregnancy. What will the nurse do to help prevent preterm delivery in the client during the second pregnancy? Suggest that the client avoid smoking. Suggest that the client increase physical activity to prevent risk. Administer progesterone (Prometrium) suppositories to the client. Administer a 17-alpha hydroxy progesterone injection to the client.

Suggest that the client avoid smoking. Rationale To prevent preterm labor the nurse can suggest health promotion activities to the client, such as avoiding smoking. This helps to promote intrauterine growth and fetal development. The nurse should suggest that the client get proper rest and care at home. The nurse should not suggest that the client increase physical activity, which could even worsen the condition. Progesterone supplements, like progesterone (Prometrium) suppositories and 17-alpha hydroxy progesterone injections, are ineffective in preventing preterm birth in clients with multifetal gestation.

A thrombosis results from the formation of a blood clot or clots inside a blood vessel and is caused by inflammation or partial obstruction of the vessel. What are three thromboembolic conditions of concern during the postpartum period? Select all that apply. Amniotic fluid embolism (AFE). Superficial venous thrombosis Deep vein thrombosis Pulmonary embolism Idiopathic thrombocytopenic purpura (ITP)

Superficial venous thrombosis Deep vein thrombosis Pulmonary embolism Rationale A superficial venous thrombosis includes involvement of the superficial saphenous venous system. With deep vein thrombosis the involvement varies but can extend from the foot to the iliofemoral region. A pulmonary embolism is a complication of deep vein thrombosis occurring when part of a blood clot dislodges and is carried to the pulmonary artery, where it occludes the vessel and obstructs blood flow to the lungs. An AFE occurs during the intrapartum period when amniotic fluid containing particles of debris enters the maternal circulation. Although AFE is rare, the mortality rate is as high as 80%. Idiopathic thrombocytopenic purpura (ITP) is not a thromboembolic conditio

Thromboembolic conditions that are of concern during the postpartum period include: Select all that apply. Amniotic fluid embolism (AFE) Superficial venous thrombosis Deep vein thrombosis Pulmonary embolism Disseminate intravascular coagulation (DIC)

Superficial venous thrombosis Deep vein thrombosis Pulmonary embolism Rationale An AFE can occur during the intrapartum period when amniotic fluid containing particles of debris enters the maternal circulation. Although AFE is rare, the mortality rate is as high as 80%. A superficial venous thrombosis includes involvement of the superficial saphenous venous system. With deep vein thrombosis the involvement varies but can extend from the foot to the iliofemoral region. A pulmonary embolism is a complication of deep vein thrombosis occurring when part of a blood clot dislodges and is carried to the pulmonary artery, where it occludes the vessel and obstructs blood flow to the lungs. DIC is an imbalance between the body's clotting and fibrinolytic systems. It's a pathologic form of clotting that consumes large amounts of clotting factors.

A pregnant woman who is 18 weeks of gestation has an elevated blood pressure of 140/98. Past medical history reveals that the woman has been treated for hypertension. Based on this information, the how would the nurse would classify this patient? Preeclamptic Gestational hypertension Superimposed preeclampsia Eclamptic

Superimposed preeclampsia Rationale Because this client already has a medical history of hypertension and is now exhibiting hypertension prior to the 20th week, she would be considered to have superimposed preeclampsia. Eclampsia is the onset of seizure activity or coma in a woman with preeclampsia. This woman is not displaying seizure activity, so this is incorrect. Gestational hypertension occurs after the 20th week of pregnancy in a client who was previously normotensive. Even though the client has chronic hypertension, the fact that she is now pregnant indicates that she would be classified as having superimposed preeclampsia.

The nurse is caring for a pregnant client with chronic hypertension. What additional complication is most likely to be seen in this client? Eclampsia Preeclampsia Gestational diabetes Superimposed preeclampsia

Superimposed preeclampsia Rationale The pregnant client suffering from chronic hypertension has a high risk of developing superimposed preeclampsia. Eclampsia is caused by the progression of preeclampsia and causes complications such as seizures, but it is not directly associated with chronic hypertension. Preeclampsia is the condition of high blood pressure, with or without proteinuria. Gestational diabetes is not directly caused by chronic hypertension.

A pregnant woman who is at 21 weeks of gestation has an elevated blood pressure of 140/98. Past medical history reveals that the woman has been treated for hypertension. On the basis of this information, the nurse would classify this patient as having: Preeclampsia. Gestational hypertension. Superimposed preeclampsia. Chronic hypertension.

Superimposed preeclampsia. Because this patient already has a medical history of hypertension and is now exhibiting hypertension prior after the 20th week of gestation, she would be considered to have superimposed pre-eclampsia. Pre-eclampsia would be the classification in a patient without a history of hypertension who was hypertensive following the 20th week of pregnancy. Gestational hypertension occurs after the 20th week of pregnancy in a patient who was previously normotensive. Even though the patient has chronic hypertension, the fact that she is now pregnant determines that she would be classified as having superimposed pre-eclampsia.

Which maternal risk is associated with placenta previa? Preeclampsia Placental abruption Surgery-related trauma Gestational hypertension

Surgery-related trauma Rationale A patient with placenta previa has a cesarean birth, and is at risk for surgery-related trauma to the sutures adjacent to the uterus. Preeclampsia is a condition in which the patient has hypertension and proteinuria after 20 weeks of gestation. The patient is at risk for a placental abruption if the patient experiences a trauma. Gestational hypertension is a hypertensive disorder in a pregnant patient and is not related to placenta previa.

Dilation and curettage (D&C)

Surgical procedure in which the cervix is widened and an instrument is inserted to scrape the uterine walls and remove uterine contents.

A neonate experiencing neonatal abstinence syndrome (NAS) is irritable and has a high-pitched cry. Which intervention should the nurse utilize when caring for the neonate? Keep the neonate awake throughout the day shift. Allow the neonates arms to be free from blankets. Talk to the neonate frequently when providing nursing care. Swaddle the neonate snuggly in blankets with the arms flexed.

Swaddle the neonate snuggly in blankets with the arms flexed. Snug swaddling decreases irritation, agitation, and startling for neonates experiencing NAS.

What are the s&s of mag sulfate toxicity?

Sweating, flushing, warmth, burning iv site should be reported immediately report >160/110, <12 RR, urinary output <25-30 ml/hr, headache, visual disturbances, decreased consciousness, epigastric pain, increasing edema, loss of DTR, abnormal maternal fetus status excreteed in urine, so if renal is not working, will cause toxicity toxicity - absent deep tendon reflexes, respiratory depression, blurred vision, slurred speech, severe muscle weakness, cardiac arrest

Interventions for Mild preeclampsia

Systolic 140-160 Diastolic 90-110 Provide bed rest Monitor BP and wt -Monitor neurological status (changes can indicate cerebral hypoxia or impending SEIZURE) -Monitor deep tendon reflexes and for presence of hyperreflexia or clonus (HTN increases CNS irritability) Administer med to reduce BP, Monitor for HELLP syndrome Monitor I&Os, urinary output 30/hr indicates adequate renal perfusion. -Increase dietary protein and carbs with no added salt.

Interventions for Severe Preeclampsia

Systolic >160 Diastolic >110 Maintain bed rest -Admin Magnesium sulfate to prevent seizures -Monitor S/S Magnesium toxicity- flushing, sweating, hypotension, depressed deep tendon reflexes, and CNS depression (resp depression) -Keep antidote Calcium gluconate available

The nurse is caring for a pregnant client prescribed levothyroxine for hypothyroidism. The client is also prescribed an iron supplement. What information does the nurse provide the client about taking these medications? Take both medications together in the morning. Take levothyroxine 1 hour after taking the iron supplement. Take the iron supplement 2 hours after taking levothyroxine. Take the two medications at different times of the day.

Take the two medications at different times of the day. Rationale The nurse should teach the client to take the medications at different times of the day. Ferrous sulfate in the iron supplement decreases the absorption of T 4, so their administration should be spaced at least 4 hours apart. If the medications are taken together, the levothyroxine will not be effective. The client should maintain a gap of at least 4 hours between both medications so that both medications are effective.

The nurse is caring for a neonate whose mother is on methadone replacement therapy. Which interventions are appropriate for the nurse to utilize when caring for the neonate? Select all that apply. Talking to the newborn in a quiet voice Nipple feeding the neonate over 30 minutes Placing stimulating color patterns in the neonates crib Applying Vaseline to the buttock with each diaper change Completing the physical assessment and vital signs at the same time

Talking to the newborn in a quiet voice A quiet voice and calm approach to the newborn decreases startling and agitation in the newborn. Applying Vaseline to the buttock with each diaper change Neonates with neonatal abstinence syndrome (NAS) often have loose stools, which cause excoriation. Application of a barrier cream will prevent skin breakdown. Completing the physical assessment and vital signs at the same time Clustering of care prevents overstimulation and promotes rest for the neonate with neonatal abstinence syndrome (NAS).

During a prenatal visit, the nurse finds that the client has decreased mobility and symptoms of preterm labor. Which nursing intervention is to be followed to prevent thrombophlebitis? Teach gentle lower extremity exercises to the client. Suggest the client to lie in the supine position in bed. Provide a calm and soothing atmosphere to the client. Give tocolytic medications as per the physician's prescription

Teach gentle lower extremity exercises to the client. Rationale The health care provider may recommend reduced activity for the client experiencing preterm labor, depending on the severity of the symptoms. As a result, the client may be at risk for thrombophlebitis due to limited activity. The nurse should teach the client how to perform gentle exercises of the lower extremities. Suggesting that the client lie in the supine position may cause supine hypotension. Instead, the nurse can suggest that the client lie in a side-lying position to help enhance placental perfusion. The nurse can provide a calm and soothing atmosphere to facilitate coping so as to reduce the client's anxiety, but this intervention does not prevent thrombophlebitis. Tocolytic medications are given to the client to inhibit uterine contractions (UCs), but they do not prevent thrombophlebitis.

Triple-marker

Test used as a screening for Down syndrome. It is performed between 16 and 18 weeks of gestation. The levels of three markers, namely MSAFP, unconjugated estriol, and hCG,in combination with maternal age are used to determine risk. The quad screen test adds an additional marker called inhibin A to increase the accuracy of screening for Down Syndrome in women more than 35 years of age.

Contraction stress test

Test used to identify the jeopardized fetus who is stable at rest but shows evidence of compromise when exposed to the stress of uterine contractions. If the resultant hypoxia of the fetus is sufficent, a deceleration of the FHR will result. Two methods used for this test are the nipple-stimulated contraction stress test and the oxytocin-stimulated contraction stress test.

Teaching for the mother with hepB about the vaccine for her baby?

That the hepatitis B vaccine will be administered to the neonate and that a second dose should be administered at 1 month after birth and a third dose at 6 months after birth discourage mother kissing neonate until neonate received vaccine

When would a woman infected with HIV first show symptoms?

The causative agent of AIDS. May first show symptoms at the time of pregnancy or possibly develop life-threatening infections because normal pregnancy involves some suppression of the maternal immune system

A woman at 26 weeks of gestation is being assessed to determine whether she is experiencing preterm labor. What finding indicates that preterm labor is occurring? Estriol is not found in maternal saliva. Irregular, mild uterine contractions are occurring every 12 to 15 minutes. Fetal fibronectin is present in vaginal secretions. The cervix is effacing and dilated to 2 cm.

The cervix is effacing and dilated to 2 cm. Rationale Cervical changes such as shortened endocervical length, effacement, and dilation are predictors of imminent preterm labor. Changes in the cervix accompanied by regular contractions indicate labor at any gestation. Estriol is a form of estrogen produced by the fetus that is present in plasma at 9 weeks of gestation. Levels of salivary estriol have been shown to increase before preterm birth. Irregular, mild contractions that do not cause cervical change are not considered a threat. The presence of fetal fibronectin in vaginal secretions between 24 and 36 weeks of gestation could predict preterm labor, but it has only a 20% to 40% positive predictive value. Of more importance are other physiologic clues of preterm labor, such as cervical changes.

A woman at 26 weeks of gestation is being assessed to determine whether she is experiencing preterm labor. What finding indicates that preterm labor is occurring? Estriol is not found in maternal saliva. Irregular, mild uterine contractions are occurring every 12 to 15 minutes. Fetal fibronectin is present in vaginal secretions. The cervix is effacing and dilated to 2 cm

The cervix is effacing and dilated to 2 cm. Cervical changes such as shortened endocervical length, effacement, and dilation are predictors of imminent preterm labor. Changes in the cervix accompanied by regular contractions indicate labor at any gestation. Irregular, mild contractions that do not cause cervical change are not considered a threat. Estriol is a form of estrogen produced by the fetus that is present in plasma at 9 weeks of gestation. Levels of salivary estriol have been shown to increase before preterm birth. The presence of fetal fibronectin in vaginal secretions between 24 and 36 weeks of gestation could predict preterm labor, but it has only a 20% to 40% positive predictive value. Of more importance are other physiologic clues of preterm labor, such as cervical changes.

A nurse is providing instruction for an obstetric client to perform a daily fetal movement count (DFMC). Which instructions could be included in the plan of care? Select all that apply. The fetal alarm signal is reached when there are no fetal movements noted for 5 hours. The client can monitor fetal activity once daily for a 60-minute period and note activity. Monitor fetal activity two times a day either after meals or before bed for a period of 2 hours or until 10 fetal movements are noted. Count all fetal movements in a 12-hour period daily until 10 fetal movements are noted. The test must be done in a clinic or hospital under the supervision of a nurse or physician.

The client can monitor fetal activity once daily for a 60-minute period and note activity. Monitor fetal activity two times a day either after meals or before bed for a period of 2 hours or until 10 fetal movements are noted. Count all fetal movements in a 12-hour period daily until 10 fetal movements are noted. Rationale Client monitoring of fetal activity once daily for a 60-minute period, and noting the activity is a protocol that can be used to perform DFMC. Monitoring fetal activity two times a day either after meals or before bed for a period of two hours or until 10 fetal movements are noted is a protocol that can be used to perform DFMC. Counting all fetal movements in a 12-hour period daily until 10 fetal movements are noted is a protocol that can be used to perform DFMC. The fetal alarm signal is noted when there are no fetal movements noted for a period of 12 hours. That the test must be done in a clinic or hospital under the supervision of a nurse or physician is incorrect. It is assessed at home and does not interfere with a woman's daily routine.

After reviewing the medical history and dietary habits of a pregnant client, the nurse suspects that the client's newborn may have risk of hyperactivity and learning disabilities. Which finding supports the nurse's opinion? The client consumed alcohol during the pregnancy. The client had three pregnancies in 2 years. The client drinks 6 ounces of coffee daily. The client is on valproic acid therapy (Depakote).

The client consumed alcohol during the pregnancy. Rationale Alcohol is a central nervous stimulant and impairs fetal neurologic development, resulting in hyperactivity and learning disabilities. Therefore, consumption of alcohol during pregnancy increases the risk of hyperactivity and learning disabilities in newborns. Drinking 12 ounces of coffee increases risk of miscarriage and intrauterine growth restriction. However, drinking 6 ounce of coffee daily will not impair the neurologic development of the fetus. If the client has three pregnancies in 2 years, it increases the risk of intrauterine growth restriction and miscarriage. Valproic acid (Depakote) is an anticonvulsant that causes neural tube defects, but not learning disabilities.

A pregnant client has painful lower abdominal cramps and a mucoid vaginal discharge. Upon further examination, the nurse concludes that the client may have a low risk of having a preterm delivery. What finding led the nurse to this conclusion? The client had a previous cesarean birth. The client has a cervical length of 40 mm. The client has preexisting diabetes mellitus. The client has symptoms of chronic hypertension.

The client has a cervical length of 40 mm. Rationale Painful, lower abdominal cramps and a mucoid vaginal discharge are symptoms of preterm labor. The cervical length is a good predictor of preterm birth. Women whose cervical length is greater than 30 mm are unlikely to experience premature birth, even if they have symptoms of preterm labor. The cervix needs to prepare itself for childbirth in terms of effacement and dilatation. A previous cesarean birth does not indicate that the woman will likely not have a preterm delivery. Preexisting diabetes and chronic hypertension are preterm birth risk factors.

A primary health care provider orders an ultrasound for a pregnant client before attempting external cephalic version (ECV). Upon assessing the client's ultrasound report, the nurse suspects that the primary health care provider will not attempt ECV. Which findings support the nurse's expectation? The client has a nuchal cord. The client is Rh negative. The client has oligohydramnios. The fetal heart rate is 120 beats per minute. The client has uterine anomalies

The client has a nuchal cord. The client has oligohydramnios. The client has uterine anomalies Rationale ECV is performed to change the fetus from a breech to a vertex presentation by applying pressure on the abdomen. ECV is contraindicated in certain conditions, including the presence of a nuchal cord, oligohydramnios, and uterine anomalies. ECV should be avoided if the ultrasound shows any of the complications mentioned. ECV is not contraindicated in Rh-negative client. Patients with an Rh-negative blood group are administered Rh immunoglobulin before performing ECV. A fetal heart rate of 120 beats per minute is considered normal, and ECV is not contraindicated in this condition.

A pregnant client has a sudden onset of seizures during the 3rd trimester of pregnancy. After reviewing the client's medical history, the nurse learns that the client had hypertension and proteinuria since 21 weeks of gestation. What will the nurse infer from these findings? The client has eclampsia. The client has preeclampsia. The client has chronic hypertension. The client has gestational hypertension.

The client has eclampsia. Rationale Eclampsia is a serious complication of pregnancy that is associated with the sudden onset of seizures. Eclampsia is usually preceded by premonitory signs and symptoms, including headache, blurred vision, abdominal pain, and altered mental status. However, convulsions can appear suddenly and without warning in a seemingly stable woman with only minimally elevated blood pressure. Preeclampsia is a condition in which the client has hypertension and proteinuria after 20 weeks of gestation, but preeclampsia is not associated with seizures. If the client has hypertension for more than 12 weeks after delivery, it indicates that client has chronic hypertension. Gestational hypertension is not associated with onset of seizures and proteinuria. Therefore, the nurse would not infer that the client has gestational hypertension.

The amniotic fluid index (AFI) in a client is 15. What should the nurse interpret form this finding? The client has normal AFI. The fetus has intrauterine growth restriction. The fetus has congenital abnormalities. The fetus has neural tube defects.

The client has normal AFI. Rationale AFI values between 10 and 25 cm are considered normal. Therefore, a 15 cm amniotic fluid index is normal and does not indicate any fetal abnormality. An AFI of less than 5 cm indicates that the patient has oligohydramnios, which is characterized by intrauterine growth restriction and congenital abnormalities. If the client has an AFI above 25 cm, it indicates that the client has polyhydramnios, which would put the fetus at risk of neural tube defects.

The amniotic fluid index (AFI) in a client is 15. What should the nurse interpret form this finding? The client has normal AFI. The fetus has intrauterine growth restriction. The fetus has congenital abnormalities. The fetus has neural tube defects.

The client has normal AFI. Rationale AFI values between 10 and 25 cm are considered normal. Therefore, a 15 cm amniotic fluid index is normal and does not indicate any fetal abnormality. An AFI of less than 5 cm indicates that the patient has oligohydramnios, which is characterized by intrauterine growth restriction and congenital abnormalities. If the client has an AFI above 25 cm, it indicates that the client has polyhydramnios, which would put the fetus at risk of neural tube defects

A client reports painless, bright-red vaginal bleeding during the second trimester of pregnancy. Upon assessment, the nurse finds that the client's urine output has decreased, the fundal height has increased, and the uterus is nontender with normal tone. What does the nurse interpret from these findings? The client has placenta previa. The client has ectopic pregnancy. The client has hydatidiform mole. The client has normal development.

The client has placenta previa. Rationale Placenta previa is an obstetric complication in which the placenta is implanted partially or completely in the lower uterine segment (near to or covering the cervix). Painless, bright-red vaginal bleeding takes place during the second trimester. Decreased urine output, greater-than-expected fundal height, and a nontender uterus with normal tone are signs of placenta previa. In ectopic pregnancy the fertilized ovum is implanted outside the uterine cavity. Hydatidiform mole is a benign proliferative growth of the placental trophoblast in which the chorionic villi develop into cystic vesicles that hang in a grapelike cluster. Decreased urine output is a complication seen in clients with diffused intravascular coagulopathy (DIC).

While assessing a postpartum woman, the nurse finds dark red blood coming from the vagina. What can the nurse infer about the client's condition by observing the blood's color? The client has partial placental separation. The client has a deep laceration of the cervix. The client has superficial lacerations of the birth canal. The client has disseminated intravascular coagulation (DIC).

The client has superficial lacerations of the birth canal. Rationale Superficial lacerations of the birth canal are characterized by dark red blood oozing from the vagina. The dark red color indicates its venous origin. Partial placental separation is characterized by spurts of blood with clots. Deep laceration of the cervix is characterized by bright redarterial blood. DIC is a condition in which the blood fails to clot or remain clotted. If the client had DIC, the color of the vaginal blood would be bright red.

After assessing a client with pregestational diabetes, the nurse suspects that the client belongs to class R of pregestational diabetes. What is done to confirm the condition? The client is referred for a urine sugar test. The client is referred for a renal function test. The client is referred for a dilated eye examination. The nurse asks the age at which the client acquired diabetes.

The client is referred for a dilated eye examination. Rationale The nurse suspects that the client belongs to class R of pregestational diabetes. This indicates that the client has symptoms of diabetic retinopathy. A dilated eye examination test would confirm that the client has retinitis proliferans. The client has pregestational diabetes; therefore, the nurse need not refer the client for a urine sugar test. A client belonging to class F of pregestational diabetes has nephropathy, and, so, should be referred for a renal function test. The nurse should find out the age at which the client acquired diabetes in order to understand whether the client belongs to category B or C of pregestational diabetes.

A client with ectopic pregnancy has received medical treatment for an ectopic pregnancy. The primary health care provider prescribed methotrexate (Amethopterin) therapy to the client. Which of the client's actions helps to prevent complications? The client takes an opioid analgesic for pain relief. The client gets her β-hCG level measured monthly. The client gets her insulin level measured monthly. The client uses a contraceptive method for 3 months.

The client uses a contraceptive method for 3 months. Rationale The client should use a contraceptive method for 3 months because it provides time for the body to heal. The client should not take strong analgesics, such as opioids, because these medications mask the symptoms of tubal rupture. The client who is on methotrexate (Amethopterin) therapy should get her β-hCG level measured weekly to make sure that β-hCG levels continue to drop steadily until they become undetectable. Insulin levels are not altered in the client who has methotrexate (Amethopterin) therapy. Therefore, the client need not get her insulin levels measured monthly.

The nurse is monitoring a pregnant client after amniotomy. Which observation would indicate a likelihood of umbilical cord compression? The fetal heart rate (FHR) confirms tachycardia. The client's vaginal drainage has a foul-smell. The client has maternal chills frequently. The fetal heart rate (FHR) has variable decelerations.

The fetal heart rate (FHR) has variable decelerations. Rationale Amniotomy is performed in a pregnant client in order to rupture the membranes artificially. After the procedure, the nurse should closely monitor the FHR. Reduced FHR and variable decelerations in FHR indicate that the client's umbilical cord is compressed. The nurse should immediately inform the primary health care provider of the client's condition. Tachycardia or increased FHR are common manifestations observed after amniotomy. Tachycardia does not require immediate clinical action. Maternal chills and foul-smelling vaginal discharge after amniotomy indicate infection of the ruptured membranes. However, this would not be a reason to expect umbilical cord compression.

The nurse is reviewing the nuchal translucency (NT) reports of a pregnant client. The report reveals that the fluid in the nape of the fetus is more than 3 mm. What is the nurse's best interpretation from this finding? The fetus has an organ disorder. The fetus has growth retardation. The fetus has a genetic disorder. The fetus has cardiac disease.

The fetus has cardiac disease. Rationale NT is a type of ultrasound screening technique done to assess genetic disorders in the developing fetus. If the fluid in the nape of the fetus is more than 3 mm, it indicates that the child has a cardiac disorder. Organ disorders, growth retardation, and genetic disorders cannot be assessed using the NT screening technique only. Fetal organ disorders are identified by transvaginal ultrasound. Growth disorders can be diagnosed by normal ultrasonography. Elevated NT along with maternal serum markers indicates the presence of genetic disease.

Diabetes in pregnancy puts the fetus at risk in several ways. What should nurses be aware of? With good control of maternal glucose levels, sudden and unexplained stillbirth is no longer a major concern. The most important cause of perinatal loss in diabetic pregnancy is congenital malformations. Infants of mothers with diabetes have the same risks for respiratory distress syndrome because of the careful monitoring. At birth, the neonate of a diabetic mother is no longer at any greater risk.

The most important cause of perinatal loss in diabetic pregnancy is congenital malformations. Rationale Congenital malformations account for 30% to 50% of perinatal deaths. Even with good control, sudden and unexplained stillbirth remains a major concern. Infants of diabetic mothers are at increased risk for respiratory distress syndrome. The transition to extrauterine life often is marked by hypoglycemia and other metabolic abnormalities.

The nurse is caring for a patient who is 24 hours postpartum and whose delivery was aided by vacuum extraction. Which assessment finding should be reported to the health care provider? The patient has dark brown blotches on one side of her face. The patient has saturated three perineal pads in the past 4 hours. The patient's fundus is midline and below the umbilicus. The nurse has noticed a trickle of bright red blood for the past 2 hours.

The nurse has noticed a trickle of bright red blood for the past 2 hours. A slow, steady trickle of bright red blood can indicate a perineal laceration. This patient's history of operative delivery increases the risk of laceration; therefore, this finding should be reported to the health care provider.

A nurse is providing instruction for an obstetrical patient to perform a daily fetal movement count (DFMC). Which instructions could be included in the plan of care? Select all that apply. The fetal alarm signal is reached when there are no fetal movements noted for 5 hours. The patient can monitor fetal activity once daily for a 60-minute period and note activity. Monitor fetal activity two times a day either after meals or before bed for a period of 2 hours or until 10 fetal movements are noted. Count all fetal movements in a 12-hour period daily until 10 fetal movements are noted.

The patient can monitor fetal activity once daily for a 60-minute period and note activity. The fetal alarm signal is reached when no fetal movements are noted for a period of 12 hours. Monitor fetal activity two times a day either after meals or before bed for a period of 2 hours or until 10 fetal movements are noted. The fetal alarm signal is reached when no fetal movements are noted for a period of 12 hours. Count all fetal movements in a 12-hour period daily until 10 fetal movements are noted. The fetal alarm signal is reached when no fetal movements are noted for a period of 12 hours.

A pregnant woman has maternal phenylketonuria (PKU) and is interested in whether or not she will be able to breastfeed her baby. Which reaction by the nurse indicates accurate information? The patient can breastfeed the baby as long as she continues to maintain a PKU-restricted diet. The patient should alternate breastfeeding with bottle feeding in order to reduce PKU levels provided to the baby. The patient should be advised to not breastfeed the infant because her breast milk will contain large amounts of phenylalanine. The patient can breastfeed for the first 3 months without any untoward effects on the infant.

The patient should be advised to not breastfeed the infant because her breast milk will contain large amounts of phenylalanine. Breastfeeding is not advised for a patient who has maternal PKU, because phenylalanine levels are high in such a patient's breast milk. Dietary restriction will not limit the amount of this substance in breast milk. Alternating feeding sources is not advised either.

A patient experienced a first-degree laceration during vaginal delivery. Which postpartum assessment finding should the nurse attribute to the patient's laceration? The patient reports an overwhelming urge to defecate. The patient rates her overall pain as a 10 on a 10-point scale. The patient's pad contains bright red blood, as well as darker lochia. The patient reports lightheadedness when assisted to a high Fowler's position.

The patient's pad contains bright red blood, as well as darker lochia. In contrast to darker lochia, the blood from a laceration is more likely to be bright red, since it originates directly adjacent to the pad.

Augmentation of Labor

The stimulation of uterine contractions after labor has started spontaneously and progress is unsatisfactory -implemented to manage hypotonic uterine dysfunction Examples of methods of augmentation: -administration of oxytocin -amniotomy -emptying the bladder -ambulation -relaxation methods -nourishment and hydration

What is used to reduce HIV exposure to a newborn?

The use of antiviral medication, reduced exposure of the neonate to maternal blood and body fluids, and early identification of HIV in pregnancy reduce the risk of transmission to the neonate

The nurse is preparing discharge education for a new mother who has demonstrated good attachment and confidence in being able to care for the newborn. Which information should the nurse include regarding postpartum mood disorders? The woman is at increased risk for postpartum posttraumatic stress disorder. Most episodes of postpartum depression begin within the first 24 hours of birth. Postpartum mood disorders are unlikely to occur in women with good attachment and confidence. The woman should not delay in seeking help if she experiences symptoms of a postpartum mood disorder.

The woman should not delay in seeking help if she experiences symptoms of a postpartum mood disorder. Instructing the woman to treat symptoms of postpartum mood disorders seriously by seeking help facilitates early detection and treatment and should be included in discharge education

McRoberts Maneuver

The woman's legs are flexed apart with her knees on her abdomen -straightens the sacrum -then suprapubic pressure can be applied to anterior shoulder

A woman presents to the emergency department complaining of bleeding and cramping. The initial nursing history is significant for a last menstrual period 6 weeks ago. On sterile speculum examination, the primary health care provider finds that the cervix is closed. The anticipated plan of care for this woman is based on a probable diagnosis of which type of spontaneous abortion? Incomplete Inevitable Threatened Septic

Threatened Rationale A woman with a threatened abortion presents with spotting, mild cramps, and no cervical dilation. A woman with an incomplete abortion presents with heavy bleeding, mild to severe cramping, and cervical dilation. An inevitable abortion presents with the same symptomatology as an incomplete abortion: heavy bleeding, mild to severe cramping, and cervical dilation. A woman with a septic abortion presents with malodorous bleeding and typically a dilated cervix.

A woman presents to the emergency department complaining of bleeding and cramping. The initial nursing history is significant for a last menstrual period 6 weeks ago. On sterile speculum examination, the primary health care provider finds that the cervix is closed. The anticipated plan of care for this woman is based on a probable diagnosis of which type of spontaneous abortion? Incomplete Inevitable Threatened Septic

Threatened A woman with a threatened abortion presents with spotting, mild cramps, and no cervical dilation. Heavy bleeding, mild to severe cramping, and cervical dilation are the presentation for both incomplete abortion and inevitable abortion. A woman with a septic abortion presents with malodorous bleeding and, typically, a dilated cervix.

During a prenatal check-up a client who is 7 months pregnant reports that she is able to feel about two kicks in an hour. The nurse refers the client for an ultrasound. What is the primary reason for this referral? To check for fetal anomalies To check gestational age To check fetal position To check for fetal well-being

To check for fetal well-being Rationale Fetal kick count is a simple method to determine the presence of complications related to fetal oxygenation and activity level. The fetal kick count during the third trimester of pregnancy is approximately 30 kicks an hour; a count lower than that is an indication of poor health of the fetus. Fetal anomalies may not affect the oxygenation levels of the fetus. The nurse already knows the gestational age of the fetus; therefore, the nurse need not refer the woman for ultrasonography to find the gestational age. Fetal position does not affect the activity level of the fetus.

The primary health care provider orders a specialized ultrasonography test for a client. What is the objective of performing the test? To determine fetal cardiac activity To assess the amniotic fluid volume To determine the presence of polyhydramnios To assess the fetal position during delivery

To determine the presence of polyhydramnios Rationale Specialized ultrasonography is used to detect physiologic and anatomic abnormalities in the fetus. Polyhydramnios, or an increased amount of amniotic fluid in the fetal sac, is a physiologic abnormality that can be assessed by specialized ultrasonography. Fetal cardiac activity is checked in a standard ultrasonographic test. Amniotic fluid volume is measured in a standard ultrasonographic test. Fetal position during delivery is determined using limited ultrasonography.

The nurse is administering glucocorticoids to a pregnant woman in preterm labor. When explaining the purpose of this medication to the client, which response by the nurse is accurate? To prevent fetal cerebral palsy To prevent early birth of the fetus To prevent gestational hypertension To prevent fetal respiratory distress syndrome

To prevent fetal respiratory distress syndrome Rationale Preterm birth causes respiratory distress in the newborn due to underdeveloped lung activity. Antenatal glucocorticoids are administered to a pregnant client who is at the risk of preterm labor to prevent fetal respiratory distress syndrome. Tocolytic agents such as magnesium sulfate (Epsom salts), are found to reduce the incidence of cerebral palsy in the child, and are unrelated to glucocorticoids. Gestational hypertension is observed in clients who have a familial history of hypertension and may not be prevented by administering glucocorticoids. Glucocorticoids have no impact on delaying preterm birth.

Goal of oxytocin administration

To produce acceptable uterine contractions in a consistent pattern of 1 contraction every 2-3 minutes lasting 80-90 seconds, strong to palpation

The nurse finds that despite gentle traction to the umbilical cord and uterine massage, the client's placenta has not expelled out even after 30 minutes of childbirth. The primary health care provider instructs the nurse to administer nitroglycerin IV (Nitrostat) to the client. What could be the reason for this instruction? Prevention pelvic hematoma To increase the effects of regional anesthesia To promote uterine relaxation Prevention of postpartum hemorrhage

To promote uterine relaxation Rationale The placenta is usually expelled out within 30 minutes after birth. Retained placenta is a condition in which the placenta is not expelled within 30 minutes after birth despite using manual measures, such as gentle traction on the umbilical cord and uterine massage. In this condition nitroglycerin IV (Nitrostat) is administered to the patient to promote uterine relaxation. Nitroglycerin IV (Nitrostat) does not affect blood coagulation; therefore, it does not prevent pelvic hematoma. Nitroglycerin IV (Nitrostat) is not an anesthetic agent; therefore, it does not provide regional anesthesia. Nitroglycerin IV (Nitrostat) does not cause uterine contractions (UCs), so it does not prevent PPH.

Toxoplasmosis 1. How is it transmitted 2. What can it cause

To the mother through raw meat or handling of cat litter of infected cats, and then transmitted to the baby across the placenta 2. Spontaneous abortion in the 1st trimester produces rash and symptoms of acute, flulike infection in the mother

terbutaline (Brethine)

Tocolytic medication administered subcutaneously to suppress uterine tachysystole."

A nurse is observing a mother care for her 2-week-old premature infant prior to discharge. The mother is concerned about the infant becoming dehydrated once home and asks which steps she should take in checking the infant and contacting the health care provider. Which patient teaching does the nurse provide? Select all that apply. Check for dry skin. Give the infant 2 oz. of water. Track the number of wet diapers daily. Weigh the infant regularly to check for gain/loss. Monitor the infant for two or three days before calling the health care provider.

Track the number of wet diapers daily. The nurse would provide this teaching because tracking the number of wet diapers provides data regarding fluid balance. After the first week of life, it is expected that the infant will have 5-6 wet diapers/day. Correct Weigh the infant regularly to check for gain/loss. The nurse would provide this teaching because weighing the infant provides data regarding fluid balance. A weight loss greater than expected can signal dehydration Check for dry skin. The nurse would provide this teaching because dry skin can be a sign of dehydration.

A 40-year-old woman with a high body mass index (BMI) is 10 weeks pregnant. Which diagnostic tool is appropriate to suggest to her at this time? Biophysical profile Amniocentesis Maternal serum alpha-fetoprotein (MSAFP) Transvaginal ultrasound

Transvaginal ultrasound Rationale An ultrasound is the method of biophysical assessment of the infant that is performed at this gestational age. Transvaginal ultrasound is especially useful for obese women, whose thick abdominal layers cannot be penetrated adequately with the abdominal approach. A biophysical profile is a method of biophysical assessment of fetal well-being in the third trimester. An amniocentesis is performed after the fourteenth week of pregnancy. A MSAFP test is performed from week 15 to week 22 of gestation (weeks 16 to 18 are ideal).

A 40-year-old woman with a high body mass index (BMI) is 10 weeks pregnant. Which diagnostic tool is appropriate to suggest to her at this time? Biophysical profile Amniocentesis Maternal serum alpha-fetoprotein (MSAFP) Transvaginal ultrasound

Transvaginal ultrasound An ultrasound is the method of biophysical assessment of the infant that is performed at this gestational age. Transvaginal ultrasound is especially useful for obese women, whose thick abdominal layers cannot be penetrated adequately with the abdominal approach. A biophysical profile is a method of biophysical assessment of fetal well-being in the third trimester. An amniocentesis is performed after the fourteenth week of pregnancy. A MSAFP test is performed from week 15 to week 22 of the gestation (weeks 16 to 18 are ideal).

Surgical Incisions

Transverse= fannenstiel in the lower abdomen - performed most often -Low transverse -Vertical incision -Classic incision (vertical and up higher) Why is the low transverse uterine incision preferred? Because it provides an option of TOLAC AND VBAC in subsequent pregnancies

Which tests are noninvasive and performed to detect chromosomal abnormalities in the fetus? Select all that apply. Amniocentesis Triple-marker test Cell-free deoxyribonucleic acid (DNA) screen Chorionic villus sampling (CVS) Nonstress test

Triple-marker test Cell-free deoxyribonucleic acid (DNA) screen Rationale The triple-marker screen is a noninvasive technique that measures the levels of three maternal serum markers: maternal serum alpha-fetoprotein (MSAFP), unconjugated estriol, and human chorionic gonadotropin (hCG). It helps in determining the increased risk of chromosomal abnormalities, Cell-free DNA screening is a noninvasive technique where maternal blood is collected for prenatal genetic diagnosis. Amniocentesis is an invasive prenatal procedure that helps identify chromosomal abnormalities in the fetus. In this test, a needle is inserted in the patient's abdominal wall under the guidance of ultrasound. Chorionic villus sampling (CVS) is an invasive technique to detect chromosomal abnormalities. While conducting CVS, a catheter is inserted into the cervix under continuous ultrasonographic guidance. A nonstress test is used evaluate the antepartum condition of the fetus. It does not help in detecting chromosomal abnormalities.

Which type of ultrasound is the standard medical scan used in pregnancy? Two-dimensional (2D) Three-dimensional (3D) Four-dimensional (4D) Five-dimensional (5D)

Two-dimensional (2D) Rationale A two-dimensional (2D) ultrasound is the standard medical scan used in pregnancy. A three-dimensional (3D) ultrasound is the ultrasound test in which sound waves are sent out at different angles. A four-dimensional (4D) ultrasound adds a fourth dimension (time) to the 3D scan. There is no five-dimensional (5D) ultrasound.

Fetal nuchal translucency (NT)"

Ultrasound test used along with maternal serum screening between 11 and 14 weeks of gestation to detect fetal heart defects and other anomalies in women with pregestational diabetes."

The labor and delivery nurse is monitoring a laboring patient and notes the presence of variable decelerations on the fetal monitor tracing. Which finding should the nurse expect based on this observation? Pressure on the fetal head Normal fetal heart activity Umbilical cord compression Uteroplacental insufficiency

Umbilical cord compression Variable decelerations present on a fetal heart monitor suggest umbilical cord compression.

Shoulder dystocia;

Uncommon obstetric emergency in which the head of the fetus is born but the anterior shoulder cannot pass under the pubic arch. Two major causes are fetopelvic disproportion related to excessive fetal size (macrosomia); maternal pelvic abnormalities

Assessment for DIC?

Uncontrolled bleeding, bruising, purpura, petechiae, and ecchymosis, presence of occult blood in excretions such as a stool, hematuria, hematemesis, vaginal bleeding, signs of shock, decresed fibrinogen level platelet count and hematocrit level, increased prothrombin time and partial thromboplastin time clotting time and fibrin degradation products

The labor and delivery nurse is admitting a woman complaining of being in labor. The nurse completes the admission database and notes that which factors may prohibit the woman from having a vaginal birth? Select all that apply. Unstable coronary artery disease Previous cesarean birth Placenta previa Initial blood pressure of 132/87 History of three spontaneous abortions

Unstable coronary artery disease Previous cesarean birth Placenta previa Rationale Indications for cesarean birth include: Maternal: (1) Specific cardiac disease (e.g., Marfan syndrome, unstable coronary artery disease). (2) Specific respiratory disease (e.g., Guillain-Barré syndrome). (3) Conditions associated with increased intracranial pressure. (4) Mechanical obstruction of the lower uterine segment (tumors, fibroids). (5) Mechanical vulvar obstruction (e.g., extensive condylomata). (6) History of previous cesarean birth; Fetal: (1) Abnormal fetal heart rate (FHR) or pattern. (2) Malpresentation (e.g., breech or transverse lie). (3) Active maternal herpes lesions. (4) Maternal human immunodeficiency virus (HIV) with a viral load of more than 1000 copies/ml (5). Congenital anomalies; Maternal-Fetal: (1) Dysfunctional labor (e.g., cephalopelvic disproportion, "failure to progress" in labor). (2) Placental abruption. (3) Placenta previa. (4) Elective cesarean birth (cesarean on maternal request). The blood pressure can be elevated because of pain and is not necessarily a contraindication to vaginal birth until further assessment is completed. Having a history of three spontaneous abortions is not a contraindication to vaginal birth.

The labor and delivery nurse is admitting a woman complaining of being in labor. The nurse completes the admission database and notes that which factors may prohibit the woman from having a vaginal birth? Select all that apply. Unstable coronary artery disease Previous cesarean birth Placenta previa Initial blood pressure of 132/87 History of three spontaneous abortions

Unstable coronary artery disease Previous cesarean birth Placenta previa Rationale Indications for cesarean birth include: Maternal: (1) Specific cardiac disease (e.g., Marfan syndrome, unstable coronary artery disease). (2) Specific respiratory disease (e.g., Guillain-Barré syndrome). (3) Conditions associated with increased intracranial pressure. (4) Mechanical obstruction of the lower uterine segment (tumors, fibroids). (5) Mechanical vulvar obstruction (e.g., extensive condylomata). (6) History of previous cesarean birth; Fetal: (1) Abnormal fetal heart rate (FHR) or pattern. (2) Malpresentation (e.g., breech or transverse lie). (3) Active maternal herpes lesions. (4) Maternal human immunodeficiency virus (HIV) with a viral load of more than 1000 copies/ml (5). Congenital anomalies; Maternal-Fetal: (1) Dysfunctional labor (e.g., cephalopelvic disproportion, "failure to progress" in labor). (2) Placental abruption. (3) Placenta previa. (4) Elective cesarean birth (cesarean on maternal request). The blood pressure can be elevated because of pain and is not necessarily a contraindication to vaginal birth until further assessment is completed. Having a history of three spontaneous abortions is not a contraindication to vaginal birth.

Nsg interventions for pregnant client with diabetes?

Use diet, insulin if prescribed, exercise and blood glucose checks to maintain blood glucose levels between 65 mg/dL and 130 mg/dL, obsereve for S/S of hyperglycemia, glycosuria, ketonuria, and hypoglycemia, monitor weight, increase calorie intake as prescribed with adequate insulin therapy, assess for maternal signs of preeclampsia, monitor for signs of infection, assess fetal status and monitor for signs of fetal compromise, instruct client to report burning and pain on urination, vaginal discharge or itching or any other signs of infections to hcp

A client diagnosed with endometritis is prescribed clindamycin (Cleocin). What comfort measures does the nurse teach the client? Select all that apply. Avoid sitz baths. Use warm blankets. Decrease fluid intake. Change perineal pads. Avoid cool compresses.

Use warm blankets. Change perineal pads. Rationale A client diagnosed with endometritis is instructed to use warm blankets to cover the body for pain relief. The client should also be instructed to change the perineal pads, so as to prevent the spread of infection. The client is advised to change the perineal pads from front to back to maintain hygiene at the infected site. Sitz baths are encouraged to maintain hygiene and provide comfort. Fluid intake is suggested to maintain the fluidity of the blood. Applying cool compresses to the site of infection is advised to relieve discomfort caused by the infection.

When should an ectopic pregnancy be resolved?

Usually within 2-3 weeks after taking methotrexate (but can take up to 6-8 weeks)

Signs and symptoms of preterm labor

Uterine activity -contractions occurring more frequently than every 10 minutes persisting for 1+ hours -contractions may be painful or painless Discomfort -lower abdominal cramping similar to gas pains, may be accompanied by diarrhea -dull, intermittent low back pain -painful, menstrual-like cramps -suprapubic pain or pressure -pelvic pressure or heaviness -urinary frequency Vaginal Discharge -change in character or amount of usual discharge -thicker (mucoid) or thinner (watery), bloody, brown, or colorless, increased amount, odor -rupture of amniotic membranes

The ultrasonography report of a pregnant woman reveals the presence of polyhydramnios. What complication would the nurse expect during the postpartum period? Hematomas Uterine atony Retained placenta Endometrial infections

Uterine atony Rationale Polyhydramnios causes the uterine muscles to stretch excessively, which may result in poor uterine contraction after delivery. This results in uterine atony in the client. Polyhydramnios does not cause hematomas. Prolonged compression of the fetal head on the vaginal walls causes vaginal hematoma. Polyhydramnios does not cause retained placenta. Polyhydramnios does not affect placental separation. Presence of polyhydramnios would not cause endometrial infections.

Match the type of spontaneous abortion with the patient description.

Vaginal bleeding at 8 weeks' gestation and no other abnormal physical signs. Threatened abortion Ruptured membranes and vaginal bleeding at 17 weeks' gestation. Inevitable abortion Heavy bleeding, rupture of membranes, and a nonviable fetus. Incomplete abortion Unremarkable early pregnancy. Ultrasound at 20 weeks' reveals no heartbeat. Missed

The antepartum nurse is caring for a new patient who seeks to attempt a vaginal delivery after a cesarean delivery. She indicates her Pfannenstiel incision scar and insists that she should be able to successfully have a vaginal birth. Which nursing education is appropriate for this patient? Pfannenstiel incisions are very likely to rupture during subsequent birth. Vaginal birth may be attempted with a Pfannenstiel incision; however, labor will most likely need to be induced. Vaginal delivery may be possible, but her previous type of uterine incision would need to be verified before attempting vaginal delivery. Because her last cesarean was performed with a Pfannenstiel incision, she should be able to successfully have a vaginal birth.

Vaginal delivery may be possible, but her previous type of uterine incision would need to be verified before attempting vaginal delivery. Skin incisions and uterine incisions do not always match, and the patient's uterine incision may contraindicate subsequent vaginal delivery. Therefore, this nursing education is appropriate for the patient.

Dark red blood

Venous -could be from varices or superficial lacerations of birth cana

A nonstress test (NST) is ordered on a pregnant woman at 37 weeks of gestation. What are the most appropriate teaching points to include when explaining the procedure to the woman? Select all that apply. After 20 minutes, a nonreactive reading indicates the test is complete. Vibroacoustic stimulation may be used during the test. Drinking orange juice before the test is appropriate. A needle biopsy may be needed to stimulate contractions. Two sensors are placed on the abdomen to measure contractions and fetal heart tones.

Vibroacoustic stimulation may be used during the test. Drinking orange juice before the test is appropriate. Two sensors are placed on the abdomen to measure contractions and fetal heart tones. Rationale Vibroacoustic stimulation is often used to stimulate fetal activity if the initial NST result is nonreactive and thus hopefully shortens the time required to complete the test. A nonreactive test requires further evaluation. The testing period is often extended, usually for an additional 20 minutes, with the expectation that the fetal sleep state will change and the test will become reactive. Care providers sometimes suggest that the woman drink orange juice or be given glucose to increase her blood sugar level and thereby stimulate fetal movements. Although this practice is common, there is no evidence that it increases fetal activity. A needle biopsy is not part of an NST. The FHR is recorded with a Doppler transducer, and a tocodynamometer is applied to detect uterine contractions or fetal movements. The tracing is observed for signs of fetal activity and a concurrent acceleration of FHR.

A nurse is educating a woman with gestational diabetes on home management of glucose levels. Which behavioral changes would the nurse suggest to the woman? Select all that apply. Eliminate all refined sugars from the diet Begin a weekly weight training session Walk 30 minutes every evening after dinner Balance the amount of dietary carbohydrates and protein Check your blood glucose as prescribed by your health care provider

Walk 30 minutes every evening after dinner A short walk is good exercise, which is important to maintain healthy blood glucose. Balance the amount of dietary carbohydrates and protein A balanced diet is essential to maintaining healthy blood glucose levels. Check your blood glucose as prescribed by your health care provider The woman will need to assess her blood glucose level as ordered by her health care provider.

The nurse cares for a mechanically ventilated 27 weeks' gestation neonate. The neonate desaturates and becomes bradycardic when suctioned and handled. Which interventions are most important for the nurse to implement when caring for this neonate? Select all that apply. Schedule suctioning for once in an hour. Warm and humidify the oxygen being administered. Nest the neonate in the supine position in the isolette. Increase the FiO2 from 30% to 50% before suctioning the neonate. Apply suctioning for 5 seconds at a time when withdrawing the suction catheter.

Warm and humidify the oxygen being administered. Warmed and humidified oxygen thins secretions making removal easier. Increase the FiO2 from 30% to 50% before suctioning the neonate. Increasing the percentage of inspired oxygen prior to suctioning may prevent bradycardia and desaturation from occurring during the suctioning. Correct Apply suctioning for 5 seconds at a time when withdrawing the suction catheter. Limiting the application of suction to 5 seconds decreases the chance of desaturation and bradycardia which can occur from over stimulation.

A neonate born at 32 weeks' gestation has a sunken anterior fontanelle and dry skin. To further assess the situation, the nurse should complete which assessments? Select all that apply. Assess the neonate's hematocrit level. Weigh all urine diapers obtained during the shift. Compare the morning weight to yesterday's weight. Auscultate the neonate's lung sounds in all fields bilaterally. Test the specific gravity of the neonate's urine in the current diaper.

Weigh all urine diapers obtained during the shift. Assessing the hourly output of the neonate would provide data that, along with the sunken anterior fontanelle and dry skin, indicate the presence of dehydration. Correct Compare the morning weight to yesterday's weight. Assessing the neonate's current weight provides data regarding fluid loss. Weight loss along with the sunken anterior fontanelle and dry skin support the presence of dehydration. Test the specific gravity of the neonate's urine in the current diaper. Assessing the specific gravity of the neonate's urine would provide data that, along with the sunken anterior fontanelle and dry skin, indicate the presence of dehydration.

The insulin dose of a client in the second trimester of pregnancy has been increased. When does the nurse expect the client's prepregnant dose of insulin to be recommended again? When the client starts lactating When the client delivers the baby When the client is in the third trimester When the client weans the baby from breastfeeding

When the client weans the baby from breastfeeding Rationale Pregnancy hormones cause increased glucose tolerance and decreased sensitivity to insulin. This results in an increased need of insulin to obtain glycemic control. In this case, the client's glucose metabolism would reach its prepregnant state only when the client stops breastfeeding the baby. During lactation, the maternal glucose is utilized and, therefore, the client would require a low dose of insulin. When the client delivers the baby, the insulin-glucose levels do not return to their nonpregnant state immediately. Therefore, the client still has to continue with the recommended high insulin dose for some time. During the third trimester, the diabetogenic effect of hormones is excessive, and maternal insulin requirements may double.

With regard to dysfunctional labor, nurses should be aware that: Women who are underweight are more at risk. Women experiencing precipitous labor are about the only women experiencing dysfunctional labor who are not exhausted. Hypertonic uterine dysfunction is more common than hypotonic dysfunction. Abnormal labor patterns are most common in older women.

Women experiencing precipitous labor are about the only women experiencing dysfunctional labor who are not exhausted. Precipitous labor lasts less than 3 hours. Short women more than 30 pounds overweight are more at risk for dysfunctional labor. Hypotonic uterine dysfunction, in which the contractions become weaker, is more common. Abnormal labor patterns are more common in women younger than 20 years.

The nurse initiates intravenous (IV) therapy on a neonate born at 32 weeks' gestation. Which interventions are most appropriate for the nurse to implement when placing the IV line? Select all that apply. Preparing the insertion site using betadine Wrapping the site with Kerlix after taping Applying tincture of benzoin around the site before applying tape Observing the site for inflammation and coolness when first flushing with fluid Placing a cotton ball on the tape when securing the extremity on to an armboard

Wrapping the site with Kerlix after taping Wrapping the IV site in gauze after placement decreases skin damage in the premature neonate. Observing the site for inflammation and coolness when first flushing with fluid Assessing for signs of infiltration when placing the IV line prevents the extensive tissue damage that can occur when a line infiltrates when being placed. Correct Placing a cotton ball on the tape when securing the extremity on to an armboard When placing an intravenous line, you should place a cotton ball on the skin before applying tape. This decreases damage to the skin from the adhesive on the tape

fetal genetic disorders and physical anomalies

a prenatal screening technique uses ultrasound measurement of fluid in the nap of the fetal neck between 10-14 weeks of gestation to identify possible fetal abnormalities A fluid collection greater than 3 mm is considered abnormal

nonreactive test

a test that does not demonstrate at least two qualifying accelerations within 20-minute window needs further evaluation if doenst meet criteria after 40 min - do BPP

After a woman with blood type Rh negative undergoes amniocentesis, the most appropriate nursing intervention is to: administer RhoD immunoglobulin. administer anticoagulant. send the patient for a computed tomography (CT) scan before the procedure. assure the mother that short-term radiation exposure is not harmful to the fetus.

administer RhoD immunoglobulin. Rationale Because of the possibility of fetomaternal hemorrhage, administering Rh oD immunoglobulin to the woman who is Rh negative is standard practice after an amniocentesis. Anticoagulants are not administered because this can increase the risk of bleeding when the needle is inserted transabdominally. A CT is not required because the procedure is ultrasound guided. The mother is not exposed to radiation during amniocentesis.

Herpes Simplex

affects External genitalia, vagina, and cervix with painful, draining lesions 2. Acyclovir 3. transmitted During birth through the infected vagina or via an ascending infection after rupture of the membranes 4. In the presence of ACTIVE vaginal herpetic lesions - no exam 5. Death or severe neuro impairment in the baby 6. C-Section 7. Vaginally with STRICT precautions taken during delivery 8. CONTACT precaution

biochemical assessment

amniocentecis (after 14 weeks)- performed to obtain amniotic fluid, which contains fetal cells; under direct ultrasound visualization, a needle is inserted transabdominally into the uterus, amniotic fluid is withdrawn into a syringe percutaneous umbelical blood sampling chorionic villus sampling maternal sampling biochemical analysis of enzymes in amniotic fluid can detect 2 things what are they? inborn errors of metabolism or fetal structural anomalies

Idiopathic/Immune Thrombocytopenic Purpura (ITP)

autoimmune disorder in which anti platelet antibodies decrease life span of platelets. What are diagnostic findings of ITP? thrombocytopenia, capillary fragility, and increased bleeding time. an autoimmune disorder in which anitplatelet antibodies decrease the lifespan of the platelets -leads to capillary fragility, increased bleeding time, and thrombocytopenia -can cause severe hemorrhage after c-section or cervical or vaginal lacerations Medical Management: -corticosteroids -platelet transfusions

idiopathic (immune) thrombocytopenia purport (ITP)

autoimmune disorder in which anti platelet antibodies decrease the life space of the plateletes

(Methergine) 0.2 mg is ordered to be administered intramuscularly to a woman who gave birth vaginally 1 hour ago for a profuse lochial flow with clots. Her fundus is boggy and does not respond well to massage. She is still being treated for preeclampsia with intravenous magnesium sulfate at 1g/hr. Her blood pressure, measured 5 minutes ago, was 155/98. In fulfilling this order, the nurse would do which of the following? a. Measure the woman's blood pressure again 5 minutes after administering the medication b. Question the order based on the woman's hypertensive status c. adminster Methergine because it is the best choice to counteract the possible uterine relaxation effects of the magnesium sulfate infusion the woman is receiving a. Tell the woman that the medication will lead to uterine cramping

b. Question the order based on the woman's hypertensive status

If postpartum bleeding persist after first and second line interventions what must be initiated?

bimanual compression, and if uterine still does not become firm the HCP performs manual exploration of uterine cavity for retained placental fragments.

Pulmonary embolism (PE)

complication of DVT occurring when part of a blood clot dislodges and is carried to the pulmonary artery, where it occludes the vessel and obstructs blood flow to the lungs. more common in postpartum period. Which thomboembolic disorder is a major cause of maternal death? pulmonary embolism (PE). Signs and symptoms of acute pulmonary embolism dyspnea, tachypnea, tachycardia, apprehension, chest pain, cough, hemoptysis, elevated temperature and syncope; usually results from dislodged deep vein thrombi. Symptoms of PE -Dyspnea and tachypnea (>20 breaths/min) -tachycardia (>100 bpm) -chest pain -coughing -presence of crackles Acute PE Medical emergency -can result in right ventricular dysfunction and pulmonary hypertension if not treated immediately Start anticoagulant (heparin IV) thearpy immediately

pulmonary embolism

complication of deep venous thrombosis (DVT) that occurs when part of a blood clot dislodges and is carried to the pulmonary artery it occludes the vessel and obstructs blood flow to the LUNGS

indications for amniocentesis

congenital anomalies (women older 35yo, women with previous child with chromosomal abnormality, family history of chromosomal anomalies)) pulmonary maturity L/S ratio ( but until no TDx FLM was used) fetal hymolitic disease

Subinvolution of the Uterus

delayed return of the enlarged uterus to normal size and function -major cause of late postpartum bleeding Causes: -retained placental fragments and pelvic infection Signs and Symptoms: -prolonged lochial discharge -irregular or excessive bleeding -hemorrhage prolonged lochial discharge, irregular or excessive bleeding and sometimes hemorrhage; a pelvic exam usually reveals larger than normal, boggy uterus. Tretament: Ergonovine (Ergotrate) or methylergonovine (Methergine) 0.2 mg q3-4 hr for 24-48 hours, D&C may be performed to remove retained placental fragments or deride placental site.

Clinical Manifestations of Miscarriages

depends on the duration of the pregnancy Signs of a threatened miscarriage early in pregnancy: -uterine bleeding -uterine contractions -abdominal pain Miscarriage that occurs before the 6th week of pregnancy: -heavy menstrual flow Miscarriage that occurs between weeks 6 and 12 -moderate discomfort and blood loss Miscarriages that occur after week 12: -severe pain (similar to labor)

What is the transvaginal ultrasound used for in the first trimester?

detect ectopic pregnancies monitor the development of the embryo identify abnormalities establish gestational age

common indications for antepartum testing

diabetes chronic hypertension preeclampsia fetal growth rstriction multiple gestation oligohydraminos preterm premature rupture of membrane late term or post term gestation previous stillbirths decreased fetal movement systemic lupus erythematous renal disease cholestasis of pregnancy

Although preterm birth is often not preventable,

early recognition of preterm labor is essential to implement interventions to reduce neonatal morbidity and mortality -transfer mother before birth to a hospital equipped to care for her preterm infant -give antibiotics during labor to prevent neonatal GBS infection -Administer glucocorticoids to women in labor to prevent respiratory distress syndrome -administer magnesium sulfate to women giving birth before 32 weeks to reduce incidence of cerebral palsy

nursing interventions for all pregnant women

education anticipatory plannin counseling for family adaptation assessment plannin gof appropriate interventions providing care to high risk women

fetal moves are not present

fetal sleep cycle reuced if woman takes depressant meds. alcohol, smoking obesity decreases perception

MRI

fetal structures ( CNS, thorax, abdomen, GU tract, MS system), overal growth placenta (position, density, presence of gestational tophoblastic disease quantity of amniotic fluid maternal structures ( uterus, cervix, adnexa, pelvis) biochemical status (pH,adenosine triphosphate content) of tissues and organs, soft tissues, metabolc and functional anomalies

ultrasound during 2-3 semester

fetal viability number, position and gestation age, growth pattern, anomalies, amniotic fluid volume, placental location and condition, presence of uterine fibroids, presence of adnexal masses cervical length

What is the initial management of excessive postpartum bleeding due to uterine atony?

firm massage of uterine fundus, eliminate bladder distention, and continuous IV infusion of 10-40 units of oxytocin with 1000 mL LR or NS.

contraction stress test (CST)

graded stress test of the fetus, and its purpose was to identify the jeopardized fetus that was stable at rest but showed evidence of compromise after stress early warning of fetal compromise semi-fowler 2 methods - nipple stimulating contraction test and oxytocin stimulated contraction test monitored with ultrasound transducer and uterine tocodynamometer

what is the function of hCG

hCG is produced during pregnancy. It can help support...progesterone

What is it HELLP associated w/?

increased risk for: Pulmonary edema Acute renal failure Disseminated intravascular coagulation (DIC) Placental abruption Liver hemorrhage or failure Adult respiratory distress syndrome Sepsis Stroke High risk for maternal death low platelet count

Total placenta previa

internal os is covered completely by the placenta when the cervix is dialated fully

Management of hypovolemic shock

involves restoring circulating blood volume and eliminating the cause of the hemorrhage The potential side effects of therapeutic interventions can further compromise the woman with a hemorrhagic disorder.

Disseminated intravascular coagulation

is a pathologic form of clotting that causes widespread external bleeding, internal bleeding, or both, and clotting. It always results from some event that triggered the clotting cascade

Preeclampsia

is a pregnancy-specific condition in which hypertension and proteinuria develop after 20 weeks of gestation in a woman who previously had neither condition. The cause of preeclampsia is unknown, and there are no known reliable tests for predicting women at risk for development of preeclampsia, although there are common risk factors associated with its development. Preeclampsia is a progressive disorder with the placenta as the root cause. It is a multisystem disease, and the pathologic changes are present long before clinical manifestations such as hypertension become evident. Women with preeclampsia (especially if it is early in onset or has severe features) have an increased risk for development of chronic hypertension and cardiovascular disease later in life.

Hemorrhagic (hypovolemic) shock

is an emergency situation in which the perfusion of body organs can become severely compromised and death can ensue

Phenylketonuria

is an inborn error of metabolism caused by an autosomal recessive trait. High levels of phenylalanine in the maternal bloodstream cross the placenta and are teratogenic to the developing fetus. Damage can be prevented or minimized by dietary restriction of phenylalanine before and during pregnancy.

A nurse providing care for the antepartum woman should understand that the contraction stress test (CST): sometimes uses vibroacoustic stimulation. is an invasive test; however, contractions are stimulated. is considered negative if no late decelerations are observed with the contractions. is more effective than nonstress test (NST) if the membranes have already been ruptured

is considered negative if no late decelerations are observed with the contractions. Rationale No late decelerations indicate a negative CST. Vibroacoustic stimulation is sometimes used with NST. CST is invasive if stimulation is by IV oxytocin but not if by nipple stimulation. CST is contraindicated if the membranes have ruptured.

A nurse providing care for the antepartum woman should understand that the contraction stress test (CST): sometimes uses vibroacoustic stimulation. is an invasive test; however, contractions are stimulated. is considered negative if no late decelerations are observed with the contractions. is more effective than nonstress test (NST) if the membranes have already been ruptured.

is considered negative if no late decelerations are observed with the contractions. Rationale No late decelerations indicate a negative CST. Vibroacoustic stimulation is sometimes used with NST. CST is invasive if stimulation is by IV oxytocin but not if by nipple stimulation. CST is contraindicated if the membranes have ruptured.

Preterm labor

is generally diagnosed clinically as regular contractions along with a change in cervical effacement or dilation or both or as presentation with regular uterine contractions and cervical dilation of at least 2 cm. Preterm birth is any birth that occurs between 20 0/7 and 36 6/7 weeks of gestation. • The cause of preterm labor is unknown and is assumed to be multifactorial; although risk factors have been identified, it is not possible to predict with certainty which women will experience preterm labor and birth Because the onset of preterm labor is often insidious and can be mistaken for normal discomforts of pregnancy, nurses should teach all pregnant women how to detect the early signs of preterm labor and assess for these symptoms at each prenatal visit.

A high risk pregnancy

is one in which the life or health of the mother, fetus, or newborn is jeopardized by circumstances coincidental with or unique to the pregnancy.

Gestational hypertension

is the onset of hypertension without proteinuria or other systemic findings diagnostic for preeclampsia after week 20 of pregnancy.

Eclampsia

is the onset of seizure activity or coma in a woman with preeclampsia who has no history of preexisting pathology that can result in seizure activity. The intent of emergency interventions for eclampsia is to prevent self-injury, enhance oxygenation, reduce aspiration risk, and establish control with magnesium sulfate.

Group B Strep (GBS)

leading Life-threatening perinatal infections 2. can occur in pregnant clientMeningitis, fasciitis, and intraabdominal abscess 3. transmission During vaginal delivey 4. Occurs within the 1st week p birth (usually 48hrs), in child can occur includes sepsis, pneumonia, or meningitis, and PERMANENT neuro disability can occur 5. diagnosis Done via vaginal AND rectal cultures at 35-37wks 6. peniccillin may be rx for during labor and birth 7. Baby will get IV abx

complications of amniocentesis in mother

leakage of amniotic fluid hemorrhage fetomaternal hemorrhage with possible maternal rH isoimmunization infection labor placental abruption inadvertent damage to the intestines or bladder amniotic fluid embolism

standard (also called basic)

level of ultrasound that evaluates fetal presentation, amniotic fluid volume, cardiac activity, placental position, fetal growth parameters, and number of fetuses. Also used to perform an anatomic survey of the fetus

postpartum hemorrhage (PPH)

loss of 500 mL of blood or more after vaginal birth or of 1000 mL of blood or more after cesarean birth additional criteria that may be used are a 10% change in hematocrit between admission for labor and postpartum or the need for erythrocyte transfusion

Incomplete abortion

loss of some of the products of conception occurs, with parts of the products retained (most often placenta is retained)

Preventing preeclampsia

low dose aspirin daily in first trimester no reliable test to predict or screen

uterine atony

marked hypotonia of the uterus it is the leading cause of early postpartum hemorrhage (PPH)

goals of managemtn ofsevere gestational hypertension with severe features

maternal safety assess degreee of fetal and maternal risk plan for birth formulate prevent eclampsia prevent placental abruption, HELLP syndrome, FUGR, fetal demise - oral hypertensive to maintain 160/110 or less - assessment for worsening -corticosteroids to enhance lung maturity (betamethasone) if less than 34 weeks intrapartum: - early identification of FHR abnormalities and prevention of maternal complication - bed rest in quite darkened environment - assessment for placental abruption, VS - emergency drugs, O2, sucction nearby - do not exceed 125 ml/hr iv fluids

assessment of severe gestational hypertension with severe features

matrnal: - monitoring BP, urine output, cerebral status, presence of epigastric pain, tenderness, labor, vaginal bleeding, platelet count, liver enzymes, serum createnine fetal: - FHR, biophysocal profile, fetal growth, amniotic fluid,

Cervical insufficiency

may be treated with a cervical or abdominal cerclage; after cervical change has been identified, the cerclage may be placed either prophylactically or as a therapeutic or rescue procedure.

The woman with hyperemesis gravidarum

may have significant weight loss, electrolyte imbalance, nutritional deficiencies, and ketonuria. Management focuses on correction of electrolyte and fluid imbalance and control of nausea and vomiting.

Obstetric emergencies

meconium-stained amniotic fluid, shoulder dystocia, prolapsed cord, rupture of the uterus, and amniotic fluid embolism) occur rarely but require immediate intervention to preserve the health or life of the mother and fetus or newborn

interventions for placental abnormalities

monitor for hemorrhage and shock prepare client for a hysterectomy

chronic hypertension

mostly non-hispanic black obese older age complications - superimposed preeclampsia, stroke, acute kideny injury, HF, placental abruption, death ( mother) complications (fetus) - IUGR, death, preterm birth smoking, alcohol stop, exercise, antihypertensive if persistently 160/105 or higher - labelol, nifedipine, methyldopa after birth monitor for complications - pulmonary edema, hypertensive ancephalopathy, renal failure can breastfeed

after evacuation of uterus

oxytocin is given after evacuation of the uterus to prevent hemorrhage? Ergot products e.g. ergonovine (methergine) or a prsotaglandin derivative (hemabate) to contract the uterus

A nurse receives the shift report when coming onto the antepartum floor. Which patient does the nurse anticipate will require a cesarean delivery? A patient who reports exhaustion A patient with complete placenta previa A patient with genital herpes who has not had an outbreak in 6 months A patient with a history of cesarean delivery with a low vertical uterine incision

patient with complete placenta previa Complete placenta previa is an indication for cesarean delivery, as this condition may result in hemorrhage during labor.

hypertensive disorders cause

perinatal morbidity and mortality - uteroplacental insufficiency and premature birth cause in mother - renal failure, coagulopathy, cardiac and live failure, placental abruption, seizures, stroke

hemorrhage assessment

persistent significant bleeding - perineal pad is soaked within 15 min restlessness, increased pulse rate, decreased BP, cool and clammy skin, ashen or grayish color complaints of weakness, lightheadedness, dyspnea

placenta increta

placenta penetrates the uterine muscle itself

placental position and function on ultrasound

placenta previa - second trimister, observe progress if resolves grading placental aging - calcium and fibrin could result in intervillous hemorrhagic infacts

goal glucose levels

premeal 60-99 postmeal (1hr) 100-129 postmeal (2 hrs) <120

Low birth weight can be caused

preterm birth, intrauterine growth restriction

Couvelaire uterus

purple or blue uterus caused by blood accumulating between the separated placenta and the uterine wall -shock may occur -complication of abruptio placentae

Chronic Villus Sampling (CVS)

removal of fetal tissue from the placenta for genetic diagnostic studies, which can be performed during either the first or second trimester 10-13 weeks genetic make up of fetus transcervically (sterile cather into cervix) or tranabdominally 18-20 gauge needle) relatively safe give rh neg mom rh D immunoglobulin after CVS

preeclampsia care

restricted activity ( not full bed rest) - diversional activities from boredom, gentle execise ( kegel, pelvic tilts, ROM), support system regular diet

cell-free DNA screening

screening method to detect the chromosomal abnormalities trisomy 21, 13, and 18. It also provides noninvasive prenatal genetic diagnosis of fetal Rh status, fetal gender, and certain paternally transmitted single gene disorders accurately predict the fetal status by measuring the amount of DNA circulating in maternal blood and comparing it with known standards; maternal blood is obtained by venipunture and sent to a commercial laboratory

Coombs test

screening tool for Rh incompatibility

Late miscarriage

second trimester loss occurring between 12 and 20 weeks of gestation

eclampsia

seizure act activity or coma in woman diagnosed w/ preeclampsia with no history of pathlogy

inevitable abortion

spotting and cramping occur & cervix begins to dilate and efface

threatened abortion

spotting and cramping occur w/out cervical change

Doppler blood flow analysis

study of blood flow in the fetus and placenta non invasively using ultrasound helps manage maternal HT, diabetes, IUGR, multiple fetuses S/D ratios greater than 3 - plasental vascular disease

cerclage

suturing of the cervix to prevent it from dilating prematurely during pregnancy, thus decreasing the chance of a spontaneous abortion -can be placed prophylactically or therapeutically -usually placed at 12-14 weeks of gestation if prophylactic (due to previous miscarriages) -Placed therapeutically at 14-23 weeks of gestation -removed in the event of PROM -if pregnancy progresses with no complications, cerclage is removed at 36 weeks rescue placed - 16-23 weeks when found cervical change (>1 dilated, prolapsed membranes)

Magnesium sulfate

the anticonvulsant of choice for preventing or controlling eclamptic seizures, requires careful monitoring of reflexes, respirations, and renal function.

partial placenta previa

the lower border of the placenta is within 3 cm of the internal cervical os, but does not fully cover it

marginal low lying placenta previa

the placenta is implanted in the lower uterus, but its lower border is more than 3 cm from the internal cervical os

Induced abortion

therapeutic or elective reasons exist for terminating pregnancy

superficial venous thrombosis

thromboembolic condition that involves superficial saphenous venous system

deep venous thrombosis (DVT)

thromboembolic condition where involvement varies but that occurs most often in the lower extremities can extend from the foot to the iliofemoral region

Administration of antenatal glucocorticosteroids to the mother

to accelerate fetal lung maturity by stimulating fetal surfactant production is now considered one of the most effective and cost-effective interventions for preventing morbidity and mortality associated with preterm labor.

The best reason to use tocolytics, or medications given to arrest labor after uterine contractions and cervical change have occurred,

to achieve sufficient time to administer glucocorticoids in an effort to accelerate fetal lung maturity.

von Willebrand disease (vWD)

type of hemophilia, deficiency or defect in blood clotting protein; probably most common of all hereditary bleeding disorders. Symptoms of vWD recurrent bleeding episodes (i.e. nose bleeds), bruising easily, prolonged bleeding time, factor VIII deficiency and bleeding from mucous membranes; can be at risk for bleeding for up to 4 weeks postpartum. Treatment for vWD Desmopressin, which promotes release of vWF and factor VIII, transfusion therapy with plasma products may be used. -A type of hemophilia -most common hereditary bleeding disorder -deficiency in a blood clotting protein- von Willebrand factor Symptoms: -recurrent bleeding episodes (e.g. nosebleeds) -bruising easily -prolonged bleeding time -factor VIII deficiency -bleeding from mucous membranes Woman can be at risk for bleeding up to 4 weeks after birth Treatment- administration of desmopressin whcih promotes release of vWF or transfusion therapy with plasma products

limited ultrasound

ultrasound performed for specific indications such as identifying fetal presentation during labor or estimating AFV (amniotic fluid volume)

Which hematologic disorder is transferred genetically from parents to offspring? Deep vein thrombosis von Willebrand disease Superficial vein thrombosis Idiopathic thrombocytopenia

von Willebrand disease Rationale Von Willebrand disease is a hereditary disorder. It is a type of hemophilia caused by the deficiency of ablood clotting factor called von Willebrand factor. Deep vein thrombosis is not a hereditary disorder. It is caused by inflammation or partial obstruction of deep veins in the lower limb. Superficial vein thrombosis is not a hereditary disorder. It is caused by the inflammation or obstruction of superficial veins of the lower limb. Idiopathic thrombocytopenia is an autoimmune disorder. It is not transferred from parents to offspring.

Pregestational Diabetes

women who have either type 1 or type 2 diabetes before pregnancy -may be complicated by vascular disease, retinopathy, nephropathy, or other diabetic complications -glycemic control may be improved during first trimester when insulin response is enhanced

Perinatal loss

•Associated with miscarriage, neonatal death, stillbirth, and therapeutic abortion. •Loss and grief may also occur with the birth of a preterm baby, a newborn with complications of birth, or a newborn with congenital anomalies; it may also occur in a client who is giving up a child for adoption. Not all interventions are appropriate for every woman and her significant family. It is crucial to consider religious and cultural health care practices and beliefs when planning care for a woman and family who have experienced

Assessment for Femoral Thrombophlebitis

■ Malaise ■ Chills and fever ■ Diminished peripheral pulses ■ Shiny white skin over affected area ■ Pain, stiffness, and swelling of affected leg

Assessment for Superficial Thrombophlebitis

■ Palpable thrombus that feels bumpy and hard ■ Tenderness and pain in affected lower extremity ■ Warm and pinkish red color over the thrombus area


Related study sets

2.4.W - Lesson Review: Matter & Periodic Table / Bonding/ Water & pH

View Set

Lisette's NCLEX MED SURG Study #3

View Set

Chapter 3 Gov Test on the President

View Set

Physics Chapter 5: Forces Questions

View Set

OB-GYN: Endometriosis and Adenomyosis

View Set

N326 Final Exam Practice Questions

View Set

Construction Materials Exam 1 Review

View Set